Sunteți pe pagina 1din 284

LAURENŢIU PANAITOPOL, VIOREL BĂNDILĂ, MIRCEA LASCU

INEGALITĂŢI

Editura GIL
ZALĂU, 1996
© 1996 Editura GIL

INEGALITĂŢI

LAURENŢIU PANAITOPOL, VIOREL BĂNDILĂ, MIRCEA LASCU

ISBN 978-606-500-041-4

Toate drepturile rezervate Editurii Gil.


Nici o parte din acest volum nu poate fi copiată fãră permisiunea scrisă a Editurii Gil.
Copyright © 1996 by Gil. All rights reserved.

Departament difuzare:

Editura GIL,
CP 44 O.P.3,
Zalău, Sălaj,
cod 450200

Tel. 0260/616314;
0744/612106
Fax.: 0260/616414
E-mail:
gil1993@zalau.astral.ro

Editor: Mircea Lascu

www.gil.ro
Cuprins

1 Puteri, radicali, probleme de ordonare 5


1.1 Enunţuri . . . . . . . . . . . . . . . . . . . . . . . . . . . . . . . 5
1.2 Probleme propuse . . . . . . . . . . . . . . . . . . . . . . . . . . 10
1.3 Soluţii . . . . . . . . . . . . . . . . . . . . . . . . . . . . . . . . 13

2 Inegalităţi clasice 37
2.1 Enunţuri . . . . . . . . . . . . . . . . . . . . . . . . . . . . . . . 37
2.2 Probleme propuse . . . . . . . . . . . . . . . . . . . . . . . . . . 47
2.3 Soluţii . . . . . . . . . . . . . . . . . . . . . . . . . . . . . . . . 56

3 Principiul trinomului 103


3.1 Enunţuri . . . . . . . . . . . . . . . . . . . . . . . . . . . . . . . 103
3.2 Probleme propuse . . . . . . . . . . . . . . . . . . . . . . . . . . 108
3.3 Soluţii . . . . . . . . . . . . . . . . . . . . . . . . . . . . . . . . 110

4 Inegalităţi cu numere complexe 125


4.1 Enunţuri . . . . . . . . . . . . . . . . . . . . . . . . . . . . . . . 125
4.2 Probleme propuse . . . . . . . . . . . . . . . . . . . . . . . . . . 128
4.3 Soluţii . . . . . . . . . . . . . . . . . . . . . . . . . . . . . . . . 131

5 Inducţie. Combinatorică. Şiruri 145


5.1 Enunţuri . . . . . . . . . . . . . . . . . . . . . . . . . . . . . . . 145
5.2 Probleme propuse . . . . . . . . . . . . . . . . . . . . . . . . . . 147
5.3 Soluţii . . . . . . . . . . . . . . . . . . . . . . . . . . . . . . . . 151

6 Funcţia exponenţială şi logaritmică 163


6.1 Enunţuri . . . . . . . . . . . . . . . . . . . . . . . . . . . . . . . 163
6.2 Probleme propuse . . . . . . . . . . . . . . . . . . . . . . . . . . 166
6.3 Soluţii . . . . . . . . . . . . . . . . . . . . . . . . . . . . . . . . 168

1
7 Inegalităţi polinomiale 179
7.1 Enunţuri . . . . . . . . . . . . . . . . . . . . . . . . . . . . . . . 179
7.2 Probleme propuse . . . . . . . . . . . . . . . . . . . . . . . . . . 182
7.3 Soluţii . . . . . . . . . . . . . . . . . . . . . . . . . . . . . . . . 187

8 Inegalităţi trigonometrice 205


8.1 Enunţuri . . . . . . . . . . . . . . . . . . . . . . . . . . . . . . . 205
8.2 Probleme propuse . . . . . . . . . . . . . . . . . . . . . . . . . . 210
8.3 Soluţii . . . . . . . . . . . . . . . . . . . . . . . . . . . . . . . . 211

9 Inegalităţi diverse 231


9.1 Enunţuri . . . . . . . . . . . . . . . . . . . . . . . . . . . . . . . 231
9.2 Probleme propuse . . . . . . . . . . . . . . . . . . . . . . . . . . 240
9.3 Soluţii . . . . . . . . . . . . . . . . . . . . . . . . . . . . . . . . 246

Indice de autori 279

2
Cuvânt ı̂nainte

Se spune că relaţia care guvernează cu adevărat matematica este cea de


inegalitate, egalitatea fiind caz special. Dacă ne gândim de pildă la problem-
atica Analizei matematice aserţiunea de mai sus este evidentă. De aceea sun-
tem de părere că un studiu sistematic al inegalităţilor este neapărat necesar.
Cunoaşterea rezultatelor de bază, a inegalităţilor remarcabile şi a tehnicilor cu
aplicabilitate largă este unul din scopurile lucrării de faţă. În intenţia de a ne
alinia programei şcolare, aşa ı̂ncât cartea de faţă să se constituie ı̂ntr-un auxil-
iar al manualelor, capitolele vizează direct temele din manuale, ceea ce rezultă
cu claritate din chiar titlurile capitolelor ”Principiul trinomului”, ”Inegalităţi
cu numere complexe”, ”Funcţia exponenţială şi logaritmică”, ”Inegalităţi poli-
nomiale”.
Lucrarea este structurată ı̂n nouă capitole, fiecare capitol cuprinzând câte
trei secţiune distincte: ”Enunţuri”, ”Probleme propuse” şi ”Soluţii”. Prob-
lemele propuse sunt marcate diferenţiat (cu simbolul ).
Dacă ı̂n finalul capitolelor şi ı̂n special la ”Inegalităţi diverse” există
exerciţii destinate ”performerilor”, trebuie spus că problemele cu care de-
butează capitolele sunt accesibile unei mase mari de cititori printre care se
numără şi elevii de gimnaziu, care pot rezolva probleme din primul capitol,
punând astfel bazele unei pregătiri solide viitoare.
În final, aducem mulţumiri colegilor noştri Marinescu Dan Ştefan, Stănean
Marius şi András Szilard pentru problemele puse la dispoziţie ı̂n vederea
editării acestei lucrări.

Autorii

3
4
Capitolul 1

Puteri, radicali, probleme de


ordonare

1.1 Enunţuri
1.1. Scrieţi ı̂n ordine crescătoare numerele: 414 , 512 , 711 şi 117 .

1.2. Arătaţi că: 1) 21987 > 31241 ; 2) 31987 > 41490 ; 3) 41987 > 51703 .

1.3. Să se arate că: 7 > 2 7.

1.4. Să se arate, fără a extrage rădăcina pătrată, că:


√ √ √ √ √ √ 20
1) 2 + 6 + 8 < 7; 2) 3 + 5 + 7 < ;
3
√ √ √ √
3) 2 + 3 + 2 2 + 6 < 5.
√ √
1.5. Care număr este mai mare: 2 3 sau 3 2 ?
√√ √ √√ √
1.6. Comparaţi 13 − 11 cu 19 − 17. Generalizare.
√ √ √ √
1.7. Care număr este mai mare: 3 2 + 3 sau 3 3 + 2?

1.8. Fie a, b, c, d > 0.


1 1 1 1 √ √ √ √
1) Dacă a + b > c + d şi + < + , atunci: a + b > c + d.
a b c d
√ √ √ √
3 6 4 5
2) Comparaţi numerele √ + √ şi √ + √ ; aceeaşi cerinţă pentru
√ √ √ 2 √ 5 3 4
2 5 3 4
numerele: √ + √ şi √ + √ .
3 6 4 5

5
1.9. Să se determine cel mai mic număr natural n astfel ı̂ncât
√ √
n − n − 1 < 0, 01.
√ √ √
1.10. Care număr este mai mare: 1001 + 999 sau 2 1000?
√ √ √
1.11. Demonstraţi inegalitatea: x + 1 + 2x − 3 + 50 − 3x ≤ 12 pentru
toate valorile lui x pentru care membrul stâng are sens.

1.12. Să se demonstreze că pentru orice x ∈ [0, 1] avem:

x x2 √ x
1+ − ≤ 1+x≤1+ .
2 8 2

1.13. Dacă x, y, z > 0, atunci: xy + yz + zx ≥ 3xyz(x + y + z).
√ √
1.14. Dacă x, y ≥ 1, atunci: x y − 1 + y x − 1 ≤ xy.
√ √ √
1.15. Dacă x, y, z ≥ 0, atunci: xy + yz + zx ≥ x yz + y zx + z xy.

1.16. Să se arate că dacă x ≥ y ≥ z ≥ 0, atunci:


√ √ √
(x + z)(y + z) + (x − z)(y − z) ≤ 2 xy.

1.17. Dacă x, y, z > 0, atunci:


√ √ √ √
x(y + z) + y(z + x) + z(x + y) ≤ 2(x + y + z).

1.18. Dacă |x|, |y| ≤ 1, atunci:


√ ( )
√ √ x+y 2
1 − x2 + 1 − y 2 ≤ 2 1 − .
2

1.19. Dacă x > 0, atunci:


√ √ √
x+1 x+2 x+3
x + (x + 1) + (x + 2)
x+2 x+3 x+4
√ √
x
+(x + 3) > 4 x2 + 3x + 1 − 4.
x+1
√ √ √ √
1.20. Să se compare numerele A = x + x + 4 şi B = x + 2 + x + 3,
unde x > 0.

6
√ √
1.21. Dacă x ∈ [−1, 1], atunci: 2, 73 < 2 + x + 2 − x < 2, 83.
√ √ √ √ √
1.22. Dacă a ≥ b ≥ |x|, atunci: a − b + a + b ≤ a − x + a + x ≤ 2 a.
√ [ ]
3 − 1 − x2 1
1.23. Să se arate că dacă x ∈ [−1, 0], atunci: ∈ ;1 .
x+4 2
1.24. Fie a, b, c ∈ R cu b + c = a > 0. Dacă x, y ∈ R şi
√ √
a − bx − cy + a + by + cx = a

să se demonstreze că |x + y| ≤ a.


√ √ √
1.25. Dacă a, b, c > 0 şi 1993 + a + 1993 + b = 2 1993 + c, atunci:

a + b ≥ 2c.

1.26. Pentru x, y ∈ R să se determine minimul expresiei


√ √ √
E(x, y) = x2 + y 2 + x2 + (y − 2)2 + (x − 2)2 + y 2

+ (x − 2)2 + (y − 2)2 .
În ce caz avem egalitate?

√ m √ m 6−1
1.27. Dacă m, n ∈ N∗ şi 23 − > 0, atunci: 23 − > .
n n mn
1.28. Demonstraţi că:

x x2 √ x x2 5
1+ − ≤ 31+x≤1+ − + x3 , pentru orice x ≥ 0.
3 9 3 9 81
√ √ √
1.29. Să se arate că pentru orice x, y > 0, avem: 3
4(x + y) ≥ 3
x+ 3 y.

1.30. Să se arate că pentru orice n ∈ N∗ , avem:



3
√ √ √
n + 3 + n < 3 n + 2 + n + 1.

3

3

3

3
1.31. Să se arate că: 43 < 3+ 9< 44.
√ √
1.32. Să se determine x ∈ N astfel ı̂ncât să avem: x <
3
3
x+ x2 < x + 1.
√ √ √ √ √
1.33. Demonstraţi că: 3 + 3 3 + 3 − 3 3 < 2 3 3.
3 3

√ √
1.34. Dacă n ∈ N, n ≥ 3, avem: n+1 n + 1 ≤ n n.

7
1.35. Demonstraţi inegalitatea:
√ √
√ √ √
n + n + n − n n ≤ 2 n n, pentru orice n ∈ N∗ − {1}.
n n n

1.36. Să se demonstreze că dacă n ∈ N∗ − {1} şi x ∈ (0, 2) atunci:


( )( )
1 1
1+ √ 1 + √ ≥ 4.
n
x n
2−x

1.37. Numerele a, b, c sunt strict pozitive. Să se compare numerele:


a b c b c a
A= + + şi B= + + .
b c a a b c

1.38. Fie a > b > 0. Care este mai mare:


1 + a + a2 + . . . + an−1 1 + b + b2 + . . . + bn−1
A= sau B= .
1 + a + a2 + . . . + an 1 + b + b 2 + . . . + bn

a c
1.39. Demonstraţi că dacă a > b > c > d > 0 şi = atunci a + d > b + c.
b d
1.40. Fie d ≥ c ≥ b ≥ a ≥ 0 şi a + b + c + d = 4. Găsiţi cea mai mare valoare
pe care o poate lua fiecare din variabilele a, b, c, d.

1.41. Se consideră numerele reale a, b cu a < b. Demonstraţi că:

(1 − x)a + bx ∈ (a, b) ⇔ x ∈ (0, 1).

1.42. Dacă a ≤ b ≤ c şi a + b + c ≥ 1, atunci: a2 + 3b2 + 5c2 ≥ 1.

1.43. Să se arate că dacă a, b > 0, atunci:

min(a, b) a2 − 2ab + 3b2 max(a, b)


≤ 2 2
≤ .
a a +b a

1.44. Dacă a, b, c, d sunt numere prime diferite, atunci:

abc + bcd + cda + abd + 173 ≤ 2abcd.

1.45. Fie x, y, z > 0. Să se arate că:

x3 + 8y 3 + 27z 3 ≤ 18xyz ⇔ x = 2y = 3z.

8
1.46. Să se arate că: x4 + y 4 + 2xy(x2 + y 2 ) + 3x2 y 2 − x2 − y 2 − 4xy + 1 ≥ 0,
pentru orice x, y ∈ R. În ce caz avem egalitatea?
1 + x2 + x4 3
1.47. Să se arate că pentru orice x > 0, avem: 3
≥ .
x+x 2
1.48. Dacă x ≥ 0, atunci: (x + 2)4 − 16x3 > 0.
( 2 )
x y2 x y
1.49. Dacă x, y > 0, atunci: 2 2
+ 2 ≥ + + 2.
y x y x
1.50. Dacă x, y ≥ 0, atunci (x + y)(x4 + y 4 ) ≥ (x2 + y 2 )(x3 + y 3 ).

1.51. Dacă a, b, c > 0, atunci:


1) (a + b + c)(ab + bc + ca) ≥ 9abc;
2) (a + b + c)(a2 + b2 + c2 ) ≥ 9abc;
3) a3 + b3 + c3 ≥ 3abc;
4) (a + b)(b + c)(c + a) ≥ 8abc.
a b c 3
1.52. Dacă a, b, c > 0, atunci: + + ≥ .
b+c c+a a+b 2
1.53. Dacă a, b, c > 0, atunci: (−a + b + c)(a − b + c)(a + b − c) ≤ abc.

1.54. Dacă x, y, z > 0, atunci:

(x + y)3 + (y + z)3 + (z + x)3 ≥ 21xyz + x3 + y 3 + z 3 .

1.55. Să se demonstreze că pentru orice x, y, z > 0, avem:


√ √
x+y x+z y+z
+ ≤ √ .
x+z x+y yz

1.56. Dacă a, b, c, d > 0, atunci:


a + b + c − d a + b − c + d a − b + c + d −a + b + c + d
+ + + ≥ 8.
d c b a

1.57. Să se arate că dacă a ≥ 2, atunci:

x4 − 2x3 + ax2 − 2x + 1 ≥ 0, pentru orice x ∈ R.

1
1.58. Demonstraţi că x4 − x + > 0, pentru orice x ∈ R.
2

9

1.59. Să se arate că: (x + y)2 + 1 ≥ xy + 3(x + y), pentru orice x, y ∈ R.
a2 − ab + b2 1
1.60. Să se arate că pentru orice a, b ∈ R − {0} avem: 3 ≥ ≥ .
a2 + ab + b2 3
1.61. Dacă a, b ∈ R, atunci: 2(a4 + b4 ) + 9 ≥ 12ab.

1.62. Să se arate că pentru orice x, y ∈ R:

2(x2 − x + 1)(y 2 − y + 1) ≥ 1 + x2 y 2 .

1.63. Dacă x, y, z ∈ R, atunci: (1 − x2 + x4 )(1 − y 2 + y 4 )(1 − z 2 + z 4 ) ≥ x2 y 2 z 2 .


2 2 2 9
1.64. Să se arate că: + + ≥ pentru orice x, y, z > 0.
x+y y+z z+x x+y+z
x2 y 2 z2 x y z
1.65. Dacă x, y, z > 0, atunci: + + ≥ + + .
y2 z 2 x2 z x y
1.66. Arătaţi că dacă x, y, z ∈ R, atunci:
3
(x + y − z)2 + (x − y + z)2 + (−x + y + z)2 + ≥ x + y + z.
4

1.67. Dacă x, y, z ∈ R, atunci:


{ }
3(x − y)2 3(y − z)2 3(z − x)2
x2 + y 2 + z 2 − xy − yz − zx ≥ max , , .
4 4 4

1.68. Fie a, b, c, d ∈ R. Demonstraţi că: a2 + b2 + c2 + d2 − a − b − c − d + 1 ≥ 0.



1.69. Dacă ad − bc = 1, atunci: a2 + b2 + c2 + d2 + ac + bd ≥ 3.

1.70. Fie x, y ∈ R astfel ı̂ncât x2 +y 2 = x+y+1. Determinaţi valoarea maximă


şi minimă a numerelor x şi y.

1.2 Probleme propuse


 1.1. Arătaţi că: 3111 < 1714 .

 1.2. Să se demonstreze că: 123321 > 321123 .

 1.3. Care dintre numerele 711991 şi 39119 este mai mare?
√ √ √ √
 1.4. Să se arate că: 8 + 3 + 5 + 2 2 < 4.

10
 1.5. Arătaţi că:
√ √ √ √ √ √ √ √ √ √
2 + 6 + 12 + 20 + 30 + 42 + 56 + 72 + 90 + 110 < 60.

2− 2
 1.6. Să se arate că: 3 < √ √ < 4.
2− 2+ 2
1
 1.7. Dacă x, y, z ≥ − şi x + y + z = 1, atunci:
4
√ √ √
4x + 1 + 4y + 1 + 4z + 1 ≤ 5.
√ √ √
6 20 2n(2n + 1) n
 1.8. Să se arate că: + +. . .+ < , pentru orice n ∈ N∗ .
5 9 4n + 1 2
√ √ √
 1.9. Determinaţi x, y ∈ Z ştiind că x − 1 + y − 1 ≥ xy.

√ √ x + y (x + y)2
 1.10. Dacă x, y ≥ 0, atunci: x y + y x ≤ + .
4 2

 1.11. Dacă x, y, z ≥ 0, atunci: x(1 + y) + y(1 + z) + z(1 + x) ≥ 6 xyz.
√ √
 1.12. Dacă x ∈ [−1, 1], calculaţi expresia: E(x) = 3 + x + 3 − x cu o
zecimală exactă.

 1.13. Fie a, b > 0 cu a + b = 6. Pentru fiecare x ∈ [−b, a] considerăm numărul


√ √
Ax = a − x + b + x − 4.

Să se arate că Ax < 0, pentru orice x ∈ [−b, a].



 1.14. Dacă ax2 + by 2 = c, unde a, b, c > 0, atunci: ax + by ≤ c(a + b).
√ √ √ √
 1.15. Să se arate că: 1 + 2 > 2 + 3.
3

√ √
 1.16. Demonstraţi că pentru n ∈ N, n ≥ 3 are loc inegalitatea: 3
n> 4
n + 1.
√ √ √ √
 1.17. Arătaţi că: 4 3 + 4 7 > 4 2 + 4 8.
√ √ 1
 1.18. Să se afle cel mai mic număr natural n, pentru care: 3
n + 1− 3
n< .
10
√ √ √
 1.19. Dacă a, b > 0, atunci: 2 a + 3 3 b ≥ 5 5 ab.
7 8 9 1993 1994
 1.20. Scrieţi ı̂n ordine crescătoare fracţiile: , , ,..., , .
10 15 20 9940 9945
Generalizare.

11
x2 2x x − y
 1.21. Dacă x > y > 0, atunci + + ≥ 8.
(x − y)y y y
√ √
 1.22. Demonstraţi că dacă x, y ∈ [−1, 1], atunci: x 1 − y 2 + y 1 − x2 ≤ 1.
1
 1.23. Fie 0 < y < x < 1. Să se demonstreze inegalitatea: x(1−x)+y(x−y) ≤ .
3
 1.24. Dacă a, b, c ∈ (0, 1), atunci:
a+b a+b b+c c+a
max(a, b) < şi < + .
1 + ab 1 + ab 1 + bc 1 + ca

 1.25. Dacă |x|, |y| < 2, atunci: |xy| − |x| − |y| + 2 > 0.

 1.26. Se dau numerele reale a1 , a2 , a3 , a4 , a5 cu a1 ≤ a2 ≤ a3 ≤ a4 ≤ a5 .


Demonstraţi că:
a1 + a2 + a3 a1 + a2 + a3 + a4 + a5
≤ .
3 5

 1.27. Dacă ak , bk ∈ R cu bk > 0, pentru orice k = 1, n, atunci:


ak a1 + a2 + . . . + an ak
min ≤ ≤ max .
1≤k≤n bk b1 + b 2 + . . . + bn 1≤k≤n bk

 1.28. Arătaţi că dacă a, b, c, d, e, f ∈ R şi a2 + b2 ≤ e2 , c2 + d2 ≤ f 2 , atunci

ac + bd ≤ |ef |.
√ √ √ √
 1.29. Să se arate că: x2 + y 2 + y 2 + z 2 + z 2 + x2 ≥ 2(x + y + z), pentru
orice x, y, z ∈ R.

 1.30. Demonstraţi că dacă x, y > 0, avem:


1 1 1 1
+ ≤ 2 + .
x2 + y y 2 + x x + x y2 + y

 1.31. Să se arate că pentru orice x, y ∈ R, avem:

2|(x + y)(1 − xy)| ≤ (1 + x2 )(1 + y 2 ).

 1.32. Dacă a, b ∈ R, să se demonstreze că: 8(a4 + b4 ) ≥ (a + b)4 .

 1.33. Să se demonstreze că pentru orice x, y ∈ R, avem: 4(x6 +y 6 ) ≥ (x2 +y 2 )3 .

12
xy yz zx
 1.34. Dacă x, y, z > 0, atunci: + + ≥ x + y + z.
z x y
|x + y| |x| |y|
 1.35. Dacă x, y ∈ R, atunci: ≤ + .
1 + |x + y| 1 + |x| 1 + |y|
 1.36. Demonstraţi că pentru orice a, b, c > 0 avem:
( ) ( )
1 1 1 1 1 1
2 2
(a + b + c ) 2
+ + ≥ (a + b + c) + + .
a2 b2 c2 a b c

 1.37. Dacă x, y, z > 0 şi x + y + z = 1, atunci:

(1 + x)(1 + y)(1 + z) ≥ 8(1 − x)(1 − y)(1 − z).

4
 1.38. Dacă x, y ∈ R şi x(x + y − 1) + y(y − 1) = 0, atunci: x + y ≤ .
3
 1.39. Dacă x, y, z ∈ R, z > 0 şi x(x − z) + y(y + z) = 0, atunci x + y ≤ z.

 1.40. Dacă x, y ∈ R şi x2 − xy + y 2 = x + y, să se arate că x + y ≤ 4.

 1.41. Dacă x ∈ [0, 1] şi y ∈ [−1, 2], atunci: −1 ≤ (x + y)(x + y − 2) ≤ 3.


( 2 ) ( )
x y2 x y
 1.42. Dacă x, y ∈ R \ {0} şi 4 + 2 − 20 + + 33 ≤ 0, atunci
y2 x y x
x = 2y sau y = 2x.

 1.43. Fie x, y ∈ R şi x2 + y 2 = 1. Să se afle maximul expresiei xy(x + y).

 1.44. Fie a, b ∈ R şi a2 + b2 = 2. Să se găsească maximul expresiei ab(a2 − b2 ).

1.3 Soluţii
1.1. Vom arăta că: 117 < 512 > 414 < 711 . Avem:

512 = 256 > 226 = 116 · 26 > 116 · 11,

căci

26 = 64 > 11; 414 > 512 ⇔ 47 > 56 ⇔ 214 > 56 ⇔

27 > 53 ⇔ 128 > 125.


Ultima inegalitate rezultă din următoarele:

73 = 49 · 7 = 343 > 46 = 256

13
şi
78 > 410 ⇔ 74 > 45 ⇔ 72 > 25 ⇔ 49 > 32.

Avem: 3111 < 3211 = (25 )11 = 255 < 256 = (24 )14 = 1614 < 1714 , deci
3111 < 1714 .

1.2. 1) Avem:

21987 = 28·248+3 = (28 )248 · 23 = 256248 · 8 > 243248 · 3

= (35 )248 · 3 = 31240 · 3 = 31241 .

2) 31987 = 32·993+1 = (32 )993 · 3 = 9993 · 3 şi 41490 = 22980 = 23·993+1 =


(23 )993 · 2 = 8993 · 2.
Din cele două reprezentări de mai sus rezultă că 31987 > 41490 .
3) Arătăm că 4 · 41987 > 4 · 51703 . Într-adevăr, avem:

4·41987 = 41988 = 47·284 = (27 )568 > 125568 = (53 )568 = 51703 ·5 > 4·51703 ,

de unde 41987 > 51703 .


11
1.3. Deoarece 74 = 2301 > 2048 = 211 , rezultă 74 > 211 , de unde 7 > 2 4 .
11 √ √
Rămâne de demonstrat că > 7 ⇔ 11 > 4 7 ⇔ 121 > 112, ceea
4
ce este evident.
√ √ 1+2 3 √ √ 2+3 5
1.4. 1) Avem 2 = 1 · 2 < = , 6= 2·3< = şi
2 2 2 2
√ √ 3
8 = 2 2 < 2 = 3.
2

√ √ √ 3 5
Însumând membru cu membru obţinem că 2+ 6+ 8 < + +3 = 7.
2 2
√ √ √ √
2) Din inegalitatea a+b ≤ 2(a + b ) rezultă 3+ 5 < 2(3 + 5) = 4
2 2

şi deci mai trebuie verificat că


√ 20 √ 8 64
7< −4 ⇔ 7< ⇔ 7< ⇔ 63 < 64,
3 3 9
ceea ce este evident.
√ √ √ √ √ √
3)
√ Avem: √ 2 + 3 + 2 2 + 6 < 2 + 2 + 3 + 6 = 2 + 3 = 5, căci
3 < 2 şi 2 < 1, 5.

14

2 3 √
1.5. Formăm raportul √ şi ı̂l ridicăm la puterea 6. Obţinem:
3 2
( √ )√6 √ √
2 3 2 18 (23 ) 2
√ = √ = √ < 1,
3 2 3 12 (32 ) 3

inegalitate adevărată deoarece atât baza cât şi exponentul


√ √ numărătorului
sunt mai mici decât ale numitorului, rezultă că 2 3 < 3 2 .
√√ √√ √

1.6. Dacă 0 ≤ a < b şi 0 < k, avem: a+k− a > b + k − b.
Pentru a demonstra această inegalitate este suficient a arăta că:
√ √ √ √
a + k − a > b + k − b,

ceea ce rezultă din identitatea:


√ √ √ √ a+k−b−k √ √
a + k − b + k − ( a − b) = √ √ − ( a − b)
a+k+ b+k
a−b a−b
=√ √ −√ √
a+k+ b+k a+ b
√ √ √ √
a+ b− a+k− b+k
= (a − b) √ √ √ √ > 0.
( a + b)( a + k + b + k)
√√ √ √√ √
Pentru a = 0, b = 17, k = 2 obţinem 13 − 11 > 19 − 17.
√ √ √ √
1.7. Vom demonstra că: 3 2 + 3 > 3 3 + 2. Avem:
√ √ √ √
1 1
3− 3− 2 ⇔ √
√ > √ √ √ ⇔
3 3
2> 3
3+ 2 9+ 6+ 34
3

√ √ √3

3
√3
3 + 2 < 9 + 6 + 4.
√ √ √ √ √ √
Dar 3 9 > 3 8 = 2 > 3 şi 3 6 + 3 4 > 1 + 1 > 2, de unde rezultă
afirmaţia făcută.
√ √ √ √ √ √
1.8. 1) Avem: a + b > c + d ⇔ a + b + 2 ab > c + d + 2 cd. Din
enunţ rezultă că:
c+d a+b 1 1 1 1 c+d
< = + < + = ,
ab ab a b c d cd
deci cd < ab. Rezultă că
√ √
(a + b) + 2 ab > (c + d) + 2 cd,

15
√ √ √ √
de unde a+ b> c+ d.
2) Avem:
3 6 4 5 27 31
+ > + ⇔ > ⇔ 324 > 310
2 5 3 4 10 12
şi
2 5 3 4 27 31
+ < + ⇔ < ⇔ 540 < 558,
3 6 4 5 18 20
folosind 1) rezultă că
√ √ √ √
3 6 4 5
+ > + .
2 5 3 4
3 4 2 5 4 5 3 6
Apoi, din + > + şi + < + rezultă
4 5 3 6 3 4 2 5
√ √ √ √
3 4 2 5
+ > + .
4 5 3 6

1.9. Avem:
√ √ √ √
n− n − 1 < 0, 01 ⇔ n − 0, 01 < n−1 ⇔
√ √
n − 0, 02 n + 0, 0001 < n − 1 ⇔ 1, 0001 < 0, 02 n ⇔
1, 0002 < 0, 0004n ⇔ 2500, 5 < n.
Rezultă că valoarea căutată este n = 2501.
√ √ √
1.10. Verificăm mai ı̂ntâi că: x + 1 + x − 1 ≤ 2 x, pentru orice x ≥ 1.
Ridicând ambii membri la pătrat avem:
√ √ √
( x + 1 + x − 1)2 ≤ 4x ⇔ x + 1 + x − 1 + 2 x2 − 1 ≤ 4x ⇔

x2 − 1 ≤ x ⇔ −1 ≤ 0.
√ √ √
Luând x = 1000 rezultă că 1001 + 999 < 2 1000.

1.11. Folosind inegalitatea a+b+c ≤ 3(a2 + b2 + c2 ), pentru orice a, b, c ≥ 0,
rezultă:
√ √ √ √
x + 1 + 2x − 3 + 50 − 3x ≤ 3(x + 1 + 2x − 3 + 50 − 3x)

= 3 · 48 = 12.

16
1.12. Ridicând ı̂n toţi membrii inegalităţii la pătrat, avem de verificat că:

x3 x4 x2
1+x− + <1+x<1+x+ .
8 64 4
Inegalitatea din stânga este echivalentă cu

x4 x3
≤ ⇔ x≤8
64 8
iar cea din dreapta este evidentă.

1.13. Avem
√ (∑ ) (∑ )2 (∑ )

xy ≥ 3xyz x ⇔ xy ≥ 3xyz x ⇔

∑ (∑ ) (∑ )
x2 y 2 + 2xyz x ≥ 3xyz x ⇔
∑ (∑ ) 1∑ 2
x2 y 2 ≥ xyz x ⇔ x (y − z)2 ≥ 0.
2
Egalitatea are loc dacă şi numai dacă x = y = z.

1.14. Inegalitatea este echivalentă cu


√ √
y−1 x−1
+ ≤ 1.
y x

a−1 1
Dar ≤ , pentru orice a ≥ 1, căci:
a 2

2 a − 1 ≤ a ⇔ 4a − 4 ≤ a2 ⇔ (a − 2)2 ≥ 0.

Egalitatea are loc dacă şi numai dacă a = 2. Rezultă că


√ √
y−1 x−1 1 1
+ ≤ + =1
y x 2 2
şi semnul egal se obţine dacă şi numai dacă x = y = 2.

1.15. Adunând inegalităţile:


xy + yz √ yz + zx √ zx + xy √
≥ y xz, ≥ z xy, ≥ x yz,
2 2 2
enunţul rezultă. Semnul egal are loc dacă şi numai dacă x = y = z.

17
1.16. Ridicând ambii membri ai inegalităţii la pătrat, avem:

2(xy + z 2 ) + 2 (x2 − z 2 )(y 2 − z 2 ) ≤ 4xy ⇔

(x2 − z 2 )(y 2 − z 2 ) ≤ xy − z 2 ⇔

x2 y 2 − z 2 (x2 + y 2 ) + z 4 ≤ x2 y 2 − 2xy 2 − 2xyz 2 + z 4 ⇔ −z 2 (x − y)2 ≤ 0,


ceea ce este evident. Egalitatea are loc dacă şi numai dacă x = y sau
z = 0.

1.17. Din inegalitatea mediilor rezultă


√ 2x + y + z
2x(y + z) ≤ ,
2
√ 2y + z + x
2y(z + x) ≤ ,
2
√ 2z + x + y
2z(x + y) ≤ ,
2
care, adunate membru cu membru, conduc la inegalitatea:
(∑ )
∑√ 4 x √ ∑√ (∑ )
2x(y + z) ≤ ⇔ 2 x(y + z) ≤ 2 x ,
2
deci ∑√ √ (∑ )
x(y + z) ≤ 2 x .

Semnul egal are loc dacă şi numai dacă

2x = y + z, 2y = z + x, 2z = x + y ⇔

3x = 3y = 3z = x + y + z ⇔ x = y = z.

1.18. Prin ridicare la pătrat inegalitatea este echivalentă cu următoarele:



1 − x2 + 1 − y 2 + 2 (1 − x2 )(1 − y 2 ) ≤ 4 − x2 − 2xy − y 2 ⇔

(1 − x2 )(1 − y 2 ) ≤ 1 − xy ⇔

(1 − x2 )(1 − y 2 ) ≤ (1 − xy)2 ⇔ 0 ≤ (x − y)2 .


Egalitatea se obţine dacă şi numai dacă x = y.

18
x+a x
1.19. Observând că > , pentru orice a > 0, primul membrul
x+a+1 x+1
al inegalităţii de demonstrat (pe care ı̂l vom nota cu E(x) ) ı̂ndeplineşte
inegalitatea:
√ √
x x √
E(x) > (x+x+1+x+2+x+3) > (4x+4) = 4 x(x + 1).
x+1 x+1
Arătăm că:
√ √ √ √
4 x(x + 1) > 4 x2 + 3x + 1 − 4 ⇔ 1 + x2 + x > x2 + 3x + 1 ⇔
√ √
1 + x2 + x + 2 x2 + x > x2 + 1 + 3x ⇔ x2 + x > x ⇔ x2 + x > x2 ,
ceea ce este evident.
1.20. Avem
√ √
A2 = x + x + 4 + 2 x(x + 4) şi B 2 = x + 2 + x + 3 + 2 (x + 2)(x + 3).

Rezultă că:
√ √
B 2 − A2 = 2( (x + 2)(x + 3) − x(x + 4)) + 1
√ √
= 2( x2 + 5x + 6 − x2 + 4x) + 1 > 1.
Prin urmare B 2 > A2 şi deci B > A.
√ √
1.21. Fie f : [−1, 1] → R, f (x) = 2 + x + 2 − x. Avem

(f (x))2 = 4 + 2 4 − x2

şi cum 0 ≤ x2 ≤ 1 rezultă 4 ≥ 4 − x2 ≥ 3 şi deci


√ √
4 + 2 3 ≤ (f (x))2 ≤ 4 + 2 4 = 8.
√ √ √ √
√ ( 3 + 1) ≤ (f√ (x))2 < (2 2)2 şi deci 1 + 3 ≤ f (x) ≤ 2 2.
Rezultă că 2

Dar 1 + 3 > 2, 73 şi 2 2 < 2, 83. Aceste inegalităţi demonstrează


complet enunţul.
1.22. Ridicând la pătrat ı̂n toţi membrii avem de verificat inegalităţile:
√ √
a − b + a + b + 2 a2 − b2 ≤ a − x + a + x + 2 a2 − x2 ≤ 4a ⇔
√ √ √
a2 − b2 ≤ a2 − x2 ≤ a2 ⇔
−b2 ≤ −x2 ≤ 0 ⇔ b2 ≥ x2 ≥ 0,
ceea ce este evident din enunţ.

19
1.23. Observăm că pentru x = −1 obţinem valoarea maximă a numărătorului
3
şi valoarea minimă a numitorului, deci maximul fracţiei este = 1;
3
pentru x = 0 obţinem valoarea minimă a numărătorului şi maximă a
2 1
numitorului. Prin urmare minimul fracţiei este = .
4 2
1.24. Existenţa radicalilor impune condiţiile

a − bx − cy ≥ 0 şi a + by + cx ≥ 0

sau, echivalent:

bx + cy ≤ a şi − a ≤ by + cx.

Relaţia din enunţ se mai scrie:


√ √
a − a + by + cx = a − bx − cy ≥ 0,

de unde

a≥ a + by + cx ⇔ a2 ≥ a + by + cx ⇔ a2 − a ≥ by + cx.

Analog, avem
√ √
a− a − bx − cy = a + by + cx ≥ 0,

de unde bx + cy ≥ a − a2 . Din cele de mai sus rezultă că

a − a2 ≤ bx + cy ≤ a şi − a ≤ by + cx ≤ a2 − a.

Prin ı̂nsumarea ultimelor două relaţii se obţine:

a−a2 −a ≤ bx+cy+by+cx ≤ a+a2 −a ⇔ −a2 ≤ (x+y)(b+c) ≤ a2 ⇔

−a2 ≤ (x + y)(b + c) ≤ a2 ⇔ |x + y| · |b + c| ≤ a2 .
Dar b + c = a ≥ 0, de unde se obţine că |x + y| ≤ a.

1.25. În inegalitatea


√ 2(u2 +√v 2 ) ≥ (u + v)2 , pentru orice u, v ∈ R punem
u = 1993 + a şi v = 1993 + b. Se obţine:
√ √
2(1993 + a + 1993 + b) ≥ ( 1993 + a + 1993 + b)2 = 4(1993 + c),

de unde 4 · 1993 + 2(a + b) ≥ 4 · 1993 + 4c, deci a + b ≥ 2c. Egalitatea


are loc dacă şi numai dacă a = b = c.

20
1.26. Considerăm ı̂ntr-un sistem de coordonate rectangular punctele O(0,0) ,
C(0,2) , B(2,0) , A(2,2) şi M(x,y) . OABC va un pătrat şi
√ √
M O = x2 + y 2 , M C = x2 + (y − 2)2 ,
√ √
M B = (x − 2)2 + y 2 , M A = (x − 2)2 + (y − 2)2 .
√ √
Avem E(x,y) = M O+M C +M B +M A ≥ OA+BC = 2·2 2 = 4 2 (din
inegalitatea triunghiului). Rezultă că E(x,y) este minimă când M, O, A şi
M, B, C sunt coliniare, deci când {M √ } = OA ∩ BC. Minimul se obţine
pentru x = y = 1 şi are valoarea 4 2.
√ m
1.27. Avem: 23 > ⇔ 23n2 − m2 > 0, deci m2 + k = 23n2 , unde k ∈ N∗ .
n
Vom arăta că k ≥ 5. Deoarece m ∈ N∗ rezultă că m = 23p ± r cu
r ∈ {0, 1, . . . , 12}.

m m2 (mod 23) m m2 (mod 23) m m2 (mod 23)


23p ± 0 0 23p ± 5 2 23p ± 9 18
23p ± 1 1 23p ± 6 3 23p ± 10 12
23p ± 2 4 23p ± 7 13 23p ± 11 8
23p ± 3 9 23p ± 8 3 23p ± 12 6
23p ± 4 16

Din tabelul anterior se observă că primul număr natural ̸= 0 care adunat
la m2 dă restul 0 la ı̂mpărţirea cu 23 este k = 5 (m = 23p+8 ⇒ m2 +5 =
√ k ≥ 5. Dar k = 23n − m ≥ 5 implică 23n ≥ m + 5,
0(mod√23)). Deci 2 2 2 2

adică 23n ≥ m2 + 5. Inegalitatea din enunţ se scrie



√ 6−1
23n > m +
m
şi pentru a rezulta e suficient să arătăm că

√ 6−1
m2 + 5 ≥ m + .
m
Într-adevăr, avem:
√ √
√ ( 6 − 1)2 √ 7−2 6
m +5 ≥ m +(2 6−2)+
2 2
⇔ 7−2 6 ≥ ⇔ m2 ≥ 1,
m2 m2
ceea ce este evident. Dacă m √≥ 2 inegalitatea
√ de mai sus este strictă.
Dacă m = 1 trebuie arătat că 23n > 6, ceea ce rezultă din n ≥ 1.

21
√ √ x x2
1.28. Fie 3
1 + x = y ≥ 1. Inegalitatea1+x≥1+ −
3
revine la
3 9
( )
y3 − 1 y3 − 1
y ≥1+ 1− ⇔
3 3
y−1 2
y−1≥ (y + y + 1)(4 − y 3 ) ⇔
9
(y − 1)[9 − (y 2 + y + 1)(4 − y 3 )] ≥ 0 ⇔
(y − 1)(y 5 + y 4 + y 3 − 4y 2 − 4y + 5) ≥ 0 ⇔ (y − 1)3 (y 3 + 3y 2 + 6y + 5) ≥ 0,
ceea ce este evident. Soluţia care urmează pentru demonstrarea celei de-
a doua inegalităţi√ este constructivă. Ea stabileşte inegalităţi din ce ı̂n ce
3
mai fixe pentru 1 + x.

Pentru x > 0 avem 3 1 + x > 1, deci
√ x x
3
1+x−1= √ √ < ,
3 2 3
(1 + x) + 1 + x + 1 3
√ x
adică 3 1 + x < 1 + . (1)
3
9 9
Pentru x ≥ , din (1) rezultă inegalitatea cerută. Fie x < , avem:
5 5
( )
√ x x 2
3
1+x− 1+ −
3 9
( )3
x x2
1+x− 1+ −
3 9
= ( ) ( )2 .
√ √ x x 2 x x2
3
(1 + x) + 1 + x 1 + −
2 3
+ 1+ −
3 9 3 9
9 √ x x 2
Cum x < (deci x < 3) şi 3 1 + x ≥ 1, 1 + − ≥ 1 avem
5 3 9
( ) 5x3 x5 ( x)
√ x x 2 − 1 − 3
3
1+x− 1+ − < 27 81 9 < 5x .
3 9 3 81
√ √
1.29. Fie 3 x = a > 0, 3 y = b > 0. Inegalitatea revine la următoarele ine-
galităţi echivalente:
4(a3 + b3 ) ≥ (a+ b)3 ⇔ 4(a+ b)(a2 − ab+ b2 ) ≥ (a+ b)(a2 +2ab+ b2 ) ⇔
4(a2 − ab + b2 ) ≥ a2 + 2ab + b2 ⇔ 3(a2 − 2ab + b2 ) ≥ 0 ⇔ 3(a − b)2 ≥ 0.
Egalitatea are loc dacă şi numai dacă a = b ⇔ x = y.

22
1.30. Avem:
√ √ √ √ √ √ √ √
3
n + 3+ n < 3 n + 2+ n + 1 ⇔ 3 n + 3− 3 n + 2 < n + 1− n ⇔
(n + 3) − (n + 2) (n + 1) − n
√ √ √ <√ √ ⇔
3
(n + 3)2 3 3
+ (n + 3)(n + 1) + (n + 2) 2 n+1+ n
1 1
√ √ √ <√√ ⇔
3
(n + 3)2 3
+ (n + 3)(n + 2) + (n + 3
2)2
n+1+ n

3
√ √ √ √
(n + 3)2 + 3 (n + 3)(n + 2) + 3 (n + 2)2 > n + 1 + n.
Dar

3
√ √ √
(n + 3)2 + 3
(n + 3)(n + 2) + 3 (n + 2)2 > 3 3 (n + 2)2
√ √ √
> 2 n + 1 > n + 1 + n,
√ √
căci 3
(n + 2)2 > n + 1 ⇔ (n + 2)4 > (n + 1)3 .

1.31. Folosind identitatea


1
a3 + b3 + c3 − 3abc = (a + b + c)[(a − b)2 + (b − c)2 + (c − a)2 ]
2
rezultă că dacă a ̸= b, b ̸= c, c ̸= a,

a + b + c > 0 ⇔ a3 + b3 + c3 > 3abc.

Prin urmare avem:


√ √ √ √ √ √
44 − 3 − 9 > 0 ⇔ 44 + 3 −3 + 3 −9 > 0 ⇔
3 3 3 3

√ √
44 − 3 − 9 > 3 3 44(−3)(−9) ⇔ 32 > 9 44 ⇔ 323 > 93 · 44 ⇔
3

√ √ √
32768 > 32076 şi 3 + 9 + 3 −49 > 0 ⇔
3 3


3 + 9 − 43 − 3 3 3 · 9(−43) > 0 ⇔

9 43 > 31 ⇔ 93 · 43 > 313 ⇔ 31347 > 29791.
3

√ √ √
1.32. Fie x ∈ N astfel ı̂ncât să avem x < 3 x+ x2 . Împărţind cu 3 x, obţinem:
3


3 √
x2 < 1 + 3 x.

Folosind observaţia de la problema precedentă, avem:


√ √ √ √
x + 1 + −x2 > 0 ⇔ x + 1 − x2 > 3 3 x · 1(−x2 ) = −3x,
3 3 3

23
√ √
deci x2 − 4x − 1 < 0. Rezultă că x ∈ (2 − 5, 2 + 5) ∩ N = √ {0, 1, 2, 3, 4}.
√ 3
Cum x = 0 şi √ x = 1 nu ı̂ndeplinesc condiţia x < x + x2 , respectiv
3

condiţia x + x2 < x + 1 rezultă că x ∈ {2, 3, 4}. Folosind şi rezultatul
3 3

problemei 1.29 se verifică acum că toate aceste valori ale lui x satisfac
cerinţele enunţului.
√ √ √ √
1.33. Fie a = 3 + 3 3 şi b = 3 − 3 3. Evident că a3 + b3 = 6 şi a > b. Cum
3 3

ab(a + b)√< a3 +√ b3 , rezultă că 3ab(a + b) < 18 ⇔ (a + b)3 < 24, deci
3 3
a + b < 24 = 2 3.
( )
1 n
1.34. Inegalitatea este echivalentă cu (n + 1) ≤ n n n+1 sau 1 + ≤ n şi
( )n n
1
rezultă din 1 + ≤ 3 ≤ n.
n
√ √ √ √
1.35. Fie x = n n + n n şi y = n n − n n. Inegalitatea de demonstrat devine
√ ( )
n
n x + y
n x + y n xn + y n
x+y ≤2 ⇔ ≤
2 2 2

şi rezultă din convexitatea funcţiei f : (0, ∞) → R, f (x) = xn sau


printr-un raţionament de inducţie matematică.
√ √
1.36. Cu notaţiile n x = a > 0, n 2 − x = b > 0 inegalitatea devine
( )( )
1 1
1+ 1+ ≥ 4,
a b

ı̂n ipoteza că an + bn = 2. Dar din inegalitatea mediilor avem:


√ an + bn
an bn ≤ = 1,
2
1
deci ab ≤ 1 ⇔ ≥ 1. Rezultă că:
ab
( )( ) √
1 1 1 1 1 1 1
1+ 1+ = 1+ + + ≥ 1+2 + ≥ 1 + 2 · 1 + 1 = 4.
a b a b ab ab ab

Semnul egal are loc dacă şi numai dacă a = b = 1 ⇔ x = 1.

1.37. Avem:
a2 c + b2 a + c2 b b2 c + ac2 + a2 b
A−B = −
abc abc

24
a2 (c − b) + a(b2 − c2 ) + bc(c − b)
=
abc
(c − b)(a2 − ab − ac + bc) (a − b)(b − c)(c − a)
= = .
abc abc
Rezultă că A ≥ B dacă şi numai dacă (a − b)(b − c)(c − a) ≥ 0 şi A ≤ B
dacă şi numai dacă (a − b)(b − c)(c − a) < 0.
1.38. Avem:
1 an 1
=1+ =1+ şi
A 2
1 + a + a + ... + a n−1 1 1 1
+ + ... +
an an−1 a
1 1
=1+ .
B 1 1 1
+ + . . . +
bn bn−1 b
1 1
Din a > b > 0 rezultă > , deci
b a
1 1 1 1 1 1
n
+ n−1 + . . . + > n + n−1 + . . . +
b b b a a a
1 1
şi prin urmare > ⇔ B > A.
A B
a c
1.39. Fie = = k > 1, rezultă că a = bk, c = dk şi deci:
b d
a + d > b + c ⇔ bk + d > b + dk ⇔
k(b − d) − (b − d) > 0 ⇔ (b − d)(k − 1) > 0,
ceea ce rezultă din b > d şi k > 1.
1.40. Pentru numărul a, din şirul de inegalităţi din ipoteză rezultă că:
3a ≤ b + c + d ⇔ 4a ≤ a + b + c + d = 4 ⇔ a ≤ 1.
Valoarea maximă pe care o poate lua a este deci 1, ı̂n cazul a = b = c =
d = 1. Pentru numărul b, avem:
2b ≤ c + d ⇔ 3b ≤ b + c + d ⇔ a + 3b ≤ a + b + c + d = 4 ⇔ a + 3b ≤ 4.
4
Deoarece a ≥ 0, valoarea maximă a lui b este şi ea se obţine când
3
4
a = 0, c = d = . Pentru c avem 2c ≤ c + d ⇔ a + b + 2c ≤ 4. Deoarece
3
a, b ≥ 0, maximul lui c este 2, când a = b = 0 şi d = 2. Pentru d avem
valoarea maximă 4, când a = b = c = 0.

25
1.41. Folosim observaţia că x ∈ (α, β) ⇔ (x − α)(β − x) > 0, unde α < β.
Vom avea astfel x ∈ (0, 1) ⇔ x(1 − x) > 0 şi

(1 − x)a + bx ∈ (a, b) ⇔ [(1 − x)a + bx − a][b − (1 − x)a − bx] > 0 ⇔

(b − a)2 x(1 − x) > 0 ⇔ x(1 − x) > 0 ⇔ x ∈ (0, 1),


ceea ce trebuia demonstrat.

1.42. Ţinând cont că a ≤ b ≤ c, avem succesiv:

a2 + 3b2 + 5c2 ≥ a2 + (b2 + 2ab) + (c2 + 2bc + 2ca)

= a2 + b2 + c2 + 2(ab + bc + ca) = (a + b + c)2 ≥ 1,


deoarece 2b2 ≥ 2ab şi 4c2 = 2c2 + 2c2 ≥ 2bc + 2ca. Egalitatea are loc
1
dacă şi numai dacă a = b = c ≥ .
3
1.43. Fără a restrânge generalitatea problemei presupunem că a ≤ b. Avem:

a a2 − 2ab − 3b2 b a2 − 2ab + 3b2 b


≤ 2 2
≤ ⇔ 1≤ ≤ .
a a +b a a2 + b2 a
Prima inegalitate se scrie succesiv:

a2 − 2ab + 3b2
1≤ ⇔ a2 + b2 ≤ a2 − 2ab + 3b2 ⇔ a ≤ b,
a2 + b2
ceeea ce este evident. A doua inegalitate se scrie succesiv:

a3 − 2a2 b + 3ab2 ≤ a2 b + b3 ⇔ a3 + 3ab2 ≤ 3a2 b + b3 ⇔ (a − b)3 ≤ 0.

1.44. Prin ı̂mpărţirea cu abcd inegalitatea este echivalentă cu următoarea:


1 1 1 1 173
+ + + + ≤ 2.
a b c d abcd
Dar a, b, c, d fiind numere prime diferite rezultă că unul dintre ele este
≥ 2, al doilea ≥ 3, al treilea ≥ 5 şi al patrulea ≥ 7. Prin urmare:
1 1 1 1 173 1 1 1 1 173
+ + + + ≤ + + + +
a b c d abcd 2 3 5 7 2·3·5·7
175 + 72 + 173 420
= = = 2.
210 210

26
1.45. Vom folosi identitatea
1
a3 + b3 + c3 − 3abc = (a + b + c)[(a − b)2 + (b − c)2 + (c − a)2 ].
2
Inegalitatea din enunţ se mai scrie:
x + 2y + 3z
0≥ [(x − 2y)2 + (2y − 3z)2 + (3z − x)2 ].
2
Deoarece x, y, z > 0 (deci x + 2y + 3z > 0) rezultă că:

(x − 2y)2 + (2y − 3z)2 + (3z − x)2 = 0,

de unde x = 2y = 3z. Reciproc, dacă x = 2y = 3z inegalitatea are loc.


1.46. Fie x2 + y 2 = a ≥ 0 şi b = xy. Inegalitatea devine

f (b) = b2 + 2b(a − 2) + a2 − a + 1 ≥ 0.

Deoarece discriminantul ecuaţiei de gradul doi ı̂n b este ∆ = 3(1 − a)


rezultă că pentru a ≥ 1 avem f (b) ≥ 0. Rămâne de studiat cazul când
a ∈ [0, 1). Avem a ≥ ±2b. Pentru b ≤ 0, avem 2b(a−2) ≥ 0 şi a2 −a+1 ≥
0 deci f (b) ≥ 0. Rădăcinile ecuaţiei f (b) = 0 sunt:
√ √
2 − a − 3(1 − a) 2 − a + 3(1 − a)
b1 = şi b2 = .
2 2
a a
Ştim că b ≤ . Arătăm că ≤ b1 şi deci b ̸∈ (b1 , b2 ), adică f (b) ≥ 0.
2 2
Inegalitatea √
a
≤ 2 − a − 3(1 − a)
2
este echivalentă cu:
( ) ( )2
√ 3a 3a 2 3a
3(1 − a) ≤ 2 − ⇔ 3 − 3a < 2 − ⇔ − 1 ≥ 0,
2 2 2
1
ceea ce este adevărat. Egalitatea se obţine pentru x = y = ± √ .
3
1.47. Avem succesiv:
1 + x2 + x4 3
≥ ⇔ 2x4 + 2x2 + 2 ≥ 3x3 + 3x ⇔
x3 + x 2
2x4 − 3x3 + 2x2 − 3x + 2 ≥ 0 ⇔ (x − 1)2 (2x2 + x + 2) ≥ 0.
Egalitatea are loc dacă şi numai dacă x = 1.

27
1.48. Inegalitatea rezultă din identitatea:

(x + 2)4 − 16x3 = x4 − 8x3 + 24x2 + 32x + 16 = x2 (x − 4)2 + 8(x2 + 4x + 2).


( )2
x2 y2 x y
1.49. Avem 2 + 2 = + − 2 şi deci notând
y x y x

x y x y
t= + ≥2 · = 2,
y x y x

inegalitatea revine la

2t2 − 4 ≥ t + 2 ⇔ 2t2 − 8 − (t − 2) ≥ 0 ⇔

2(t − 2)(t + 2) − (t − 2) ≥ 0 ⇔ (t − 2)(2t + 3) ≥ 0,


ceea ce este evident. Egalitatea are loc dacă şi numai dacă

t = 2 ⇔ x = y.

1.50. Avem succesiv:

(x+y)(x4 +y 4 )−(x2 +y 2 )(x3 +y 3 ) = x5 +xy 4 +x4 y+y 5−x5−x2 y 3−x3 y 2−y 5

= xy 4 + x4 y − x2 y 3 − x3 y 2 = xy 2 (y 2 − x2 ) − x2 y(y 2 − x2 )
= (y 2 − x2 )(xy 2 − x2 y) = xy(x + y)(x − y)2 ≥ 0.
Egalitatea are loc dacă şi numai dacă x = 0 sau y = 0 sau x = y.

1.51. 1) Ţinând cont de inegalitatea mediilor, avem:


√ √3

3
a+b+c ≥ 3 abc, ab+bc+ca ≥ 3 a2 b2 c2 şi a2 +b2 +c2 ≥ 3 a2 b2 c2 .
3

Înmulţind membru cu membru prima şi a doua inegalitate rezultă 1).


Din prima şi a treia rezultă 2). O demonstraţie directă pentru 3) rezultă
din identitatea:
1
a3 + b3 + c3 − 3abc = (a + b + c)[(a − b)2 + (b − c)2 − (c − a)2 ].
2
Inegalitatea 4) rezultă din inegalitatea mediilor:
√ √ √
(a + b)(b + c)(c + a) ≥ 2 ab · 2 bc · 2 ca = 8abc.

În toate inegalităţile semnul egal se obţine dacă şi numai dacă a = b = c.

28
1.52. I-a soluţie. Ţinând cont de inegalitatea mediilor, avem:
(∑ ) (∑ 1 )
(a + b) ≥9 ⇔
a+b
(∑ )(∑ 1 ) (
1 1 1
)
9
2 a ≥ 9 ⇔ (a+b+c) + + ≥ ⇔
a+b a+b b+c c+a 2
c a b 9 a b c 3
1+ + +1+1+ ≥ ⇔ + + ≥ .
a+b b+c a+c 2 b+c a+c a+b 2

A II-a soluţie. În inegalitatea Cauchy-Buniakovski:

(x21 + x22 + x23 )(y12 + y22 + y32 ) ≥ (x1 y1 + x2 y2 + x3 y3 )2 ,

punând √ √ √
a b c
x1 = , x2 = , x3 = ,
b+c c+a a+b
√ √ √
y1 = a(b + c), y2 = b(c + a), y3 = c(a + b),
obţinem:
( )
a b c
+ + (ab + ac + bc + ba + ca + cb) ≥ (a + b + c)2 ,
b+c c+a a+b
de unde:
( )
a b c a2 + b2 + c2 + 2ab + 2bc + 2ca
+ + ≥
b+c c+a a+b 2ab + 2bc + 2ca
3(ab + bc + ca) 3
≥ = .
2(ab + bc + ca) 2
A III-a soluţie. Notând b + c = x, c + a = y, a + b = z, deducem că
x+y+z
a+b+c= ,
2
de unde:
−x + y + z x−y+z x+y−z
a= , b= , x= (cu x, y, z > 0).
2 2 2
Înlocuind ı̂n inegalitatea din enunţ, avem:
−x + y + z x − y + z x + y − z 3
+ + ≥ ⇔
2x 2y 2z 2

29
( ) ( ) (x
x y y z z)
+ + + + + ≥ 6.
y x z y z x
Ultima inegalitate este adevărată deoarece
x y y z x z
+ ≥ 2, + ≥2 şi + ≥2
y x z y z x
(simple consecinţe ale inegalităţii mediilor).

1.53. Fără a restrânge generalitatea problemei presupunem că a ≥ b ≥ c.


Avem a − b + c > 0, a + b − c > 0, iar pentru −a + b + c apar două
situaţii:
1) Dacă −a + b + c < 0, atunci inegalitatea din enunţ este evidentă căci
membrul stâng al ei este negativ iar cel drept este pozitiv.
2) Dacă −a + b + c > 0, atunci notăm −a + b + c = x, a − b + c = y,
a + b − c = z şi obţinem
y+z z+x x+y
a+b+c=x+y+z ⇔ a= , b= , c=
2 2 2
şi inegalitatea din enunţ devine:
1
xyz ≤ (x + y)(y + z)(z + x),
8
unde x, y, z > 0. Ultima inegalitate este demonstrată ı̂n problema
1.51,4).

1.54. Efectuând calculele avem succesiv:


∑ (∑ )
(x3 + y 3 + 3x2 y + 3xy 2 ) ≥ 21xyz + x3 ⇔
∑ ∑
x3 + 3 xy(x + y) ≥ 21xyz.
Dar din inegalitatea mediilor rezultă:

x3 ≥ 3xyz şi
∑ ∑
3xy(x + y) = 3 (x2 y + xy 2 )

≥3·6· 6
x2 y · xy 2 · y 2 z · yz 2 · z 2 x · zx2 = 18xyz.
Adunând inegalităţile de mai sus, enunţul rezultă. Egalitatea are loc
dacă şi numai dacă x = y = z.

30
1.55. Inegalitatea este echivalentă cu:
y+z 2x + y + z
√ ≥√ .
yz (x + z)(x + y)
y z
Fie = a > 0, = b > 0. Inegalitatea se scrie succesiv ı̂n următoarele
x x
forme echivalente:
a+b 2+a+b
√ ≥√ ⇔
ab (1 + a)(1 + b)

(a + b)2 (1 + a)(1 + b) ≥ ab(2 + a + b)2 ⇔


(a + b)2 (ab + a + b + 1) ≥ ab[(a + b)2 + 4(a + b) + 4] ⇔
(a + b)2 (a + b) + ab(a + b)2 + (a + b)2 ≥ ab(a + b)2 + 4ab(a + b) + 4ab ⇔
(a + b)(a − b)2 + (a − b)2 ≥ 0 ⇔ (a − b)2 (a + b + 1) ≥ 0,
ceea ce este evident.
Egalitatea are loc dacă şi numai dacă a = b ⇔ x = y = z.

1.56. Avem succesiv:


∑a+b+c−d ∑a+b+c
≥8 ⇔ −4≥8 ⇔
d d
∑a+b+c ∑a+b+c
≥ 12 ⇔ + 4 ≥ 16 ⇔
d d
∑a+b+c+d ( ∑ ) (∑ 1 )
≥ 16 ⇔ a ≥ 42 ,
d a
ceea ce rezultă din inegalitatea mediilor. Semnul egal se obţine dacă şi
numai dacă a = b = c = d.

1.57. Avem:

x4 − 2x3 + x2 + x2 − 2x + 1 + (a − 2)x2 = x2 (x − 1)2 + (x − 1)2 + (a − 2)x2

= (x − 1)2 (x2 + 1) + (a − 2)x2 ≥ 0,


pentru a ≥ 2. Egalitatea are loc dacă şi numai dacă x = 1 şi a = 2.
( ) ( )
1 1 2 1 2
1.58. Avem x − x + = x −
4 2
+ x− , de unde rezultă enunţul.
2 2 2

31
1.59. Inegalitatea se scrie succesiv sub următoarele forme echivalente:

4(x + y)2 + 4 ≥ 4xy + 4 3(x + y) ⇔

4(x + y)2 + 4 ≥ (x + y)2 − (x − y)2 + 4 3(x + y) ⇔
√ √
3(x+y)2 −4 3(x+y)+4+(x−y)2 ≥ 0 ⇔ [ 3(x+y)−2]2 +(x−y)2 ≥ 0.
Egalitatea are loc dacă şi numai dacă x = y şi

√ 3
2 3x = 2 ⇔ x = y = .
3

ab b2 3b2
1.60. Din a2 + ab + b2 = a2 + 2 + + > 0 rezultă că inegalităţile din
2 4 4
enunţ sunt echivalente succesiv cu următoarele:
a2 − ab + b2
3≥ ⇔ 3a2 + 3ab + 3b2 ≥ a2 − ab + b2 ⇔ 2(a + b)2 ≥ 0 şi
a2 + ab + b2
a2 − ab + b2 1
≥ ⇔ 3a2 − 3ab + 3b2 ≥ a2 + ab + b2 ⇔ 2(a − b)2 ≥ 0.
a2 + ab + b2 3
Semnul egal rezultă dacă şi numai dacă b = −a ı̂n prima inegalitate şi
dacă şi numai dacă b = a ı̂n a doua.
1.61. Aplicând succesiv inegalităţile 2(x2 + y 2 ) ≥ (x + y)2 şi x2 + y 2 ≥ 2xy,
avem:
2(a4 + b4 ) + 9 ≥ (a2 + b2 )2 + 9 ≥ (2ab)2 + 32 ≥ 2 · 2ab · 3 = 12ab.
Egalitatea se obţine dacă şi numai dacă

3
a = b şi 2ab = 3 ⇔ a = b = .
2

1.62. Fie x = 1 + a şi y = 1 + b. Avem:


x2 − x + 1 = (1 + a)2 − (1 + a) + 1 = a2 + a + 1 şi y 2 − y + 1 = b2 + b + 1.
Inegalitatea de demonstrat revine la următoarea:
2(a2 + a + 1)(b2 + b + 1) ≥ 1 + (1 + a)2 (1 + b)2 .
Efectuând calculele, inegalitatea se reduce la
a2 b2 + (a − b)2 ≥ 0.
Egalitatea are loc dacă şi numai dacă a = b = 0 ⇔ x = y = 1.

32
1.63. Avem evident:

(1 − x2 )2 ≥ 0 ⇔ 1 − x2 + x4 ≥ x2

şi deci
(1 − x2 + x4 )(1 − y 2 + y 4 )(1 − z 2 + z 4 ) ≥ x2 y 2 z 2 .
Egalitatea se obţine dacă şi numai dacă

x2 = y 2 = z 2 = 1 ⇔ x, y, z ∈ {−1, 1}.

1.64. Din inegalitatea mediilor rezultă:


( )( )
2 2 2 x+y y+z z+x
+ + + + ≥ 32 ,
x+y y+z z+x 2 2 2

deci:
2 2 2 9
+ + ≥ .
x+y y+z z+x x+y+z
Egalitatea are loc dacă şi numai dacă x+y = y+z = z+x ⇔ x = y = z.

1.65. Folosind inegalitatea a2 + b2 + c2 ≥ ab + bc + ca, avem:


( )2 ( ) ( )
x2 y 2 z 2 x y 2 z 2 x y y z z x x y z
2
+ 2+ 2 = + + ≥ · + · + · = + + .
y z x y z x y z z x x y z x y
x y z
Egalitatea are loc dacă şi numai dacă = = . Notând cu m valoarea
y z x
comună a acestor rapoarte, obţinem

x = my, y = mz, z = mx ⇔ xyz = m3 xyz ⇔

m3 = 1 ⇔ m = 1 deci x = y = z.
1
1.66. Deoarece a2 + b2 + c2 ≥ (a + b + c)2 , pentru orice a, b, c ∈ R rezultă că
3
notând x + y + z = 2p, avem:
(∑ )2
∑ (p − x) p2
(p − x)2 ≥ = .
3 3
Dar inegalitatea din enunţ este echivalentă cu următoarea:
∑ 3
4 (p − x)2 + ≥ 2p.
4

33
4p2 3
Va trebui să demonstrăm că + ≥ 2p pentru a rezulta enunţul.
3 4
Avem: √
4p2 3 4p2 3
+ ≥2 · = 2p,
3 4 3 4
conform inegalităţii mediilor. Egalitatea are loc dacă şi numai dacă

2p 3 1
p − x = p − y = p − z şi √ = ⇔ x=y=z= .
3 2 2

1.67. Fără a restrânge generalitatea problemei, presupunem că x ≤ y ≤ z.


Inegalitatea din enunţ se scrie echivalent astfel:
3
x2 + y 2 + z 2 − xy − yz − zx ≥ (z − x)2 ⇔
4
4x2 + 4y 2 + 4z 2 − 4xy − 4yz − 4zx ≥ 3z 2 − 6zx + 3x2 ⇔
x2 + 4y 2 + z 2 + 2xz − 4xy − 4yz ≥ 0 ⇔
4y 2 − 4y(x + z) + (x + z)2 ≥ 0 ⇔ (2y − x − z)2 ≥ 0,
inegalitatea adevărată pentru orice x, y, z ∈ R.
1.68. Avem:

4(a2 + b2 + c2 + d2 − a − b − c − d + 1) = (4a2 − 4a + 1) + (4b2 − 4c + 1)

+(4d2 − 4d + 1) = (2a − 1)2 + (2b − 1)2 + (2c − 1)2 + (2d − 1)2 ≥ 0.


1
Egalitatea are loc dacă şi numai dacă a = b = c = d = .
2
1.69. Folosind inegalitatea mediilor şi identitatea

(ad − bc)2 + (ac + bd)2 = (a2 + b2 )(c2 + d2 ),

avem:

a2 + b2 + c2 + d2 + ac + bd ≥ 2 (a2 + b2 )(c2 + d2 ) + ac + bd

= 2 (ac + bd)2 + 1 + (ac + bd).
Notând ac + bd = x, avem:
√ √ √
(2 x2 + 1 + x)2 = 4x2 + 4 + 4x x2 + 1 + x2 = (2x + x2 + 1)2 + 3 ≥ 3,

de unde √ √
2 (ac + bd)2 + 1 + (ac + bd) ≥ 3.

34
1.70. Ipoteza se mai poate scrie sub următoarele forme echivalente:

4x2 + 4y 2 = 4x + 4y + 4 ⇔ 4x2 − 4x + 1 + 4y 2 − 4y + 1 = 6 ⇔

(2x − 1)2 + (2y − 1)2 = 6.


Rezultă că:
6 − (2x − 1)2 = (2y − 1)2 ≥ 0,
deci
√ √ √
(2x − 1)2 ≤ 6 ⇔ |2x − 1| ≤ 6 ⇔ − 6 ≤ 2x − 1 ≤ 6 ⇔
[ √ √ ]
1− 6 1+ 6
x∈ , .
2 2
[ √ √ ]
1− 6 1+ 6
Analog, obţinem că y ∈ , . Valorile sunt luate efectiv;
2 2

1 1± 6
de exemplu: x = , y = .
2 2

35
36
Capitolul 2

Inegalităţi clasice

2.1 Enunţuri
2.1. a) Să se arate că pentru orice x ∈ R are loc inegalitatea

x4 + x3 + x2 + x + 1 > 0.

b) Dacă x ≥ 0, atunci: (x − 1)4 ≤ x4 + x3 + x2 + x + 1 ≤ (x + 1)4 .



x(1 − x2 ) 1

2.2. Demonstraţi că: ≤ , pentru orice x ∈ R.
(1 + x2 )2 4

2.3. Să se arate că pentru orice x ∈ R şi pentru orice n ∈ N are loc inegali-
tatea:
x2n − 2nx + 2n − 1 ≥ 0.

1
2.4. Dacă x ∈ R, atunci: x2n ± x2n−1 ± x2n−2 ± . . . ± x + 1 > .
2
2.5. Fie a ∈ R, |a| ≤ 4. Dacă x, y ∈ R, atunci: x4 + y 4 + axy + 2 ≥ 0.

2.6. Dacă x, y ∈ R, atunci: (1 + x2 + 2x4 )(1 − y 2 + 2y 4 ) ≥ 7x2 y 2 .

2.7. Să se arate că pentru orice x, y, z ∈ R avem:


∑ (∑ )
x6 + 3x2 y 2 z 2 ≥ 2 x3 y 3 .

(∑ )2 (∑ )3
2.8. Dacă x, y, z ∈ R, atunci: x3 − 3xyz ≤ x2 .

37
2.9. Să se demonstreze că pentru orice x, y, z ∈ R are loc inegalitatea:

xy(y − z)(z − x) + yz(z − x)(x − y) + xz(x − y)(y − z) ≤ 0.

2.10. Să se arate că pentru orice x, y, z ∈ R∗ avem:


∑ ( x2 y2
)2 ∑ (
x y 2
)
+ 2 ≥ + .
y2 x y x

2.11. Să se demonstreze că pentru orice x, y, z ∈ R are loc inegalitatea:

x(x + y)3 + y(y + z)3 + z(z + x)3 ≥ 0.

2.12. Demonstraţi inegalitatea:


√ √ √
(a + c)2 + b2 + (a − c)2 + b2 ≥ 2 a2 + b2 , unde a, b, c ∈ R.

2.13. Să se arate că dacă a, b, c, d ∈ R sunt numere distincte, atunci:

(a − b)(b − c)(c − d)(d − a) + (a − c)2 (b − d)2 > 0.

2.14. Să se arate că pentru orice x1 , x2 , . . . , xn ∈ R (unde n ∈ N, n ≥ 2) are


loc inegalitatea:

n ∑
n
2 ∑ n
x2k + xk + xi xj + ≥ 0.
n−1 8
k=1 k=1 1≤i<j≤n

2.15. Fie x1 , x2 , . . . , xn ∈ R. Să se arate că:



n ∑
n
n
x2k − xk xk+1 ≥ max{(x1 −x2 )2 , (x2 −x3 )2 , . . . , (xn −x1 )2 },
2(n − 1)
k=1 k=1

unde xn+1 = x1 , iar n ∈ N∗ − {1}.

2.16. Dacă xk , yk ∈ R, k = 1, n, atunci:


( n )( n ) ( n )( n ) ( n )
∑ ∑ ∑ ∑ ∑
4 x2k yk2 + 4 xk yk xk + 4n x2k
k=1 k=1 k=1 k=1 k=1
( )2 ( n )2 ( n )( n )

n ∑ ∑ ∑
>4 xk yk + xk +4 x2k yk .
k=1 k=1 k=1 k=1

38
( )( )
2a 2b
2.17. Dacă a, b > 0, atunci: +1 + 1 ≥ 9. În ce caz avem egalitate?
b a
2.18. Arătaţi că pentru orice a, b > 0 cu a + b = 1, avem:
( )2 ( )2
1 1 25
a+ + b+ ≥ .
a b 2

2.19. Fie a, b, x, y > 0. Să se demonstreze inegalitatea:


( )2 ( y )2
x
a +b + a + b ≥ 2(a + b)2 .
y x

2.20. Fie x, y ≥ 0. Arătaţi că:


√ √ √ √
3(x − 3
xy + y) ≥ 4 xy − 1 şi 3(x − 3 xy + 1) ≥ 4 x − y.

În ce caz avem egalitate?


√ a−b
2.21. Dacă a, b > 0 şi n ∈ N, atunci: abn−1 ≤ b +
n
. În ce caz avem
n
egalitate?

2.22. Fie a, b, c > 0 şi a = min(a, b, c). Să se arate că:


( )3
b+c
a + b + c − 3abc ≥ 2
3 3 3
−a .
2

2.23. Dacă a, b, x, y, z > 0, atunci:

x y z 3
+ + ≥ .
ay + bz az + bx ax + by a+b

2.24. Să se arate că pentru orice x, y, z > 0 are loc inegalitatea:

x y z 3
+ + ≥ .
x + 2y + 2z 2x + y + 2z 2x + 2y + z 5

2.25. Dacă x, y, z > 0 şi a ≥ 1, atunci:

x y z 3
+ + ≤ .
ax + y + z x + ay + z x + y + az a+2

39
2.26. Să se arate că pentru orice x, y, z > 0 are loc inegalitatea:
∑ x 9
≥ .
(y + z)(y + z − x) 2(x + y + z)

2.27. Dacă x, y, z ≥ 0, atunci există inegalitatea:


x y z
+ + ≤ 1.
xy + x + 1 yz + y + 1 zx + z + 1

2.28. Dacă x, y, z > 0, atunci:


x2 y2 z2 3
2 2
+ 2 2
+ 2 2
≥ .
y(x + xy + y ) z(y + yz + z ) x(z + zx + x ) x+y+z

2.29. Demonstraţi că pentru x, y, z ≥ 0 avem:



(x − y)2 (x + y − z) ≥ 0.

2.30. Demonstraţi că pentru a, b, c ≥ 0 are loc inegalitatea:

a3 + b3 + c3 + 3abc ≥ ab(a + b) + bc(b + c) + ca(c + a).

1
2.31. Dacă a, b, c ≥ 0, atunci: a3 + b3 + c3 + abc ≥ (a + b)(b + c)(c + a).
2
∑ ∑ ∑ ∑
2.32. Pentru a, b, c ≥ 0 avem: a3 ≥ a2 b şi a4 ≥ a3 b.

2.33. Arătaţi că pentru a, b, c > 0 are loc inegalitatea:


( )( )( )
4a 4b 4c
+1 +1 + 1 > 25.
b+c c+a a+b

2.34. Dacă a, b, c > 0, atunci:


a2 b2 c2 a+b+c
+ + ≥ .
b+c c+a a+b 2

a2 + bc b2 + ac c2 + ab
2.35. Dacă a, b, c > 0, atunci: + + ≥ a + b + c.
b+c a+c a+b
2.36. Demonstraţi, pentru a, b, c > 0, inegalitatea:

∑ a3 a
2 2
≥ .
a + ab + b 3

40
2.37. Demonstraţi inegalitatea:
1 1 1 1
+ 3 + 3 ≤ , unde a, b, c > 0.
a3 3 3 3
+ b + abc b + c + abc c + a + abc abc

2.38. Să se arate că pentru orice a, b, c > 0 avem:


∑ a3 ∑ a2 + b2 ∑
≥ ≥ a.
bc 2c

2.39. Fie a, b, c > 0. Să se demonstreze inegalitatea:


1 1 1
(−a + b + c)3 + (a − b + c)3 + (a + b − c)3 ≥ a2 + b2 + c2 .
a b c

2.40. Dacă a, b, c > 0, atunci:


a3 b3 c3 3(ab + bc + ca)
+ 2 + 2 ≥ .
− bc + c
b2 2 c − ca + a 2 a − ab + b2 a+b+c

a − b b − c c − a

2.41. Dacă a, b, c > 0, atunci: + + < 1.
a + b b + c c + a
2.42. Să se arate că pentru orice x, y, z > 0 are loc inegalitatea:
√ √ √
x y z
+ + > 2.
y+z z+x x+y

2.43. Să se arate că pentru orice x, y, z > 0 are loc inegalitatea:
x y
√ +√
(x2 + y 2 )(x2 + z2) (y 2 + z 2 )(y 2 + x2 )

z 1 x2 + y 2 + z 2
+√ ≤ · .
(z 2 + x2 )(z 2 + y 2 ) 2 xyz

2.44. Demonstraţi inegalitatea:


√ √
x y z 3
+ + 3 > , unde x, y, z > 0.
y z x 2

2.45. Dacă a, b, c > 0, atunci:


( ) ( √ )
1 1 1 3 (a + b)(b + c)(c + a)
(a + b + c) + + ≥3 1+ ≥ 9.
a b c abc

41
2.46. Să se arate că:
√ √ √ √
xy + yz + zx ≥ 2 4 xyz(x + y + z), pentru orice x, y, z ≥ 0.

Când are loc egalitatea?

2.47. Fie x, y, z > 0 şi n ∈ N∗ . Să se arate că:

xn+1 (y + z) + y n+1 (z + x) + z n+1 (x + y) ≥ 2xyz(xn−1 + y n−1 + z n−1 ).

2.48. Să se arate că dacă a, b, c, d > 0, atunci:


a b c d
+ + + ≥ 1.
3b + c 3c + d 3d + a 3a + b

2.49. Demonstraţi că pentru orice a, b, c, d > 0 are loc inegalitatea:


a b c d
+ + + ≥ 2.
b+c c+d d+a a+b

2.50. Dacă x1 , x2 , x3 , x4 , x5 ≥ 0, atunci:

(x1 + x2 + x3 + x4 + x5 )2 ≥ 4(x1 x2 + x2 x3 + x3 x4 + x4 x5 + x5 x1 ).

2.51. Arătaţi că dacă a, b, c, d, e > 0, atunci:


( a )4 ( b )4 ( c )4 ( d )4 ( e )4 b c d e a
+ + + + ≥ + + + + .
b c d e a a b c d e

2.52. Dacă a, b, c, d, e, f > 0, atunci:


a b c d e f
+ + + + + ≥ 3.
b+c c+d d+e e+f f +a a+b

2.53. Demonstraţi că pentru a1 , a2 , . . . , an > 0 are loc inegalitatea:



n
ak − ak+2
≥ 0, unde an+1 = a1 , an+2 = a2 .
ak+1 + ak+2
k=1

2.54. Dacă a1 , a2 , . . . , an > 0, atunci:


a1 a2 an n
+ + ... + > .
a2 + a3 a3 + a4 a1 + a2 4

42
2.55. Dacă a1 , a2 , . . . , an > 0 atunci:
(a1 + a2 + . . . + an )2 a1 a2 an
2 2 2
≤ + + ... + .
2(a1 + a2 + . . . + an ) a2 + a3 a3 + a4 a1 + a2

2.56. Să se demonstreze inegalitatea:


x31 x32 x3n
+ + . . . +
x21 + x1 x2 + x22 x22 + x2 x3 + x23 x2n + xn x1 + x21
1
≥ (x1 + x2 + . . . + xn ), unde x1 , x2 , . . . , xn > 0.
3
2.57. Dacă a1 , a2 , . . . , an > 0, atunci:

(1 + a1 )(1 + a2 ) . . . (1 + an ) ≥ (1 + n
a1 a2 . . . an )n .

2.58. Fie a1 , a2 , . . . , an > 0 şi k > p ≥ 0 două numere naturale. Demonstraţi


inegalitatea:
( )
ak1 + ak2 + . . . + akn a1 + a2 + . . . + an k−p
≥ .
ap1 + ap2 + . . . + apn n

2.59. Dacă a1 , a2 , . . . , an > 0, atunci:


√ √
a1 + a2 + . . . + an √ ( ai − aj )2
− a1 a2 . . . an ≥ max
n
.
n 1≤i<j≤n n

2.60. Fie a1 , a2 , . . . , an > 0 şi an+1 = a1 . Demonstraţi inegalitatea:



n
a2k 1∑
n
≥ ak .
ak + ak+1 2
k=1 k=1

2.61. Să se arate că:



n ∑
n ∑
n
(ak + bk )2 a2k b2k
k=1 k=1 k=1
≤ + ,

n ∑
n ∑
n
pk (ak + bk ) pk ak pk bk
k=1 k=1 k=1


n ∑
n
unde pk , ak , bk ∈ R iar pk ak > 0 şi pk bk > 0.
k=1 k=1

43
2.62. Fie a, b > 0 cu a2 + b2 = 1. Să se demonstreze că:
1 √
a+b+ ≥ 2 + 2.
ab

2.63. Demonstraţi că dacă x, y > 0 şi x3 + y 3 = x − y, atunci x2 + y 2 < 1.

2.64. Dacă x, y, z > 0 şi x + y + z = 1, atunci:


( )( )( )
1 1 1
1+ 1+ 1+ ≥ 64.
x y z

2.65. Dacă x, y, z ∈ (0, 1) şi x + y + z = 1, atunci:


1 1 1
+ + ≥ 3(x2 + y 2 + z 2 ) + 8.
x y z

2.66. Fie a, b, c ∈ (0, 1) şi a + b + c = 2. Să se arate că

4(a2 + b2 + c2 ) ≥ 8 − 9abc.

În ce caz avem egalitate?

2.67. Dacă a, b, c ∈ R şi a + b + c = 1, atunci

10|a3 + b3 + c3 − 1| ≤ 9|a5 + b5 + c5 − 1|.

2.68. Dacă a, b, c > 0 şi a2 + b2 + c2 = 1, atunci:


a5 + b5 b5 + c5 c5 + a5
+ + ≥ 3(ab + bc + ca) − 2.
ab(a + b) bc(b + c) ca(c + a)
∑ 1
2.69. Dacă a, b, c ∈ R şi = 2, atunci:
1 + a2
5
abc(a + b + c − abc) ≤ .
8
2.70. Fie a, b, c, d ∈ R astfel ı̂ncât a + c = 1 şi b + d = 1.
Să se arate că: a2 b + c2 d ≥ bd şi ab2 + cd2 ≥ ac.

2.71. Fie a, b, c, d, e ∈ R astfel ı̂ncât

a + b + c + d + e = 8 şi a2 + b62 + c2 + d2 + e2 = 16.

Să se găsească valoarea maximă a lui e.

44
2.72. Dacă x, y, z > 0 şi xyz = 1, atunci:
x y z
+ + ≥ x + y + z.
y z x

2.73. Dacă x, y, z > 0 şi xyz = 1, atunci:


x y z 3
+ + ≥ .
xy + 1 yz + 1 zx + 1 2

2.74. Dacă x1 , x2 , . . . , xn > 0 şi x1 · x2 · . . . · xn = 1, atunci:


 n−1 ( n )n−2
( )n−1 ∑ ∑ 1
2  1  ≥n ,
n−1 xi xj xk
1≤i<j≤n k=1

pentru orice n ∈ N, n ≥ 2.

2.75. Dacă 0 < a < b şi x1 , x2 , x3 ∈ [a, b], atunci:


( )
1 1 1 (2a + b)(2b + a)
9 ≤ (x1 + x2 + x3 ) + + ≤ .
x1 x2 x3 ab

2.76. Fie x, y, z ∈ [0, 1]. Demonstraţi că: x + y + z − xy − yz − zx ≤ 1.

2.77. Demonstraţi că pentru orice x, y, z ∈ [0, 1] avem:


∑ x ∏
+ (1 − x) ≤ 1.
y+z+1

x y z
2.78. Dacă x, y, z ∈ [0, 1], atunci: + + ≤ 2.
yz + 1 zx + 1 xy + 1
2.79. Dacă x1 , x2 , x3 ∈ [0, 1], atunci:

(x1 + x2 + x3 + 1)2 ≥ 4(x21 + x22 + x23 ).

Generalizare.

2.80. Fie x, y, z ∈ (1, 3). Să se arate că dacă xy + yz + zx = 26, atunci:
8, 75 < x + y + z < 9.

2.81. Fie x, y, z, t ∈ [1, 3]. Să se arate că:

(x + y + z + t)2 ≥ 3(x2 + y 2 + z 2 + t2 ).

45
2.82. Dacă x, y, z, t ∈ [−1, ∞) şi x + y + z + t = 2, demonstraţi că:
1
x3 + y 3 + z 3 + t3 ≥ .
2

n ∏
n
2.83. Dacă 0 < xk < 1 (k = 1, n), atunci: xk − xk < n − 1.
k=1 k=1


n
1 n
2.84. Dacă x1 , x2 , . . . , xn ≥ 1, atunci: ≥ √ .
1 + xk 1 + x1 x2 . . . xn
n
k=1

2.85. Demonstraţi că dacă x > y > 0, atunci:


(x − y)2 x+y √ (x − y)2
< − xy < .
8x 2 8y
2.86. Dacă a, b, c ∈ R şi a ≥ b ≥ c, atunci avem:
7
a2 + b2 + c2 − ab − bc − ca ≥ (a − b)(b − c).
3
2.87. Dacă a ≥ b ≥ c ≥ 0, atunci:
a3 b b3 c c3 a ab3 bc3 ca3
+ + ≥ + + .
a3 + b3 b3 + c3 c3 + a3 a3 + b3 b3 + c3 c3 + a3
2.88. Dacă x1 , x2 , y1 , y2 ∈ R şi x21 ≤ y1 , x22 ≤ y2 , atunci:
1
(x1 − x2 )(x2 y1 − x1 y2 ) ≤ (y1 − y2 )2 .
4
2.89. Dacă a, b, c, x, y, z, t > 0 şi x ≥ y, z ≤ t, atunci:
x y z t 4
+ + + ≥ .
ay + bz + ct az + bt + cx at + bx + cy ax + by + cz a+b+c
2.90. Dacă a ≥ b ≥ c > 0, arătaţi că:
an b(a − b) + bn c(b − c) + cn a(c − a) ≥ 0, pentru orice n ∈ N∗ .

2.91. Demonstraţi că dacă 0 < a1 ≤ a2 ≤ . . . ≤ an , atunci:


[ ]

n
1 1 ∑n
≤ n(a1 + an ) − ak .
ak a1 an
k=1 k=1

2.92. Fie f : R → (0, ∞) o funcţie strict crescătoare şi a1 ≤ a2 ≤ . . . ≤ an . Să


se demonstreze inegalitatea:
f (a1 ) f (a2 ) f (an ) f (a2 ) f (a3 ) f (a1 )
+ + ... + ≥ + + ... + .
f (a2 ) f (a3 ) f (a1 ) f (a1 ) f (a2 ) f (an )

46
2.2 Probleme propuse
 2.1. Să se arate că pentru orice x ∈ R avem: x8 − x5 + x2 − x + 1 > 0.

 2.2. a) Fie x, y ∈ R. Să se arate că dacă sunt numere naturale de aceeaşi
paritate, atunci:
xm + y m xn + y n xm+n + y m+n
· ≤ .
2 2 2

x + y x2 + y 2 x3 + y 3 x6 + y 6
b) Dacă x, y ∈ R, atunci: · · ≤ .
2 2 2 2
( )
x4 + x3 y + x2 y 2 + xy 3 + y 4 x+y 4
 2.3. Dacă x, y ∈ R, atunci: ≥ .
5 2
 2.4. Dacă x, y ∈ R sunt astfel ı̂ncât x + y > 0, atunci:

x4 + y 4 x+y x5 + y 5 x+y
3 2 2 3
≥ şi 4 3 2 2 3 4
≥ .
x + x y + xy + y 4 x + x y + x y + xy + y 5

 2.5. Să se arate că dacă x, y, z ∈ R, atunci:

(x + y − z)2 + (−x + y + z)2 + (x − y + z)2 ≥ xy + yz + zx.

 2.6. Dacă x, y, z ∈ R, atunci:

x2 + y 2 + z 2
min{(x − y)2 , (y − z)2 , (z − x)2 } ≤ .
2
Generalizare.

 2.7. Să se arate că pentru orice x, y, z ∈ R avem:


√ √
2 · (x + y)2 + (y + z)2 + (z + x)2 ≥ |x| + |y| + |z| + |x + y + z|

≥ |x + y| + |y + z| + |z + x|.

 2.8. Demonstraţi că pentru orice x, y, z ∈ R are loc inegalitatea:


3
x2 + y 2 + z 2 − xy − yz − zx ≥ (x − y)2 .
4

 2.9. Dacă a, b, c, d ∈ R, atunci: 2(a2 − ab + b2 )(c2 − cd + d2 ) ≥ a2 c2 + b2 d2 .


În ce condiţii avem egalitatea?

47
√ √
2a − a2 + 2b − b2
 2.10. Să se arate că dacă a, b ∈ (0, 2), atunci: √ ≤ 1.
4(a + b) − (a + b)2
∑1( 2 1
)
1
 2.11. Să se arate că pentru orice x, y, z > 0 avem: 1− − ≤ .
x y x 4
 2.12. Să se demonstreze inegalitatea:
x y z 1
+ + ≥ ,
x + 2y + 3z y + 2z + 3x z + 2x + 3y 2

unde x, y, z > 0.

 2.13. Să se demonstreze că dacă x, y, z > 0, atunci:


x y z 3
+ + ≥ .
x + 2y + z y + 2z + x z + 2x + y 4

 2.14. Să se arate că pentru orice x, y, z > 0 avem:


xy yz zx
+ 2 + 2 ≤ 1.
x2 + xy + y 2 y + yz + z 2 z + zx + x2

 2.15. Să se arate că pentru orice x, y, z > 0 avem:


x y z 9
+ + ≤ .
(x + y)(x + z) (y + z)(y + x) (z + x)(z + y) 4(x + y + z)

 2.16. Dacă x, y, z ≥ 0, atunci: x8 + y 8 + z 8 ≥ x2 y 2 z 2 (xy + yz + zx).

 2.17. Să se demonstreze că pentru orice x, y, z > 0 avem:


∑ 3
(x + y)2 z ≤ (x + y)(y + z)(z + x).
2

 2.18. Fie a, b, c > 0. Să se demonstreze că:


( )
1 1 1
2 2
(a + b + c )2
+ + ≥ 3(a + b + c).
a b c

 2.19. Să se arate că dacă a, b, c > 0, atunci:

(a + b)(b + c)(c + a) a+c b+c


≥ + .
4abc b+c a+c

48
ab bc ca a+b+c
 2.20. Dacă a, b, c > 0, atunci: + + ≤ .
a+b b+c c+a 2
 2.21. Dacă a, b, c > 0, atunci:

ab + bc + ca + a2 ab + bc + ca + b2 ab + bc + ca + c2
+ + ≤ a + b + c.
b + c + 2a a + c + 2b a + b + 2c

 2.22. Dacă a, b, c > 0 arătaţi că:

9abc ab2 bc2 ca2 a2 + b2 + c2


≤ + + ≤ .
2(a + b + c) a+b b+c c+a 2

 2.23. Dacă a, b, c > 0, atunci:


∑ a2 + b2 ∑ ∑ a+b ∑1
≥ a şi ≤ .
a+b a2 + b2 a
[ ]
1 1 1
 2.24. Dacă a, b, c > 0, atunci: (1 + abc) + + ≥ 3.
a(b + 1) b(c + 1) c(a + 1)
∑ ∑ √
 2.25. Demonstraţi că pentru x, y, z ≥ 0 avem: x3 ≥ x2 yz.

(x + y + z)2 √ √ √
 2.26. Dacă x, y, z ≥ 0, atunci: ≥ x yz + y zx + z xy.
3
 2.27. Dacă x, y, z > 0, demonstraţi că:
( ) √ √
3 x2 y 2 z2 x y z
+ 2 + 2 +1 ≥ +2 +33 ≥ 6.
2 y2 z x y z x

 2.28. Fie a, b, c, d > 0. Arătaţi că:



√ √ 2 2
a2 + c2 + b2 + d 2 ≥ .
1 1
+
a+b c+d
√ √ √ 2
 2.29. Dacă a, b, c > 0, atunci: 3
ab + 3
bc + 3
ca − 1 ≤ (a + b + c).
3
 2.30. Dacă a, b, c, x, y, z ≥ 0, atunci:
√ √ √
(a + x)(b + y)(c + z) ≥ abc + 3 xyz.
3 3

Generalizare.

49
( )
1
 2.31. Fie x, y, z ∈ R şi h1 , h2 , h3 ∈ 0, cu h1 + h2 + h3 = 1. Demonstraţi
2
că: ∑
h1 h2 h3 (x + y + z)2 ≥ xyh3 (1 − 2h3 ).
Când are loc egalitatea?
 2.32. Dacă a, b, c, d > 0, atunci:
a b c d 4
+ + + ≥ .
b+c+d c+d+a d+a+b a+b+c 3
Generalizare.
 2.33. Dacă a, b, c > 0 şi n ∈ N∗ , atunci:
a2n+1 b2n+1
+
b2n − b2n−1 c + . . . + c2n c2n − c2n−1 a + . . . + a2n
c2n+1 ab + bc + ca
+ ≥3 .
a −a
2n 2n−1 b + ... + b 2n a+b+c
 2.34. Fie a1 , a2 , . . . , an > 0. Să se demonstreze inegalitatea:
(
n(n − 1) 1 1 1
+ + ... +
2 a1 a2 a1 a3 a1 an
)
1 1 1
+ + ... + + ... +
a2 a3 a2 an an−1 an
( )
1 1 1 1
≥4 + + ... + + ... + .
a1 + a2 a1 + a3 a1 + an an−1 + an
 2.35. Demonstraţi că dacă a1 , a2 , . . . , an > 0 şi a1 + a2 + . . . + an = 1, atunci:
a21 a22 a2n 1
+ + ... + ≥ .
a1 + a2 a2 + a3 an + a1 2

 2.36. Dacă a1 , a2 , . . . , an > 0, atunci:

(1 + a1 )2 (1 + a2 )3 . . . (1 + an )n+1 ≥ a1 a2 . . . an (n + 1)n+1 .

 2.37. Dacă a1 , a2 , . . . , an > 0, atunci:


√k a a . . . a +. . .+ k a
√ √
1 2 k n−k+1 . . . an a1 a2 . . . ak +. . .+an−k+1 . . . an
≤ k
,
Cnk Cnk
pentru n ≥ k.

50
 2.38. Fie (ain )n≥1 , i ∈ {1, 2, . . . , p} şiruri de numere reale strict pozitive.
Demonstraţi că:
 
∑  (∑ ( p )p )
 n 1  n ∑
  aik ≥ pp · n2 , pentru orice n ∈ N∗ .
 ∏p 
k=1  k=1 i=1
aik
i=1

 2.39. Fie ak , bk , ck > 0, pentru orice k = 1, n. Să se demonstreze inegalitatea:



n ∑
n ∑
n ∑
n
a2k b2k c2k (ak + bk + ck )2
k=1 k=1 k=1 k=1
+ + ≥ .

n ∑
n ∑
n ∑n
ak bk ck (ak + bk + ck )
k=1 k=1 k=1 k=1

( )2 ( )2
1 1 25
 2.40. Dacă a, b > 0 şi a + b = 1, atunci: a+ + b+ ≥ .
a b 2
 2.41. Dacă x, y > 0 şi x5 + y 5 = x − y ≥ 0, atunci: x4 + y 4 < 1.
7
 2.42. Fie x, y < 0 astfel ı̂ncât 7(x+y) = 3(x3 −y 3 ). Să se arate că: x2 +y 2 > .
3
 2.43. Arătaţi că dacă a, b, c ≥ 0 şi a + b + c = 1, atunci:
ab bc ca 1
+ + ≤ .
c+1 a+1 b+1 4
xy yz zx
 2.44. Găsiţi minimul expresiei E = + + , dacă
z x y
x, y, z > 0 şi x2 + y 2 + z 2 = 1.

 2.45. Fie x, y, z ∈ R astfel ı̂ncât x + y + z = 3. Să se arate că:

x2 y 2 + y 2 z 2 + z 2 x2 ≥ 3xyz.

 2.46. Dacă numerele reale x, y, z ı̂ndeplinesc condiţiile

x + y + z = 2 şi xy + yz + zx = 1,
[ ]
4
atunci: x, y, z ∈ 0, .
3

51
 2.47. Dacă x, y, z ∈ R şi x + y + z = 5, xy + yz + zx = 3, atunci
[ ]
13
x, y, z ∈ −1, .
3

 2.48. Dacă x, y, z > 0 şi x4 + y 4 + z 4 = 3, atunci:

x + y + z ≤ 3, xy + yz + zx ≤ 3 şi xyz ≤ 1.

 2.49. Dacă a, b, c > 0 şi abc = 1, atunci:


( )6
a+b+c
(a + b)(b + c)(c + a) ≤ .
2

 2.50. Dacă a, b, c ∈ R şi a + b + c = 2, atunci:


1
a2n + b2n + c2n > , pentru orice n ∈ N.
22n−2

 2.51. Dacă a, b, c > 0 şi a + b + c = 1, atunci:

a2 + b2 b2 + c2 c2 + a2
+ + ≥ 1.
a+b b+c c+a


n ∑
n
 2.52. Demonstraţi că dacă ak = bk = 1 iar ak , bk > 0 (i = 1, 2, . . . , n),
k=1 k=1

n
a2
atunci: k
≥ 1.
bk
k=1

x y 1
 2.53. Dacă x, y, z > 0 şi xyz = 1, atunci: + + ≥ x + y + 1.
y z x
 2.54. Dacă x1 , x2 , . . . , xn > 0 şi x1 x2 . . . xn = 1, atunci:
( ) ( ) ( )
x2 2 x3 2 x1 2
x1 + + x2 + + . . . + xn + ≥ 4n.
x3 x4 x2

1
 2.55. Dacă x, y ∈ R şi x + y ≤ − , atunci x2 ≥ y şi y 2 ≥ x.
4
√ ( 2 )2
1 x2 + y 2 x + y x − y2
 2.56. Dacă x, y ≥ , atunci: − ≤ .
2 2 2 2

52
x y a b
 2.57. Dacă x, y ∈ [a, b], unde a, b > 0, atunci: 2 ≤ + ≤ + .
y x b a
 2.58. Fie xk ∈ [0, 1], pentru orice k = 1, n. Demonstraţi că:

x1 + x2 + . . . + xn − (x1 x2 + x2 x3 + . . . + xn x1 )

nu depăşeşte:
[n]
a) 1, pentru n = 3; b) 2, pentru n = 4; c) , pentru n ≥ 3.
2
( )
1
 2.59. Fie x, y, z ∈ , ∞ . Să se arate că cel mai mare dintre numerele
3

3x − y 2 + z 3 , 3y − z 2 + x3 , 3z − x2 + y 3
1
nu poate fi mai mare decât .
27
 2.60. Fie xk , αk ∈ R astfel ı̂ncât:


 α1 = x1 + x2 . . . xn



 α1 α2 = x2 + x3 + . . . + xn
(1) ..
 .



 α1 α2 . . . αn−1 = xn−1 + xn

α1 α2 . . . αn = xn
Să se arate că sistemul de inegalităţi:
(2) 0 < αk < 1, k = 1, n este echivalent cu:
(3) x1 + x2 + . . . + xk < 1, xk > 0, k = 1, n.
4
 2.61. Demonstraţi că pentru x > y ≥ 0, avem: x + ≥ 3.
(x − y)(y + 1)2
a2 c2 b2 c2
 2.62. Dacă a > b > c > 0, atunci: + 2 > + 2 .
b2 a2 a2 b2
 2.63. Să se demonstreze inegalitatea:

(x + y + z)(xy + yz + zx) ≥ 9xyz + (y − x)(z − x)2 ,

unde 0 < x ≤ y ≤ z.
 2.64. Se consideră numerele reale x, x1 , x2 , y, y1 , y2 astfel ı̂ncât x21 + x22 ≤ x2 şi
y12 + y22 ≤ y 2 . Să se arate că:

(x1 + y1 )2 + (x2 + y2 )2 ≤ 4 max(x2 , y 2 ).

53
 2.65. Dacă a1 , a2 , a3 , a4 , a5 , a6 ∈ R şi a1 < a2 < a3 < a4 < a5 < a6 , atunci:

(a1 + a2 + a3 + a4 + a5 + a6 )2 > 12(a1 a6 + a2 a5 + a3 a4 ).

 2.66. Fie a ≤ b ≤ c. Dacă f continuă pe [a, c] şi f ′ crescătoare pe (a, c), atunci:

(a − b)f (c) + (b − c)f (a) + (c − a)f (b) ≤ 0.

 2.67. Demonstraţi că dacă a1 < a2 < . . . < an şi 1 ≤ k < n (k, n ∈ N), atunci:
a1 + a2 + . . . + ak a1 + a2 + . . . + an ak+1 + ak+2 + . . . + an
< < .
k n n−k

 2.68. Se dau două şiruri de numere reale astfel ı̂ncât:

a1 > a2 > a3 > . . . > an şi b1 > b2 > b3 > . . . > bn .

Să se arate că: a1 b1 + a2 b2 + . . . + an bn > a1 bn + a2 bn−1 + . . . + an b1 .


k ∑
k
 2.69. Dacă a1 ≥ a2 ≥ . . . ≥ an ≥ 0, b1 , b2 , . . . , bn ∈ R şi ai ≤ bi , pentru
i=1 i=1
orice k = 1, n, atunci:

n ∑
n
a2i ≤ b2i .
i=1 i=1

 2.70. Dacă 0 < x1 < x2 < . . . < xn ≤ 1, atunci:


x1 + x2 + . . . + xn 1 1 1
x1 + x2 + . . . + xn ≥ + + ... + .
+ x1 x2 . . . xn 1 + x1 1 + x2 1 + xn
n

 2.71. Dacă x1 , x2 , . . . , xn ∈ R şi x1 ≥ x2 ≥ . . . ≥ xn > 0, atunci:


(x3 + x4 + . . . + xn )(x1 − x2 ) (x1 + x4 + . . . + xn )(x2 − x3 )
+ + ...+
x1 + x2 x2 + x3
(x1 +x2 + . . . +xn−2 )(xn−1 −xn ) (x2 +x3 + . . . + xn−1 )(xn −x1 )
+ + ≥ 0.
xn−1 +xn xn + x1
1 √
 2.72. Dacă a, b, c > 0 astfel ı̂ncât a2 +b2 +c2 = 1, atunci a+b+c+ ≥ 4 3.
abc
1 √
 2.73. Dacă a, b, c > 0 astfel ı̂ncât a3 +b3 +c3 = 1, atunci a+b+c+ ≥ 3+ 9.
3

abc

54
 2.74. Dacă k, n ∈ N∗ , a1 , . . . , an > 0 astfel ı̂ncât ak1 + ak2 + . . . + akn = 1, atunci
1 √
k √
a1 + a2 + . . . + an + ≥ nk−1 + k n.
a1 a2 . . . an

 2.75. Dacă a, b > 0 astfel ı̂ncât a2 + b2 ≤ 2, atunci

1 2 √ 2 √
a+b+ ≥√ + 2(a2 + b2 ) ≥ 2 2
+ 2(a2 + b2 ).
ab 2
ab(a + b ) 2 a +b

2 a+b √
 2.76. Dacă a, b > 0, atunci + ≥ 2 ab.
1 1 2
+
a b
 2.77. Dacă a, b, c ∈ (0, 1) astfel ı̂ncât a2 + b2 + c2 = 1 să se arate că

12 4 abc(1 − a)(1 − b)(1 − c) ≤ 4 + (a + b + c − 1)2 .

 2.78. Dacă x, y, z, w ∈ R astfel ı̂ncât xy = 4, z 2 + w2 = 4, atunci


(x − z)2 + (y − w)2 ≥ 1, 6.

 2.79. Dacă xi ∈ R astfel ı̂ncât 0 ≤ xi ≤ 1, pentru orice i = 1, n, arătaţi că


 1
( ) 
 dacă n este par
1∑ 2
n
1∑
n 2  4
xi − xi ≤
n n 

1 1
i=1 i=1  − 2 dacă n este impar
4 4n

 2.80. Pentru x, y, z > 0, fir


yz zx xy
A= 2
+ 2
+ şi
(y + z) (z + x) (x + y)2
yz zx xy
B= + + .
(y + x)(z + x) (z + y)(x + y) (x + z)(y + z)

a) Arătaţi că A + 3B ≥ 3. În ce caz avem egalitate?


b) Arătaţi că 4A + 8B ≥ 9. În ce caz avem egalitate?
 2.81. Dacă x, y, z > 0, arătaţi că:
x+y+z xy + yz + zx
√ ≥√ √ √ .
3 3 y + yz + z + z 2 + zx + x2 + x2 + xy + y 2
2 2

În ce caz avem egalitate?

55
 2.82. Dacă a, b, c > 0, arătaţi că:
( ) √ 2 √ √
√ 1 1 1 b + bc + c2 c2 + ca + a2 a2 + ab + b2
3 + + ≤ + + .
a b c bc ca ab
Când avem egalitate?
 2.83. Dacă a, b, c > 0, arătaţi că:
3 a b c a+b b+c c+a 9
i) ≤ + + ≤ + + − ;
2 b+c c+a a+b c a b 2
a b c 5√
ii) √ +√ +√ ≤ a + b + c.
a+b b+c c+a 4

2.3 Soluţii
2.1. a) Inegalitatea se mai scrie sub următoarele forme echivalente:
4x4 + 4x3 + 4x2 + 4x + 4 > 0 ⇔
4x4 + 4x3 + x2 + 2x2 + x2 + 4x + 4 > 0 ⇔
x2 (2x + 1)2 + 2x2 + (x + 2)2 > 0.
Semnul egal nu poate avea loc căci ar trebui simultan ca x = 0 şi x = −2.
b) Inegalitatea (x − 1)4 ≤ x4 + x3 + x2 + x + 1 este echivalentă cu
următoarele:
x4 − 4x3 + 6x2 − 4x + 1 ≤ x4 + x3 + x2 + x + 1 ⇔ 0 ≤ 5x(x2 − x + 1),
ceea ce are loc deoarece
( )2
1 3
x −x+1=
2
x− + > 0.
2 4
Inegalitate x4 + x3 + x2 + x + 1 ≤ (x + 1)4 = x4 + 4x3 + 6x2 − 4x + 1 se
reduce la 0 ≤ x(3x2 + 5x + 3), ceea ce rezultă din x ≥ 0.
2.2. Inegalitatea este echivalentă cu următoarea:
4|x| · |1 − x2 | ≤ (1 + x2 )2 .
Fie |x| = a ≥ 0. Avem de demonstrat că: 4a(1 − a2 ) ≤ (1 + a2 )2 , dacă
a ≤ 1 şi 4a(a2 − 1) ≤ (1 + a2 )2 , dacă a ≥ 1. Dacă a = 0, inegalitatea este
evidentă. Dacă 0 < a ≤ 1, inegalitatea de demonstrat se scrie succesiv:
a4 + 2a2 + 1 ≥ 4a − 4a3 ⇔ a4 + 4a3 + 2a2 − 4a + 1 ≥ 0 ⇔

56
( ) ( ) ( )
1 1 1 2 1
a + 2 +2+4 a−
2
≥0 ⇔ a− +4 a− +4≥0 ⇔
a a a a
( )2
1
a − + 2 ≥ 0.
a
√ √ √
Semnul egal are loc ⇔ a = 2 − 1 ⇔ |x| = 2 − 1 ⇔ x = ±( 2 − 1).
Dacă a > 1, inegalitatea se reduce (analog inegalităţii de mai sus) la
( )2
1
a− −2 ≥ 0.
a
√ √
Egalitatea se obţine pentru a = 2 + 1 ⇔ x = ±( 2 + 1).

2.3. Inegalitatea este echivalentă cu următoarele:

(x2 )n −1−2n(x−1) ≥ 0 ⇔ (x2 −1)(x2n−2 +x2n−4 +x2 +1)−2n(x−1) ≥ 0

sau

(x−1)[(x2n−1 +x2n−3 +. . .+x3 +x)+(x2n−2 +x2n−4 +. . .+x2 +1)−2n] ≥ 0

⇔ (x − 1)(x2n−1 + x2n−2 + x2n−3 + . . . + x3 + x2 + x + 1 − 2n) ≥ 0 ⇔

(x−1)[(x2n−1 −1)+(x2n−2 −1)+. . .+(x3 −1)+(x2 −1)+(x−1)] ≥ 0 ⇔

(x−1)2 [(x2n−2 +x2n−3 +. . .+x+1)+(x2n−3 +x2n−4 +. . .+x+1)+. . . +

+(x2 + x + 1) + (x + 1) + 1] ≥ 0.
Dacă x ≥ 0 inegalitatea de mai sus este evidentă. Dacă x < 0, inegali-
tatea din enunţ rezultă din faptul că x2n , −2nx şi 2n − 1 sunt numere
pozitive.

2.4. Înmulţind inegalitatea dată cu 2 şi grupând convenabil, avem:

x2n + (x2n ± 2x2n−1 + x2n−2 ) + (x2n−2 ± 2x2n−3 + x2n−4 ) + . . . +

+(x2 ± 2x + 1) + 1 > 1 ⇔

x2n + x2n−2 (x ± 1)2 + x2n−4 (x ± 1)2 + . . . + (x ± 1)2 > 0,


inegalitate adevărată deoarece membrul stâng este o sumă de pătrate
perfecte.

57
2.5. Expresia din membrul stâng al inegalităţii se mai scrie:
( )
a2 a2
x4 + y 4 + axy + 2 = (x4 − 2x2 y 2 + y 4 ) + 2x2 y 2 + axy + +2−
8 8
( ( )
a )2 a2
= (x − y ) + 2 xy +
2 2 2
+ 2− .
4 8
Inegalitatea rezultă deoarece fiecare termen este nenegativ.
2.6. Dacă xy = 0 inegalitatea are loc căci
( )2
1 3
1 − y + 2y = y +
2 4 4
−y 2
+ > 0.
2 4

Fie xy ̸= 0. Împărţind cu x2 y 2 inegalitatea de demonstrat devine:


( )( )
1 1
2
+ 2x + 1 + 2y − 1 ≥ 7.
2
x2 y2
Dar inegalitatea mediilor implică

1 1 √
+ 2x2 ≥ 2 · 2x2
= 2 2
x2 x2
1 1 1
(semnul egal are loc ⇔ 2
= 2x2 ⇔ x2 = √ ⇔ x = ± √
4
).
x 2 2
Rezultă că:
( )( )
1 1 √ √
+ 2x2
+ 1 + 2y 2
− 1 ≥ (2 2 + 1)(2 2 − 1) = 7.
x2 y2
∑ ∑
2.7. Deoarece x3 y 3 ≤ |x|3 · |y|3 putem considera că x, y, z ≥ 0. Vom
folosi următoarea formă particulară a inegalităţii lui Abel:
Dacă a1 ≥ a2 ≥ a3 ≥ 0 şi b1 ≥ 0, b1 + b2 ≥ 0, b1 + b2 + b3 ≥ 0, atunci
a1 b1 + a2 b2 + a3 b3 ≥ 0 (demonstraţia inegalităţii rezultă din identitatea
a1 b1 + a2 b2 + a3 b3 = a3 (b1 + b2 + b3 ) + (a2 − a3 )(b1 + b2 ) + (a1 − a2 )b1 ≥ 0).
Din cauza simetriei putem presupune că x ≥ y ≥ z ≥ 0. Inegalitatea de
demonstrat se scrie:

x2 [x4 − (y 3 + z 3 )x + y 2 z 2 ] ≥ 0.

Luând

a1 = x2 , a2 = y 2 , a3 = z 2 , b1 = x4 − (y 3 + z 3 )x + y 2 z 2 ,

58
b2 = y 4 − (x3 + z 3 )y + x2 z 2 , b3 = z 4 − (x3 + y 3 )z + x2 y 2 ,
rămâne de arătat că b1 ≥ 0, b1 + b2 ≥ 0 şi b1 + b2 + b3 ≥ 0. Avem:

b1 = x4 −(y 3 +z 3 )x+y 2 z 2 ≥ x4 −xy 3 −xz 3 +yz 3 = x(x3 −y 3 )−z 3 (x−y)

= (x − y)(x3 + x3 y + xy 2 − z 3 ) ≥ 0;
b1 + b2 = x4 + y 4 − z 3 (x + y) − xy(x2 + y 2 ) + z 2 (x2 + y 2 )
= x3 (x − y) − y 3 (x − y) + z 2 (x2 − xz + y 2 − yz)
= (x − y)2 (x2 + xy + y 2 ) + z 2 [x(x − z) + y(y − z)] ≥ 0
şi ∑ ∑ ∑
b1 + b 2 + b3 = x4 − x3 (y + z) + x2 y 2 .

Din x2 (2x − y − z)2 ≥ 0 deducem
∑ (∑ )
∑ ∑ x2 y 2 + xyz x
x4 − x3 (y + z) + ≥ 0.
2
∑ (∑ )
x2 y 2 xyz x
Cum ≥ rezultă prin adunare că:
2 2
∑ ∑ ∑
x4 − x3 (y + z) + x2 y 2 ≥ 0,

deci b1 + b2 + b3 ≥ 0 şi deci a1 b1 + a2 b2 + a3 b3 ≥ 0.


Observaţie. O a doua demonstraţie a problemei rezultă din inegalitatea
lui Popovici de la problema 9.52.
∑ ∑ ∑
2.8. Notăm x2 = p, xy = q, x3 − 3xyz = m. Cum p − q ≥ 0,
rezultă: (∑ )2 (∑ ∑ )2
m2 = x x2 − xy
(∑ ∑ ) (∑ ∑ )2
= x2 + 2 xy x2 − xy = (p + 2q)(p − q)2 ,

şi deci:

p3 − m2 = p3 − (p + 2q)(p − q)2 = p3 − (p3 − 3pq 2 + 2q 3 ) = 3pq 2 − 2q 3

= q 2 (3p − 2q) = q 2 [2(p − q) + p] ≥ 0


căci p − q ≥ 0 şi p ≥ 0.

59
2.9. Avem succesiv:

xy(y − z)(z − x) + z(x − y)[x(y − z) + y(z − x)]

= xy(y − z)(z − x) + z(x − y)(xy − xz + yz − xy)


= xy(y − z)(z − x) − z 2 (x − y)2 = −z 2 (x − y)2 + xy(yz − z 2 − xy + xz)
= −z 2 (x − y)2 − x2 y 2 + xyz(x + y) − xyz 2
1
= − [(4z 2 (x − y)2 + 4xyz 2 + 4x2 y 2 − 4xyz(x + y)]
4
1
= − [4z 2 (x − y)2 + z 2 (x + y)2 − z 2 (x − y)2 + 4x2 y 2 − 4xyz(x + y)]
4
1
= − [3z 2 (x − y)2 + z 2 (x + y)2 − 4xyz(x + y) + 4x2 y 2 ]
4
1
= − [3z 2 (x − y)2 + (z(x + y) − 2xy)2 ] ≤ 0.
4
Egalitatea are loc dacă şi numai dacă x = y = z.
x y y z z x x2 y 2
2.10. Fie + = a, + = b, + = c. Rezultă 2 + 2 = a2 − 2, etc. şi
y x z y x z y x
inegalitatea de demonstrat devine:
∑ ∑ ∑ ∑ ∑
(a2 − 2)2 ≥ a2 ⇔ a4 − 4 a2 + 12 ≥ a2 ⇔
∑ ∑
(a4 − 5a2 + 4) ≥ 0 ⇔ (a2 − 1)(a2 − 4) ≥ 0.

x2 + y 2 |x|2 + |y|2
Dar |a| = = ≥ 2, deci a2 ≥ 4 şi prin urmare
|xy| |xy|

(a2 − 1)(a2 − 4) ≥ 0.

2.11. Deoarece dacă notăm E(x, y, z) = x(x + y)3 + y(y + z)3 + z(z + x)3 ,
atunci E(x, y, z) = E(−x, −y, −z), pentru orice x, y, z ∈ R, rămân de
studiat următoarele cazuri:
i) x, y, z pozitive;
ii) x ≤ 0 şi y, z ≥ 0, adică două numere pozitive şi al treilea nega-
tiv; evident alegerea făcută nu influenţează generalitatea demonstraţiei.
Cazul i) este evident. Rămâne de studiat cazul ii). Înlocuind pe x cu −x
obţinem

E = x(x − y)3 + y(y + z)3 + z(z − x)3 , unde x, y, z ≥ 0.

60
Dacă x ≥ y ⇒ −y ≥ −z, deci

E ≥ x(x − z)3 + y(y + z)3 + z(z − x)3 = (x − z)3 (x − z) + y(y + z)3

= (x − z)4 + y(y + z)3 ≥ 0, evident.


Dacă z ≥ x, avem:

E = [y(x − y)3 + y(y + z)3 ] + x(x − y)3 − y(x − y)3 + z(z − x)3

= y[(x − y)3 + (y + z)3 ] + (x − y)4 + z(z − x)3


= y(x−y+y+z)[(x−y)2 −(x−y)(y+z)+(y+z)2 ]+(x−y)4 +z(z−x)3 ≥ 0,
deoarece a2 − ab + b2 ≥ 0, pentru orice a, b ∈ R şi am luat a = x − y,
b = y + z. Dacă z ≤ x ≤ y, atunci

E = x(x − y)3 + y(y + z)3 + z(z − x)3

şi deoarece −x ≥ −y şi −z ≥ −y rezultă că

x(x − y)3 = −x(y − x)3 ≥ −y(y − x)3 = y(x − y)3

iar
z(z − x)3 = −z(x − z)3 ≥ −y(x − z)3 = y(z − x)3 ,
deci
E ≥ y(x − y)3 + y(y + z)3 + y(z − x)3
= y[x3 − 3x2 y + 3xy 2 − y 3 + y 3 + 3y 2 z + 3yz 2 + z 3 + z 3 − 3z 2 x + 3zx2 − x3 ]
= y[3xy(y − x) + 2z 3 + 3yz(y + z) + 3zx(x − z)] ≥ 0
deoarece y ≥ x, x ≥ z. În fine, dacă z ≤ y ≤ x, notând y = z + a,
x = z + a + b, unde a, b ≥ 0, avem:

E = (z + a + b)b3 + (z + a)(2z + a)3 − z(a + b)3

= 8z 4 + 20az 3 + [18a2 z 2 − 3a(ab + b2 − 2a2 )z + a4 + ab3 + b4 ] ≥ 0


căci discriminantul trinomului de gradul doi ı̂n z, din paranteza dreaptă,
este negativ.
2.12. Folosind inegalitatea lui Minkowski, avem:
√ √ √
(a + c)2 + b2 + (a − c)2 + b2 ≥ (a + c + a − c)2 + (b + b)2
√ √
= 4(a2 + b2 ) = 2 a2 + b2 .
Semnul egal are loc dacă şi numai dacă c = 0.

61
2.13. Fie x = a − b, y = b − c, z = c − d, t = d − a; deci x + y + z + t = 0 şi
x, y, z, t ∈ R∗ . Inegalitatea de demonstrat devine:

xyzt + (x + y)2 (x + t)2 > 0 ⇔ xyzt + [x2 + x(y + t) + yt]2 > 0.

Înlocuind y + t = −(x + z) ı̂n ultima inegalitate avem:

xyzt + (yt − xz)2 > 0 ⇔ y 2 t2 + x2 z 2 > xyzt.

Dacă xyzt < 0, inegalitatea este evidentă. Dacă xyzt > 0, atunci din
(yt − xz)2 > 0 rezultă y 2 t2 + x2 z 2 > 2xyzt > xyzt.
( n )2
∑ ∑ ∑
n
2.14. Deoarece 2 xi xj = xk − x2k , inegalitatea devine:
1≤i<j≤n k=1 k=1

n−2∑ 2 ∑
n n
s2 n
xk + s + + ≥ 0, unde s = xk .
n−1 n−1 8
k=1 k=1


n
1 2
Deoarece x2k ≥ s este suficient să arătăm că:
n
k=1

n−2 2 s2 n 2 n
(n−1) s +s+ + ≥ 0 ⇔ s2 +s+ ≥ 0 ⇔ (4s+n)2 ≥ 0,
n n−1 8 n 8
ceea ce este evident. Egalitatea se obţine dacă şi numai dacă
1
x1 = x2 = . . . = xn = − .
4

2.15. Fără a restrânge generalitatea problemei vom presupune că:

(x1 − x2 )2 = max{(x1 − x2 )2 , (x2 − x3 )2 , . . . , (xn − x1 )2 }.

Avem de arătat că:



n ∑
n
n
2 x2k − 2 xk xk+1 ≥ (x1 − x2 )2 ⇔
n−1
k=1 k=1


n ∑
n
n
(x2k + x2k+1 ) − 2 xk xk+1 ≥ (x1 − x2 )2 ⇔
n−1
k=1 k=1

n
(n − 1) (xk − xk+1 )2 ≥ n(x1 − x2 )2 ⇔
k=1

62

n
(n − 1) (xk − xk+1 )2 ≥ (x1 − x2 )2 .
k=2
Dar ţinând cont de inegalitatea Cauchy-Buniakovski, avem:

k(b21 + b22 + . . . + b2k ) ≥ (b1 + b2 + . . . + bk )2

şi luând bi = xi+1 − xi+2 , i = 1, k iar k = n − 1, rezultă:



n
(n − 1) (xk − xk+1 )2 ≥ (x2 − x1 )2 .
k=2

2.16. Deoarece x2 − x + 1 > 0, pentru orice x ∈ R, luând x = λak + bk (cu


ak , bk ∈ R şi k = 1, n, λ ∈ R), rezultă:

n
[(λak + bk )2 − (λak + bk ) + 1] > 0,
k=1

pentru orice λ ∈ R ⇔
( n )
∑ ∑n ∑n ∑n
λ2 a2k +λ (2ak bk −ak )+ b2k − bk +n > 0, pentru orice λ ∈ R.
k=1 k=1 k=1 k=1

Deoarece trinomul de gradul doi ı̂n λ este pozitiv pentru orice λ ∈ R,


rezultă că ∆ < 0, deci:
[ n ]2 ( n )( n )
∑ ∑ ∑ ∑n
(2ak bk − ak ) − 4 2
ak bk −
2
bk + n < 0 ⇔
k=1 k=1 k=1 k=1
( )2 ( )( n )

n ∑
n ∑
n ∑
2 ak bk − ak <4 a2k b2k
k=1 k=1 k=1 k=1
( )( n )

n ∑ ∑
n
−4 a2k bk + 4n a2k ⇔
k=1 k=1 k=1
( )2 ( n )( n ) ( )2

n ∑ ∑ ∑
n
4 ak bk −4 ak bk ak + ak
k=1 k=1 k=1 k=1
( )( n ) ( )( n )

n ∑ ∑
n ∑ ∑
n
<4 a2k b2k −4 a2k bk + 4n a2k ⇔
k=1 k=1 k=1 k=1 k=1

63
( n )2 ( )( n ) ( n )2
∑ ∑
n ∑ ∑
4 ak bk +4 a2k bk + ak
k=1 k=1 k=1 k=1
( )( n ) ( )( n )

n ∑ ∑
n ∑ ∑
n
<4 a2k b2k +4 ak bk ak + 4n a2k ,
k=1 k=1 k=1 k=1 k=1
adică enunţul.

2.17. Înmulţind cu ab ambii membri, avem succesiv:

(2a + b)(2b + a) ≥ 9ab ⇔ 2a2 + 2b2 + 5ab − 9ab ≥ 0.

Egalitatea are loc dacă şi numai dacă a = b. Altă soluţie se obţine prin
inegalitatea mediilor:
2a + b a+a+b √
3 2b + a √
3
= ≥ a2 b; ≥ b2 a,
3 3 3
deci
2a + b 2b + a √ 3
· ≥ a3 b3 = ab.
3 3
( )
x2 + y 2 x+y 2
2.18. În inegalitatea evidentă: ≥ , punând
2 2
1 1
x=a+ , y =b+ ,
a b
obţinem:
( ) ( )
1 2 1 2
a+ + b+ ( ) ( )
a b 1 1 1 2 1 1 1 2
≥ a+ +b+ = 1+ +
2 4 a b 4 a b
( ) ( )
1 a+b 2 1 1 2
= 1+ = 1+ .
4 ab 4 ab
( )
a+b 2 1 1 1
Însă ab ≤ = , deci ≥4 ⇔ 1+ ≥ 5. Rezultă că:
2 4 ab ab
( ) ( ) ( )
1 2 1 2 1 1 2 1 2 25
a+ + b+ ≥ 1+ ≥ ·5 = .
a b 2 ab 2 2
1
Egalitatea are loc pentru a = b = .
2

64
2.19. Inegalitatea din enunţ se scrie succesiv:
a2 x2 2abx a2 y 2 2aby
+ + b 2
+ + + b2 ≥ 2a2 + 2b2 + 4ab ⇔
y2 y x2 x
( 2 ) ( )
2 x y2 x y
a + 2 + 2ab + ≥ 2a2 + 2 · 2ab,
y2 x y x
x2 y 2 x y
şi rezultă din 2
+ 2 ≥ 2, + ≥ 2. Egalitatea are loc dacă şi numai
y x y x
dacă x = y.

2.20. I-a soluţie. Ţinând cont de inegalitatea mediilor rămâne de demonstrat


că:
√ √ √ √ √
3(2 xy − 3 xy) ≥ 4 xy − 1 sau 2 xy + 1 ≥ 3 3 xy
ceea ce rezultă astfel:

√ √ √ √ √ √
2 xy + 1 = xy + xy + 1 ≥ 3 3 xy · xy · 1 = 3 3 xy.

Cea de-a doua inegalitate se demonstrează analog. Egalitatea are loc


⇔ x = y = 1.
√ √
A II-a soluţie. Adunând x + y + 1 ≥ 3 3 xy cu 2(x + y) ≥ 4 xy rezultă
enunţul.
√ √
2.21. Notăm n a = x > 0, n b = y > 0. Inegalitatea devine:
xn − y n
xy n−1 ≤ y n + ⇔ nxy n−1 ≤ ny n + xn − y n ⇔
n
xn − y n − ny n−1 (x − y) ≥ 0 ⇔
(x − y)(xn−1 + xn−2 y + . . . + y n−1 − ny n−1 ) ≥ 0 ⇔
(x − y)[(xn−1 − y n−1 ) + (xn−2 y − y n−1 ) + . . . + (xy n−2 − y n−1 )] ≥ 0 ⇔
(x − y)2 [(xn−2 + xn−3 y + . . . + y n−2 )
+y(xn−3 + xn−4 y + . . . + un−3 ) + . . . + y n−2 ] ≥ 0,
ceea ce este evident. Semnul egal are loc dacă şi numai dacă

x = y ⇔ a = b.

Observaţie. Scrisă ı̂n loc de n cu n + 1 inegalitatea are forma:


√ nb + a
abn ≤
n+1
.
n+1

65
Ea poate fi folosită la demonstrarea prin inducţie a inegalităţii mediilor,
făcând trecerea de la pasul n la n + 1. Astfel dacă presupunem că ea are
loc pentru a1 , a2 , . . . , an > 0 şi an+1 este un număr pozitiv, atunci:
√ √

n+1
an+1 (an . . . a1 ) = n+1 an+1 ( n a1 a2 . . . an )n

n n a1 a2 . . . an + an+1 a1 + a2 + . . . + an + an+1
≤ ≤ .
n+1 n+1
2.22. Fie b − a = x ≥ 0 şi c − a = y ≥ 0. Rezultă că b − c = x − y şi ţinând
cont de identitatea
1
a3 + b3 + c3 − 3abc = (a + b + c)[(a − b)2 + (b − c)2 + (c − a)2 ],
2
inegalitatea devine succesiv:

2(3a + x + y)[x2 + y 2 + (x − y)2 ] ≥ (x + y)3 ⇔

6a(2x2 + 2y 2 − 2xy) + 4(x + y)(x2 − xy + y 2 ) ≥ (x + y)3 ⇔


12a(x2 − xy + y 2 ) + 3(x3 + y 3 ) ≥ 3xy(x + y) ⇔
4a(x2 − xy + y 2 ) + x3 + y 3 ≥ xy(x + y) ⇔
4a(x2 − xy + y 2 ) + (x + y)(x2 − xy + y 2 − xy) ≥ 0 ⇔
4a(x2 − xy + y 2 ) + (x + y)(x − y)2 ≥ 0,
( y )2 3y 2
ceea ce evident are loc căci a > 0 şi x2 − xy + y 2 = x − + ≥ 0.
2 4
Egalitatea are loc dacă şi numai dacă x = y ⇔ a = b = c.

2.23. Punând
√ √ √
x y z
x1 = , x2 = , x3 = ,
ay + bz az + bx ax + by
√ √ √
y1 = x(ay + bz), y2 = y(az + bx), y3 = z(ax + by)
ı̂n inegalitatea lui Cauchy-Buniakovski, obţinem:
( )
x y z
+ + [x(ay + bz) + y(az + bx) + z(ax + by)]
ay + bz az + bx ax + by

≥ (x + y + z)2

66
de unde
x y z (x + y + z)2
+ + ≥ .
ay + bz az + bx ax + by (a + b)(xy + yz + zx)

Cum (x + y + z)2 ≥ 3(xy + yz + zx), pentru orice x, y, z > 0 deducem


că:
(x + y + z)2 3(xy + yz + zx) 3
≥ = .
(a + b)(xy + yz + zx) (a + b)(xy + yz + zx) a+b
Semnul egal are loc dacă şi numai dacă x = y = z.

2.24. I-a soluţie. Notând x + 2y + 2z = a, 2x + y + 2z = b, 2x + 2y + z = c


avem
1
x + y + z = (a + b + c)
5
şi rezolvând acest sistem obţinem
1 1 1
x = (2b + 2c − 3a), y = (2c + 2a − 3b), z = (2a + 2b − 3c).
5 5 5
a b
Înlocuind ı̂n inegalitatea din enunţ pe x, y, z şi ţinând cont că + ≥ 2,
b a
pentru orice a, b > 0, obţinem succesiv:
2b + 2c − 3a 2c + 2a − 3b 2a + 2b − 3c
+ +
5a 5b 5c
[( ) ( ) ( ]
2 a b b c c a) 9 12 9 3
= + + + + + − ≥ − = .
5 b a c b a c 5 5 5 5

A II-a soluţie. Notând x + y + z = a > 0, inegalitatea din enunţ devine:


x y z 3 a a a 9
+ + ≥ ⇔ + + ≥ .
2a − x 2a − y 2a − z 5 2a − x 2a − y 2a − z 5
Aplicând inegalitatea mediilor succesiv, avem:

a a a a a a
+ + ≥33 · ·
2a − x 2a − y 2a − z 2a − x 2a − y 2a − z

3a 3a
= √ ≥
3
(2a − x)(2a − y)(2a − z) (2a − x) + (2a − y) + (2a − z)
3
9a 9
= = .
5a 5

67
Egalitatea are loc dacă şi numai dacă x = y = z.
A III-a soluţie. În inegalitatea Cauchy-Buniakovski:

(x21 + x22 + x23 )(y12 + y22 + y32 ) ≥ (x1 y1 + x2 y2 + x3 y3 )2 ,

pentru orice xi , yi ∈ R, i = 1, 3 punând


√ √
x y
x1 = , x2 = ,
x + 2y + 2z 2x + y + 2z
√ √
z
x3 = , y1 = x(x + 2y + 2z),
2x + 2y + z
√ √
y2 = y(2x + y + 2z), y3 = z(2x + 2y + z),
obţinem:
( )
x y z
+ + [x(x + 2y + 2z)
x + 2y + 2z 2x + y + 2z 2x + 2y + z

+y(2x + y + 2z) + z(2x + 2y + z)] ≥ (x + y + z)2 ⇔


x y z
+ +
x + 2y + 2z 2x + y + 2z 2x + 2y + z
(x + y + z)2

(x + y + z)2 + 2(xy + yz + zx)
şi astfel rămâne a arăta că:
(x + y + z)2 3
≥ ⇔
(x + y + z)2 + 2(xyy z + zx) 5

5(x + y + z)2 ≥ 3(x + y + z)2 + 6(xy + yz + zx) ⇔


x2 + y 2 + z 2 ≥ xy + yz + zx ⇔ (x − y)2 + (y − z)2 + (z − x)2 ≥ 0.

2.25. Pentru a = 1 inegalitatea este evidentă. Fie a > 1. Notăm

ax + y + z = A(a + 2), x + ay + z = B(a + 2), x + y + az = C(a + 2).


A(a + 1) − (B + C)
Rezultă x + y + z = A + B + C şi x = (căci a > 1).
a−1
Inegalitatea devine:
A(a + 1) − (B + C)
∑ a−1 3

A(a + 2) a+2

68
şi se mai scrie succesiv:
∑ A(a + 1) − (B + C)
≤ 3(a − 1) ⇔
A
∑B+C ∑B+C
3(a + 1) − ≤ 3(a − 1) ⇔ ≥ 6,
A A
ceea ce este evident căci
∑B+C ∑(A B
) ∑
= + ≥ 2 = 6.
A B A
Egalitatea are loc dacă şi numai dacă x = y = z.
x
2.26. Funcţia f (x) = cu 0 < x < s fiind convexă, vom avea:
(s − x)(2s − x)
( 3 )

9 xi
∑ 3
xi i=1
≥( )( ).
(s − xi )(2s − xi ) ∑3 ∑
3
i=1
3s − xi 6s − xi
i=1 i=1

Luând x1 = x, x2 = y, x3 = z şi 3s = x + y + z, rezultă enunţul.


Egalitatea are loc dacă şi numai dacă x = y = z.
2.27. Eliminând numitorii, inegalitatea se scrie:
∑ ∏
x(yz + y + 1)(zx + z + 1) ≤ (xy + x + 1).

Dar
∑ ∑
x(yz+y+1)(zx+z+1) = x(z 2 xy+yz 2 +yz+xyz+yz+y+zx+z+1)
(∑ ) (∑ ) ∑ ∑ ∑
= xyz xy + 2xyz x + 6xyz + 2 xy + x2 z + x.
Prin calcul direct se obţine:
∏ (∑ ) (∑ ) ∑
(xy + x + 1) = x2 y 2 z 2 + xyz xy + 2xyz x + x2 z
∑ ∑
+4xyz + 2 xy + x + 1.
Astfel inegalitatea de verificat se reduce la

0 ≤ x2 y 2 z 2 − 2xyz + 1 = (xyz − 1)2 ,

ceea ce este evident. Egalitatea are loc dacă şi numai dacă xyz = 1.

69
2.28. Demonstrăm mai ı̂ntâi inegalitatea:

x2 2x − y
2 2
≥ , pentru orice x, y > 0.
y(x + xy + y ) 3xy

Eliminând numitorii ea se scrie sub formele echivalente:

3x3 ≥ (2x − y)(x2 + xy + y 2 ) ⇔ x3 ≥ x2 y + xy 2 − y 3 ⇔

(x + y)(x2 − xy + y 2 ) ≥ xy(x + y) ⇔ x2 − xy + y 2 ≥ xy ⇔ (x − y)2 ≥ 0.


Vom avea:
∑ ∑ ∑
∑ x2 ∑ 2x − y 2 xz − yz xy
≥ = = .
y(x2 + xy + y 2 ) 3xy 3xyz 3xyz
(∑ ) ( ∑ )
Cum x xy ≥ 9xyz (din inegalitatea mediilor), inegalitatea
din enunţ rezultă. Egalitatea se obţine numai pentru x = y = z.

2.29. Putem presupune că x ≤ y ≤ z. Fie y = x + p, z = y + q cu p, q ≥ 0.


Inegalitatea se reduce succesiv la următoarele:

p2 (x − q) + q 2 (z + p) + (p + q)2 (x + q) ≥ 0 ⇔

[p2 + (p + q)2 ]x + q 2 (z + p) + (2pq + q 2 )q ≥ 0.


Egalitatea are loc dacă şi numai dacă x = y = z.

2.30. Fără a micşora generalitatea putem presupune că a ≥ b ≥ c. În acest


caz avem c(a − c)(b − c) ≥ 0, de unde c3 + abc ≥ ac2 + bc2 . E suficient
să mai demonstrăm că:

a3 + b3 + 2abc ≥ ab(a + b) + a2 c + b2 c ⇔ (a − b)2 (a + b − c) ≥ 0,

ceea ce este evident. Egalitatea are loc pentru a = b = c.


Observaţie. Adăugând 2abc ı̂n ambii membri ai inegalităţii demonstrate,
ea se mai poate scrie sub forma:
∑ ∏
a3 + 5abc ≥ (a + b).

2.31. Din observaţia la inegalitatea precedentă avem:


∑ ∏
a3 + abc ≥ (a + b) − 4abc.

70
Rămâne de demonstrat că
∏ 1∏ ∏
(a + b) − 4abc ≥ (a + b) ⇔ (a + b) ≥ 8abc,
2
ceea ce rezultă din inegalitatea mediilor:
√ √ √
(a + b)(b + c)(c + a) ≥ 2 ab · 2 bc · 2 ca = 8abc.

Egalitatea are loc dacă şi numai dacă a = b = c.

2.32. Fie k, l ≥ 0. Inegalitatea evidentă:

(ka + lb)(a − b)2 + (kb + lc)(b − c)2 + (kc + la)(c − a)2 ≥ 0

este echivalentă cu:


(∑ ) (∑ ) (∑ )
(k + l) a3 + (l − 2k) a2 b + (k − 2l) ab2 ≥ 0.

k ∑ ∑
Luând l = rezultă a3 ≥ a2 b. Asemănător pornind de la
2
echivalenţa:

(lb2 + ka2 + mab)(a − b)2 + (kb2 + lc2 + mbc)(b − c)2

+(kc2 + la2 + mca)(c − a)2 ≥ 0 ⇔


(∑ ) (∑ ) (∑ )
(k + 1) a4 + (m − 2k) a3 b + (m − 2l) ab3
(∑ )
+(k + l − 2m) a2 b2 ≥ 0

k 2k ∑ ∑
cu m, l, k ≥ 0 şi luând l = ,m= , rezultă că a4 ≥ a3 b.
3 3
2.33. Putem presupune că a + b + c = 1
 a 
∏ ( 4a ) ∏

4
a+b+c 
(deoarece +1 =  b c
+ 1).
b+c
+
a+b+c a+b+c
Inegalitatea se scrie:
∏ 3a + 1 ∏ ∏
> 25 ⇔ (3a + 1) > 25 (1 − a) ⇔
1−a
∑ ( ∑ ) ∑
27abc+9 ab+3+1 > 25 1 + ab − 1 − abc ⇔ 4 ab−13abc < 1.

71
4
Cazul I: c ≤ (se poate alege din cauza simetriei). Vom demonstra că:
13
∑ ( ) ( )
a+b 2 a+b 2
4 ab − 13abc ≤ 4 + 4c(a + b) − 13 c.
2 2
Avem:
∑ ( )2 ( )2
a+b a+b
4 ab − 13abc ≤ 4 + 4c(a + b) − 13 c ⇔
2 2

16ab − 52abc ≤ 4a2 + 4b2 + 8ab − 13a2 c − 13b2 c − 26abc ⇔


4
4(a − b)2 − 13c(a − b)2 ≥ 0 ⇔ c ≤ ,
13
a+b 1−c 1
deci ea are loc. Notând =u ⇔ = u ≤ . Rezultă că:
2 2 2
4ab + 4c(a + b) − 13abc ≤ 4u2 + 8uc − 13u2 c < 1,

căci ultima inegalitate prin ı̂nlocuirea lui c = 1 − 2u se reduce la

(2u − 1)(13u2 − 6u + 1) < 0.

4 1 ∑
Cazul II: Dacă a, b, c > . Fie a < (din cauza faptului că a = 1 şi
13 3
simetriei). Avem b = 1 − a − c şi după unele calcule inegalitatea revine
la:
c2 (13a − 4) + c(13a2 − 17a + 4) − (2a − 1)2 < 0 ⇔
1−a
c2 (13a − 4) + c(a − 1)(13a − 4) − (2a − 1)2 < 0, unde < 1.
2
Notând f (c) = c2 (13a − 4) + c(a − 1)(13a − 4) − (2a − 1)2 , avem f (1) =
1
9a2 − 1 < 0 (din a < ), f (0) = −(2a − 1)2 < 0, deci f (c) < 0, pentru
3
orice c ∈ (0, 1) ⊂ (c1 , c2 ) unde c1 , c2 sunt rădăcinile ecuaţiei f (c) = 0.

2.34. Fără a restânge generalitatea problemei presupunem că a ≥ b ≥ c.


1 1 1
Rezultă că a2 ≥ b2 ≥ c2 şi ≥ ≥ , deci tripletele
( b +)c c+a a+b
1 1 1
(a2 , b2 , c2 ) şi , , sunt la fel ordonate. Aplicând ine-
b+c c+a a+b
galitatea lui Cebâşev, avem:
( 2 ) ( )
a b2 c2 1 1 1
3 + + ≥ (a + b + c )
2 2 2
+ + .
b+c c+a a+b b+c c+a a+b

72
Dar
1 1 1 1 9
a2 + b2 + c2 ≥ (a + b + c)2 şi + + ≥ ,
3 b+c c+a a+b 2(a + b + c)

de unde deducem că:


( )
1 1 1 3
2 2
(a + b + c ) 2
+ + ≥ (a + b + c).
b+c c+a a+b 2

Egalitatea are loc dacă şi numai dacă a = b = c.

2.35. I-a soluţie. Adunând a + b + c ı̂n ambii membri şi grupând convenabil,
inegalitatea din enunţ se transcrie:
∑ (a + b)(a + c) (∑ )
≥2 a .
b+c
Cu substituţiile
∑ a + b = x, b + c = y, c + a = z inegalitatea devine
xy ∑
≥ x, adică problema 1.15.
z
A II-a soluţie. Vom demonstra mai ı̂ntâi inegalitatea

a2 + bc b2 + ac
+ ≥ a + b.
b+c a+c

Într-adevăr, avem:

a2 + bc b2 + ac
−b+ −a≥0 ⇔
b+c a+c

a2 + bc − b2 − bc b2 + ac − a2 − ac
+ ≥0 ⇔
b+c a+c
( )
a2 − b2 a2 − b2 1 1
− ≥ 0 ⇔ (a − b )
2 2
− ≥0 ⇔
b+c a+c b+c a+c
a+c−b−c a+b
(a2 − b2 ) ≥ 0 ⇔ (a − b)2 ≥ 0.
(b + c)(a + c) (b + c)(a + c)
Adunând inegalitatea demonstrată cu ı̂ncă două inegalităţi analoage se
obţine enunţul.
A III-a soluţie. În inegalitatea evidentă
∑ ∑
(xy)2 ≥ (xy)(yz) = xyz(x + y + z),

73
luând x = a + b, y = b + c, z = c + a, obţinem:

(a + b)2 (a + c)2 ≥ (a + b)(b + c)(c + a)(a + b + c) · 2 ⇔

∑ (a + b)(a + c) (∑ ) ∑ a2 + ab + ac + bc (∑ )
≥2 a ⇔ ≥2 a ⇔
b+c b+c
∑ a2 + bc ∑ (∑ ) ∑ a2 + bc ∑
+ a≥2 a ⇔ ≥ a.
b+c b+c
Semnul egal are loc dacă şi numai dacă a = b = c.
∑ a3 − b3 ∑
2.36. Observăm că 2 2
= (a − b) = 0 şi deci membrul stâng al
a + ab + b
1 ∑ a3 + b3
inegalităţii din enunţ se poate scrie sub forma . Dar
2 a2 + ab + b2
a3 + b3 = (a + b)(a2 − ab + b2 ) şi

a2 − ab + b2 1 2(a2 − 2ab + b2 ) 2(a − b)2


− = = ≥ 0.
a2 + ab + b2 3 a2 + ab + b2 a2 + ab + b2
Prin urmare
a3 + b3 a+b
2 2
≥ .
a + ab + b 3
Adunând cu alte două inegalităţi analoage, se obţine:

∑ a3 1 ∑ a+b a
≥ = .
a2 + ab + b2 2 3 3
Egalitatea se obţine dacă şi numai dacă a = b = c.

2.37. Avem a3 + b3 = (a + b)(a2 − ab + b2 ) ≥ ab(a + b) de unde rezultă că:

a3 + b3 + abc ≥ ab(a + b) + abc = ab(a + b + c),

şi deci
1 c
≤ .
a3 + b3 + abc abc(a + b + c)
Sumând obţinem:

∑ 1 ∑ c a 1
3 3
≤ = (∑ ) = .
a + b + abc abc(a + b + c) abc a abc

Egalitatea are loc dacă şi numai dacă a = b = c.

74
2.38. Avem:

∑ a3 ∑ a2 + b2 ∑ a4 ab(a2 + b2 )
≥ ⇔ ≥ ⇔
bc 2c abc 2abc
(∑ ) ∑
2 a4 ≥ ab(a2 + b2 ),

ceea ce rezultă din a4 +b4 ≥ ab(a2 +b2 ) prin sumarea ciclică. Inegalitatea
folosită se demonstrează astfel:

a4 + b4 ≥ a3 b + ab3 ⇔ a4 − a3 b + b4 − ab3 ≥ 0 ⇔

a3 (a − b) − b3 (a − b) = (a − b)(a3 − b3 ) = (a − b)2 (a2 + ab + b2 ) ≥ 0.


A doua inegalitate din enunţ se mai scrie:
( 2 ) ( 2 ) ( 2 )
a b2 b c2 c a2
+ + + + + ≥ a + b + c.
2b 2a 2c 2b 2a 2c
Se verifică mai ı̂ntâi inegalitatea:
a2 b2 a+b
+ ≥ .
2b 2a 2
Avem succesiv:
a2 b2 a+b
+ ≥ ⇔ a3 + b3 ≥ ab(a + b) ⇔
2b 2a 2
(a + b)(a2 − ab + b2 ) ≥ ab(a + b) ⇔ a2 − ab + b2 ≥ ab ⇔ (a − b)2 ≥ 0.
Adunând inegalitatea demonstrată cu ı̂ncă două analoage se obţine
enunţul.

2.39. Datorită omogenităţii putem∑considera că a + b + c = 1 (ı̂mpărţim ı̂n


2 a
inegalitate cu (a + b + c) şi = 1). Inegalitatea se scrie:
a+b+c
∑ (1 − 2a)3 ∑
≥ a2 .
a
Dar
∑1 (∑ ) [ ∑ ]
1 1
(1 − 2a) ≥
3
(1 − 2a)3
a 3 a
din inegalitatea lui Cebâşev. Deoarece are loc identitatea
∑ (∑ )3
x3 = x − 3(x + y)(y + z)(z + x),

75
vom obţine că:
∑ (1 − 2a)3 (∑ )
1 1
≥ (1 − 24abc)
a 3 a
( )
1 ∑ 1 (∑ ) 24 (∑ ) 1 (∑ )
= a − ab ≥ · 9 − 8 ab
3 a 3 3
(∑ ) (∑ 1 )
căci a ≥ 9 din inegalitatea mediilor. Enunţul va rezulta
a
dacă arătăm că
(∑ ) ∑ (∑ ) (∑ )2
3−8 ab ≥ a ⇔ 3−6
2
ab ≥ a ⇔

(∑ )2
(∑ ) ∑ 1 a
3−6 ab ≥ 1 ⇔ ab ≤ = ⇔
3 3
∑ ∑ ∑ ∑
3 ab ≤ 2 ab + a2 ⇔ (a − b)2 ≥ 0,
ceea ce este evident.

2.40. Fie
∑ a3
P = ,
b2 − bc + c2
∑ b3 + c3 ∑ (b + c)(b2 − bc + c2 ) ∑ (∑ )
Q= = = (b+c) = 2 a .
b2 − bc + c2 b2 − bc + c2
Obţinem
(∑ ) (∑ 1
)
3
P +Q= a
b2 − bc + c2
1 (∑ ) (∑ 1
)
1 (∑ √ )2
= (b + c)(b2 − bc + c2 ) ≥ b + c ,
2 b2 − bc + c2 2
conform inegalităţii lui Cauchy-Buniakovski. Rezultă că:
1 (∑ √ )2 (∑ ) ∑ √ √ (∑ )
P ≥ b+c −2 a = a+b· b+c− a .
2
Să notăm

√ √ ab
Pa = c+a· b + a − a = √( ∑ ) .
2
ab + a + a

76
Încă două expresii analoage le notăm prin Pb , Pc . Avem P ≥ Pa +Pb +Pc .
Însă (∑ )2 (∑ )
a ≥3 ab ,
deci:
v
∑ ) u (∑ )2
(∑ 
ab t u
ab 3 a 
Pa ≥ v ( = (∑ )2  + a2 − a

u ∑ )2 3
u a
t a
+ a2 + a
3
şi ı̂ncă două inegalităţi analoage pentru Pb şi Pc . Inegalitatea din enunţ
va rezulta dacă mai arătăm că:
(√ )
∑ (a + b + c)2
+ a2 − a ≥ a + b + c,
3
sau √ ( )2
∑ 1 a
+ ≥ 2.
3 a+b+c

1
Considerând funcţia f (x) = + x2 , se constată că este convexă, deci:
3
( )
f (x1 ) + f (x2 ) + f (x3 ) x1 + x2 + x3
≥f .
3 3
a b c
Alegând x1 = , x2 = , x3 = , obţinem:
a+b+c a+b+c a+b+c
√ ( )2 ( )
∑ 1 a 1
+ = f (x1 ) + f (x2 ) + f (x3 ) ≥ 3f = 2.
3 a+b+c 3
Egalitatea are loc dacă şi numai dacă a = b = c.

2.41. Vom demonstra că pentru orice a, b, c ≥ 0, avem:



a − b b − c c − a (a − b)(b − c)(c − a)

a + b + b + c + c + a = (a + b)(b + c)(c + a) .

Ţinând cont că c − a = (c − b) + (b − a), rezultă că:


( ) ( )
a−b b−c c−a 1 1 1 1
+ + = (a−b) − +(b−c) −
a+b b+c c+a a+b c+a b+c c+a

77
( )
(a − b)(b − c) 1 1 (a − b)(b − c)(c − a)
= − = .
c+a b+c a+b (a + b)(b + c)(c + a)
Deoarece |a − b| < a + b, |b − c| < b + c şi |c − a| < c + a, inegalitatea
din enunţ e complet demonstrată.
a+1 √ 1 2
2.42. Dacă a > 0, atunci ≥ a şi deci √ ≥ . Rezultă că
2 a a+1
√ a 2a
a= √ ≥ ,
a a+1
egalitatea având loc dacă şi numai dacă a = 1. Punând pe rând ı̂n locul
x y z
lui a pe , şi , rezultă enunţul.
y+z z+x x+y
x
∑√ x ∑ 2y + z ∑ 2x
≥ x = = 2.
y+z +1 x+y+z
y+z
Inegalitatea e strictă căci relaţiile x = y + z, y = z + x, z = x + y nu pot
avea loc simultan căci x + y + z ≥ 0.

2.43. Avem (x2 + y 2 )(x2 + z 2 ) ≥ 2xy · 2xz = 4x2 yz, de unde rezultă că
x 1
√ ≤ √ .
(x2 + y 2 )(x2 + z2) 2 yz

Deci:
∑ √
∑ x 1∑ 1 1 x yz
√ ≤ √ = · .
(x2 + y 2 )(x2 + z 2 ) 2 yz 2 xyz

Dar ∑ √ ∑√ 1∑ ∑
x yz = xy · xz ≤ (xy + xz) ≤ x2 .
2
Egalitatea are loc dacă şi numai dacă x = y = z.

2.44. Inegalitatea rezultă din inegalitatea mediilor:


√ √ √ √ √ √ √
x y 3
z x 1 y 1 y 13 z 13 z 13 z
+ + = + + + + +
y z x y 2 z 2 z 3 x 3 x 3 x
√ √ √
x 1 y 1 z 1 6 2 · 3
6 6 √
6 3
≥66 · · · · =6 24 · 33 > .
6
= =
y 4 z 27 x 2 ·3
2 2 2 ·3
2 3 2

78
2.45. Avem √ √ √
(a + b)(b + c)(c + a) ≥ 2 ab · 2 bc · 2 ca = 8abc
şi deci √
(a + b)(b + c)(c + a)
3

≥ 1 + 8 = 3,
3
1+
abc
de unde rezultă a doua inegalitate din enunţ. Prima inegalitate rezultă
tot din inegalitatea mediilor:
( )
1 1 1 a+b b+c c+a
(a + b + c) + + =3+ + +
a b c c a b

3 a + b b+c c+a
≥3+3 · · .
c a b
În ambele inegalităţi se atinge semnul egal dacă şi numai dacă a = b = c.
2.46. Fără a restrânge generalitatea problemei presupunem că x ≥ y ≥ z.
√ √ √
Astfel avem succesiv: 2 xy ≥ xy ≥ z deci x + y + 2 xy ≥ x + y + z,
√ √ √ √ √
adică ( x+ y)2 ≥ x+y +z, de unde x+ y ≥ x + y + z. Înmulţind
√ √ √ √
această inegalitate cu z, obţinem: xz + yz ≥ xz + yz + z 2 . Dar
√ √
( 4 xy − 4 xz + yz + z 2 )2 ≥ 0 implică
√ √ √
xy + xy + yz + z 2 ≥ 2 4 xyz(x + y + z).

Adunând membru cu membru ultimele inegalităţi se obţine inegalitatea


din enunţ. Egalitatea se obţine dacă şi numai dacă x = y = z = 0.
2.47. Vom demonstra ı̂n prealabil că
xn y n
+ ≥ xn−1 + y n−1 , pentru orice x, y > 0 şi n ∈ N∗ .
y x
y
Eliminând numitorii şi notând t = > 0, inegalitatea devine succesiv:
x
xn+1 + xn+1 tn+1 ≥ xn+1 t + xn+1 tn ⇔ tn+1 + 1 ≥ tn + t ⇔

tn (t − 1) − (t − 1) ≥ 0 ⇔ (t − 1)(tn − 1) ≥ 0 ⇔
(t − 1)2 (tn−1 + tn−2 + . . . + t + 1) ≥ 0.
Semnul egal se obţine dacă şi numai dacă t = 1 ⇔ x = y. Această
inegalitate se ı̂nmulţeşte cu z şi scriind ı̂ncă două inegalităţi analoage şi
adunând membru cu membru cele trei inegalităţi, inegalitatea rezultă.
Egalitatea va avea loc dacă şi numai dacă x = y = z.

79

a √
2.48. Luând a1 = , b1 = a(3b + c), etc. ı̂n inegalitatea Cauchy-
3b + c
Buniakovski:

(a21 + a22 + a23 + a24 )(b21 + b22 + b23 + b24 ) ≥ (a1 b1 + a2 b2 + a3 b3 + a4 b4 )2 ,

obţinem: (∑ ) (∑
a ) (∑ )2
a(3b + c) ≥ a ,
3b + c
deci: (∑ )2
∑ a a
≥ .
3b + c 3(ab + bc + cd + da) + 2(ac + bd)
Rămâne de demonstrat că:

(a + b + c + d)2 ≥ 3(ab + bc + cd + da) + 2(ac + bd) ⇔

a2 + b2 + c2 + d2 ≥ ab + bc + cd + da = a(b + d) + c(b + d) = (a + c)(b + d).


Ultima inegalitate rezultă astfel:
(a + c)2 + (b + d)2
a2 + b2 + c2 + d2 ≥ ≥ (a + c)(b + d)
2
(am folosit: 2(x2 +y 2 ) ≥ (x+y)2 şi x2 +y 2 ≥ 2xy, pentru orice x, y ∈ R).
2.49. I-a soluţie. Avem (folosind inegalitatea mediilor):
( ) ( )
a c b d
+ + +
b+c d+a c+d a+b
a(d + a) + c(b + c) b(a + b) + d(c + a)
= +
(b + c)(d + a) (c + a)(a + b)
4(ad + a2 + bc + c2 ) 4(ab + b2 + cd + d2 )
≥ +
(a + b + c + d)2 (a + b + c + d)2
4(a2 + b2 + c2 + d2 + ab + bc + cd + da)
=
(a + b + c + d)2
(a + b + c + d)2 + (a − c)2 + (b − d)2
=2 ≥ 2.
(a + b + c + d)2
A II-a soluţie. Folosind inegalitatea Cauchy-Buniakovski, avem:
∑ (√ a )2 ∑ √ (∑
a
) (∑ )
· 2
( a(b + c)) = a(b + c)
b+c b+c

80
(∑ )
a
= (2ac + 2bd + ab + bc + cd + da)
b+c
(∑ √ )2 ( ∑ )
a √ 2
≥ · a(b + c) = a .
b+c
Dar a2 + c2 ≥ 2ac, b2 + d2 ≥ 2bd implică

(a + b + c + d)2 ≥ 4ac + 4bd + 2(ab + bc + cd + da).


∑ a
Ţinând cont de această inegalitate, rezultă că ≥ 2.
b+c
2.50. Se observă că la o permutare circulară a indicilor, atât membrul stâng
cât şi membrul drept rămân invarianţi. De aceea celui mai mare dintre
cele 5 numere i se poate atribui orice indice. Este comod să se considere
că x2 este cel mai mare dintre aceste numere: x2 ≥ xi , i = 1, 5. Trecând
ambii membri ai inegalităţii ı̂n stânga, se obţine:

x21 + x22 + x23 + x24 + x25 − 2x1 x2 − 2x2 x3 − 2x3 x4 − 2x5 x1

+2x1 x3 + 2x1 x4 + 2x2 x4 + 2x2 x5 + 2x3 x5


= (x1 − x2 + x3 )2 + (x4 − x5 )2 + 2x4 (x1 + x2 − x3 ) + 2x5 (x2 + x3 − x1 ).
Având ı̂n vedere că x2 ≥ x3 şi x2 ≥ x1 şi că toate numerele xi erau
pozitive, se vede că expresia obţinută este pozitivă.

2.51. Din (x1 − x2 )2 + (x2 − x3 )2 + (x3 − x4 )2 + (x4 − x5 )2 + (x5 − x1 )2 ≥ 0


rezultă că pentru orice x1 , x2 , x3 , x4 , x5 ∈ R, avem:

x21 + x22 + x23 + x24 + x25 ≥ x1 x2 + x2 x3 + x3 x4 + x4 x5 + x5 x1 .


( a )2 ( b )2 ( c )2 ( d )2 ( e )2
Înlocuind x1 , x2 , x3 , x4 , x5 respectiv cu , , , ,
b c d e a
se obţine:
( a )4 ( b )4 ( c )4 ( d )4 ( e )4
+ + + + ≥
b c d e a
( a )2 ( b )2 ( c )2 ( d )2 ( e )2
+ + + +
c d e a b
( a )2 ( c )2 ( e )2 ( ) ( )
b 2 d 2
= + + + + .
c e b d a

81
a c d
În aceeaşi inegalitate ı̂nlocuind x1 , . . . , x5 respectiv cu , , . . . , ,
c e a
rezultă că:
( a )2 ( c )2 ( e )2 ( b )2 ( d )2
+ + + +
c e b d a
a c c e e b b d d a a b c d e
≥ · + · + · + · + · = + + + + ,
c e e b b d d a a c e a b c d
ceea ce trebuia demonstrat.

2.52. Aplicând inegalitatea Cauchy-Buniakovski, avem:


(∑ )
a
[a(b + c) + b(c + d) + c(d + e) + d(e + f ) + e(f + a) + f (a + b)]
b+c

≥ (a + b + c + d + e + f )2 ,
sau
(∑ ) (∑ )2
a
[(a + d)(b + e) + (b + e)(f + c) + (f + c)(a + d)] ≥ a .
b+c

Cu notaţia a + d = p, b + c = q, f + c = r, obţinem
(∑ )
a
(pq + qr + rp) ≥ (p + q + r)2 .
b+c

Dar ∑ ∑ ∑
(p + q + r)2 = p2 + 2 pq ≥ 3 pq,
deci (∑ ) (∑ ) (∑ )
a
pq ≥ (p + q + r)2 ≥ 3 pq ,
b+c
∑ a
de unde rezultă că ≥ 3.
b+c
2.53. Din inegalitatea mediilor avem:

∑n
ak + ak+1 a1 + a2 a2 + a3 an + a1
≥nn · · ... · = n.
ak+1 + ak+2 a2 + a3 a3 + a4 a1 + a2
k=1

Apoi:

n
ak+1 ∑ ak+1 + ak+2 ∑
n
ak+2
n
= −
ak+1 + ak+2 ak+1 + ak+2 ak+1 + ak+2
k=1 k=1 k=1

82

n
ak+2
=n− .
ak+1 + ak+2
k=1
Prin urmare, avem:
( )

n
ak ∑ ak+1
n ∑
n
ak+2
≥n− =n− n−
ak+1 + ak+2 ak+1 + ak+2 ak+1 + ak+2
k=1 k=1 k=1


n
ak+2
= ,
ak+1 + ak+2
k=1
de unde rezultă enunţul.

2.55. Inegalitatea rezultă din inegalitatea Cauchy-Buniakovski şi inegalitatea


mediilor. Avem (notând an+1 = a1 şi an+2 = a2 ):
( n )2 ( n √ )2
∑ ∑√ ak
ak = ak (ak+1 + ak+2 ) ·
ak+1 + ak+2
k=1 k=1
( )( n )

n ∑ ak
≤ ak (ak+1 + ak+2 )
ak+1 + ak+2
k=1 k=1
( )
a1 an
= (a1 a2 + a1 a3 + . . . + an a1 + an a2 ) + ... +
a2 + a3 a1 + a2
( 2 )
a2 + a21 a2n + a22 ∑
n
a1 + a22 a21 + a23 ak
≤ + + ... + n +
2 2 2 2 ak+1 + ak+2
k=1
( )
a1 an
= 2(a21 + a22 + . . . + a2n ) + ... + ,
a2 + a3 a1 + a2
de unde rezultă inegalitatea din enunţ. Egalitatea are loc dacă şi numai
dacă a1 = a2 = . . . = an .

2.56. Avem:
x31 − x32 x32 − x33 x3n − x31
+ + . . . +
x21 + x1 x2 + x22 x22 + x2 x3 + x23 x2n + xn x1 + x21
= (x1 − x2 ) + (x2 − x3 ) + . . . + (xn − x1 ) = 0,
deci
x31 x32 x3n
+ + . . . + =
x21 + x1 x2 + x22 x22 + x2 x3 + x23 x2n + xn x1 + x21

83
x32 x33 x31
+ + . . . + .
x21 + x1 x2 + x22 x22 + x2 x3 + x23 x2n + xn x1 + x21
Rezultă că:
x31 x3n
+ . . . +
x21 + x1 x2 + x22 x2n + xn x1 + x21
( )
1 x31 + x32 x3n + x31
= + ... + 2
2 x21 + x1 x2 + x22 xn + xn x1 + x21
[ ]
1 (x1 + x2 )(x21 − x1 x2 + x22 ) (xn + x1 )(x2n − xn x1 + x21 )
= + ... +
2 x21 + x1 x2 + x22 x2n + xn x1 + x21

1 1 x1 + x2 + . . . + xn
≥ · [(x1 + x2 ) + (x2 + x3 ) + . . . + (xn + x1 )] =
2 3 3
căci pentru orice x, y ∈ R∗ avem

x2 − xy + y 2 1
2 2
≥ .
x + xy + y 3

Egalitatea are loc dacă şi numai dacă x1 = x2 = . . . = xn .

2.57. Demonstrăm mai ı̂ntâi inegalitatea


√ √ √
n
(b1 + a1 )(b2 + a2 ) . . . (bn + an ) ≥ n
b 1 b2 . . . b n + n
a1 a2 . . . an ,

pentru orice a1 , a2 , . . . , an > 0, b1 , b2 , . . . , bn > 0. Inegalitatea se mai


scrie √ √
b1 bn a1 an
1≥ n
... + n ...
b1 + a1 bn + an a1 + b1 an + bn
şi rezultă din inegalitatea mediilor:

a1 an b1 bn
n= + ... + + + ... +
a1 + b1 an + bn b1 + a1 bn + an
√ √
a1 an b1 bn
≥nn ... +nn ... .
a1 + b1 an + bn b1 + a1 bn + an
Luând b1 = b2 = . . . = bn = 1 şi ridicând la puterea n ı̂n ambii membri
rezultă că ı̂n inegalitatea din enunţ semnul egal are loc dacă şi numai
dacă a1 = a2 = . . . = an .

84
2.58. Folosind identitatea:

(n − 1)(ak1 + ak2 + . . . + akn ) = [(ak−1


1 − ak−1
2 )(a1 − a2 )

+(ak−1
1 − ak−1
3 )(a1 − a3 ) + . . . + (an−1 − an )(an−1 − an )]
k−1 k−1

+(ak−1
2 + . . . + ak−1 k−1
n )a1 + (a1 + ak−1
3 + . . . + ak−1
n )a2 + . . . +

+(ak−1
1 + . . . + ak−1
n−1 )an .

Deoarece expresiile din parantezele drepte sunt ı̂n permanenţă pozitive,


vom avea:

(n−1)(ak1 +ak2 +. . .+akn ) ≥ (ak−1 k−1 k−1 k−1


2 +. . .+an )a1 +. . .+(a1 +. . .+an−1 )an ,

de unde

(n − 1)(ak1 + ak2 + . . . + akn ) ≥ (ak−1


1 + . . . + ak−1
n )(a1 + . . . + an )

−(ak1 + ak2 + . . . + akn )

şi deci

n(ak1 + ak2 + . . . + akn ) ≥ (ak−1


1 + . . . + ak−1
n )(a1 + a2 + . . . + an ).

Dacă ı̂n această inegalitate ı̂nlocuim exponenţii cu valorile k − 1,


k − 2, . . . , p + 1 şi ı̂nmulţim membru cu membru inegalităţile obţinute,
rezultă:

nk−p (ak1 + ak2 + . . . + akn ) ≥ (ap1 + ap2 + . . . + apn )(a1 + a2 + . . . + an )k−p ,

de unde
( )k−p
ak1 + ak2 + . . . + akn a1 + a2 + . . . + an
≥ .
ap1 + ap2 + . . . + apn n

Semnul egal are loc dacă şi numai dacă a1 = a2 = . . . = an . În cazul
p = 0 se obţine cunoscuta inegalitate
( )k
ak1 + ak2 + . . . + akn a1 + a2 + . . . + an
≥ .
n n

85
2.59. Vom arăta că:
√ √
a1 + a2 + . . . + an √ ( ai − aj )2
− a1 a2 . . . an ≥
n
,
n n
pentru orice i, j = 1, n, i < j de unde va rezulta enunţul. Folosind
inegalitatea mediilor, avem:

a1 + a2 + . . . + an √ ai + aj − 2 ai aj
− n a1 a2 . . . an ≥ ⇔
n n
√ √
a1 + . . . + ai−1 + ai+1 + . . . + aj−1 + aj+1 + . . . + an + ai aj + ai aj
n

≥ a1 a2 . . . an
n

ceea ce rezultă din faptul că


√ √
n √ √
a1 . . . ai−1 ai+1 . . . aj−1 aj+1 . . . an ai aj ai aj = n a1 a2 . . . an .

a2i + a2j ai + aj
2.60. Din ≥ şi identitatea
ai + aj 2
( ) ( )
a21 a2n a22 a21
+ ... + − + ... +
a1 + a2 an + a1 a1 + a2 an + a1


n
a2k − a2k+1 ∑
n
= = (ak − ak+1 ) = 0,
ak + ak+1
k=1 k=1
rezultă că:

n
a2k 1 ∑ a2k + a2k+1
n
1∑
n
= ≥ (ak + ak+1 )
ak + ak+1 2 ak + ak+1 4
k=1 k=1 k=1

1 ∑ n
a1 + a2 + . . . + an
= ·2· ak = .
4 2
k=1
Egalitatea are loc dacă şi numai dacă a1 = a2 = . . . = an .

2.61. Din inegalitatea lui Minkowski deducem:


v v 2
u n u n

n
u∑ u∑
(ak + bk )2 ≤ t a2k + t b2k  .
k=1 k=1 k=1

86
Vom arăta că:
 n 
∑ ∑ n
v v 2 [ ] 2 2
u n u n ak bk 
u∑ 2 u∑ ∑n
 
t ak + t b2k  ≤ pk (ak + bk )  k=1
+ k=1 .
∑n ∑
n 
k=1 k=1 k=1  
pk ak p k bk
k=1 k=1


n ∑
n ∑
n
Deoarece pk (ak + bk ) = pk ak + pk bk , diferenţa dintre membrul
k=1 k=1 k=1
drept şi cel stâng se scrie:
   

n ∑
n
( ) p k bk  ( n ) pk ak 

n ∑
n ∑
n
  ∑  
a2k + b2k + a2k 
∑
k=1
n
+
 b 2 
k ∑
k=1
n


k=1 k=1 k=1   k=1  
pk ak p k bk
k=1 k=1
v( )( n )
u n

n ∑
n u ∑ ∑
− a2k − b2k − 2t a2k b2k
k=1 k=1 k=1 k=1
v
u
v
u∑ 2
u ∑n
u
n
u( n ) p k bk u pk ak ( n )
u ∑ u ∑ 
u k=1 u k=1 2 
= u a2k − u b  ≥ 0.
u ∑n u∑ n k

t k=1 p a t p b
k k
k=1  k k
k=1 k=1

Inegalitatea devine egalitate dacă există r, s ∈ R astfel ı̂ncât r·ak = s·bk ,


pentru orice k ∈ {1, 2, . . . , n}.

2.62. Avem (a + b)2 = a2 + b2 + 2ab = 1 + 2ab, de unde a + b = 1 + 2ab şi
astfel inegalitatea din enunţ devine
√ 1 √
1 + 2ab + ≥ 2 + 2.
ab
1
Notând ab = x > 0 avem 2x = 2ab ≤ a2 + b2 = 1, deci x ≤ . Vom arăta
( ] 2
1 √ 1
că funcţia f : 0, → R, f (x) = 1 + 2x + este descrescătoare,
2 x
adică x ≥ y ⇒ f (x) ≤ f (y). Dar
√ 1 √ 1 2(x − y) x−y
f (x)−f (y) = 1 + 2x+ − 1 + 2y − = √ √ −
x y 1 + 2x + 1 + 2y xy

87
[ ] [ √ √ ]
2 1 2xy − 1 + x − 1 + y
= (x−y) √ √ − = (x−y) √ √ <0
1 + x+ 1 + y xy xy( 1 + x + 1 + y)
1 1 1 √ √ 1
căci 2xy ≥ 2 · · = < 1 + x + 1 + y. Cum x ≤ rezultă
( ) 2 2 2 2
1 √
f (x) ≥ f = 2 + 2 şi astfel inegalitatea din enunţ rezultă.
2

2.63. I-a soluţie. Evident că x > y > 0. Din enunţ avem

x3 + y 3 x3 − y 3 x3 + y 3
= 1 şi < = 1.
x−y x−y x−y

Rezultă că x2 + xy + y 2 < 1 şi prin urmare x2 + y 2 < x2 + xy + y 2 < 1.


A II-a soluţie. Avem:

(x2 + y 2 − 1)(x − y) = x3 − y 3 − xy(x − y) − (x − y)

= x3 − y 3 − xy(x3 + y 3 ) − (x3 + y 3 )

= −2y 3 − xy(x3 + y 3 ) = −y[2y 2 + x(x3 + y 3 )] < 0,

deoarece x, y > 0. Cum x − y = x3 + y 3 > 0, rezultă că x2 + y 2 < 0,


adică x2 + y 2 < 1.

2.64. Avem
∏( 1
) ∑1 ∑ 1 1
1+ =1+ + +
x x xy xyz

∑1 x 1 ∑1 2
=1+ + + =1+ + .
x xyz xyz x xyz
∑1 32
Dar ≥ ∑ = 9 (din inegalitatea mediilor). Problema revine la a
x x
verifica dacă
2 2
1+9+ ≥ 64 ⇔ ≥ 54 ⇔ 1 ≥ 27xyz
xyz xyz

ceea ce rezultă din 1 = (x + y + z)3 ≥ 27xyz. Egalitatea are loc dacă şi
1
numai dacă x = y = z = .
3

88
∑ ∑
2.65. Din x + y + z = 1 rezultă x2 = 1 − 2 xy. Inegalitatea devine

xy ∑ 1 11
≥ 11 − 6 xy ⇔ +6≥ ∑ .
xyz xyz xy

∑ √ 1 1
Dar xy ≥ 3 3 x2 y 2 z 2 , deci ∑ ≥ √
3
. Este suficient să
xy 3 x2 y 2 z 2
demonstrăm că:
1 11
+6≥ √ .
xyz 3
3 x2 y 2 z 2
√ 1
Din 1 = x + y + z ≥ 3 3 xyz rezultă că a = √
3 xyz
≥ 3. Va trebui să
verificăm inegalitatea:

3a3 − 11a2 + 18 ≥ 0 ⇔ (a − 3)[3(a − 3)a + 7a − 6 ≥ 0

ceea ce este adevărat pentru a ≥ 3.

2.66. I-a soluţie. Inegalitatea abc ≥ (−a + b + c)(a − b + c)(a + b − c) adevărată


pentru orice a, b, c > 0 devine, ı̂n ipotezele din enunţ:

abc ≥ (2 − 2a)(2 − 2b)(2 − 2c) ⇔

abc ≥ 8 − 8(a + b + c) + 8(ab + bc + ca) − 8abc ⇔


16 − 8(ab + bc + ca) ≥ 8 − 9abc.
Dar 16 = 4(a + b + c)2 şi ı̂nlocuind avem:

4(a2 + b2 + c2 ) + 8(ab + bc + ca) − 8(ab + bc + ca) ≥ 8 − 9abc ⇔

4(a2 + b2 + c2 ) ≥ 8 − 9abc.

A II-a soluţie. Dacă a, b, c ∈ (0, 1) şi a + b + c = 2 deducem că:

a+b+c
a<1 ⇔ a< ⇔ a < b + c.
2
Analog se deduce că b < c + a şi c < a + b, deci se poate construi un
triunghi cu laturile de lungimi a, b, c. Din inegalitatea

(cos A + cos B − 1)2 + (sin A − sin B)2 ≥ 0

89
care este echivalentă cu
3
cos A + cos B + cos C ≤ ,
2
obţinem:
∑ b2 + c2 − a2 3 ∑ (b + c)2 − a2 9
≤ ⇔ ≤ ⇔
2bc 2 2bc 2
∑ 9abc ∑ 9abc
a(b + c − a) ≤ ⇔ a(2 − 2a) ≤ ⇔
2 2
4(a2 + b2 + c2 ) ≥ 8 − 9abc.
2
Egalitatea are loc dacă şi numai dacă a = b = c = .
3
2.67. Ţinând cont de identitatea

a3 + b3 + c3 = (a + b + c)3 − 3(a + b)(b + c)(c + a)

şi de relaţia din enunţ, avem succesiv:

|a3 + b3 + c3 − 1| = |(a + b + c)3 − 3(a + b)(b + c)(c + a) − 1|

= 3|(1 − a)(1 − b)(1 − c)|.


De asemenea:
|a5 + b5 + c5 − 1|

= |(a + b + c)5 − 5(a + b)(b + c)(c + a)(a2 + b2 + c2 + ab + bc + ca) − 1|


1
= 5|(1 − a)(1 − b)(1 − c)| · [(a + b + c)2 + a2 + b2 + c2 ].
2
Dar 3(a2 + b2 + c2 ) ≥ (a + b + c)2 şi deci:

1
5|(1 − a)(1 − b)(1 − c)| · [(a + b + c)2 + a2 + b2 + c2 ]
2
[ ]
5 (a + b + c)2
≥ |(1 − a)(1 − b)(1 − c)| (a + b + c) +2
2 3
10
= |(1 − a)(1 − b)(1 − c)|.
3

90
2.68. Avem:
a5 + b5 = (a + b)(a4 − a3 b + a2 b2 − ab3 + b4 )
= (a + b)[a4 + b4 + a2 b2 − ab(a2 + b2 )]
≥ (a + b)[2a2 b2 + a2 b2 − ab(a2 + b2 )] = ab(a + b)[3ab − (a2 + b2 )],
deci a5 + b5 ≥ ab(a + b)(3ab + c2 − 1). Prin urmare:
∑ a5 + b5 ∑ ∑ (∑ )
≥3 ab + a2 − 3 = 3 ab − 2.
ab(a + b)

2.69. Avem ∑ ∑
1
= 2 ⇔ 2a2 2 2
b c + a2 b2 = 1
1 + a2
şi
5 1∑ 2 2 5
abc(a + b + c) − a2 b2 c2 ≤ ⇔ abc(a + b + c) + a b −1 ≤ ⇔
8 2 8
9 3
(ab + bc + ca)2 ≤ ⇔ |ab + bc + ca| ≤ .
4 2
a2 b 2 c 2
Fie 2
= x2 , 2
= y2, = z 2 cu x, y, z ≥ 0 rezultă
1+a 1+b 1 + c2
x y z
|a| = √ , |b| = √ , |c| = √
y2 + z2 x2+ z2 x2 + y 2

şi deoarece |ab + bc + ca| ≤ |a||b| + |b||c| + |c||a| e suficient a demonstra


că: ∑
3 xy 3
|ab| + |bc| + |ca| ≤ ⇔ √ √ ≤ .
2 z 2 + x2 · z 2 + y 2 2
Dar, din inegalitatea mediilor, avem:
( )
x y 1 x2 y2
√ ·√ ≤ +
2
z +x2 z2 + y2 2 z 2 + x2 z 2 + x2

şi ı̂ncă două inegalităţi analoage care adunate demonstrează ultima ine-
galitate.

2.70. Metoda de demonstraţie pentru ambele inegalităţi fiind aceeaşi, vom da


soluţia primei inegalităţi din enunţ. Din ipoteză deducem că c = 1 − a
şi d = 1 − b; prin urmare:

a2 b + c2 d − bd = a2 b + (1 − a)2 (1 − b) − b(1 − b)

91
= a2 b + (1 − a)2 − b(1 − 2a + a2 ) − b + b2
= (1 − a)2 + b2 − 2b(1 − a) = (1 − a − b)2 ≥ 0,
ceea ce trebuia demonstrat.
2.71. Din enunţ rezultă că 8 − e = a + b + c + d şi 16 − e2 = a2 + b2 + c2 + d2 .
Dar inegalitatea Cauchy-Buniakovki implică
4(a2 + b2 + c2 + d2 ) ≥ (a + b + c + d)2 ⇔ 4(16 − e2 ) ≥ (8 − e)2 ⇔
64 − 4e2 ≥ 64 − 16e + e2 ⇔ 5e2 − 16e ≤ 0 ⇔ e(5e − 16) ≤ 0.
16
Din ultima inegalitate deducem că 0 ≤ e ≤ , deci maximul lui e este
5
16 16
. Egalitatea se obţine dacă şi numai dacă a = b = c = d = .
5 5
2.72. I-a soluţie. Inegalitatea este echivalentă cu
xy 2 + yz 2 + zx2 ≥ x + y + z.
Folosind inegalitatea mediilor avem:
3(xy 2 + yz 2 + zx2 ) = (2xy 2 + yz 2 ) + (2yz 2 + zx2 ) + (2zx2 + xy 2 )
√ √ √
≥ 3( 3 x2 y 5 z 2 + 3 x5 y 2 z 2 + 3 x2 y 2 z 5 ) = 3(x + y + z),
ceea ce trebuia demonstrat.
A II-a soluţie. Demonstrăm mai ı̂ntâi inegalitatea:
x3 + y 3 + z 3 ≥ x2 y + y 2 z + z 2 x.
Avem:
3(x3 + y 3 + z 3 ) = (x3 + x3 + y 3 ) + (y 3 + y 3 + z 3 ) + (z 3 + z 3 + x3 )
≥ 3(x2 y + y 2 z + z 2 x)
din inegalitatea mediilor. Prin ı̂mpărţirea cu xyz, deducem:
x2 y2 z2 x y z
+ + ≥ + +
yz xz xy z x y
x z y
şi cu substituţiile = a, = b, = c, obţinem:
z y x
a c b
+ + ≥ a + b + c.
c b a
Punând a = x, b = z, c = y deducem inegalitatea din enunţ.

92
c a b
2.73. Fie x = , y = . Atunci z = şi inegalitatea din enunţ devine:
b c a
c a b
b + c + a ≥3 ⇔ c
+
a
+
b 3
≥ .
a b c 2 a + b b + c c + a 2
+1 +1 +1
b c a

Ultima inegalitate a fost demonstrată la 1.52. Cazul de egalitate are loc


dacă şi numai dacă a = b = c = 1.

2.74. Inegalitatea din enunţ se mai scrie:


  n−1 ( )n−2
( )n−1 ∑ ∑
n
2 x1 x2 . . . xn  1  1
≥ n x1 . . . xn
n−1 xi xj xi
1≤i<j≤n i=1

( )n−1 (∑ )n−1 (∑ )n−2


2
⇔ x1 x2 . . . xn−2 ≥n x1 x2 . . . xn−1 ⇔
n−1
( )n−1 (∑ )n−1 (∑ )n−2
2
n n−2
x1 x2 . . . xn−2 ≥ nn−1 x1 x2 . . . xn−1
n−1
( ) n−1
(∑ ) n−1
n−2 2 (n−1)(n−2)
(n−1)(n−2)
⇔ n (n−1)(n−2) x1 x2 . . . xn−2
n−1

n−1 (∑ ) n−2
(n−1)(n−2)
≤ n (n−1)(n−2) x1 x2 . . . xn−1 ⇔

( ) 1
(∑ ) 1 (∑ ) 1
1 2 n−2
n−2 1 n−1
n n−1 x1 x2 . . . xn−2 ≥ n n−2 x1 x2 . . . xn−1
n−1
∑ ∑  1
 1
n−2
n−1
 x x
1 2 . . . xn−2  x x
1 2 . . . xn−1
⇔   ≥   ⇔
n(n − 1) n
2
∑  1 ∑  1
n−2 n−1
x1 x2 . . . xn−2 x1 x2 . . . xn−1
  ≥  
Cnn−2 Cnn−1

ceea ce reprezintă inegalitatea lui MacLaurin.

93
2.75. Prima inegalitate este o simplă consecinţă a inegalităţii mediilor. Pentru
a demonstra a doua inegalitate presupunem (fără a restrânge generali-
tatea problemei) că x1 ≤ x2 ≤ x3 . Arătăm că:
( ) ( )
1 1 1 1 1 1
(x1 + x2 + x3 ) + + ≤ (a + x2 + x3 ) + + .
x1 x2 x3 a x2 x3

Inegalitatea de mai sus este echivalentă succesiv cu următoarele:


( ) ( )
1 1 1 1 1
1 + x1 + + (x2 + x3 ) + (x2 + x3 ) +
x2 x3 x1 x2 x3
( ) ( )
1 1 1 1 1
≤1+a + + (x2 + x3 ) + (x2 + x3 ) + ⇔
x2 x3 a x2 x3

x1 (x2 + x3 ) x2 + x3 a(x2 + x3 ) x2 + x3
+ ≤ + ⇔
x2 x3 x1 x2 x3 a
x1 1 a 1
+ ≤ + ⇔ (x1 − a)(x1 a − x2 x3 ) ≤ 0,
x2 x3 x1 x2 x3 a
inegalitate adevărată deoarece 0 < a ≤ x1 ≤ x2 ≤ x3 ≤ b. În continuare
vom demonstra că:
( ) ( )
1 1 1 1 1 1
(a + x2 + x3 ) + + ≤ (a + x2 + b) + + .
a x2 x3 a x2 b

Efectuând calculele ca mai sus ea se reduce la

(x3 − b)(x3 b − ax2 ) ≤ 0,

care are loc. Analog se verifică şi inegalitatea:


( ) ( )
1 1 1 1 1 1
(a + x2 + b) + + ≤ (a + b + a) + + ⇔
a x2 b a b a

(x2 − a)(x2 − b) ≤ 0.

2.76. Fie f : [0, 1] → R, f (x) = x+y+z−x(y+z)−yz = x(1−y−z)+y+z−yz.


Fiind funcţie de gradul ı̂ntâi, f este monotonă şi deci maximul ei se
atinge ı̂n 0 sau 1. Dar f (0) = y + z − yz = 1 − (1 − y)(1 − z) ≤ 1 şi
f (1) = 1 − yz ≤ 1. Prin urmare f (x) ≤ 1, pentru orice x ∈ [0, 1].

94
2.77. Fără a micşora generalitatea putem presupune că 0 ≤ x ≤ y ≤ z ≤ 1.
Atunci vom avea: ∑ x x+y+z

y+z+1 x+y+1
şi va mai trebui verificată inegalitatea:
x+y+z
+ (1 − x)(1 − y)(1 − z) ≤ 1.
x+y+1
Avem succesiv:
x+y+z x+y+1 z−1
+(1−x)(1−y)(1−z)= + +(1−x)(1−y)(1−z)
x+y+1 x+y+1 x+y+1
( )
1−z
=1− [1 − (1 + x + y)(1 − x)(1 − y)].
x+y+1
Inegalitatea de demonstrat rezultă din cea de mai sus după ce observăm
că:

(1 + x + y)(1 − x)(1 − y) ≤ (1 + x + y + xy)(1 − x)(1 − y)

= (1 + x)(1 + y)(1 − x)(1 − x) = (1 − x2 )(1 − y 2 ) ≤ 1.

2.78. I-a soluţie. Fără a restrânge generalitatea problemei presupunem că x ≤


y ≤ z, de unde rezultă că:
1 1 1
≥ ≥
xy + 1 zx + 1 yz + 1
şi astfel obţinem:
x y z x+y+z
+ + ≤ .
yz + 1 zx + 1 xy + 1 xy + 1
Rămâne de demonstrat că: x + y + z ≤ 2xy + 2. Dar (1 − x)(1 − y) ≥ 0,
pentru orice x, y ∈ [0, 1] implică 2xy ≥ xy ≥ x + y − 1 şi deci 2xy + 1 ≥
x + y. Cum 1 ≥ z, prin adunarea ultimelor două inegalităţi obţinem
2xy + 2 ≥ x + y + z.
A II-a soluţie. Considerăm funcţia f : [0, 1] → R,
x y z
f (x) = + +
yz + 1 zx + 1 xy + 1
pentru care
1 yz yz
f ′ (x) = − 2

yz + 1 (zx + 1) (xy + 1)2

95
şi f ′ (x) > 0, f ′′ (x) > 0 pe [0, 1]. Rezultă că extremele lui f se obţin ı̂n
x = 0 sau x = 1 şi avem f (0) = y + z ≤ 2 şi

1 y z 1 y z
f (1) = + + ≤ + + ≤ 1 + 1 = 2.
yz + 1 z + 1 y + 1 yz + 1 y + z y + z

Egalitatea se obţine dacă şi numai dacă x = 0 şi y = z = 1 sau y = 0 şi


x = z = 1 sau z = 0 şi x = y = 1.

2.79. Vom demonstra următorul rezultat:


Dacă x1 , x2 , . . . , xn ∈ [0, 1], atunci:

(x1 + x2 + . . . + xn + 1)2 ≥ 4(x21 + x22 + . . . + x2n ).

Într-adevăr, deoarece x1 , x2 , . . . , xn ∈ [0, 1] avem

x21 ≤ x1 , x22 ≤ x2 , . . . , x2n ≤ xn ,

deci
4(x21 + x22 + . . . + x2n ) ≤ 4(x1 + x2 + . . . + xn ).

Cum 4a ≤ (a + 1)2 , pentru orice a ∈ R, vom avea:

4(x1 + x2 + . . . + xn ) ≤ (x1 + x2 + . . . + xn + 1)2 ,

de unde enunţul rezultă. Pentru n = 3 obţinem inegalitatea din enunţ.

2.80. Avem 1 < x < 3 ⇔ (x − 1)(3 − x) > 0 ⇔ 4x > x2 + 3. Prin urmare:

4(x + y + z) > (x2 + y 2 + z 2 ) + 9 ≥ xy + yz + zx + 9 = 26 + 9 = 35,

35
de unde se obţine că x + y + z > = 8, 75. Din x, y, z < 3 rezultă şi a
4
doua inegalitate: x + y + z < 3 + 3 + 3 = 9. Aşadar x + y + z ∈ (8, 75; 9).

2.81. Din 1 ≤ x ≤
( 3 rezultă
) (x − 1)(x − 3) ≤ 0 deci (x − 1)(x − 3) ≤ 0, adică
∑ ∑
x2 ≤ 4 x − 12. Rămâne de arătat că

(∑ )2 [ (∑ ) ] [(∑ ) ]2
x ≥3 4 x − 12 ⇔ x − 6 ≥ 0,

ceea ce este evident.

96
2.82. I-a soluţie. Deoarece x ≥ −1 rezultă
( )
1 2 3x 1
(x + 1) x − ≥ 0, deci x3 ≥ − , pentru orice x ≥ −1.
2 4 4
Rezultă că:
3 1 3 1
x3 + y 3 + z 3 + t3 ≥ (x + y + z + t) − 4 · = · 2 − 1 = ,
4 4 4 2
deoarece x + y + z + t = 2. Semnul egal se obţine pentru
1
x=y=z=t= .
2

A II-a soluţie. Folosind inegalităţile lui Cebâşev şi Cauchy-Buniakovski


pentru sistemele la fel ordonate (x, y, z, t) şi (x2 , y 2 , z 2 , t2 ), avem:
∑ 1 (∑ ) (∑ 2 ) 1 (∑ ) 1 (∑ )2 1 1 1
x3 ≥ x x ≥ x · x = ·2· ·4 = .
4 4 4 4 4 2

2.83. Demonstrăm mai ı̂ntâi inegalitatea:



n ∑
n
(1 − bk ) > 1 − bk
k=1 k=1

cu 0 < bk < 1, k = 1, n. Pentru n = 2 inegalitatea este evidentă.


Presupunând-o adevărată pentru n, avem:
( )

n+1 ∑
n ∑
n+1 ∑
n ∑
n+1
(1−bk ) > 1 − bk (1−bn+1 ) = 1− bk +bn+1 bk > 1− bk .
k=1 k=1 k=1 k=1 k=1

Inegalitatea a fost demonstrată prin inducţie matematică. Înlocuind bk =


1 − xk , avem xk ∈ (0, 1) şi

n ∑
n
xk > 1 − n + xk ,
k=1 k=1

de unde rezultă enunţul.


1
2.84. Fie f : [0, ∞) → R, f (x) = . Deoarece
1 + ex
ex (ex − 1)
f ′′ (x) = ≥0
(ex + 1)3

97
funcţia f este convexă.
Fie x1 , x2 , . . . , xn ≥ 1. Din inegalitatea lui Jensen avem (luând ak =
ln xk ≥ 0):

n
1 n ∑
n
1 n
≥ ∑
n ⇔ ≥ √ .
1 + eln xk ln xk 1 + xk 1 + x1 x2 . . . xn
n
k=1 k=1 k=1
1+e n

2.85. Inegalitatea este succesiv echivalentă cu următoarele:


√ √ √ √ √ √ √ √ √ √
( x − y)2 ( x + y)2 ( x − y)2 ( x − y)2 ( x + y)2
< < ⇔
8x 2 8y
√ √ √ √
( x + y)2 ( x + y)2 √ √
<1< ⇔ 4y < ( x + y)2 < 4x,
4x 4y
√ √ √ √
ceea ce este evident căci 2 y < x + y < 2 x.
2.86. Prin ı̂nmulţire cu 2 inegalitatea de demonstrat se scrie:
14
(a − b)2 + (b − c)2 + (c − a)2 ≥ (a − b)(b − c).
3
Fie a − b = x ≥ 0, b − c = y ≥ 0. Rezultă că a − c = x + y şi va trebui
demonstrat că:
14
x2 + y 2 + (x + y)2 ≥ xy ⇔ 3(x2 + y 2 ) − 4xy ≥ 0 ⇔
3
( )
2y 2 5 2
3 x− + y ≥ 0,
3 3
ceea ce este evident. Semnul egal se obţine dacă şi numai dacă

x = y = 0 ⇔ a = b = c.

2.87. Inegalitatea din enunţ se mai scrie


(a − b)ab (b − c)bc (c − a)ca
E= + + ≥ 0.
a2 − ab + b2 b2 − bc + c2 c2 − ca + a2
Presupunând că a2 − ab + b2 este cel mai mare dintre cei trei numitori
rezultă:
ab(a − b) + bc(b − c) + ca(c − a) (a − b)(b − c)(a − c)
E≥ = ≥ 0,
a − ab + b
2 2 a2 − ab + b2
deoarece ţinând cont că a ≥ b ≥ c, rezultă că (a − b)(b − c)(a − c) ≥ 0.

98
2.88. Notăm y1 = x21 +a, y1 = x22 +b unde a, b ≥ 0. Inegalitatea este echivalentă
cu:

4(x1 − x2 )[x1 x2 (x1 − x2 ) + ax2 − bx1 ] ≤ [(x21 − x22 ) + (a − b)]2 ⇔

4x1 x2 (x1 − x2 )2 + 4(x1 − x2 )(ax2 − bx1 )


≤ (x1 − x2 )2 (x1 + x2 )2 + 2(x21 − x22 )(a − b) + (a − b)2 ⇔
0 ≤ (x1 − x2 )4 + 2(x1 − x2 )2 (a + b) + (a − b)2 ⇔
0 ≤ (x1 − x2 )4 + 2(x1 − x2 )2 (a + b) + (a − b)2 ,
ceea ce este evident căci a, b ≥ 0.

2.89. În inegalitatea Cauchy-Buniakovski:


( )( 4 ) ( )2

4 ∑ ∑
4
x2i yi2 ≥ xi y i ,
i=1 i=1 i=1

pentru orice xi , yi ∈ R, i = 1, 4, luând


√ √
x
x1 = , y1 = x(ay + bz + ct),
ay + bz + ct

etc., obţinem:
∑ x ∑ (∑ )2
x(ay + bz + ct) ≥ x ,
ay + bz + ct

de unde: (∑ )2
∑ x x
≥∑ .
ay + bz + ct x(ay + bz + ct)

Rămâne să mai verificăm că:


(∑ )2
x 4
∑ ≥ ,
x(ay + bz + ct) a+b+c

sau efectuând calculele:

(a + b + c)(x + y + z + t)2 ≥ 4(a + c)(x + z)(y + t) + 8b(xz + yt).

99
Această inegalitate rezultă din următoarele:
(a+c)(x+y+z+t)2 ≥ 4(a+c)(x+z)(y+t) şi (x+y+z+t)2 ≥ 8(xz+yt).
Prima dintre ele se reduce la (u + v)2 ≥ 4uv, unde u = x + z, v = y + t.
A doua rezultă din 4(x + y)(z + t) ≥ 8(xz + yt), care este echivalentă cu
(y − x)(z − t) ≥ 0.
2.90. Fără a restrânge generalitatea problemei presupunem că a ≥ b ≥ c şi
astfel distingem două cazuri:
1) a ≥ b ≥ c ≥ 1, Notăm cu xn = an b(a − b) + bn c(b − c) + cn a(c − a).
Vom arăta prin inducţie că xn ≥ 0, pentru orice n ∈ N∗ .
i) Notând x = a − b, y = b − c atunci x, y ≥ 0 şi
x1 = bx(b + x) + by(b − y) + (x + y)(b − y)(b + x) = xy(x + y) ≥ 0.

ii) Presupunând că xn ≥ 0 vom arăta că xn+1 ≥ 0. Într-adevăr:


xn+1 − xn = an+1 b(a − b) + bn+1 c(b − c) + cn+1 a(c − a) − xn
= c[an b(a − b) + bn c(b − c) + cn a(c − a)] + an+1 b(a − b) + bn+1 c(b − c)
−an bc(a − b) − bn c2 (b − c) − xn
= (c − 1)xn + an b(a − b)(a − c) + bn c(b − c)2 ≥ 0,
căci c ≥ 1 şi a ≥ b ≥ c.
2) a ≥ b ≥ c şi c ∈ (0, 1) ı̂n acest caz există t > 1 astfel ı̂ncât tc > 1 şi
deci ta ≥ tb ≥ tc > 1. Dar pentru numerele ta, tb, tc proprietatea este
adevărată, deci:
(ta)n tb(ta − tb) + (tb)n tc(tb − tc) + (tc)n ta(tc − ta) ≥ 0
sau
tn+2 [an − b(a − b)nb c(b − c) + cn a(c − a)] ≥ 0,
de unde inegalitatea rezultă şi ı̂n acest caz.
2.91. Înmulţind cu a1 an rezultă:

n ∑
n n (
∑ )
a1 an a1 an
− n(a1 + an ) + ak = − a1 − an + ak
ak ak
k=1 k=1 k=1


n
1 ∑ 1 n
= (a1 an − a1 ak − an ak + a2k ) = (a1 − ak )(an − ak ) ≤ 0.
ak ak
k=1 k=1

100
2.92. Din ipoteză rezultă că f (a1 ) ≤ f (a2 ) ≤ . . . ≤ f (an ). Notând xk = f (ak ),
k = 1, n, inegalitatea revine la
x1 x2 xn x2 x3 x1
+ + ... + ≥ + + ... +
x2 x3 x1 x1 x2 xn
ı̂n ipoteza 0 < x1 ≤ x2 ≤ . . . ≤ xn , adică problema 9.54.

101
102
Capitolul 3

Principiul trinomului

3.1 Enunţuri
3.1. Fie expresia E(x) = (x − 1)2 + (x − 2)2 + . . . + (x − n)2 . Să se arate că
pentru orice x ∈ R avem
(n − 1)n(n + 1)
E(x) ≥ .
12

3.2. Să se determine numărul real a astfel ı̂ncât max |x2 + a| să fie minim,
x ∈ [−1, 1].
3.3. Să se arate că dacă numerele reale a, b, c verifică relaţia ac = 2(b + d),
atunci cel puţin una din ecuaţiile x2 + ax + b = 0, x2 + cx + d = 0 are
soluţii reale.
3.4. Să se demonstreze că dacă ecuaţiile

ax2 + bx + c = 0, a1 x2 + b1 x + c1 = 0, a, a1 ∈ (0, ∞), b, b1 , c, c1 ∈ R

nu au rădăcini reale atunci nici ecuaţia

(a + a1 )x2 + (b + b1 )x + (c + c1 ) = 0

nu are rădăcini reale.


3.5. Fie a, b, c trei numere reale nenule. Să se arate că cel puţin una din
ecuaţiile

ax2 + 2bx + c = 0, bx2 + 2cx + a = 0, cx2 + 2ax + b = 0

are rădăcini reale.

103
3.6. Dacă a1 , a2 , b1 , b2 , c1 , c2 ∈ R astfel ı̂ncât a1 a2 > 0, a1 c1 ≥ b21 , a2 c2 ≥ b22
atunci
(a1 + a2 )(c1 + c2 ) ≥ (b1 + b2 )2 .
√ √
7− 52 x+3 7 + 52
3.7. Dacă x ∈ R, atunci ≤ 2 ≤ .
3 x −x+1 3
3.8. Să se arate că inegalitatea

x21 + x22 + x23 > 2α1 x2 x3 + 2α2 x1 x3 + 2α3 x1 x2

are loc pentru orice x1 , x2 , x3 ∈ R dacă şi numai dacă

α12 + α22 + α32 ≤ 1 − 2α1 α2 α3

şi atunci α12 + α22 + α32 ≥ 6α1 α2 α3 .

3.9. Se consideră funcţia f : R → R,

f (x) = (x − a1 )(x − a2 ) + (x − a2 )(x − a3 ) + (x − a3 )(x − a1 ),

a1 , a2 , a3 ∈ R. Să se arate că dacă f (x) ≥ 0, pentru orice x ∈ R, atunci


a1 = a2 = a3 .
x4 − 2ax3 + 3a2 x2 − 2a3 x + a4 + 9
3.10. Fie f : R → R dată de f (x) = . Să se
x2 − ax + a2
determine valoarea minimă a acestei funcţii ştiind că a ∈ [−2, 2] \ {0}.

3.11. Dacă x, y, z astfel ı̂ncât x + y + z = 4, xy + yz + xz = 4 atunci


[ ]
8
x, y, z ∈ 0, .
3

a+b+c √
3.12. Dacă a, b, c ∈ R∗+ , atunci ≥ abc.
3

3
3.13. Dacă x, y ∈ R astfel ı̂ncât x2 ≥ 2y, atunci
√ √ √
( 3 + 1)(2 x2 − 2y − x) ≥ x2 − 2y + x.

3.14. Dacă a, b, c, d ∈ R astfel ı̂ncât (a − c)(bc − ad) > (b − d)2 , atunci

a2 − 4b > 0 şi c2 − 4d > 0.

104
3.15. Fie k ∈ (0, 2) şi a, b, c, d ∈ R, a ≤ b ≤ c, atunci

(a + b + c + d)2 ≥ 4k(ac + bd).

3.16. Dacă a, b, c, d ∈ [0, e] astfel ı̂ncât e = max{a, b, c, d} şi a + c = b + d,


atunci 4|ac − bd| ≤ e2 .

3.17. Dacă a, b, c ∈ [0, 1], atunci a2 + b2 + c2 ≤ a2 b + b2 c + c2 a + 1.

3.18. Dacă x, y, z ∈ R, atunci:

2(x2 − x + 1)(y 2 − y + 1) ≥ x2 y 2 + 1

3(x2 − x + 1)(y 2 − y + 1)(z 2 − z + 1) ≥ x2 y 2 z 2 + xyz + 1.

3.19. Fie f, g : R → R două funcţii de gradul al doilea cu coeficienţii termenilor


de gradul doi egali cu 1. Să se determine toate funcţiile de gradul al doilea
h : R → R cu proprietatea:

f (x) + g(x)
min(f (x), g(x)) ≤ h(x) ≤ , pentru orice x ∈ R.
2

3.20. Numerele a, b, c, d, p, q satisfac condiţia ab + cd = 2pq. Demonstraţi că,


dacă ac ≥ p2 > 0, atunci bd < q 2 .

3.21. Fie a, b, c ∈ R, a ̸= 0 şi f : R → R, f (x) = ax2 +bx+c. Dacă există α ∈ R


astfel ı̂ncât af (α) < 0, atunci ecuaţia ax2 + bx + c = 0 are rădăcinile
reale şi distincte, deci ∆ > 0.

3.22. Numerele reale a, b, c satisfac relaţia a2 + ab + ac ≤ 0. Demonstraţi că

b2 ≥ 4ac.

3.23. Fie a, b, c ∈ R, a ̸= 0 şi f : R → R, f (x) = ax2 + bx + c. Dacă există


α, β ∈ R astfel ı̂ncât f (α) · f (β) < 0, atunci ∆ > 0.

3.24. Fie A, B, C, D numere negative şi A > B + D. Dacă

p(x) = Ax2 + (C − B − D)x − C

să se arate că p nu are rădăcini ı̂n (0, 1).

105
3.25. Fie numerele reale ai , bi , i = 1, n astfel ı̂ncât

a1 + b1 = a2 + b2 = . . . = an + bn = 1.

Să se arate că:

(a21 + a22 + . . . + a2n )(b21 + b22 + . . . + b2n − n) ≤ (a1 b1 + a2 b2 + . . . + an bn )2 .

3.26. Se consideră numerele reale a, b, c cu proprietăţile

a+b+c=0 şi a2 + b2 + c2 = 1.
1
Să se arate că dacă a ≤ b ≤ c, atunci b2 ≤ .
6
bk
3.27. Dacă bk ∈ R, ak ∈ R∗ şi m ≤ ≤ M , pentru orice k = 1, n, atunci
ak

n ∑
n ∑
n
b2k + mM ak bk ≤ (M + m) ak bk .
k=1 k=1 k=1

3.28. Fie P (x) = ax2 + bx + c un polinom care admite două rădăcini reale şi
distincte ı̂n intervalul (0, 2). Dacă a, b, c ∈ Z astfel ı̂ncât a > 0, atunci
a ≥ 2, b ≤ −3 şi c ≥ 1.

3.29. Fie b, c ∈ R şi f (x) = x2 + bx + c. Să se arate că cea mai mare valoare a
1
lui |f | ı̂n intervalul [−1, 1] nu este mai mică decât .
2
3.30. Fie a, b, c ∈ R, a ̸= 0, f : R → R, f (x) = ax2 + bx + c şi α ∈ R.
|a|
i) Demonstraţi că max{|f (α − 1)|, |f (α)|, |f (α + 1)|} ≥ .
2
ii) Dintre toate polinoamele f cu proprietatea că |f (x)| ≤ 2, pentru
orice x ∈ [0, 2] să se determine polinomul pentru care a are cea mai mică
valoare.

3.31. Dacă |ax2 + bx + c| ≤ 1, pentru orice x ∈ [−1, 1], atunci |a| + |b| + |c| ≤ 3.

3.32. Dacă |ax2 + bx + c| ≤ 1, pentru orice x ∈ [−1, 1], atunci a2 + b2 + c2 ≤ 5.

3.33. Fie a, b, c lungimile laturilor unui triunghi. Atunci pentru orice x, y, z


numere reale care satisfac relaţia ax + by + cz = 0 are loc inegalitatea
xy + yz + zx ≤ 0.

106
3.33. Fie a, b, c, ai ∈ R, i = 1, n, astfel ı̂ncât b2 ≥ ac şi a > n. Să se demonstreze
că:

(b − a1 − a2 − . . . − an )2 > (a − n)(c − a21 − a22 − . . . − a2n ).


n ∑
n
3.34. Fie a, b, ai , bi ∈ R, i = 1, n, a2 > 2 a22k+1 şi a2 > 2 a22k atunci:
k=1 k=1
( )2 { ( )

2n ∑
n ∑
n
ab − ak bk ≥ min a2 − 2 a22k−1 , b2 − 2 b22k−1 ,
k=1 k=1 k=1
( ) ( )}

n ∑
n
a2 − 2 a22k , b2 − 2 b22k .
k=1 k=1

3.35. Dacă ai , bi ∈ R+ şi ai ≤ bi , pentru orice i = 1, n, atunci:


 2   
∑ ∑ ∑
 (ai bi + bi bj ) ≥ 4  ai bj   bi aj  .
1≤i≤j≤n 1≤i≤j≤n 1≤i≤j≤n

3.36. Se consideră polinoamele

P (x) = 2x2 + ax + b, Q(x) = x2 + cx + d

astfel ı̂ncât a2 + c2 + 4d ≤ 2ac + 4b, a, b, c, d ∈ R. Să se arate că dacă


P (x) = 0 are rădăcini reale, atunci şi Q(x) = 0 are rădăcini reale.
Afirmaţia reciprocă este adevărată?

3.37. În binomul f (x) = ax2 + bx + c, toţi coeficienţii sunt pozitivi şi

a + b + c = 1.

Demonstraţi că pentru orice x1 , x2 , . . . , xn > 0 cu x1 · x2 · . . . · xn = 1 ⇒

f (x1 ) · f (x2 ) · . . . · f (xn ) ≥ 1.

3.38. Fie a, b, c laturile unui triunghi obtuzunghic. Să se demonstreze că are
loc inegalitatea:

(am − bn − cp)2 ≥ (a2 − b2 − c2 )(m2 − n2 − p2 ), pentru orice m, n, p ∈ R.

Să se studieze ı̂n ce caz are loc egalitatea.

107
3.2 Probleme propuse
b2 − 4ac
 3.1. i) Dacă a > 0, atunci ax2 + bx + c ≥ − , pentru orice x ∈ R.
4a
b2 − 4c
ii) Dacă a < 0, atunci ax2 + bx + c ≤ − , pentru orice x ∈ R.
4a
 3.2. Să se arate că dacă x + y + z = a, atunci f = xy + yz + zx admite
un maxim. Să se afle maximul lui f şi valorile lui x, y, z pentru care f
devine maxim.

 3.3. Fie funcţia f : R → R,

f (x) = (ax − a1 )2 + (ax − a2 )2 + . . . + (ax − a10 )2

astfel ı̂ncât a ̸= 0 şi ai ∈ R, i = 1, 10.


i) Să se arate că valoarea minimă a lui f nu depinde de a.

ii) Dacă a21 + a22 + . . . + a210 = 1, atunci |a1 + a2 + . . . + a10 | ≤ 10.

 3.4. Fiind date n numere reale a1 , a2 , . . . , an să se determine x astfel ı̂ncât


suma (x − a1 )2 + (x − a2 )2 + . . . + (x − an )2 să aibă cea mai mare valoare
posibilă.

 3.5. Se consideră funcţia f : R → R, definită √


prin f (x) = x2 + ax + 1, cu
a ∈ (−2, 2). Să se arate că f (x) + m ≥ f (x − m)f (x + m), pentru
2

orice x real şi m parametru real.

 3.6. Să se demonstreze că oricare ar fi a, b ∈ [4, ∞) cel puţin una din ecuaţiile:
x2 + ax + b = 0 şi x2 + bx + a = 0 are rădăcini reale.

 3.7. Se consideră funcţia f : R → R, f (x) = ax2 + bx + c unde a, b, c sunt


numere naturale astfel ı̂ncât a > b > 0, iar f (0) şi |f (−1)| sunt numere
prime. Ştiind că trinomul admite rădăcini raţionale demonstraţi că:
i) a > 9;
ii) pentru a < 12 există o singură funcţie f care verifică condiţiile din
ipoteză.

 3.8. Dacă a, b, x, y ∈ R şi a2 + b2 + ab ≤ 3, atunci ax + by ≤ x2 + y 2 − xy.

 3.9. Se consideră polinoamele

P (x) = 2x2 + ax + b, Q(x) = x2 + cx + d

108
astfel ı̂ncât a2 + c2 + 4d ≤ 2ac + 4b, a, b, c ∈ R. Să se arate că dacă
P (x) = 0 are rădăcini reale atunci şi Q(x) are rădăcini reale. Afirmaţia
reciprocă este adevărată?

 3.10. Numerele a, b, c, d, p, q satisfac condiţia ab + cd = 2pq. Dacă ac ≥ p2 > 0


atunci bd ≤ q 2 .

 3.11. Fie a > 0, b ∈ R; dacă x ∈ R astfel ı̂ncât

|a + x + b| ≤ 1, |4a + 2x + b| ≤ 1, 9a + 6x + 4b + 4 ≥ 0

să se arate că |x| ≤ 24.

 3.12. Pentru a, b, c ∈ R se consideră

a2 (b + c) + b2 (a + c) + c2 (a + b) (a2 + b2 + c2 )abc
P (x) = x2 − x+
a+b+c a+b+c

cu rădăcinile x1 şi x2 . Dacă x1 , x2 ∈ (−∞, 1] ∪ [4, ∞) să se arate că a, b, c


nu pot fi numere distincte din [1, 2].

 3.13. Fie a, b, c numere ı̂ntregi şi |a| ≤ 360. Dacă f (x) = ax2 + bx + c, să se
demonstreze ( că f nu)poate avea rădăcinile distincte, amândouă situate
1 1
ı̂n intervalul , .
10 9

a2 b2
 3.14. Găsiţi minimul funcţiei f : (0, 1) → R, f (x) = + dacă a, b > 0.
x 1−x
Pentru ce valori ale lui x se atinge minimul?

 3.15. Găsiţi cel mai mic k ∈ R pentru care oricare ar fi a, b, c ∈ R este


adevărată afirmaţia: dacă pentru orice x ∈ [−1, 1] avem |ax2 +bx+c| ≤ h,
atunci |a| + |b| + |c| ≤ kh.

 3.16. Fie ax2 + bx + c =√ 0, o ecuaţie de gradul al doilea unde a, b, c ∈ Z. Să se


arate că numărul 3 3 nu poate fi rădăcină a acestei √ ecuaţii.
√ Să se arate
că există coeficienţii a, b, c ∈ Z∗ astfel ı̂ncât 0 < a 3 9 + b 3 3 + c < 0, 01.

 3.17. Dacă a, b, c, d ∈ [1, α], α > 1 atunci

(a + b + c + d)2 ≥ t(a2 + b2 + c2 + d2 )

unde t ∈ R+ şi t(α + 1)2 ≤ 16α.

109
 3.18. Să se determine numerele reale a, b, c astfel ı̂ncât să avem:
|ax2 + bx + c| ≤ |cx2 + bx + a|, pentru orice x ∈ R.

 3.19. Se consideră funcţia f : R → R,


f (x) = |(m − 1)x2 − 2(m + 1)x + m + 1|, m ∈ R.
Să se arate că f (x) ≤ 4, pentru orice x ∈ [−1, 1] dacă şi numai dacă
f (−1) ≤ 4.

3.3 Soluţii
3.1. Expresia E(x) se mai scrie:
E(x) = (x2 − 2x + 1) + (x2 − 2 · 2x + 22 ) + . . . + (x2 − 2nx + n2 )
= nx2 − 2x(1 + 2 + . . . + n) + (12 + 22 + . . . + n2 ).

n
n(n + 1) ∑ 2 n(n + 1)(2n + 1)
n
Ţinând cont că k= şi k = avem
2 6
k=1 k=1

n(n + 1) n(n + 1)(2n + 1)


E(x) = nx2 − 2x · +
2 6
şi astfel, după efectuarea calculelor inegalitatea din enunţ se reduce la
[2x − (n + 1)]2 ≥ 0 inegalitate adevărată pentru orice x ∈ R.
3.2. Notăm f (a) = max |x2 + a|. Dacă a > 0 atunci f (a) = 1 + a (atins ı̂n
x∈[−1,1]
punctele x = ±1). Dacă a ∈ [−1, 0], atunci f (a) = max(1 + a, −a). Dacă
a < −1, atunci f (a) = −a şi ı̂n consecinţă
 1

 1 + a dacă a ≥
2
f (a) =

 −a 1
dacă a <
2
1
Minimul lui f (a) este atins pentru a = .
2
3.3. Fie D1 şi D2 discriminanţii celor două ecuaţii. Dacă D1 < 0 şi D2 < 0
atunci D1 + D2 < 0, dar
D1 +D2 = a2 −4b+c2 −4d = a2 +c2 −4(b+d) = a2 +c2 −2ac = (a−c)2 ≥ 0,
contradicţie. Astfel rezultă că cel puţin una din ecuaţii are rădăcini reale.

110
3.4. Avem b2 − 4ac < 0, b21 − 4a1 c1 < 0 şi cum a, a1 > 0 rezultă c, c1 > 0 deci
( 2 )
b b21
4(a + a1 )(c + c1 ) > + (c + c1 ) ≥ (b + b1 )2
c c1

deci concluzia problemei este adevărată.

3.5. Presupunem că nici una din ecuaţii nu are rădăcini reale. Astfel suntem
conduşi la inegalitatea b2 −ac < 0, c2 < ab, a2 < bc de unde prin adunare
ajungem la
1
a2 + b2 + c2 < ab + bc + ca sau [(a − b)2 + (b − c)2 + (c − a)2 ] < 0
2
care este falsă ı̂n R. Deci presupunerea făcută este falsă şi ı̂n consecinţă
cel puţin una din ecuaţiile

ax2 + 2bx + c = 0, bx2 + 2cx + a = 0, cx2 + 2ax + b = 0

are rădăcini reale.

3.6. Fie
∆1
f1 = a1 x2 + 2b1 x + c1 , = b21 − a1 c1 ≤ 0,
4
∆2
f2 = a2 x2 + 2b2 x + c2 , = b22 − a2 c2 ≤ 0.
4
Din a1 a2 > 0 rezultă că a1 şi a2 au acelaşi semn şi cum ∆1 ≤ 0, ∆2 ≤ 0
trinoamele f1 , f2 au acelaşi semn pentru orice x ∈ R, de exemplu

f1 (x) ≥ 0, f2 (x) ≥ 0, pentru orice x ∈ R.

Fie f = f1 + f2 = (a1 + a2 )x2 + 2(b1 + b2 )x + (c1 + c2 ) ≥ 0, pentru orice


x ∈ R rezultă

= (b1 + b2 )2 − (a1 + a2 )(c1 + c2 ) ≤ 0.
4

x+3
3.7. Considerând funcţia f : R → R, f (x) = şi calculând Im f
x2 −x+1
deducem inegalitatea din enunţ.

3.8. Avem succesiv:

x21 + x22 + x23 ≥ 2α1 x1 x2 x3 + 2α2 x3 x2 + 2α3 x1 x2 , x1 , x2 , x3 ∈ R ⇒

111
x21 − (2α2 x3 + 2α3 x2 )x1 − 2α1 x2 x3 + x22 + x23 ≥ 0, x1 , x2 , x3 ∈ R ⇒
(α2 x3 + α3 x2 )2 ≤ x22 + x23 − 2α1 x2 x3 , x1 , x2 , x3 ∈ R ⇒
(1 − α32 )x22 + (−2α1 x3 − 2α2 α3 x3 )x2 + x23 − α22 x23 ≥ 0, x1 , x2 , x3 ∈ R ⇒
|α3 | = 1 şi α1 + α2 α3 = 0, 1− α22 ≥0 sau
α32 ≤ 1 şi α12 + α22 + α33 + 2α1 α2 α3 ≤ 1 ⇔
α12 + α22 + α32 + 2α1 α2 α3 ≤ 1 şi α1 , α2 , α3 ∈ [−1, 1].
Ultima afirmaţie rezultă din faptul că:

α12 + α22 + α32 + 2α1 α2 α3 ≤ 1 ⇔ (α1 + α2 α3 )2 ≤ (1 − α22 )(1 − α32 ).

În continuare vom arăta că dacă numerele α1 , α2 , α3 verifică condiţia

α12 + α22 + α32 ≤ 1 − 2α1 α2 α3

atunci
α12 + α22 + α33 ≥ 6α1 α2 α3 .
Distingem două cazuri:
1) Dacă α1 α2 α3 ≥ 0 din α12 + α22 + α32 + 2α1 α2 α3 ≤ 1 rezultă

α12 ≤ 1, α22 ≤ 1, α32 ≤ 1 ⇔ α1 , α2 , α3 ∈ [−1, 1]

deci

x21 + x22 + x23 ≥ 2α1 x2 x3 + 2α2 x3 x1 + 2α3 x1 x1 , pentru orice x1 , x2 , x3 ∈ R

deci şi pentru x1 = α1 , x2 = α2 , x3 = α3 rezultă că

α12 + α22 + α32 ≥ 6α1 α2 α3 .

2) Dacă α1 α2 α3 < 0 atunci evident avem

α12 + α22 + α32 ≥ 6α1 α2 α3 .

3.9. Avem succesiv

f (x) ≥ 0, pentru orice x ∈ R ⇔

3x2 − 2(a1 + a2 + a3 )x + a1 a2 + a2 a3 + a3 a1 ≥ 0 pentru orice x ∈ R ⇔


∆ ≤ 0 ⇔ (a1 + a2 + a3 )2 − 3(a1 a2 + a2 a3 + a3 a1 ) ≤ 0 ⇔
1
[(a1 − a2 )2 + (a2 − a3 )2 + (a3 − a1 )2 ] ≤ 0 ⇔ a1 = a2 = a3 .
2

112
9
3.10. Observăm că f (x) = x2 − ax + a2 + .
− ax + a2
x2
Deoarece x2 − ax + a2 > 0 pentru orice x ∈ R putem aplica inegalitatea
mediilor, deci:

9
f (x) ≥ 2 (x2 − ax + a2 ) 2 = 6.
x − ax + a2
Deducem
√ că valoarea minimă a lui f este 6 şi se obţine pentru x =
a ± 3(4 − a2 )
şi aceasta există deoarece 4 − a2 ≥ 0, a ̸= 0.
2
3.11. Avem { {
y+z =4−x y+z =4−x

yz = 4 − x(y + z) yz = 4 − 4x + x2
şi formând ecuaţia de gradul al doilea ı̂n t cu rădăcinile y, z deducem

t2 − (4 − x)t + x2 − 4x + 4 = 0.

Dar cum t ∈ R, avem:

∆t = (4 − x)2 + 4(x2 − 4x + 4) ≥ 0
[ ] [ ]
8 8
de unde deducem că x ∈ 0, şi apoi analog deducem y, z ∈ 0, .
3 3
3.12. Fără a restânge
√ generalitatea problemei presupunem că 0 < a ≤ b ≤ c.
Notăm 3 abc = k, trebuie să demonstrăm că

a + b + c ≥ 3k ⇔ a2 b + ab2 + abc ≥ 3abk ⇔ ab2 + (a2 − 3ak)b + k 3 ≥ 0.

Considerăm funcţia f (x) = ax2 + (a2 − 3ak)x + k 3 , x ∈ [0, c]. Această


1
funcţie ı̂şi atinge minimul ı̂n (3k − a) şi are valoarea
2
4ak 3 − (a2 − 3ak)2 (k − a)2 (4k − a)
= ≥0
4a 4

deoarece k 3 = abc ≥ a3 ⇒ k ≥ a ⇒ 4k ≥ a. În particular f (b) ≥ 0.


y
3.13. Considerăm trinomul de gradul al doilea ı̂n z: z 2 − zx + şi fie u, v
2
rădăcinile sale, deci
√ √
2u = x + x2 − 2y, 2v = x − x2 − 2y.

113
Observând că x = u + v şi x2 − y = (u + v)2 − 2uv = u2 + v 2 inegalitatea
devine
√ √
( 3 + 1)(2 u2 + v 2 − u − v) ≥ 2u pentru orice u, v ∈ R sau
√ √
2 u2 + v 2 ≥ u 3 + v pentru orice u, v ∈ R
inegalitate ce se demonstrează uşor.

3.14. Fie trinoamele x2 +ax+b şi x2 +cx+d. Presupunem că cele două trinoame
au o rădăcină comună x0 . Scăzând membru cu membru relaţiile

x20 + ax0 + b = 0 şi x20 + cx0 + d = 0

găsim (a − c)x0 + (b − d) = 0. Condiţia din enunţ conduce la a ̸= c şi


d−b
deci x0 = . Notăm primul trinom cu g şi al doilea cu h, obţinem
a−c
( )
d−b 2 d−b (b − d)2 − (a − c)(bc − ad)
g(x0 ) = +a +b= < 0,
a−c a−c (a − c)2

deci g şi h nu au rădăcini comune. Deoarece g(x0 ) < 0 trinomul g are


două rădăcini reale şi distincte. Analog, trinomul h are două rădăcini
reale şi distincte deoarece h(x0 ) < 0.

3.15. Considerăm funcţia f : R → R, f (x) = (x + a + b + c)2 − 4kbx − 4kac.


Rezultă

f (x) = x2 + 2(a + b + c − 2kb)x + (a + b + c)2 − 4ack.

Discriminantul trinomului este:

∆ = 16k[(k − 1)b2 − (a + c)b + ac].

Cum k > 0 rămâne de arătat că (k − 1)b2 − (a + c)b + ac ≤ 0. Cum


k − 1 ≤ |k − 1| este suficient să se demonstreze că

g(b) = |k − 1|b2 − (a + c)b + ac ≤ 0

pentru orice k ∈ (0, 2) şi pentru orice b ∈ [a, c].


Pentru k = 1, g(b) = −b2 − (b − a)(c − b) ≤ 0. Pentru k ∈ (0, 1) ∪ (1, 2),
|k − 1| > 0 şi maximul lui g(b) se atinge ı̂n a sau ı̂n c. Cum g(a) =
(|k − 1| − 1)a2 ≤ 0 şi g(c) = (|k − 1| − 1)c2 < 0 rezultă că g(b) ≤ 0 pentru
orice b ∈ [a, c].

114
3.16. Vom arăta că −4(ac − bd) ≤ e2 şi astfel inegalitatea din enunţ va rezulta
din motive de simetrie. Înlocuind d = a + c − b avem:

−4(ac − bd) = −4(ac − ba − bc + b2 ) = −4(a − b)(c − b).

Cum funcţia f : [0, e] → R, f (b) = (a − b)(c − b) are un minim absolut


a+c
pentru ymin = şi notând (pentru a şi c fixate) a + c = α avem
2
a+c
0 ≤ b ≤ min{e, α} şi deoarece ∈ [0, e] obţinem că
2
1
(a − b)(c − b) ≥ − (a − c)2 adică − 4(ac − bd) ≤ (a − c)2 ≤ e2 .
4

3.17. Considerăm funcţia f : [0, 1] → R,

f (x) = x2 (1 − b) − c2 x + b2 + c2 − b2 c − 1.

Dacă b = 1, f (x) = −c2 x + c(c − 1) ≤ 0 deoarece x, b, c ∈ [0, 1]. Dacă


b ∈ [0, 1) atunci 1 − b > 0 şi maximul lui f se realizează pentru x = 0
sau x = 1.

f (0) = b2 + c2 − b2 c − 1 = (1 − c)(b2 − 1 − c) ≤ 0.

f (1) = b(b − 1 − bc) ≤ 0


şi deci f (x) ≤ 0, pentru orice x ∈ [0, 1] adică f (a) ≤ 0 şi de aici rezultă
enunţul.

3.18. (1) 2(x2 − x + 1)(y 2 − y + 1) ≥ x2 y 2 + 1.


Avem:
2(x2 − x + 1)(y 2 − y + 1) − x2 y 2 − 1
= x2 y 2 − 2xy(x + y) + 2(x2 + y 2 + xy) − 2(x + y) + 1
= (x − y)2 + x2 y 2 − 2xy(x + y) + (x + y)2 + 2xy − 2(x + y) + 1
= (x − y)2 + (xy − x − y + 1)2 = (x − y)2 + (x − 1)2 (y − 1)2 ≥ 0.
Cazul de egalitate are loc dacă şi numai dacă x = y = 1. Ţinând cont
de inegalitatea (1) este suficient să demonstrăm
(2) 3(x2 y 2 + 1)(z 2 − z + 1) ≥ 2(x2 y 2 z 2 + xyz + 1).
Ordonând după puterile lui z inegalitatea (2) se mai scrie:
(3) (x2 y 2 + 3)z 2 − (3x2 y 2 + 2xy + 3)z + 3x2 y 2 + 1 ≥ 0.

115
Discriminantul ataşat trinomului are expresia:
Dz = (3x2 y 2 + 2xy + 3)2 − 4(x2 y 2 + 3)(3x2 y 2 + 1) = −3(xy − 1)4
şi deoarece coeficientul lui z 2 este strict pozitiv şi Dz ≤ 0 rezultă că
inegalitatea (3) este adevărată pentru orice x, y, z ∈ R. În inegalitatea
din enunţ avem egalitate dacă şi numai dacă x = y = 1.
3.19. Fie f : R → R, f (x) = x2 + b1 x + c1 , g(x) = x2 + b2 x + c2 . Determinăm
numerele reale x astfel ı̂ncât f (x) = g(x). Obţinem ecuaţia
(b1 − b2 )x + (c1 − c2 ) = 0.
Distingem trei cazuri:
1) b1 = b2 şi c1 = c2 şi ı̂n acest caz f (x) = g(x), pentru orice x ∈ R.
2) b1 = b2 = b şi c1 ̸= c2 şi obţinem f (x) ̸= g(x), pentru orice x ∈ R.
3) b1 ̸= b2 şi ı̂n acest caz există un unic x0 astfel ı̂ncât f (x0 ) = g(x0 ).
Deci:
I) Unica soluţie este h(x) = f (x) = g(x) pentru orice x ∈ R.
II) Fără a restrânge generalitatea, presupunem că c1 < c2 (condiţiile
sunt simetrice ı̂n raport cu f şi g). Deci f (x) < g(x) pentru orice x ∈ R;
f (x) = min(f (x), g(x)), pentru orice x ∈ R.
Dacă h(x) = αx2 + βx + γ atunci trebuie să avem
c1 + c2
x2 + bx + c1 ≤ αx2 + βx + γ ≤ x2 + bx + , pentru orice x ∈ R.
2
Din prima inegalitate scrisă astfel:
(1 − α)x2 + (b − β)x + c1 − γ ≤ 0 pentru orice x ∈ R
rezultă 1 − α ≤ 0 deci α ≥ 1, iar din a doua inegalitate α ≤ 1, de unde
α = 1 şi obţinem pentru orice x ∈ R
c1 + c2
x2 + bx + c1 ≤ x2 + βx + γ ≤ x2 + bx + .
2
Prima inecuaţie de gradul I are soluţii ı̂n R dacă şi numai dacă b =
β şi γ ≥ c1 iar a doua inecuaţie de gradul I are soluţii ı̂n R dacă şi
c1 + c2
numai dacă b = β şi γ ≤ . Obţinem deci
2
[ ]
c1 + c2
h(x) = x + bx + γ cu γ ∈ c1 ,
2
2

116
şi se observă că h satisface cele două inegalităţi din enunţ ı̂n acest caz.
III) Deoarece ecuaţia f (x) = g(x) are o soluţie unică x0 , atunci inecuaţia
de gradul I

f (x) = x2 + b1 x + c1 ≤ x2 + b2 x + c2 = g(x)

are soluţia de forma (−∞, x0 ] sau [x0 , ∞). Din simetria condiţiei din
enunţ, putem presupune că f (x) ≤ g(x) ⇔ x ∈ (−∞, x0 ). Deci
{
f (x), x ≤ x0
min(f (x), g(x)) =
g(x), x > x0
Avem pentru x ≤ x0

f (x) = x2 + b1 x + c1 ≤ αx2 + βx + γ = h(x)


b1 + b2 c1 + c2 f (x) + g(x)
≤ x2 + x+ = .
2 2 2
iar pentru x ≥ x0

g(x) = x2 + b2 x + c2 ≤ αx2 + βx + γ = h(x)


b1 + b2 c1 + c2 f (x) + g(x)
≤ x2 + x+ = .
2 2 2
Obţinem 1 ≤ α ≤ 1 ⇔ α = 1. Deci
b1 + b2 c1 + c2 f (x) + g(x)
h(x) = x2 + βx + γ ≤ x2 + x+ =
2 2 2
pentru orice x ∈ R, cu egalitate pentru x0 , deoarece
f (x0 ) + g(x0 )
min(f (x0 ), g(x0 )) = f (x0 ) = g(x0 ) =
2
Aşadar
b1 + b 2 c1 + c2
βx + γ ≤ x+ pentru orice x ∈ R
2 2
adică
( )
b1 + b2 c1 + c2
x β− +γ− ≤ 0 pentru orice x ∈ R
2 2
b 1 + b2
rezultă β = . Cum pentru x0 avem egalitate rezultă că şi
2
c1 + c2 f +g
γ= deci h= .
2 2

117
ab + cd (ab + cd)2
3.20. Fie p = pentru q ̸= 0; ac ≥ p2 ⇔ q 2 ≥ şi apoi
2q 4ac
(ab + cd)2
q 2 ≥ bd rezultă astfel: q 2 ≥ ≥ bd deoarece (ab − cd)2 ≥ 0 şi
4ac
ac > 0. Dacă q = 0, ab + cd = 0 şi ac > 0. Din ab + cd = 0 rezultă că b
şi d au semne diferite şi deci bd ≤ 0 ≤ q 2 .
3.25. Fie
P (x)= (a21 +a22 +. . .+a2n )x2 +2(a1 b1 +a2 b2 +. . .+an bn )x+b21 +b22 +. . .+b2n−n.
Astfel P (x) se mai scrie
P (x) = (a1 x + b1 )2 + (a2 x + b2 )2 + . . . + (an x + bn )2 − n.
Deoarece a1 + b1 = a2 + b2 = . . . = an + bn = 1 rezultă că P (1) = 0 deci
rădăcinile lui P sunt reale.
3.26. Avem
1 1 1
a + c = −b şi ac = [(a + c)2 − a2 − c2 ] = [b2 − (1 − b)2 ] = b2 −
2 2 2
1
deci trinomul f (x) = x2 +bx+b2 − are rădăcinile a şi c. Cum a ≤ b ≤ c,
2
rezultă că f (b) ≤ 0 deci
1 1
b2 + b2 + b2 − ≤ 0 ⇔ b2 ≤ .
2 6
3.27. Considerăm polinomul
P (x) = (x − M )(x − m) = x2 − (M + m)x + mM,
bk
cu rădăcinile m şi M . Deoarece numerele sunt situate ı̂ntre rădăcinile
ak
polinomului, ( )
bk
P ≤ 0 pentru orice k = 1, n
ak
deci
( )2 ( )
b1 b1
− (M + m) + mM ≤ 0/a21
a1 a1
( ) ( )
b2 2 b2
− (M + m) + mM ≤ 0/a22
a2 a2
.......................................
( )2 ( )
bn bn
− (M + m) + mM ≤ 0/a2n .
an an

118
Rezultă că

b2k − (M + m)ak bk + mM a2k ≤ 0 pentru orice k = 1, n.

Adunând membru cu membru aceste inegalităţi obţinem



n ∑
n ∑
n
b2k + mM a2k ≤ (M + m) ak bk .
k=1 k=1 k=1

3.28. P (0) · P (2) = a2 x1 (2 − x1 )x2 (2 − x2 ) dar x1 (2 − x1 ) ≤ 1 şi x2 (2 − x2 ) ≤ 1


şi de aici deducem x1 (2 − x1 )x2 (2 − x2 ) ≤ 1 (rădăcinile fiind distincte).
Deoarece P (0) · P (2) ∈ Z+ avem 1 ≤ P (0) · P (2) < a2 de unde deducem
că a2 > 1 dar având ı̂n vedere că a > 0 rezultă a ≥ 2. De asemenea din
b
x1 + x2 > 0 rezultă că − > 0 şi de aici deducem că b este negativ, iar
a
din x1 x2 ∈ (0, 4) rezultă că c ≥ 1. Ţinând cont că a ≥ 2 şi c ≥ 1, din
b2 − 4ac ≥ 0 avem b2 > 8 şi din b < 0 rezultă că b ≤ −3.

3.29. Presupunem că cea mai mare valoare a lui f ı̂n intervalul [−1, 1] este
1
mai mică decât . Astfel
2
1 1 1
|f (−1)| = |1 − b + c| < , |f (0)| = |c| < , |f (1)| = |1 + b + c| < .
2 2 2
1
Considerăm polinomul g(x) = f (x) − f0 (x) unde f0 (x) = x2 − . Avem
2
1 1
g(x) = bx + c + şi g(−1) = −b + c + < 0,
2 2
1 1
g(0) = c + > 0, g(1) = b + c + < 0.
2 2
Prin urmare g are două rădăcini reale şi distincte x1 ∈ (−1, 0), x2 ∈
(0, 1). Fiind polinomul de gradul I, g(x) = 0 pentru orice x ∈ R ⇔
1
f (x) = f0 (x) pentru orice x ∈ R. Dar atunci |f (0)| = |f0 (0)| = , ı̂n
2
1
contradicţie cu |f (0)| < .
2
3.30. a) f (α − 1) = a(α2 − 2α + 1) + b(α − 1) + c = f (α) − 2aα + a − b

f (α + 1) = a(α2 + 2α + 1) + b(α + 1) + c = f (α) + 2aα + a + b ⇒

f (α − 1) + f (α + 1) = 2f (α) + 2a ⇒

119
f (α − 1) + f (α + 1) − 2f (α)
a= ⇒
2
|f (α − 1)| + |f (α + 1)| + 2|f (α)| M + M + 2M
|a| ≤ ≤ = 2M (∗)
2 2
unde M = max{|f (α − 1)|, |f (α)|, |f (α + 1)|}.
b) Din a) ⇒ |a| ≤ 2 max{|f (α−1)|, |f (α)|, |f (α+1)|} ≤ 4, deci a ≥ −4.
Încercăm să determinăm polinoamele de gradul II cu a = −4. Deoarece
ı̂n (∗) avem egalitate

⇒ |f (0)| = |f (1)| = |f (2)| = 2.

În plus,
f (0) + f (2) − 2f (1)
a= = −4,
2
deci ı̂n mod necesar f (0) = f (2) = −2, f (1) = 2. Obţinem sistemul
  
 c = −2  c = −2  c = −2
a+b+c=2 ⇔ a+b=4 ⇔ a+b=4 ⇔
  
4a + 2b + c = −2 4a + 2b = 0 2a + b = 0

 c = −2
a = −4 ⇒ f (x) = −4x2 + 8x − 2.

b=8

Rămâne să verificăm că | − 4x2 + 8x − 2| ≤ 2, pentru orice x ∈ [0, 2].


b
Deoarece − = 1 ∈ [0, 2], | − 4x2 + 8x − 2| ≤ 2 este echivalentă cu
2a
|f (0)| ≤ 2, |f (1)| ≤ 2, |f (2)| ≤ 2.

3.31. |f (0)| = |c| ≤ 1 (1)


|f (1)| = |a + b + c| ≤ 1 (2)
|f (−1)| = |a − b + c| ≤ 1 (3)
Din (2) şi (3) rezultă |a + c| + |b| ≤ 1 (4)
dar |a + c| ≥ |a| − |c| (5).
Din (4) şi (5) |a| − |c| + |b| ≤ 1 sau |a| + |b| + |c| ≤ 1 + 2|c| ≤ 3.
Observaţie. 3 este cel mai mic majorant deoarece f = 2x2 − 1 verifică
condiţiile enunţului şi realizează |a| + |b| + |c| = 3.

120
ax + by
3.32. Din ipoteză obţinem z = − , astfel că inegalitatea de demonstrat
c
xy + yz + zx ≤ 0 este echivalentă cu

ax + by
xy − (x + y) ≤0
c
adică
cxy − (x + y)(ax + by) ≤ 0 sau
(1) ax2 + (a + b − c)xy + by 2 ≥ 0.
Dacă y = 0, inegalitatea (1) este evidentă. Dacă y ̸= 0, ı̂mpărţind cu
x
numărul strict pozitiv y 2 şi punând = t, inegalitatea (1) este echiva-
y
lentă cu
(2) at2 + (a + b − c)t + b ≥ 0.
Avem:

∆ = (a + b − c)2 − 4ab = a2 + b2 + c2 − 2ab − 2ca

= (a2 − ab − ac) + (b2 − ab − ac) + (c2 − ac − bc)


= −[a(b + c − a) + b(a + c − b) + c(a + b − c)]
care este un număr strict negativ, datorită inegalităţilor cunoscute pe
care le satisfac lungimile a, b, c ale laturilor triunghiului. Deci inegalitatea
(2) este adevărată pentru orice t ∈ R, ceea ce ı̂ncheie soluţia problemei.

3.33. Utilizăm funcţia


n
f (x) = ax + 2bx + c −
2
(x − ak )2
k=1

şi se poate deduce că f (x0 ) ≤ 0 (unde x0 este o rădăcină reală a ecuaţiei
ax2 +2bx+c = 0) şi conform cu principiul trinomului rezultă inegalitatea
din enunţ.

3.34. Fie f : R → R,

f (x) = (ax − b)2 − (a1 x − b1 )2 − . . . − (a2n x − b2n )2


( ) ( )

2n ∑
2n ∑
2n
= a2 − a2k x2 − 2 ab − ak bk x + b2 − b2k
k=1 k=1 k=1

121
= f1 (x) + f2 (x) unde f1 , f2 ∈ R[x].
( ) ( )
1 ∑n ∑2n
f1 (x) = a2 − 2 a22k−1 x2 − ab − ak bk x
2
k=1 k=1
( )
1 ∑
n
1 1
+ b2 − 2 b22k−1 = A1 x2 − Bx + C1
2 2 2
k=1
( ) ( )
1 ∑
n ∑
2n
f2 (x) = a −2
2
a22k x −
2
ab − ak bk x
2
k=1 k=1
( )
1 ∑
n
1 1
+ b2 − 2 b22k = A2 x2 − Bx + C2 .
2 2 2
k=1

Cum f1 şi f2 au coeficienţi dominanţi pozitivi


( ) (deci şi f = f1 + f2 )
b
iar f are şi valori nepozitive (deoarece f ≤ 0) deducem că ∆f1 ≥
a
0 sau ∆f2 ≥ 0, adică B 2 ≥ A1 C1 sau B 2 ≥ A2 C2 şi atunci B 2 ≥
min(A1 C1 , A2 C2 ) care este inegalitatea din enunţ.

3.35. Considerăm funcţia



f (x) = (ai x − bi )(bj x − aj )
1≤i≤j≤n

şi observăm că f (1) ≤ 0, dar cum coeficientul dominant este pozitiv
rezultă, conform principiului trinomului, inegalitatea din enunţ.

a2
3.36. Avem a2 − 8b ≥ 0 deci 4b ≤ . Inegalitatea din enunţ dă
2

a2 a2
a2 + c2 + 4d ≤ 2ac + sau 4d ≤ − − c2 + 2ac.
2 2
Discriminantul lui Q este

a2 (2c − a)2
∆ = c2 − 4d ≥ c2 + + c2 − 2ac = ≥ 0.
2 2
Reciproca este falsă. Fie a = c > 0, b > 0, d < 0. Inegalitatea se verifică,
Q are rădăcini reale dar P nu are rădăcini reale.

122

3.37. Demonstrăm că f (x)f (y) ≥ (f ( xy))2 pentru orice x, y > 0. Notând

xy = z, avem:

f (x)f (y) − f (z)2 = a2 (x2 y 2 − z 4 ) + b2 (xy − z 2 ) + c2 (1 − 1)

+ab[x2 y + xy 2 − 2z 3 ] + ac(x2 + y 2 − 2z 2 ) + bc(x + y − 2z)


√ √ √ √
= abz( x − y)2 + ac(x − y)2 + bc( x − y)2 ≥ 0.

3.38. Această problemă este ı̂n esenţă o problemă de algebră (inegalitatea


Aczel). Este cunoscut că laturile a, b, c ale unui triunghi obtuzunghic
satisfac inegalitatea a2 > b2 + c2 . Această condiţie este necesară şi su-
ficientă pentru demonstrarea inegalităţii lui Aczel. Considerăm funcţia
f : R → R,

f (x) = (ax − m)2 − (bx − n)2 − (cx − p)2 = (a2 − b2 − c2 )x2

−2x(am − bn − cp) + m2 − n2 − p2
şi cum a2 > b2 +c2 deducem că ∆f ≥ 0, obţinându-se ı̂n final inegalitatea
din enunţ. Dacă relaţia din enunţ trece la egalitate atunci este necesară
şi suficientă existenţa a două numere reale k, t astfel ı̂ncât:
a b c k
= = = .
m n p t

Demonstraţie. Dacă relaţia din enunţ se verifică la egalitate, atunci


m
∆f = 0. Prin urmare f (x) ≥ 0, pentru orice x ∈ R. Pentru s =
a
(a ̸= 0) obţinem

f (s) = −(bs − n)2 − (cs − p)2 ≤ 0.

Din cele două inegalităţi obţinem


a b c k
f (s) = 0 ⇒ = = =
m n p t
cu notaţiile anterioare.

123
124
Capitolul 4

Inegalităţi cu numere
complexe

4.1 Enunţuri
4.1. Fie z1 , z2 ∈ C. Să se arate că |z1 + z2 | ≤ |z1 | + |z2 |. Cazul de egalitate
are loc dacă şi numai dacă z1 · z2 ≥ 0.


4.2. Fie z1 , z2 ∈ C. Să se arate că |z1 | − |z2 | ≤ |z1 − z2 |.

4.3. Fie z ∈ C. Dacă |1 + z| ≤ 1 şi |1 + z 2 | ≤ 1 să se arate că |z| ≤ 1.

4.4. Fie z1 , z2 ∈ C. Să se arate că |z1 | + |z2 | ≤ |z1 + z2 | + |z1 − z2 |. Când are
loc egalitatea?
( )
1 1
4.5. Dacă z ∈ C şi Re < atunci |z| > 1.
1−z 2

1 z
4.6. Dacă z ∈ C şi Re (z) < atunci < 1.
2 1 − z

4.7. Să se arate că pentru orice z ∈ C cu |z| = 1 avem |1 + z| > 1 sau
|1 + z 2 | > 1.

∑n
k
4.8. Dacă z ∈ C şi Re (z) = 0 atunci |1 + z| ≥ 1 şi z ≥ 1.

k=0

4.9. Dacă z ∈ C verifică |z 2 + 1| ≤ 1 şi |z 2 − 1| ≤ 1 să se arate că z = 0.

125
4.10. Dacă z1 , z2 ∈ C, |z1 | = |z2 | şi z1 + z2 > 0 atunci z1 · z2 > 0. Reciprocele
sunt adevărate?

4.11. Fie ω o rădăcină nereală de ordinul trei a unităţii. Demonstraţi că dacă
z ∈ C astfel ı̂ncât |z − 1| ≤ 1 şi |z − ω| ≤ 1 atunci |z| ≤ 1.

4.12. Dacă z1 , z2 , z3 ∈ C astfel ı̂ncât |z1 | = |z2 | = |z3 | = 1, atunci:

|z1 − z2 | + |z2 − z3 | + |z3 − z1 | ≥ |z1 − z2 ||z2 − z3 ||z3 − z1 |.

În ce caz avem egalitate?

4.13. Fie f ∈ C[x], f = xn + an−1 xn−1 + . . . + a1 x + a0 . Să se arate că există


z ∈ C astfel ı̂ncât |z| = 1 şi |f (z)| ≥ 1.

4.14. Fie z1 , z2 , . . . , zn ∈ C∗ astfel ı̂ncât |z1 | = |z2 | = . . . = |zn | = r. Să se


arate că numărul
( )
1 1 1
(z1 + z2 + . . . + zn ) + + ... +
z1 z2 zn

este real şi cuprins ı̂n intervalul [0, n2 ].

4.15. Dacă z1 , z2 , z3 ∈ C astfel ı̂ncât |z1 | = |z2 | = |z3 | = 1 demonstraţi că


există α1 , α2 , α3 ∈ {−1, 1} astfel ı̂ncât
√ √ √
|α1 z1 z2 + α2 z2 z3 + α3 z3 z1 | ≤ 1.

4.16. Dacă z1 , z2 ∈ C \ R şi |z1 | = |z2 | = 1, iar a, b ∈ R cu a + b = 1, atunci:

|z1 + z2 |
|az1 + bz2 | ≥ .
2

3 1 1
4.17. Fie z ∈ C∗ astfel ı̂ncât z + 3 ≤ 2. Să se arate că z +
≤ 2.
z z
z−u
4.18. Fie u, v ∈ C astfel ı̂ncât |u| ≤ 1, |v| = 1 şi fie ω = v cu z ∈ C. Să
1 − uz
se arate că
|ω| ≤ 1 ⇔ |z| ≤ 1.

4.19. Dacă z ∈ C şi |z| = 1 atunci 2 ≤ |1 + z| + |1 + z 2 | + |1 + z 3 |.



4.20. Dacă z ∈ C astfel ı̂ncât |z| = 1, atunci |1 + z| + |1 + z 2 | ≥ 2.

126

1
4.21. Dacă z ∈ C şi |z| < 1, |z − 1| < 1 atunci z − < 1.
2
4.22. Fie z1 , z2 , z3 ∈ C astfel ı̂ncât

|z1 z2 + z2 z3 + z3 z1 | = a > 0 şi |z1 z2 z3 | = b > 0.


3b
Să se arate că există k ∈ {1, 2, 3} astfel ı̂ncât |zk | ≤ .
a
4.23. Fie z1 , z2 , z3 , z4 ∈ C cu |z1 | = |z2 | = |z3 | = |z4 | = 1 şi 0 ≤ arg z1 <
arg z2 < arg z3 < arg z4 ≤ π. Să se arate că |z1 + z4 | ≤ |z2 + z3 |.

4.24. Dacă z1 , z2 , z3 ∈ C distincte două câte două astfel ı̂ncât |z1 | = |z2 | =
|z3 | = r, să se afle minimul expresiei:

9|z1 |2 − |z1 + z2 + z3 |2
E= .
|z1 − z2 ||z2 − z3 ||z3 − z1 |

4.25. Dacă z ∈ C∗ astfel ı̂ncât |z 2 + 4| = 4|z| atunci



|z|4 + 8Re (z 2 ) ≤ 16(11 + 8 2).

4.26. Dacă A = {z ∈ C : z = x + iy, x, y ∈ Z, x · y = 0} atunci oricare ar


fi n ∈ N, n multiplu de 4 şi oricare ar fi z0 , z1 , . . . , zn ∈ A, toate diferite
ı̂ntre ele, avem:
n(n + 2)(n + 4)(3n2 + 12n − 16)
z04 + z14 + . . . + zn4 ≥ .
3840


n
4.27. Fie z1 , z2 , . . . , zn ∈ C astfel ı̂ncât |zk | = 0 şi a1 , a2 , . . . , an ∈ R.
k=1
Atunci:
∑n 1 ∑
n

ak zk ≤ (M − m) |zk |
2
k=1 k=1

unde M = max{ak , k = 1, n} şi m = min{ak , k = 1, n}.

4.28. Fie z = x + iy un număr complex. Să se demonstreze inegalitatea:


{ } √ √
|x + y − 1| + |x + y + 1|
2 · max |z|, √ ≤ | z 2 − 1 − z| + | z 2 − 1 + z|
2 2

≤ 2 · min{(1 + |z|), x2 + y 2 + 1}.

127
4.29. Se consideră ecuaţia z n + nz + a = 0, unde a ∈ R şi n ∈ N, n ≥ 2. Să se
arate că dacă ω este o rădăcină complexă, dar nereală, a acestei ecuaţii,
atunci |ω| > 1.

4.30. Să se arate că pentru orice n ∈ N, n ≥ 2, ecuaţia 2n−1 (z n +1)−(z +1)n =
0 are toate rădăcinile de acelaşi modul.

n
4.31. Se dau numerele complexe z1 , z2 , . . . , zn cu proprietatea |zi | ≥ 1.
i=1
Demonstraţi că există J ⊂ {1, 2, . . . , n} astfel ı̂ncât:


1
zj ≥ .
4
j∈J

z1 + z2 + . . . + zn
4.32. Fie z1 , z2 , . . . , zn ∈ C şi z0 = . Demonstraţi că pentru
n
orice z ∈ C avem:

n ∑
n
|z0 − zk | ≤
2
|z − zk |2 .
k=1 k=1

4.2 Probleme propuse



 4.1. Pentru orice z ∈ C are loc: |Re z| + |Im z| ≤ |z| 2.

 4.2. Fie z1 , z2 ∈ C astfel ı̂ncât z1 ̸= z2 şi |z1 | = |z2 |. Să se arate că:
1
|z1 + z2 | < |z1 |.
2

 4.3. Fie ecuaţia az 2 + bz + c = 0, a, b, c ∈ C∗ cu |a| = |b| = |c|. Să se arate că:


√ √
5+1 5−1
≥ |zk | ≥ , k = 1, 2
2 2
unde zk sunt rădăcinile ecuaţiei.

 4.4. Fie z0 = r(cos α + i sin α), r > 0, α ∈ R şi z ∈ C astfel ı̂ncât

|z − r| ≤ r şi |z − z0 | ≤ r.
α

Să se arate că: |z| ≤ 2r cos .
2

128

6
 4.5. Dacă zi ∈ C, i = 1, 6 verifică relaţiile zik = 0, k = 1, 2 atunci
i=1

( )2

6 ∑
6
3 |zk |2 ≤ 2 zk .
i=1 i=1

 4.6. Dacă z1 , z2 , . . . , zn ∈ C astfel ı̂ncât |zk | = 1, pentru orice k = 1, n atunci


pentru orice z ∈ C:


n
|z − zk | ≥ n(|z| − 1)2 .
k=1

 4.7. Dacă z1 , z2 , z3 sunt numere complexe având acelaşi modul r, atunci

|z1 − z2 | · |z1 − z3 | + |z2 − z3 | · |z2 − z1 | + |z3 − z1 | · |z3 − z2 | ≤ 9r2 .

 4.8. Fie z1 , z2 ∈ C. Să se arate că

2|z1 + z2 | · |1 + z1 | · |1 + z2 | ≤ |2 + z1 + z2 |2 .

 4.9. Fie z ∈ C. Să se rezolve inecuaţia z 2 + z ≤ 0.

 4.10. Fie n ∈ N∗ şi z1 , z2 , . . . , zn ∈ C∗ . Să se arate că există A ⊆ {z1 , . . . , zn }


astfel ı̂ncât

∑ n
√ ∑
|zk | ≤ 4 2 zi .
k=1 zi ∈A

 4.11. Dacă z ∈ C astfel ı̂ncât |z| = 1 şi Re z ≥ 0 atunci |z + z 2 + z 3 | ≥ 1. Când


are loc egalitatea?

 4.12. Se dau numerele complexe z1 , . . . , zn cu |zk | ≤ 1, pentru orice k = 1, n.


Demonstraţi că există ε1 , . . . , εn ∈ {−1, 1} astfel ı̂ncât

|ε1 z1 + . . . + εp zp | ≤ 2 pentru orice p = 1, n.

 4.13. Fie polinomul f = x4 −x3 +ax2 +bx+c, a, b, c ∈ R cu rădăcinile complexe


nereale z1 , z1 , z2 , z2 . Dacă |z1 |2 = 2Re z2 atunci
√polinomul are cel puţin
două rădăcini de modul mai mare sau egal cu 2 − 1.

129
 4.14. Fie n ≥ 2 un număr natural şi z ∈ C \ {1} astfel ı̂ncât z n = 1. Să se
arate că:
2
≤ |1 − z| ≤ 2.
n−1

 4.15. Dacă z1 , z2 , z3 ∈ C, atunci are loc:

|z1 + z2 | + |z2 + z3 | + |z3 + z1 | ≤ |z1 | + |z2 | + |z3 | + |z1 + z2 + z3 |.

 4.16. Să se arate că dacă zk ∈ C, k = 1, n atunci:


n
∑ ∑
n ∑

zk + (n − 2) |zk | ≥ |zi + zj |.

k=1 k=1 1≤i≤j≤n

 4.17. Fie z1 , z2 , z3 ∈ C, nu toate reale, astfel ı̂ncât |z1 | = |z2 | = |z3 | = 1 şi
2(z1 + z2 + z3 ) − 3z1 z2 z3 ∈ R. Să se arate că
π
max{arg z1 , arg z2 , arg z3 } ≥ .
6

 4.18. Dacă z1 , z2 , z3 ∈ C distincte două câte două de modul r să se arate că:

1 1 1 1
+ + ≥ 2.
|z1 − z2 | · |z1 − z3 | |z2 − z1 | · |z2 − z3 | |z3 − z1 | · |z3 − z1 | r

 4.19. Fie z1 , z2 ∈ C cu |z1 | = |z2 | şi |z1 + tz2 | ≥ |z2 | pentru orice t ∈ R. Să se
arate că z12 + z22 = 0.
√ 1
 4.20. Fie z ∈ C, |z| = 1. Să se arate că 2|1 − z| + |1 − z 4 | ≥ |1 − z 2 |. Pentru
4
ce valori ale lui z are loc egalitatea?

 4.21. Fie (a, b) ∈ C × C şi r ∈ R, r > 0. Dacă |z| ≤ r, să se determine


max |az + b|.

 4.22. Dacă z ∈ C şi 1 ≤ |z| ≤ 6 să se determine sup |z 2 + 5z + 1|.

 4.23. Fie z1 , z2 , . . . , zn numere complexe diferite ı̂ntre ele cu proprietatea că:

min{|zi − zj |, i, j = 1, n, i ̸= j} ≥ max{|zi |, i = 1, n}.

Determinaţi valoarea maximă a lui n.

130
 4.24. Fie a, b, c ∈ R cu a > 0 şi funcţia f : C → R, f (z) = a|z|2 + |bz + c|,
pentru orice z ∈ C.
a) Să se arate că pentru orice z1 , z2 ∈ C şi orice α ∈ [0, 1] avem

f (αz1 + (1 − α)z2 ) ≤ αf (z1 ) + (1 − α)f (z2 ).

b) Să se arate că pentru orice z1 ∈ C cu |z1 | < 1 există z2 ∈ C cu |z2 | ≤ 1


astfel ı̂ncât f (z1 ) < f (z2 ).

n
 4.25. Dacă zi ∈ C, i = 1, n astfel ı̂ncât zi = 0 şi |zi | ̸= 0, pentru orice
i=1

n
i = 1, n atunci pentru orice z ∈ C avem |zi − z| ≥ n.
i=1

 4.26. Dacă z ∈ C să se demonstreze inegalitatea (|z|2 + 1)n ≥ |z 2n − 1|. În ce


caz avem egalitate?


4 ∑
4
 4.27. Dacă zi ∈ C, i = 1, 4 astfel ı̂ncât zi = zi2 = 0, determinaţi
i=1 i=1
valoarea maximă a expresiei

|4(z18 + z28 + z38 + z48 ) − (z14 + z24 + z34 + z44 )2 |.

4.3 Soluţii
4.1. I) Fie x1 = a1 + ib1 şi x2 = a2 + ib2 . Atunci inegalitatea este echivalentă
cu:
√ √ √
(a1 + a2 )2 + (b1 + b2 )2 ≤ a21 + a22 + b21 + b22 ⇔

a1 a2 + b1 b2 ≤ (a21 + b21 )(a22 + b22 )
care este adevărată ı̂n conformitate cu inegalitatea C-B-S.
II) Inegalitatea din enunţ este echivalentă cu

|z1 + z2 |2 ≤ |z1 |2 + |z2 |2 + 2|z1 | · |z2 | ⇔

(z1 + z2 )(z1 + z2 ) ≤ z1 · z1 + z2 · z2 + 2|z1 | · |z2 | ⇔


2Re (z1 · z2 ) ≤ 2|z1 | · |z2 | ⇔ Re (z1 · z2 ) ≤ |z1 · z2 |

131
inegalitate care este evidentă. Egalitatea are loc dacă şi numai dacă
z1 · z2 ≥ 0 ⇔ există α ∈ R, α > 0 astfel ı̂ncât z1 = αz2 .
III) Fie P1 şi P2 punctele din plan care au afixele z1 respectiv z2 şi O
originea sistemului de coordonate şi P punctul de afix z1 + z2 . Atunci
ı̂n triunghiul, eventual degenerat, OP1 P avem: OP ≤ OP1 + P1 P ; cum
P1 P = OP2 deducem |z1 + z2 | ≤ |z1 | + |z2 |.

4.2. Din inegalitatea |z1 + z2 | ≤ |z1 | + |z2 | avem:

|z1 | ≤ |z2 | + |z1 − z2 | şi |z2 | ≤ |z1 | + |z2 − z1 |

de unde deducem inegalitatea:




|z1 | − |z2 | ≤ |z1 − z2 |.

4.3. Avem 2|z| = |2z| = |(z + 1)2 − (z 2 + 1)| ≤ |1 + z|2 + |1 + z 2 | ≤ 2 de unde


se deduce uşor că |z| ≤ 1.
1 1
4.4. |z1 + z2 | + |z1 − z2 | = (|z1 + z| + |z1 − z2 |) + (|z1 + z2 | + |z1 − z2 |)
2 2
1
≥ [2|z1 | + 2|z2 |] = |z1 | + |z2 |
2
(unde ı̂n demonstraţie s-a folosit inegalitatea |z1 + z2 | ≤ |z1 | + |z2 |).
4.5. Fie z = a + bi. Atunci:
( )
1 1 1 − a + bi 1 1−a
= = deci Re = ,
1−z 1 − a − bi (1 − a) + b
2 2 1−z (1 − a)2 + b2
( )
1 1 1−a 1
Re < ⇔ < ⇔
1−z 2 (1 − a) + b
2 2 2
2 − 2a < 1 − 2a + a2 + b2 ⇔ a2 + b2 > 1 ⇔ |z| > 1.

4.6. Fie z = a + bi atunci



z 1 1

1 − z < 1 ⇔ a +b < (1−a) +b ⇔ 2a < 1 ⇔ a < 2 ⇔ Re (z) < 2
2 2 2 2

care este adevărată conform ipotezei.


4.7. Admiţând că |z + 1| < 1 şi |z 2 + 1| < 1 atunci, ı̂n conformitate cu: ”dacă
z ∈ C şi |1 + z| ≤ 1 şi |1 + z 2 | ≤ 1 atunci |z| ≤ 1” avem |z| < 1 ceea ce
contrazice ipoteza, deci |z + 1| ≥ 1 sau |z 2 + 1| ≥ 1.

132
4.8. Dacă z = a + bi cu a ≥ 0, atunci:
√ √
|1 + z| = (1 + a)2 + b2 = a2 + b2 + 2a + 1.

Cum a ≥ 0 deducem că |1 + z| ≥ 1. Analog se arată că



∑n

z k ≥ 1.

k=0

4.9. Luând z = a + bi atunci inegalităţile |z 2 + 1| ≤ 1 şi |z 2 − 1| ≤ 1 sunt


echivalente cu

(a2 − b2 + 1)2 + 4a2 b2 ≤ 1 şi (a2 − b2 − 1)2 + 4a2 b2 ≤ 1

de unde, prin adunarea lor vom avea:

2(a4 + 2a2 b2 + b4 ) ≤ 0

adică a2 + b2 ≤ 0 de unde z = 0.
4.10. Fie z1 = a1 + b1 i şi z2 = a2 + b2 i. Atunci relaţiile din enunţ se reduc la:

a21 + b21 = a22 + b22 ; a1 + a2 > 0 şi b1 + b2 = 0,

de unde a21 = a22 de unde a1 = a2 > 0. Atunci

z1 z2 = a1 a2 − b1 b2 + i(a1 b2 + a2 b1 ) = a21 + b21 > 0.

Pentru z1 = −1 şi z2 = −3 avem z1 · z2 > 0, dar |z1 | ̸= |z2 . Pentru


z1 = −1 − i şi z2 = −1 + i avem z1 z2 > 0 dar z1 + z2 < 0. Deci
reciprocele sunt false.
4.11. Punctele de afix z cu |z − 1| ≤ 1 se află ı̂n interiorul sau pe
cercul C(A, 1), unde A are afixul 1, iar punctele pentru care
|z − ω| ≤ 1 se află ı̂n interiorul sau pe cercul C(B, 1) cu B de afix ω.
Aceste cercuri au ı̂n comun punctul M .
Cum OABM este romb şi m(AOB) \ =

120 (OB se obţine din OA prin
rotire cu 120◦ ) deducem că OM = 1. M
Punctele care ı̂ndeplinesc ambele ine- B
galităţi se află ı̂n zona comună din inte-
riorul celor două cercuri şi ı̂n consecinţă A
O
|z| ≤ OM = 1.

133
4.12. Dacă două din numere sunt egale, inegalitatea este evidentă, cu egali-
tate dacă sunt toate egale ı̂ntre ele. Dacă numerele sunt distincte două
câte două atunci fie A, B, C punctele care au afixele z1 , z2 , z3 . Atunci
inegalitatea din enunţ este echivalentă cu

AB + AC + BC ≥ AB · AC · BC.

Cum R = 1 este echivalentă cu

(sin A + sin B + sin C) ≥ 4 sin A sin B sin C.

Din inegalitatea mediilor avem:



sin A + sin B + sin C ≥ 3 sin A sin B sin C.
3


3 3
Cum sin A sin B sin C ≤ din aceste inegalităţi obţinem:
8
sin A + sin B + sin C ≥ 4 sin A sin B sin C.

4.13. Fie polinomul g = f (x) · x ∈ C[x] şi ε0 , ε1 , . . . , εn rădăcinile de ordinul


n + 1 ale unităţii. Atunci
{
p p p n + 1 dacă p = n + 1
ε0 + ε1 + . . . + εn =
0 dacă 0 ≤ p ≤ n, p ∈ N.

În consecinţă

n ∑
n
g(εk ) = n + 1 de unde |g(εk )| ≥ n + 1
k=0 k=0

şi ı̂n consecinţă există un S ∈ {0, 1, . . . , n} astfel ı̂ncât |g(εS )| ≥ 1 de


unde |εS | · |f (εS )| ≥ 1 adică |f (εS )| ≥ 1. În consecinţă z = εS verifică
cerinţele problemei.
( )
1 1 1
4.14. Fie ω = (z1 + z2 + . . . + zn ) + + ... + . Atunci
z1 z2 zn
( )
1 1 1
ω = (z1 + z2 + . . . + zn ) + + ... + .
z1 z2 zn
r2
Cum zk · zk = r2 pentru orice k = 1, n ⇒ zk = şi ı̂n consecinţă
zk
( )
1 1 1 1
ω=r 2
+ + ... + · (z1 + z2 + . . . + zn ) = ω
z1 z2 zn r2

134
deci ω = ω de unde ω ∈ R.
Fie zk = r(cos αk + i sin αk ) pentru orice k = 1, n. Atunci

ω = [cos α1 + . . . + cos αn + i(sin α1 + . . . + sin αn )]

×[cos α1 + . . . + cos αn − i(sin α1 + . . . + sin αn )]


= (cos α1 + . . . + cos αn )2 + (sin α1 + . . . + sin αn )2 ∈ R+ .
Pe de altă parte

ω =n+2· cos(αi − αj ) ≤ n + 2(n − 1 + n − 2 + . . . + 1)
1≤i≤j≤n

n(n − 1)
=n+2· = n2 .
2
Deci ω ∈ [0, n2 ].

4.15. Deoarece |z1 | = |z2 | = |z3 | = 1 avem:

z1 = cos t1 + i sin t1 ; z2 = cos t2 + i sin t2 ; z3 = cos t3 + i sin t3 şi

z1 z2 = cos(t1 + t2 ) + i sin(t1 + t2 );
z2 z3 = cos(t2 + t3 ) + i sin(t2 + t3 );
z3 z1 = cos(t3 + t1 ) + i sin(t3 + t1 ).
Prin urmare:
( ) ( )
√ t1 + t2 + 2k1 π t1 + t2 + 2k1 π
z1 z2 = cos +i sin = cos θ1 +i sin θ1
2 2
( ) ( )
√ t2 + t3 + 2k2 π t2 + t3 + 2k2 π
z2 z3 = cos +i sin = cos θ2 +i sin θ2
2 2
( ) ( )
√ t3 + t1 + 2k3 π t3 + t1 + 2k2 π
z3 z1 = cos +i sin = cos θ3 +i sin θ3
2 2
unde k1 , k2 , k3 ∈ {0, 1}. Dar
√ √ √
|α1 z1 z2 + α2 z2 z3 + α3 z3 z1 |2 = (α1 cos θ1 + α2 cos θ2 + α3 cos θ3 )2

+(α1 sin θ1 + α2 sin θ2 + α3 sin θ3 )2


= α12 cos2 θ1 + α22 cos2 θ2 + α23 cos2 θ3 + 2α1 α2 cos θ1 cos θ2

135
+2α2 α3 cos θ2 cos θ3 +2α3 α1 cos θ3 cos θ1 +α12 sin2 θ1 +α22 sin2 θ2+α23 sin2 θ3

+2α1 α2 sin θ1 sin θ2 + 2α2 α3 sin θ2 sin θ3 + 2α3 α1 sin θ3 sin θ1

= 3 + 2α1 α2 cos(θ1 − θ2 ) + 2α2 α3 cos(θ2 − θ3 ) + 2α3 α1 cos(θ3 − θ1 ) ≤

3 + 2(α1 α2 + α2 α3 + α3 α1 ).
Luând α1 , α2 , α3 ∈ {−1, 1} astfel ı̂ncât α1 α2 + α2 α3 + α3 α1 = −1 (de
exemplu α1 = −1, α2 = −1, α3 = 1) rezultă că:
√ √ √
|α1 z1 z2 + α2 z2 z3 + α3 z3 z1 |2 ≤ 1 ⇒
√ √ √
|α1 z1 z2 + α2 z2 z3 + α3 z3 z1 |2 ≤ 1.

4.16. Fie A, B ∈ C(O, 1) punctele care au afixele z1 şi z2 .

Atunci există un unic punct C pe A


M
dreapta AB de afix az1 +bz2 cu a, b ∈ R B
şi a + b = 1. Evident OC = |az1 + bz2 |.
Inegalitatea din enunţ revine la a arăta O C
că OC ≥ OM , ceea ce este evident ı̂n
baza faptului că OA = OB.

4.17. Avem echivalenţele:



3 1 2
z + 1 ≤ 2 ⇔ z + · z +
1
− 1 ≤2 ⇔
z3 z z2

( )

z + 1 1 2

· z + − 3 ≤ 2.
z z
( )2
1 2
1 1
Dacă z + > 2 atunci cum z + − 3 > z + − 3 va rezulta
z z z
că ( )2

1 1
z + · z + − 3 > 2 · 1 = 2
z z

1

ceea ce este ı̂n contradicţie cu inegalitatea din enunţ. Deci z + ≤ 2.
z

136
4.18. Avem: |ω| ≤ 1 ⇔ |ω|2 ≤ 1 ⇔ ω · ω ≤ 1 ⇔

z−u z−u
v· ·v· ≤1
1−u·z 1 − uz

⇔ v · v(z − u)(z − u) ≤ 1 − uzuz + uu + zz ⇔


zz + uu − 1 − uu · zz ≤ 0 ⇔
|z|2 + |u|2 − 1 − |u|2 · |z|2 ≤ 0 ⇔ (|z|2 − 1)(1 − |u|2 ) ≤ 0.
Cum |u| ≤ 1 ultima inegalitate este echivalentă cu |z|2 ≤ 1 adică |z| ≤ 1.

4.19. 2 = |1 + z + 1 − z| ≤ |1 + z| + |1 − z| = |1 + z| + |1 + z 3 − (z + z 3 )|

≤ |1 + z| + |1 + z 3 | + |z| · |1 + z 2 | = |1 + z| + |1 + z 2 | + |1 + z 3 |.

Egalitatea are loc dacă şi numai dacă z = −1.


Interpretare geometrică: Dacă dintr-un
punct B al unui cerc se construiesc trei A2
A1
arce congruente BA1 , A1 A2 şi A2 A3 A3
şi dacă A este punctul diametral opus
lui B, atunci suma lungimilor vectorilor A B
AA1 , AA2 , AA3 este mai mare decât di- O
ametrul cercului, egalitatea având loc
dacă şi numai dacă A1 = A.

4.20. Dacă notăm |1 + z| = u atunci u ∈ [0, 2],



1

|1 + z | = |z| · z + = |z + z| = 2|Re z|.
2
z

Cum u2 = (1 + z)(1 + z) = 1 + 2Re (z) + |z|2 = 2 + 2Re (z) deducem √


că 2Re (z) = u2 − 2. Inegalitatea din enunţ devine: u + |u2 − 2| ≥ 2
când u ∈ [0, 2] care este evidentă. Egalitatea are loc dacă şi numai dacă
z = ±i.

1 1 1 1 1

4.21. z − = |2z − 1| = |z + z − 1| ≤ [|z| + |z − 1|] < (1 + 1) = 1 ⇒
2 2 2 2 2


z − 1 < 1.
2

137
4.22. Din inegalitatea |z1 + z2 | ≤ |z1 | + |z2 | avem:

a = |z1 z2 + z2 z3 + z3 z1 | ≤ |z1 ||z2 | + |z2 ||z3 | + |z3 ||z1 |.

Cum b = |z1 ||z2 ||z3 | > 0 avem:

a 1 1 1
≤ + + (1)
b |z1 | |z2 | |z3 |

3b
Dacă |zk | > pentru orice k ∈ {1, 2, 3} atunci
a
1 1 1 a a
+ + <3· =
|z1 | |z2 | |z3 | 3b b

ceea ce contrazice inegalitatea (1). În consecinţă există k ∈ {1, 2, 3} astfel


3b
ı̂ncât |zk | ≤ .
a
4.23. Fie A1 , A2 , A3 , A4 punctele de afixe z1 , z2 , z3 , z4 .
Condiţiile din enunţ impun figura. Fie
P mijlocul coardei A1 A4 şi Q mijlocul Q A2
A3
coardei A2 A3 . Evident OP ≤ OQ
A4 P
de unde, ţinând seama de faptul că A1
z1 + z4
afixul lui P este iar a lui Q
2 O
z2 + z3
este deducem inegalitatea din
2
enunţ.

4.24. Fie A, B, C punctele de pe cercul C(O, r) care au afixele z1 , z2 , z3 . Cum


afixul ortocentrului △ABC este z1 + z2 + z3 , expresia din enunţ devine:
√ √
9r2 − OH 2 9r2 − 9r2 + a2 + b2 + c2 a2 + b2 + c2 4 3S 3
= = ≥ =
abc abc abc 4rS r

(am folosit inegalitatea ”În orice triunghi ABC: a2 + b2 + c2 ≥ 4 3S”).

4.25. Cum z ∈ C∗ relaţia din enunţ se scrie:



z 2
+ = 2. (1)
2 z

138
z 2

Cum |u + v| ≥ |u| − |v| avem: 2 ≥ − de unde |z|2 − 4|z| − 4 ≤ 0.
2 √z
Dar |z + 4| = 4|z| deci |z + 4| ≤ 8(1 + 2). Cum
2 2

(z 2 + 4)(z 2 + 4) = |z|4 + 16 + 4(z 2 + z 2 ) = |z|4 + 16 + 8Re (z 2 )


deducem că
√ √
|z|4 + 8Re (z 2 ) ≤ 64(1 + 2)2 − 16 = 16(11 + 8 2)
adică inegalitatea din enunţ.
4.26. Fie n ∈ N, n multiplu de 4 şi z0 , . . . , zn ∈ A, oricare două diferite ı̂ntre
ele. Deoarece zk ∈ A ⇒ zk4 ∈ N pentru orice k = 0, n şi deoarece
numerele sunt distincte două câte două rezultă pentru orice p ∈ N pot
exista cel mult 4 numere zk (k = 0, n) astfel ı̂ncât zk4 = p. În consecinţă:
n4
z04 + z14 + . . . + zn4 ≥ 04 + 14 + . . . + .
44
Cum
p(p + 1)(2p + 1)(3p2 + 3p − 1)
0 4 + 1 4 + . . . + p4 =
30
n
pentru orice p ∈ N şi ∈ N vom avea:
4
n(n + 2)(n + 4)(3n2 + 12n − 16)
z04 + z14 + . . . + zn4 ≥ .
3840
4.27. Fără a leza generalitatea problemei presupunem că
a1 = min{ak , k = 1, n} şi an = max{ak , k = 1, n}
de unde, pentru orice k ∈ {1, . . . , n} vom avea: a1 ≤ ak ≤ an şi
a1 − an ≤ 2ak − an − a1 ≤ an − a1 sau |2ak − an − a1 | ≤ an − a1 .
Cum z1 + . . . + zn = 0 avem:
1
|a1 z1 + . . . + an zn | = |2a1 z1 + . . . + 2an zn |
2
1
= |(2a1 − an − a1 )z1 + . . . + (2an − an − a1 )zn |
2
1 1
≤ |2a1 − an − a1 ||z1 | + . . . + |2an − an − a1 ||zn |
2 2
1
≤ (an − a1 )[|z1 | + . . . + |zn |]
2
ceea ce trebuia demonstrat.

139
4.28. Un calcul simplu conduce la:
√ √
| z 2 − 1 − z| + | z 2 − 1 + z| = |z − 1| + |z + 1|.
Folosind inegalitatea modulului pentru sume:
|z − 1| + |z + 1| ≤ 2(1 + |z|) (1)
Pe de altă parte:
√ √
|z − 1| + |z + 1| = (x − 1)2 + y 2 + (x + 1)2 + y 2

(x − 1)2 + y 2 + (x + 1)2 + y 2 √
≤2 = 2 x2 + y 2 + 1 (2)
2
Din (1) şi (2) deducem partea a doua a inegalităţii. Pe de altă parte:
|z − 1| + |z + 1| ≥ 2|z| (3)
Cum √ √
|z − 1| + |z + 1| = (x − 1)2 + y 2 + (x + 1)2 + y 2
|x + y − 1| |x + 1 + y| |x + y − 1| + |x + y + 1|
≥ √ + √ = √ (4)
2 2 2
Din (3) şi (4) obţinem prima parte a inegalităţii.
4.29. Fie ω ∈ C o rădăcină nereală a ecuaţiei. Atunci:
ω = r(cos α + i sin α) cu α ∈ (0, 2π) \ {π}.
Atunci:
ω n + nω + a = 0
de unde
rn sin nα + rn sin α = 0,
adică
rn−1 sin nα = −n sin α
de unde
rn−1 | sin nα| = n| sin α|.
Cum pentru n ≥ 2 şi α ∈ (0, 2π) \ {π} avem
| sin nα| < n| sin α|
(ceea ce se poate arăta prin inducţie), de unde
n| sin α| < rn−1 n| sin α| adică rn−1 > 1 ⇔ r > 1.

140
4.30. Fie ω o soluţie a ecuaţiei şi r = |ω|. Atunci

2n−1 (ω n + 1) = (ω + 1)n şi 2n−1 (ω n + 1) = (ω + 1)n ,

de unde
2n−1 (r2n + ω n ) = (r2 + ω)n .
Scăzând aceste relaţii obţinem:

2n−1 (r2n − 1) = (r2 + ω)n − (ω + 1)n sau


n
2 n−1
(r 2n
− 1) = (r − 1)
2
(r2 + ω)n−k (ω + 1)k−1 (1)
k=1

rn − 1
Presupunem prin absurd că r ̸= 1. Se observă că: > 0. Relaţia
r−1
(1) devine:

2n − 1 ∑n
r
2n−1 2 = (r2 + ω)n−k (ω + 1)k−1
r −1
k=1


n ∑
n
≤ 2
(r + r) n−k
(r + 1) k−1
= (r + 1) n−1
rn−k
k=1 k=1

de unde
r2n − 1 n−1 r − 1
n
2n−1 ≤ (r + 1) .
r2 − 1 r−1
rn − 1
Împărţind prin se obţine:
r−1

2n−1 (rn + 1) ≤ (r + 1)n .

Însă pentru orice x ∈ R∗+ avem:

(x + 1)n ≤ 2n−1 (xn + 1),

având egalitate dacă x = 1. Aşadar

2n−1 (rn + 1) = (r + 1)n , de unde r = 1.

ceea ce contrazice presupunerea noastră. Deci r = 1.

141
4.31. Fie zk = ak + bk i, k = 1, n,

M = {z1 , . . . , zn }, M1 = {z ∈ M, Re (z) ≥ 0},

M2 = {z ∈ M, Re (z) ≤ 0}, M3 = {z ∈ M, Im (z) ≥ 0},


M4 = {z ∈ M, Im (z) ≤ 0}.
Deoarece

n
|a + bi| ≤ |a| + |b| şi |zi | ≥ 1
i=1
rezultă că
∑ ∑ ∑ ∑
|ak | + |ak | + |bk | + |bk | ≥ 1
k∈M1 k∈M2 k∈M3 k∈M4

deci cel puţin unul din cele patru numere este mai mare sau egal cu
1
, presupunem că primul este acesta (similar se demonstrează pentru
4
celelalte). Deoarece numerele din M1 au partea reală de acelaşi semn
vom avea:

∑ ∑ ∑
1
|ak | = ak deci ak ≥ .
k∈M1
k∈M1 4 k∈M1

Alegem J = {j ∈ N, zj ∈ M1 } şi astfel




∑ ∑ ∑ 1

zj ≥
Re (zj ) = ak ≥ .

j∈J j∈J k∈M1 4


n
4.32. Fie f : C → R, f (z) = |z − zk |2 . Avem
k=1


n ∑
n
f (z) = (z − zk )(z − zk ) = (|z|2 + |zk |2 − zzk − zzk )
k=1 k=1


n ∑
n ∑
n ∑
n
= n|z|2 + |zk |2 − z zk − z zk = n|z|2 + |zk |2 − nzz0 − nzz0 .
k=1 k=1 k=1 k=1
Deci

n
f (z0 ) = n|z0 |2 + |zk |2 − nz0 z0 − nz0 z0 .
k=1

142
Atunci

f (z) − f (z0 ) = n|z|2 − nzz0 − nzz0 + n|z0 |2 = n[zz − zz0 − zz0 + z0 z0 ]

= n[(z − z0 )(z − z0 )] = n|z − z0 |2 ≥ 0.

143
144
Capitolul 5

Inducţie. Combinatorică.
Şiruri

5.1 Enunţuri
5.1. Demonstraţi că oricare ar fi n ∈ N, n ≥ 1,
√ 1 1
2( n + 1 − 1) < 1 + √ + . . . + √ .
2 n

5.2. Demonstraţi că pentru oricare n ∈ N avem:


√ √ √
2 − 2 + 2 + ... + 2 1
√ √ <
√ 3
2 − 2 + 2 + ... + 2

unde numărătorul conţine n iar numitorul n − 1 radicali, n ≥ 2.

5.3. Fie a1 , a2 , . . . , a12 o progresie geometrică crescătoare cu termeni pozitivi.


Demonstraţi că a12 − a1 ≥ 11(a7 − a6 ). Generalizare.

5.4. Dacă a1 , a2 , a3 sunt termenii unei progresii aritmetice cu termeni pozi-


tivi, atunci
1 1 2

n a
+ √
n a
≥ √
n a
pentru orice n ∈ N, n ≥ 2.
1 2 3

5.5. Demonstraţi că oricare ar fi n ∈ N, n ≥ 2:


n(n−1)
2 2 ≥ n!

145
5.6. Demonstraţi inegalitatea:
1 · 3 . . . (2n − 1) 1
<√ , n ∈ N∗ .
2 · 4 . . . (2n) 2n + 1

(n + 1)2n
5.7. Să se demonstreze că ≥ 1 · 4 · 7 . . . (3n + 1), n ∈ N∗ .
n
√ √ √ √
√ √
5.8. Care din numerele 2 + 3 + 2 + . . . şi 3 + 2 + 3 + . . .
conţinând fiecare câte n semne radical este mai mare?
√ √
∗ √
5.9. Să se arate că oricare ar fi n ∈ N \ {1} avem: 2 · 3 · . . . n < 3.
5.10. Dacă a1 , a2 , . . . , an sunt numere reale pozitive şi
1
a1 · a2 · . . . · =
2n2 +n
atunci (1 + 22 a1 )(1 + 24 a2 ) · . . . · (1 + 22n an ) ≥ 2n .
an
5.11. Dacă x + y = a > 0, atunci xn + y n ≥ .
2n−1
( )n
1
5.12. Să se demonstreze că 2− ≥ n oricare ar fi n ≥ 1.
n
5.13. Să se demonstreze că
2 √ √ √ √ 2 √
n n + 1 < 1 + 2 + 3 + . . . + n < (n + 1) n.
3 3

n
5.14. Să se demonstreze inegalitatea (1 + Cnk ) ≥ 1 + 2n .
k=0


n √
k n(n + 1)
5.15. Să se demonstreze inegalitatea k! > .
2e
k=0

x4n + 9
5.16. Fie şirul (xn )n≥1 definit prin x1 = 2, xn+1 = oricare ar fi n ≥ 1.
10xn
4 5
Să se arate că < xn ≤ oricare ar fi n ≥ 2.
5 4
1
5.17. Fie şirul (xn )n∈N∗ definit astfel: xn+1 = − x2n + xn + 1, x1 ∈ (1, 2).
2
√ 1
Să se arate că |xn − 2| < n oricare ar fi n natural, n ≥ 3.
2

146
5.18. Fie numerele naturale x1 < x2 < . . . < xn . Arătaţi că:

(x1 + x2 + . . . + xn )2 ≤ x31 + x32 + . . . + x3n .

5.19. Fiind date numerele reale a1 , a2 , . . . , an se formează numerele

1∑
n
bk = ai , j = 1, n.
k
i=1

Dacă oricare ar fi n ∈ N∗ se notează



n ∑
n
Bn = (ai − bi )2 , Dn = (ai − bn )2
i=1 i=1

demonstraţi că oricare ar fi n ∈ N∗ avem inegalităţile Bn ≤ Dn ≤ 2Bn .


1 1 1 1 1
5.20. Fie hn = 1 + + . . . + , Arătaţi că: 2 + 2 + . . . + < 2.
2 n h1 2h2 nh2n
√ √ √ 6n!
5.21. Dacă n ∈ N, n ≥ 6, atunci 1 · 2! · 3
3! · . . . · n
n! ≤ .
2n

1 ∑
2n+1

2n+1+k ≥ A3n+2 .
A2n+1 2n+1
5.22. Dacă n este un număr natural, atunci:
2n + 1
k=1

5.23. Fie (an )n∈N∗ un şir de numere reale cu proprietatea că există M > 0 un
număr real, astfel ı̂ncât |an+1 − an | ≤ M oricare ar fi n ≥ 1. Să se arate
că oricare ar fi n ∈ N∗ are loc inegalitatea:
( )2

n
1 ∑
n
M2
a2k − ak ≤ n(n + 1)(n − 1).
n 12
k=1 k=1

5.2 Probleme propuse


1 1 3 2n − 1 1
 5.1. Să se arate că: √ < · · . . . · < √ , n ∈ N∗ .
2 n 2 4 2n 2n
 5.2. Demonstraţi inegalitatea:
√ √ √
1+ n √ √ 4n n + 3 n − 1
n ≤ 1 + 2 + ... + n ≤ , n ∈ N∗ .
2 6

147
 5.3. Demonstraţi inegalitatea:
n
C2n+k · C2n−k
n
≤ (C2n
n 2
) ,
unde k, n ∈ N, 0 ≤ k ≤ n.
 5.4. Dacă x ∈ R şi n ∈ N∗ , atunci C2n
n (1 + x + . . . + x2n−1 ) ≥ (x + 1)2n+1 .

 5.5. Să se demonstreze că oricare ar fi n ∈ N∗ avem:


√ √

1 + 2 + . . . + n < 2.

√ ( )3
√ p
 5.6. Dacă p, n ∈ N astfel ı̂ncât n ≤ p, atunci: p+1
p+1 −
An+1 p Anp ≤ .
p+1

n
 5.7. Fie n numere reale cu xi ≤ n. Atunci oricare ar fi k = 1, n există k
i=1
termeni dintre aceştia cu suma mai mare sau egală cu k.
 5.8. Se consideră şirul (an )n≥1 definit astfel an = an−1 + an−2 + an−3 oricare
ar fi n ≥ 4 cu proprietatea că:
( ) ( ) ( )
5 81 729
a1 ∈ 1, , a2 ∈ 2, , a3 ∈ 4, .
9 25 125
( )n−1 ( )n
9 9
Arătaţi că < an < , oricare ar fi n ≥ 4.
5 5
{ }
1 1
 5.9. Fie {a1 , a2 , . . . , an } = {b1 , b2 , . . . , bn } = 1, , . . . , .
2 n
4
Dacă a1 + b1 ≥ a2 + b2 ≥ . . . ≥ an + bn atunci am + bm ≤ oricare ar
m
fi m = 1, n.
 5.10. Care este mai mare 3133 sau 3331 ? 19751977 sau 19771975 ? Generalizare.
√ √ √ √ √
 5.11. Arătaţi că n + n − 1 + . . . + 2 + 1 ≤ n+1, oricare ar fi n ∈ N∗ .

 5.12. Dacă 0 ≤ a0 ≤ a1 ≤ . . . ≤ an şi x ≥ 0 atunci:


(x0 + a1 x + . . . + an xn )2
( )
1 1 1 2n−1
≤ (a0 + a1 + . . . + an ) 1 + x + . . . +
2 n
x + ... + x +x2n
.
2 n+1 2

148
 5.13. Fie {an }n∈N şi {bn }n∈N două şiruri de numere pozitive ordonate crescă-
tor. Să se arate că:

n ∑
n ∑
n ∏
n
b

n
b ∏
n
ak bn−k+1 ≤ ak bσ(k) ≤ ak bk , ai n−i+1 ≤ ai σ(i) ≤ abi i ,
k=1 k=1 k=1 i=1 i=1 i=1

unde σ este o permutare a numerelor {1, 2, . . . , n}.

 5.14. Să se arate că:

1 1 1 (n! · n + 1)2
+ + ... + ≤ .
1! − 2! 3! − 2! (n + 1)! − n! 4(n − 1)[(n + 1)! − 1]

 5.15. Fie şirul de numere reale (xn )n≥1 definit prin


n
x1 = 1 şi xn+1 = 1 + , pentru orice n ≥ 1.
xn
√ √
Arătaţi că n ≤ xn ≤ n − 1 + 1 pentru orice n ∈ N∗ .
1
 5.16. Fie şirul (xn )n≥1 definit prin xn+1 = xn + , x1 = 1. Arătaţi că:
xn
√ √ 1
2n ≤ xn ≤ 2n − 2 + 2 − √ , pentru orice n ∈ N, n ≥ 2.
2

 5.17. Se dau 2n numere reale x1 ≥ x2 ≥ . . . ≥ xn şi y1 ≥ y2 ≥ . . . ≥ yn . Dacă


z1 , z2 , . . . , zn este o permutare a numerelor y1 , y2 , . . . , yn să se demon-
streze inegalitatea:


n ∑
n
(xi − yi )2 ≤ (xi − zi )2 .
i=1 i=1

1 · 4 · 7 · . . . · (3n − 2)
 5.18. Dacă an = oricare ar fi n ∈ N∗ să se arate că:
2 · 5 · 8 · . . . · (3n − 1)

1 1
√ ≤ an ≤ √
3
.
3n + 1 3n + 1

 5.19. Fie (an )n∈N un şir astfel ı̂ncât 2an < an−1 + an+1 pentru orice n ∈ N∗ .
Să se arate că pentru orice p ∈ N∗ există q ∈ N∗ , astfel ı̂ncât ap < aq .

149

n
pk − 2
 5.20. Dacă an = , p ∈ N∗ , să se arate că
pk − 1
k=1

1 1
√ ≤ an ≤ √ .
pn + 1 p
(2p − 1)n + 1

 5.21. Dacă ai ∈ N∗ \ {1}, i = 1, n, atunci

(a1 + a2 + . . . + an )! ∏∑ n n
≥ max aσ(i) .
a1 !a2 ! . . . an !
k=2 i=k

( )2

n
 5.22. Dacă n ∈ N, n ≥ 7, atunci 2 n2 < Cnk
2
< 3n .
k=0

 5.23. Să se demonstreze că pentru orice n ∈ N, n ≥ 2 avem:


( n )( n ) √
∑ 1 ∑ 1 (1 + 3 3)2 2
−k
k k k ≤ √ ·n .
k=1 k=1
333

 5.24. Se consideră progresia aritmetică (an )n∈N∗ , unde 0 < a1 < a2 . Să se
arate că pentru orice p, n ∈ N∗ , 2 ≤ 2p ≤ n, există inegalitatea

n
1 n+1−p
≥ .
ak ak+2p−1 ap an+1
k=1

În ce caz avem egalitate?

 5.25. Să se arate că oricare ar fi numărul natural n ≥ 2 şi numerele reale
x1 , x2 , . . . , xn ∈ [0, 1], avem:


n ∑
xk − xk xl ≤ 1.
k=1 1≤k<l≤n


r
 5.26. Dacă n1 , n2 , . . . , nr ∈ N∗ , ni ≥ 2, pentru orice i = 1, r şi xi = n
i=1
arătaţi că
Cn21 + Cn22 + . . . + Cn2r ≤ Cn−r+1
2

cu inegalitate strictă dacă r > 1.

150
√ √
3 ∏
n
4k + 1 5
 5.27. Arătaţi că: < < .
4n + 3 4k + 3 4n + 5
k=1

 5.28. Fie x1 , . . . , xn numere reale distincte şi notăm cu



n
n
(1 + x2k ) 2
k=1
S= ,
P (k)

unde P (k) = |xk − xj |. Arătaţi că S ≥ n. În ce caz avem egalitate?
j̸=k

5.3 Soluţii
5.1. I. În mod evident
√ √ 1
k + 1 − k < √ , pentru orice k ∈ N∗ .
2 k
Dând lui k valori de la 1 la n şi ı̂nsumând aceste inegalităţi obţinem
( )
√ √ √ √ √ √ 1 1 1
2 − 1 + 3 − 2 + ... + n − 1 − n < 1 + √ + ... + √
2 2 n
adică
√ 1 1
2( n + 1 − 1) < 1 + √ + . . . + √
2 n
ceea ce reprezintă inegalitatea din enunţ.
II. Vom demonstra prin inducţie după n. Pentru n = 1 avem:
√ 2 √ √
(2 2 − 1) < 1 ⇔ √ <1 ⇔ 2< 2+1 ⇔ 1< 2
2+1
ceea ce este adevărat. Presupunem adevărată afirmaţia pentru n şi
demonstrăm că este adevărată pentru n + 1, adică:
√ 1 1 1
2( n + 2 − 1) < 1 + √ + . . . + √ + √ .
2 n n+1
1 1 1 1
Fie an+1 = 1 + √ + . . . + √ + √ . Atunci an+1 = an + √ .
√ 2 n n+1 n+1
Cum an > 2( n + 1 − 1) deducem că
√ 1
an+1 > 2( n + 1 − 1) + √ .
n+1

151
Vom dovedi ı̂n continuare că
√ 1 √
2( n + 1 − 1) + √ > 2( n + 2 − 1).
n+1
Această inegalitate este echivalentă cu
√ √ 1 2 1
2( n + 2 − n + 1) < √ ⇔ √ √ <√ ⇔
n+1 n+2+ n+1 n+1
√ √
n+1< n+2

ceea ce este adevărat şi deci an+1 ≥ 2( n + 2 − 1). Conform principiului
inducţiei deducem justeţea afirmaţiei din enunţ.
√ √ √
5.2. Fie an = 2 + 2 + . . . + 2 unde numărul radicalilor este n. Atunci
√ √
an+1 = 2 + an şi a1 = 2. Vom demonstra că an < 2, pentru orice
n ∈ N∗ . Pentru n = 1 afirmaţia este adevărată.√Presupunem √ an < 2 şi
demonstrăm că an+1 < 2. Într-adevăr an+1 = 2 + an < 2 + 2 = 2
deci an < 2. Conform principiului inducţiei deducem că an < 2 pentru
orice n ∈ N∗ . Inegalitatea din enunţ se mai scrie:
2 − an 1
< .
2 − an−1 3

Cum an , an−1 < 2 inegalitatea se mai scrie

3(2 − an ) < 2 − an−1 ⇔ 6 − 3an < 2 − an−1 ⇔


√ √
6 − 3 2 + an−1 < 2 − an−1 ⇔ 4 + an−1 < 3 2 + an−1 ⇔
a2n−1 − an−1 − 2 < 0 ⇔ (an−1 − 2)(an−1 + 1) < 0
ceea ce este adevărat.

5.3. Fie q raţia progresiei cu q > 1. Inegalitatea revine la:

a1 q 11 − a ≥ 11(a1 q 6 − a1 q 5 ) ⇔ q 11 − 1 ≥ 11(q 6 − q 5 ) ⇔

q 10 + q 9 + . . . + q + 1 ≥ 11q 5
care este adevărată ı̂n baza inegalităţii mediilor. Într-adevăr,

q 10 + q 9 + . . . + q + 1 √ √
> 11 q 1+2+...+10 = 11 q 55 = q 5 .
11

152
5.4. Din inegalitatea mediilor avem:
√ √ √
n a1 + n a3 √ √ a1 + a3 √
≥ 2n
a1 a3 şi n
a2 = n
≥ 2n a1 a3 .
2 2

Înmulţind aceste inegalităţi avem:


√ √ √
3 √ √
n a + n a
1 1 1 2
· n a2 ≥ 2n (a1 a3 )2 = n a1 a3 sau √ + √ ≥ √ .
2 n a
1
n a
3
n a
2

5.5. Vom demonstra afirmaţia prin inducţie după n. Pentru n = 2 inegali-


tatea revine la 2 ≥ 2! = 2 ceea ce este adevărat. Presupunând inegali-
tatea adevărată pentru n, arbitrar vom demonstra că este adevărată şi
pentru n + 1. Pentru n + 1 inegalitatea revine la:
n(n+1)
2 2 ≥ (n + 1)!
n(n+1) n(n−1)
Dar 2 2 = 2 2 · 2n ≥ n!2n = n!(1 + 1)n ≥ n!(n + 1) = (n + 1)! (Am
folosit ipoteza de inducţie şi inegalitatea lui Bernoulli). Deci afirmaţia
este adevărată pentru n + 1. Conform principiului inducţiei deducem că
n(n+1)
2 2 ≥ n! pentru orice n ∈ N, n ≥ 2.

5.6. Vom demonstra prin inducţie. Pentru n = 1 inegalitatea revine la


1 1
<√
2 3
care este adevărată. Presupunând
1 3 2n − 1 1
· · ... · <√
2 4 2n 2n + 1
adevărată pentru n ∈ N, n ≥ 1, arbitrar vom arăta că
1 3 2n − 1 2n + 1 1
· · ... · · <√ .
2 4 2n 2n + 2 2n + 3
1 3 2n − 1
Notând an = · · ... · atunci
2 4 2n

2n + 1 1 2n + 1 2n + 1
an+1 = an · <√ · = .
2n + 2 2n + 1 2n + 2 2n + 2

153

Cum (2n + 1)(2n + 3) < 2n + 2 (consecinţă a inegalităţii mediilor)
1
deducem că an+1 < √ adică afirmaţia este adevărată şi pentru
2n + 3
n + 1. Conform principiului de inducţie rezultă justeţea afirmaţiei din
enunţ.
Observaţie. Se poate demonstra prin inducţie că
1 3 2n − 1 1
· · ... · <√ , pentru orice n ∈ N, n ≥ 2.
2 4 2n 3n + 1

5.7. Vom demonstra prin inducţie după n. Pentru n = 1 inegalitatea revine


la 22 ≥ 4 care este adevărată.
Presupunând inegalitatea adevărată pentru n, arbitrar, n ≥ 1 vom
demonstra că este adevărată şi pentru n + 1, adică:
(n + 2)2n+2
1 · 4 · 7 · . . . · (3n + 1)(3n + 4) ≤ .
n+1
Conform presupunerii avem:
(n + 1)2n
1 · 4 · 7 · . . . · (3n + 1)(3n + 4) ≤ · (3n + 4).
n
Rămâne să arătăm că
(n + 1)2n (n + 2)2n+2
· (3n + 4) ≤ .
n n+1
Această ultimă inegalitate este echivalentă cu:
( )2n+1
3n + 4 n+2
(3n + 4)(n + 1) 2n+1
≤ n(n + 2) ⇔
2n+2
≤ ⇔
n(n + 2) n+1
( )2n+1
3n + 4 1
≤ 1+ .
n(n + 2) n+1
Cum din inegalitatea lui Bernoulli avem
( )2n+1
1 2n + 1
1+ ≥1+ ,
n+1 n+1
2n + 1 3n + 4
iar 1 + ≥ deducem că inegalitatea este adevărată şi pen-
n+1 n+2
tru n + 1. În consecinţă, conform principiului inducţiei inegalitatea este
adevărată pentru orice n ∈ N∗ .

154
5.8. Fie an primul număr şi bn cel de-al doilea. Atunci
√ √
a1 = 2, b1 = 3 şi a2n+1 = 2 + bn , b2n+1 = 3 + an .

Demonstrăm prin inducţie că an < bn < an + 1 pentru orice n ∈ N∗ .


Se observă că pentru n = 1 inegalitatea are loc. Presupunând că ea este
adevărată pentru un n ∈ N∗ arbitrar, avem:

a2n+1 = 2 + bn < 3 + an = b2n+1

deci
an+1 < bn+1 şi (an+1 + 1)2 = a2n+1 + 2an+1 + 1

= 3 + bn + 2an+1 > 3 + an = b2n+1

adică an+1 +1 > bn+1 . Deci afirmaţia este adevărată pentru orice n ∈ N∗
şi ı̂n consecinţă an < bn pentru orice n ∈ N∗ .
√ √
√ √
5.9. Fie ak = k (k + 1) . . . (n − 1) n. Va trebui demonstrat că a2 < 3.
Dacă presupunem a2 ≥ 3 atunci vom demonstra că ak > k + 1. Într-
√ a2 ≥ 3 deci afirmaţia este adevărată pentru k = 2. Deoarece
adevăr,
ak = ak+1 k rezultă

a2k (k + 1)2 k 2 + 2k + 1
ak+1 = ≥ = > k + 2.
k k k

Aşadar, dacă a2 ≥ 3 ar rezulta an−1 ≥ n adică


√ √ √
(n − 1) n ≥ n ⇔ (n − 1) n ≥ n2

ceea ce este evident fals. Prin urmare a2 < 3.


a a
5.11. Fie x = + t şi y = − t. Atunci
2 2
(a )n ( a )n an an−1 an an−1
xn +y n = +t + − t = n +Cn1 n−1 t+. . .+ n −Cn1 n−1 t+. . .
2 2 2 2 2 2

an an−2 an
=2· n
+ 2Cn2 n−2 t2 + . . . ≥ n−1 .
2 2 2

155
( )n [ ( )]
1 1 n
5.12. 2− = 1+ 1−
n n
( ) ( ) ( )
1 1 2 1 n
=1+n 1− + Cn 1 −
2
+ ... + 1 −
n n n
( )
1
≥1+n 1− = 1 + n − 1 = n.
n
5.13. Vom demonstra dubla inegalitate prin inducţie după n. Pentru n = 1
2 √ 2
avem · 2 < 1 < · 2 care este evidentă. Admitem că este adevărată
3 3
dubla inegalitate pentru un n arbitrar şi demonstrăm că este adevărată
şi pentru n + 1. În acest caz avem de arătat că
2 √ √ √ √ √ 2 √
(n + 1) n + 2 < 1 + 2 + 3 + . . . + n + n + 1 < (n + 2) n + 1.
3 3
√ √ √
Fie an = 1 + 2 + 3 + . . . + n. Atunci

an+1 = an + n + 1.
2 √ 2 √
Cum an verifică n n + 1 < an < (n + 1) n deducem că
3 3
2 √ √ 2 √ √
n n + 1 + n + 1 < an+1 < (n + 1) n + n + 1.
3 3
Vom dovedi ı̂n continuare că
2 √ 2 √ √
(n + 1) n + 2 < n n + 1 + n + 1 (1)
3 3
şi
2 √ 2 √ √
(n + 2) n + 1 > (n + 1) n + n + 1 (2)
3 3
(1) este echivalentă cu
2√ 2 √
(n + 1)(n + 2) < n + 1 ⇔ 2 (n + 1)(n + 2) < 2n + 3 ⇔
3 3
4n2 + 12n + 8 < 4n2 + 12n + 9
care este evidentă.
(2) este echivalentă cu
2 2√ √
(n + 2) > (n + 1)n + 1 ⇔ 2n + 1 > 2 n(n + 1) ⇔
3 3

156
4n2 + 4n + 1 > 4n2 + 4n
care este de asemenea evidentă.
Din (1) şi (2) deducem că
2 √ 2 √
(n + 1) n + 2 < an+1 < (n + 2) n + 1
3 3
adică dubla inegalitate este adevărată pentru n+1. Conform principiului
inducţiei deducem justeţea inegalităţii din enunţ.

5.14. Din inegalitatea lui Bernoulli avem:



n ∑
n
(1 + Cnk ) ≥1+ Cnk = 1 + 2n .
k=0 k=0

( )k
k
5.15. Vom demonstra ı̂n prealabil că pentru orice k ∈ k! > N∗ ,
. Pentru
e
( )k
k
k = 1 inegalitatea este adevărată. Admitem k! > şi demonstrăm
e
( )
k + 1 k+1
(k + 1)! > . Avem
e
( )k
k
(k + 1)! = k!(k + 1) > (k + 1) .
e
Vom dovedi că:
( )k ( ) ( ) ( )
k k + 1 k+1 k + 1 k+1 1 k
(k + 1) > ⇔ e> ⇔ e> 1+
e e k k

k k
care este evidentă. În baza inegalităţii stabilite avem: k! > pentru
e
orice k ∈ N∗ , de unde

n √
k

n
k n(n + 1)
k! > = .
e 2e
k=1 k=1

5.16. Folosind inducţia deducem că xn > 0, pentru orice n ∈ N, n ≥ 1 şi apoi,
din inegalitatea mediilor avem:

x4n + 9 x3 3 3 3
xn+1 = = n+ + +
10xn 10 10xn 10xn 10xn

157

33 · x3n 2√ 4
≥44
4
= 27 > .
10 · xn
4 3 5 5
x4n + 9 (x2 − 1)(x2n − 9)
Cum xn+1 −xn = −xn = n prin inducţie deducem
10xn 10xn
5
că xn < pentru orice n ≥ 2.
4
5.17. Din formula de recurenţă avem xn+1 = f (xn ) unde f : R → R.
1 3 1
f (x) = − x2 + x + 1 = − (x − 1)2 .
2 2 2
3
Dar = f (1) > x2 = f (x1 ) > f (2) = 1.
2
( )
11 3 3
=f < x3 = f (x2 ) < f (1) = .
8 2 2
11 √ 3 √ 1 1
Cum < 2 < rezultă că |x3 − 2| < = 3 . Vom demonstra
8 2 8 2
că pentru orice n ∈ N, n ≥ 2 prin inducţie după n. Pentru n = 3 este
√ 1
evidentă conform celor de mai sus. Dacă |xn − 2| < n vom arăta că
2
√ 1
|xn+1 − 2| < n+1
2

√ √ 1 1 √ √
2
|xn+1 − 2| = xn − 2 + (2 − xn ) = |xn − 2||xn + 2 − 2|.
2 2
√ 1 √
Cum |xn − 2| < n deducem că |xn + 2 − 2| < 1 şi ı̂n consecinţă
2
√ 1 1 1
|xn+1 − 2| < · n · 1 = n+1 .
2 2 2
Deci afirmaţia din enunţ este demonstrată.
5.18. Dacă x1 = 0 atunci inegalitatea revine la o inegalitate de acelaşi tip cu
n − 1 variabile şi ı̂n consecinţă putem admite că x1 > 0. Vom demonstra
inegalitatea prin inducţie. Pentru n = 1 avem x21 ≤ x31 care este adevărat.
Admite
(x1 + . . . + xn )2 ≤ x31 + . . . + x3n
şi fie xn+1 ∈ N astfel ı̂ncât xn < xn+1 . Avem
xn (xn + 1) xn+1 (xn+1 − 1)
x1 + . . . + xn ≤ 1 + 2 + . . . + xn = ≤
2 2

158
de unde
2(x1 + . . . + xn )xn+1 + x2n+1 ≤ x3n+1 .
Cum (x1 + . . . + xn )2 ≤ x31 + . . . + x3n prin adunarea acestor inegalităţi
avem:

(x1 + x2 + . . . + xn + xn+1 )2 ≤ x31 + . . . + x3n + x3n+1 ,

adică inegalitatea este adevărată şi pentru n + 1.

5.19. Vom demonstra prin inducţie după n. Pentru n = 1 avem B1 = D1 . Pre-


supunem adevărată inegalitatea pentru un n arbitrar şi vom demonstra
că este adevărată şi pentru n + 1. Fie


n
f (x) = (x − ai )2 = n(x − bn )2 + f (bn )
i=1

şi se deduce imediat că

f (bn+1 ) = n(bn+1 − bn )2 + f (bn )

de unde
f (bn+1 ) − f (bn ) = n(bn+1 − bn )2 > 0.
Adunând ı̂n ambii membri ai inegalităţii Bn ≤ Dn pe (an+1 − bn+1 )2
obţinem:

Bn+1 ≤ Dn + (an+1 − bn+1 )2 + f (bn+1 ) − f (bn ) = Dn+1

deci Bn+1 ≤ Dn+1 .


Partea a doua a inegalităţii se demonstrează tot prin inducţie.

5.20. Pentru k ≥ 2 avem:


1 1 1 1
− = > .
hk−1 hk k · hk−1 · hk k · h2k

Însumând aceste inegalităţi de la k = 2 obţinem:


1 1 1 1
+ ... + < − <1
2h22 nh2n h1 h2n

de unde rezultă inegalitatea din enunţ.

159
√ k
5.21. Prin inducţie se arată că pentru orice k ∈ N, k ≥ 6, k
k! ≤ . În
2
consecinţă
∏n √ ∏
n
k n!
k! ≤
k
= n−5 (1)
2 2 5!
k=6 k=6
√ k+1
Pentru k ∈ {1, 2, . . . , 5} se verifică k
k! ≤ deci
2
√ √ √ √ 2·3·4·5·6
1· 2! · 3! · 4! · 5! ≤ (2)
25
Din (1) şi (2) deducem că

n √
n!
k! ≤ · 6.
k

2n
k=6

5.22. Fie funcţia f : [0, ∞) → R, f (x) = (x + 1)(x + 2) . . . (x + 2n + 1). Evident


f este convexă pe [0, ∞) şi ı̂n consecinţă:
 

2n+1 ∑
2n+1
 x k  f (xk )
 k=1  k=1
f ≤
 2n + 1  2n + 1

pentru orice x1 , . . . , xn ∈ [0, ∞). Luând xk = k se obţine


2n+1
f (k) ≥ (2n + 1)f (n + 1)
k=1

de unde rezultă inegalitatea din enunţ.

5.23. Condiţia din enunţ devine:

−M (n + 1) + M n ≤ an+1 − an ≤ M (n + 1) − M n pentru orice n ∈ N∗

sau
an+1 − M (n + 1) ≤ an − M n
şi
an+1 + M (n + 1) ≥ an + M n pentru orice n ∈ N∗ .
În consecinţă şirurile (bn )n≥1 şi (cn )n≥1 definite

bn = an − M n şi cn = an + M n

160
au monotonii diferite. Conform inegalităţii lui Cebâşev vom avea:
( n )( n )
∑n
1 ∑ ∑
bk ck ≤ bk ck
n
k=1 k=1 k=1

de unde se deduce inegalitatea din enunţ.

161
162
Capitolul 6

Funcţia exponenţială şi


logaritmică

6.1 Enunţuri
6.1. Care număr este mai mare: log3 4 sau log4 5?
6.2. Să se arate că: log2 3 + log3 4 + log4 5 > 4.

6.3. Demonstraţi inegalitatea: log17 71 > 7 17.
6.4. Arătaţi că dacă b > a > 1 şi k > 0, atunci: loga b > loga+k (b + k).
a2
6.5. Demonstraţi că dacă a, b ∈ (1, ∞) şi 1 < c ≤ , atunci:
b

lg a ≥ lg b · lg c.
√ √
6.6. Demonstraţi inegalitatea: a loga b ≤ ab.

3 2 √
6.7. Dacă a, b, c > 1, atunci: a loga b·logb c ≤ 3 abc.
6.8. Dacă a, b ∈ (0, 1) sau a, b ∈ (1, ∞), atunci:
(loga b)2 + (logb a)2 ≥ loga b + logb a.

6.9. Să se arate că pentru orice n ∈ N∗ are loc inegalitatea: n ≥ lg(n3 + 9).
6.10. Să se arate că pentru orice n ∈ N∗ are loc inegalitatea:
( )
1 1 1 1 1
1 + + + . . . − ln n ≥ 1+ .
2 3 n 2 n

163
6.11. Demonstraţi inegalitatea:

ln2 n > ln(n − 1) ln(n + 1) pentru orice n ∈ N, n ≥ 2.

2x x(x + 2)
6.12. Demonstraţi că pentru orice x ≥ 0 avem: ≤ ln(1 + x) ≤ .
x+2 2(x + 1)
2 101
6.13. Care număr este mai mare: sau ln ?
201 100
b−a b
6.14. Demonstraţi că dacă 0 < a < b, atunci: 2 · < ln .
b+a a
b b2 − a2
6.15. Demonstraţi că dacă 0 < a < b, atunci: ln < .
a 2ab
6.16. Să se determine m > 0, minim, cu proprietatea că inegalitatea:
x
≤ ln(1 + x)
1 + mx
are loc pentru orice x ≥ 0.

6.17. Demonstraţi că dacă x ≥ 0, a ≥ 1 şi m ∈ (0, 2], atunci:


mx
ln(x + a) ≥ .
1 + ma

1
6.18. Demonstraţi că dacă x ≥ 1, atunci: 1 − ≤ ln x ≤ x − 1.
x
( )
1 1
6.19. Arătaţi că pentru orice x > 0 are loc inegalitatea: ln 1 + > .
x x+1
x−1 x
6.20. Să se demonstreze că: √ ≤ ln x, pentru orice x ∈ (0, 1] şi ln x ≤
x e
pentru orice x > 0.

6.21. Să se arate că pentru 0 < a < b are loc inegalitatea:
√ b−a a+b
ab < < .
ln b − ln a 2

6.22. Pentru ce valori m > 0 inegalitatea ln(1 + x) ≥ x − mx2 , are loc pentru
orice x ≥ 0?

164
6.23. Determinaţi m ∈ R astfel ı̂ncât inegalitatea ln x ≤ mx să aibă loc pentru
orice x > 0.
a b a+b
6.24. Dacă a, b, c, d > 0, atunci: a · ln + b · ln ≥ (a + b) · ln .
c d c+d
6.25. Să se arate că dacă a, b > 0 şi a2 + b2 = 1, atunci:

a · ln a + b · ln b + (a + b) · ln(a + b) ≤ 0.

6.26. Se dau numerele pozitive M şi m1 , m2 , . . . , mn astfel ı̂ncât

M > m 1 + m2 + . . . + mn .

Arătaţi că:
m1 m2 m3 mn
+ + + ... +
M M − m1 M − m1 − m2 M − m1 − m2 − . . . − mn−1
M
< ln .
M − m1 − m2 − . . . − mn

6.27. Să se arate că pentru orice x > 0 are loc inegalitatea: ex−1 ≥ x. Când
are loc egalitatea?
x2
6.28. Să se arate că pentru orice x > 0 are loc inegalitatea: ex > 1 + x + .
2
6.29. Să se arate că pentru orice x ≥ 0 are loc inegalitatea: ex ≥ 1 + x2 .

6.30. Să se găsească ordinea dintre numerele xy şi y x dacă:


1) 0 < x < y < e; 2) e < x < y.

6.31. Dacă x, y > 0, atunci: xy + y x > 1.

6.32. Dacă x > 0, atunci: xx ≥ ex−1 .

6.33. Găsiţi minimul expresiei xx + y y când x, y ∈ (0, 1) şi x + y = 1.

6.34. Să se demonstreze inegalitatea xy−1 > y x−1 când 0 < x < y < 1 sau
1 < x < y.

6.35. 1) Demonstraţi că pentru orice n ∈ N∗ − {1} are loc inegalitatea:


( )n
n n2
> .
n−1 n2 − 1

165
2) Demonstraţi că pentru x, y > 0 are loc inegalitatea:
( ) ( )y
x + 1 y+1 x
≥ .
y+1 y
2 2 2
6.36. Dacă x, y, z ∈ R, atunci: 2x + 2y + 2z ≥ 2xy + 2yz + 2zx .

6.37. Să se demonstreze că pentru orice x, z, y > 0 avem:


x+y+z
xx · y y · z z ≥ (xyz) 3 .

6.2 Probleme propuse


 6.1. Arătaţi că log3 2 + log4 3 + log5 4 + log2 5 > 4.

 6.2. Demonstraţi că pentru orice x > 0, x ̸= 1 are loc inegalitatea:


x ln x 1
0≤ ≤ .
x −1
2 2

 6.3. Demonstraţi că pentru orice x > −2 are loc inegalitatea:


( )
1 x
ln 1 − + < 0.
x+3 (x + 2)(x + 3)

 6.4. Demonstraţi că pentru orice x > 0 are loc inegalitatea:

ex > 1 + (x + 1) ln(x + 1).

1+x
 6.5. Demonstraţi că pentru orice x ∈ (0, 1) avem: e2x < .
1−x
 6.6. Demonstraţi inegalitatea:
( ) ( )
1 1 1 1
1 + + ... + ln(n + 1) > 1 + + . . . + ln n,
2 n−1 2 n
pentru orice n ∈ N∗ − {1}.
n+1 n−1
 6.7. Fie x > 1 şi n ∈ N∗ . Arătaţi că xn − 1 ≥ n(x 2 −x 2 ).

 6.8. Să se demonstreze că dacă a ∈ Q ∩ (0, 1] şi x ∈ (0, 1) ∪ (1, ∞) atunci:
1 − xa (1 + x)a
≤ .
1−x 1+x

166
c c c c
 6.9. Fie b > a > 1 şi c ∈ R. Să se compare cb + ba cu aa + bb .

 6.10. Comparaţi numerele log1987 1988 şi log1988 1989.

( ( )2
y) y
 6.11. Dacă 0 < x < y < 1 şi x + y < 1 atunci ey < 1+ 1− .
x x+y−1

ln(1 + x) + ex (1 − x)
 6.12. Dacă x > 0, atunci < 1.
x+1

 6.13. Fie a, b, c, d ∈ R astfel ı̂ncât a ≥ b ≥ c ≥ d > 0. Arătaţi că:

an − bn + cn − dn ≥ (a − b + c − d)n , pentru orice n ∈ N∗ .

 6.14. Dacă a, b, c > 0, atunci

[( ) a+b ( ) b+c ( ) c+a ] 13 ( ) a+b+c


1 a+b 2 b+c 2 c+a 2 a+b+c 3
[a b c ] ≥
a b c 3 ≥
2 2 2 3

z y z x y z z
 6.15. Fie e ≤ x ≤ y ≤ z. Arătaţi că: max{xy , xz , y x , y z , z x , z y } = xy .

x
 6.16. Dacă x > 0 şi y satisface relaţia ey = , atunci x > 2y.
1 − e−x

1 x 1 1 1
 6.17. Dacă x > 0, atunci − < − x < .
2 8 x e −1 2

 6.18. Care număr este mai mare π π · ee sau eπ ?


n−1
1 √ ∑ 1 n
 6.19. Dacă n ∈ N, n ≥ 2, atunci ≤ ln n ≤ .
2k + 1 2k + 1
k=1 k=1

 6.20. Arătaţi că e + ln 4 > 4.

 6.21. Dacă α > 0 şi β ≥ 1 arătaţi că

( e )β·αβ β −αβ
( α )β·eβ β −eβ
≤ ee şi ≤ eα .
α e

167
6.3 Soluţii
3x 4
6.1. Fie log3 4 = x > 1 şi log4 5 = y > 1. Avem 3x = 4, 4y = 5 şi deci y
=
4 5
3x−1 16
sau y−1 = > 1. Rezultă că 3x−1 > 4y−1 şi cum x − 1, y − 1 > 0
4 15
obţinem că x − 1 > y − 1, de unde x > y, adică log3 4 > log4 5.

1
6.2. Deoarece 55 > 210 · 3 rezultă log2 5 > 2 + · log2 3. Notând log2 3 = a,
5
avem:
2 log 5 2 a 11 2
E = log2 3+log3 22 +log4 5 = a+ + 2 > a+ +1+ = a+ +1.
a 2 a 10 10 a
Vom arăta că:
11 2
a + + 1 > 4 ⇔ 11a2 − 30a + 20 > 0 ⇔
10 a
( √ )( √ )
15 − 5 15 + 5
11 a − a− > 0.
11 11

15 + 5
E suficient să arătăm că log2 3 = a > . Din 81 > 80 rezultă
11
1 log2 5
că 4 log2 3 > 4 + log2 5. Din log2 5 > 2 + log2 3 şi log2 3 > 1 +
5 4
30
obţinem log2 3 > . Rămâne de arătat că
19

30 15 + 5 √ √
> ⇔ 45 > 19 5 ⇔ 9 5 > 19 ⇔ 405 > 361,
19 11
ceea ce este evident.

7 3
6.3. Arătăm că 17 < < log17 71. Avem:
2
√ 3 37
⇔ 17 < 7 ⇔ 17 · 128 < 81 · 27 ⇔ 2176 < 2187
7
17 <
2 2
iar
3 3
< log17 71 ⇔ 17 2 < 71 ⇔ 173 < 712 ⇔ 4193 < 5041.
2

168
b b+k
6.4. Pentru b > a > 1 şi k > 0 vom avea > şi deci
a a+k
b b+k b+k
loga > loga > loga+k .
a a+k a+k
Rezultă că loga b − 1 > loga+k (b + k) − 1 şi deci loga b > loga+k (b + k).

6.5. Din enunţ rezultă că logaritmii ce apar ı̂n inegalitate sunt pozitivi.
Folosind faptul că funcţia logaritm zecimal este strict crescătoare şi
apelând la inegalitatea mediilor, vom avea:

2 lg a = lg(a2 ) ≥ lg(bc) = lg b + lg c ≥ 2 lgb · lg c.

Egalitatea va avea loc dacă şi numai dacă

a2 = bc şi b = c ⇔ a = b = c.

6.6. Avem: √ √ 1+loga b √ √


a loga b = a 1·loga b ≤ a 2 = aloga ab = ab.
Semnul egal se obţine pentru a = b.
Observaţie. Din inegalitatea demonstrată rezultă că:
√ √ √
a loga b + b logb a ≤ 2 ab ≤ a + b.

3

3 loga a+loga b+loga c
6.7. a log2a b·logb c =a loga a·loga b·loga c
≤a 3


3 √
3
= aloga abc
= abc.

Egalitatea are loc dacă şi numai dacă a = b = c.

6.8. Cu notaţia x = loga b > 0 inegalitatea din enunţ devine:


( )
1 1 1 2 1
x + 2 ≥x+ 2
⇔ x+ −2≥x+ ⇔
x x x x
( )( )
1 1
x+ +1 x + − 2 ≥ 0.
x x

1 1 1
Cum x > 0, deci x + + 1 > 0 şi x + ≥ 2 x · = 2, inegalitatea
x x x
rezultă.

169
6.9. Inegalitatea din enunţ este echivalentă cu 10n ≥ n3 + 9, pentru orice
n ∈ N∗ . Pentru n = 1 se obţine semnul egal. Demonstraţia inegalităţii
se face prin inducţie. Avem:

10n+1 = 10n · 10 ≥ 10(n3 + 9) ≥ (n + 1)3 + 9

căci ultima inegalitate se reduce la 9n3 + 80 ≥ 3n2 + 3n ceea ce este


evident:
9n3 + 80 = 3n3 + 3n3 + 3n3 + 80 ≥ 3n2 + 3n.
1 1 1
6.10. Arătăm că şirul an = 1 + + . . . + − ln n − este crescător şi cum
2 n 2n
1
a1 = rezultă inegalitatea din enunţ. Avem:
2
1 1 1
an+1 − an = − ln(n + 1) − + ln n +
n+1 2(n + 1) 2n
( ) ( )
1 1 1 1
= + − ln 1 + > 0,
2 n n+1 n
1
căci luând x = 1 + ı̂n inegalitatea de la 6.12 rezultă
n
( ) ( )
1 1 1 1
ln 1 + < +
n 2 n+1 n

şi deci an+1 > an .

6.11. Aducem inegalitatea dată la forma:

ln n ln(n + 1)
>
ln(n − 1) ln n

ln x
şi considerăm funcţia f : (2, ∞) → R, f (x) = . Avem:
ln(x − 1)
( )
ln(x − 1) ln x x−1
− x ln − ln(x − 1)

f (x) = x x − 1 =
x
< 0,
ln2 (x − 1) x(x − 1) ln2 (x − 1)
( )
x−1
ln <0 .
x
Prin urmare f este descrescătoare pe (2, ∞) şi deci f (n) > f (n + 1).

170
6.12. Inegalităţile sunt echivalente cu:

2(x − 1) x2 − 1
≤ ln x ≤ , pentru orice x ≥ 1
x+1 2x

şi apar din demonstraţiile problemelor 6.14 şi 6.15.


101 2
6.13. Avem ln = ln 1, 01 > după cum rezultă din inegalitatea de la
100 201
6.12 luând x = 100.
b x−1
6.14. Notând = x > 1 inegalitatea revine la ln x > 2 · . Avem:
a x+1
( )2
′ 1 x + 1 − (x − 1) 1 4 x−1
f (x) = − 2 · 2
= − = > 0,
x (x + 1) x (x + 1)2 x+1

2(x − 1)
unde am notat prin f funcţia f : [1, ∞) → R, f (x) = ln x − .
x+1
Deoarece f este strict crescătoare, din x > 1 rezultă f (x) > f (1) = 0.

1
6.15. Inegalitatea se reduce la a demonstra că funcţia f (x) = x − − 2 ln x
x
b
este pozitivă pentru x = > 1. Aceasta rezultă din faptul că
a

1 2 (x − 1)2
f ′ (x) = 1 + − = >0 şi f (1) = 0.
x2 x x2

1
6.16. Vom arăta că valoarea căutată este m = . Pentru aceasta arătăm că:
2
x
1) x ≤ ln(1 + x), pentru orice x ≥ 0.
1+
2
1
2) Dacă m < inegalitatea nu are loc pentru orice x ≥ 0.
2
2x
1) Considerând funcţia f : [0, ∞) → R, f (x) = ln(1 + x) − avem:
x+2

1 4 x2
f ′ (x) = − = ≥ 0,
1 + x (x + 2)2 (x + 1)(x + 2)2

deci f este strict crescătoare. Cum x ≥ 0 rezultă f (x) ≥ f (0) = 0.

171
x
2) Fie f : [0, ∞) → R, f (x) = ln(1 + x) − . Rezultă că:
1 + mx
( )
2 2m − 1
xm x +
1 1 m2
f ′ (x) = − = .
1 + x (1 + mx)2 (1 + x)(1 + mx)2

1 2m − 1 1 − 2m
Cum m < , ecuaţia x + 2
= 0 are soluţie x0 = > 0 şi
2 m m2
din tabelul de variaţie al funcţiei

x 0 x0 ∞
f ′ (x) − − 0 + +
f (x) 0 ↘ f (x0 ) < 0 ↗

rezultă că f (x) < 0 pe (0, x0 ).


mx
6.17. Fie f : [0, ∞) → R, f (x) = ln(x + a) − . Avem:
x + ma
1 ma x[x + ma(2 − m)]
f ′ (x) = −m· 2
= ≥0
x+a (x + ma) (x + a)(x + ma)2

căci 2 − m ≥ 0 iar x, m, a ≥ 0. Funcţia f fiind crescătoare rezultă că


f (x) ≥ f (0) = 0.

6.18. Inegalităţile rezultă considerând funcţiile


1
f, g : [1, ∞) → R, f (x) = ln x − 1 + şi g(x) = x − 1 − ln x
x
pentru care avem:
1 1 x−1 1 x−1
f ′ (x) = − 2 = ≥ 0, g ′ (x) = 1 − = ≥ 0.
x x x2 x x
Rezultă că f (x) ≥ f (1) = 0 şi g(x) ≥ g(1) = 0, pentru orice x ≥ 1.
1
6.19. Înlocuind x cu , inegalitatea se reduce la următoarea:
x
x
ln(1 + x) > , pentru orice x > 0.
x+1
2x x
Ea rezultă din 6.12, observând că ≥ .
x+2 x+1

172
x
6.20. Avem ln x ≤ ⇔ x − e ln x ≥ 0.
e
Fie f : (0, ∞) → R, f (x) = x − e ln x. Derivata
e x−e
f ′ (x) = 1 − =
x x
are rădăcina x = e care este punct de minim global pentru f . Rezultă
că f (x) ≥ f (e) = 0, pentru orice x > 0. Pentru a doua inegalitate
considerăm funcţia
x−1
g : (0, ∞) → R, g(x) = ln x − √ .
x
Deoarece
√ x−1
x− √ √
1 2 x ( x − 1)2
g ′ (x) = − =− √ ≤ 0,
x x 2x x

g este descrescătoare. Din x ≤ 1 rezultă g(x) ≥ g(1) = 0, pentru orice


x ∈ (0, 1].
b
√ b−a b b−a b −1
6.21. Avem ab < ⇔ ln < √ ⇔ ln < a√ . Notând
ln b − ln a a ab a b
a
b
= x2 > 1 ultima inegalitate se scrie
a
x2 − 1
2 ln x <
x
b 2(b − a)
şi a fost demonstrată la 6.15. Inegalitatea ln > apare ı̂n 6.14.
a b+a
6.22. Fie f : (−1, ∞) → R, f (x) = ln(1 + x) + mx2 − x. Avem

1 2mx2 + (2m − 1)x


f ′ (x) = + 2mx − 1 = .
1+x 1+x
Dacă 2m − 1 ≥ 0, atunci f ′ (x) > 0, pentru orice x ∈ (0, ∞) deci f
este strict crescătoare. Rezultă că f (x) > f (0) = 0, pentru orice x > 0.
1 − 2m
Dacă 2m − 1 < 0, f ′ admite rădăcinile x1 = 0 şi x2 = . Din
2m
tabelul de variaţie rezultă că x = 0 este punct de maxim local şi deci

173
( )
1 − 2m
f (x) < f (0) = 0 pentru x ∈ 0, . Aşadar enunţul are loc pentru
2m
1
m≥ .
2
ln x
6.23. Va trebui ca m ≥ , pentru orice x > 0. Funcţia f : (0, ∞) → R,
x
ln x 1 − ln x
f (x) = are derivata f ′ (x) = şi cum f ′ (x) > 0 pentru
x ′ x2
x ∈ (0, e), f (x) < 0 pentru x ∈ (e, ∞) rezultă că x = e este punct de
ln x 1 1
maxim pe (0, ∞). Deci: ≤ , pentru orice x > 0. Rezultă că m ≥ .
x e e
a b
6.24. Fie = x > 0 şi = y > 0. Avem a = cx şi b = dy iar inegalitatea se
c d
reduce la următoarea:
c d cx + dy cx + dy
· x · ln x + · y · ln y ≥ · ln .
c+d c+d c+d c+d
Considerăm funcţia f : (0, ∞) → R, f (x) = x ln x. Deoarece f ′′ (x) =
c
x−1 > 0 funcţia f este convexă pe (0, ∞) şi deci, luând λ = ∈
c+d
d
(0, 1), 1 − λ = şi x1 = x, x2 = y ı̂n proprietatea
c+d
λf (x1 ) + (1 − λ)f (x2 ) ≥ f (λx1 + (1 − λ)x2 )

(definitorie pentru o funcţie convexă), rezultă:


c d cx + dy cx + dy
· x · ln x + · y · ln y ≥ · ln .
c+d c+d c+d c+d

6.25. Inegalitatea este echivalentă cu


a b
· ln a + · ln b + ln(a + b) ≤ 0.
a+b a+b
Deoarece funcţia f : (0, ∞) → R, f (x) = ln x este concavă pe (0, ∞), are
loc inegalitatea

λf (x1 ) + (1 − λ)f (x2 ) ≤ f (λx1 + (1 − λ)x2 ), pentru orice x1 , x2 > 0.


a
Luând λ = şi x1 = a, x2 = b, rezultă:
a+b
( )
a b a b
ln a + ln b ≤ ln ·a+ ·b
a+b a+b a+b a+b

174
a2 + b2 1
= ln = ln = − ln(a + b).
a+b a+b
Observaţie.
( π) Din inegalitatea demonstrată rezultă că pentru orice α ∈
0, avem:
2
sin α · ln(sin α) + cos α · ln(cos α) + (sin α + cos α) · ln(sin α + cos α) ≤ 0.

1
6.26. Considerând funcţia f : [0, 1) → R, f (x) = ln −x = − ln(1−x)−x,
1−x
avem
1 x
f ′ (x) = −1= >0
1−x 1−x
1
şi deci f (x) > f (0) = 0, adică: ln > x, pentru orice x ∈ (0, 1).
1−x
m M
Aşadar dacă M > m > 0, atunci < ln . Definim
M M −m
Mk = M − (m1 + m2 + . . . + mk ) şi M0 = M (k = 1, 2, . . . , n).

Avem Mi − mi+1 = Mi+1 > 0 şi deci Mi > mi+1 > 0, de unde se obţine:
m1 m2 mn M0 M1 Mn−1 M0
+ + ... + < ln + ln + . . . + ln = ln ,
M0 M1 Mn−1 M1 M2 Mn Mn
ceea ce echivalează cu enunţul problemei.
6.27. Considerăm funcţia f : R → R, f (x) = ex−1 − x. Avem f ′ (x) = ex−1 − 1
şi deci f ′ (x) = 0 ⇔ ex−1 = e0 ⇔ x = 1. Din tabelul de variaţie al
funcţiei:

x −∞ 1 ∞
f ′ (x) − − 0 + +
f (x) ∞ ↘ 0 ↗ ∞

rezultă că f (x) ≥ f (1) = 0, pentru orice x ∈ R. Egalitatea are loc dacă
şi numai dacă x = 1.
x1 + x2 + . . . + xn
Aplicaţie. Fie x1 , x2 , . . . , xn > 0 şi A = . Avem:
n
x1 x1 xn xn
e A −1 ≥ , . . . , e A −1 ≥
A A
de unde rezultă
x1 +x2 +...+xn
−n x1 · x2 · . . . · xn
e A ≥ .
An

175
x1 + x2 + . . . + xn
Dar = n. Rezultă că
A
x1 x2 . . . xn
1≥ ⇔ x1 x2 . . . xn ≤ An ⇔
An
√ x1 + x2 + . . . + xn
n
x1 x2 . . . xn ≤ ,
n
adică inegalitatea mediilor. Semnul egal are loc dacă şi numai dacă
x1 x2 xn
= = ... = = 1 ⇔ x1 = x2 = . . . = xn .
A A A

x2
6.28. Fie f : (0, ∞) → R, f (x) = ex − 1 − x − . Avem
2
x
f ′ (x) = ex − 1 − iar
2
1 1 1
f ′′ (x) = ex −> 1 − = > 0, pentru orice x > 0.
2 2 2
Rezultă că f este funcţie crescătoare, deci f ′ (x) > f ′ (0) = 0. Obţinem

că f este crescătoare, deci f (x) > f (0) = 0, pentru orice x > 0.
ex
6.29. Considerăm funcţia f : [0, ∞) → R, f (x) = . Avem:
x2 + 1
ex (x2 + 1) − ex · 2x ex (x − 1)2
f ′ (x) = = ≥ 0, pentru orice x ∈ [0, ∞).
(x2 + 1)2 (x2 + 1)2

Rezultă că f (x) ≥ f (0) = 1, pentru orice x ≥ 0 deoarece f este funcţie


crescătoare.
1
6.30. Considerăm funcţia f : (0, ∞) → R, f (x) = x x . Avem:

( )′ 1
1 · x − ln x 1 − ln x
f ′ (x) = x
1 1 1
x ln x =x · x
x
2
= xx · ,
x x x2

deci f ′ (x) > 0 ⇔ x ∈ (0, e) şi f ′ (x) < 0 ⇔ x ∈ (e, ∞). Prin urmare:
0 < x < y < e implică xy < y x şi e < x < y implică xy > y x .

6.31. Inegalitatea este evidentă dacă x ≥ 1 sau y ≥ 1. Pentru x, y ∈ (0, 1) vom


demonstra inegalitatea ajutătoare:
x
xy > .
x + y − xy

176
Fie f : (0, 1] → R, f (x) = (x + y − xy)xy−1 − 1. Avem

f ′ (x) = (1 − y)xy−1 + (x + y − xy)(y − 1)xy−2 = y(x − 1)(1 − y)xy−2 ≤ 0

căci x − 1 ≤ 0. Rezultă că f este strict descrescătoare pe (0, 1] şi deci


x < 1 implică f (x) > f (1) = 0. Vom avea:
x y x+y
xy + y x > + = > 1.
x + y − xy x + y − xy x + y − xy

x−1
6.32. Avem xx ≥ ex−1 ⇔ x ln x ≥ x − 1 ⇔ ln x ≥ . Inegalitatea
x
x−1
rezultă considerând funcţia f : (0, ∞) → R, f (x) = ln x − pentru
x
x−1
care f ′ (x) = . Punctul x = 1 fiind punct de minim global pe (0, ∞),
x2
avem f (x) ≥ f (1) = 0.

6.33. Fie f : (0, 1) → R, f (x) = xx . Cum ln f (x) = x ln x, rezultă

f ′ (x) = f (x)(1 + ln x) şi


[ ]
f (x) 1
f ′′ (x) = f ′ (x)(1 + ln x) + = f (x) (1 + ln x)2 + > 0.
x x
f fiind funcţie convexă, vom avea:
( )
f (x) + f (y) f (x) + f (1 − x) x+1−x
= ≥f
2 2 2
( ) ( )1 √
1 1 2 1 1
=f = = =√ .
2 2 2 2

Minimul căutat este deci 2.

6.34. Avem:
ln x ln y
xy−1 > y x−1 ⇔ (y − 1) ln x > (x − 1) ln y ⇔ > .
x−1 y−1
ln x
Fie f : (0, 1) → R, f (x) = . Avem
x−1
x−1
− ln x
f ′ (x) = x <0
(x − 1)2

177
căci notând
x−1 1 1 1−x
g(x) = − ln x ⇒ g ′ (x) = 2 − = >0
x x x x2
deci g(x) < g(1) = 0. Obţinem că f este descrescătoare pe (0, 1) şi prin
urmare
ln x ln y
x < y ⇒ f (x) > f (y) ⇔ > .
x−1 y−1
La fel se demonstrează inegalitatea pentru 1 < x < y.
(x + 1)b+1
6.35. 2) Considerăm funcţia f : (0, ∞) → R, f (x) = , (b > 0).
xb
Avem: ( ) ( )
x+1 1 b−1 b
f ′ (x) = 1+ 1−
x x x
şi deci x = b este un punct de minim global pentru f . Rezultă f (x) ≥
f (b), pentru orice x > 0. Luând x > 0, b = y > 0 inegalitatea rezultă.
Punctul 1) rezultă din 2) pentru y = n − 1 şi x = n.

6.36. Avem:
2 2
√ √ √
2x + 2y ≥ 2 2x2 · 2y2 = 2 2x2 +y2 ≥ 2 22xy = 2 · 2xy

şi analog
2 2 2 2
2y + 2z ≥ 2 · 2yz , 2z + 2x ≥ 2 · 2zx .
Însumând aceste inegalităţi se obţine enunţul. Egalitatea are loc numai
dacă x = y = z.

6.37. Logaritmând ambii membri ai inegalităţii de demonstrat obţinem:


x+y+z
xx · y y · z z ≥ (xyz) 3 ⇔

3x ln x + 3y ln y + 3z ln z ≥ (x + y + z)(ln x + ln y + ln z) ⇔
(x − y)(ln x − ln y) + (y − z)(ln y − ln z) + (z − x)(ln z − ln x) ≥ 0,
ceea ce este adevărat deoarece a − b şi ln a − ln b au acelaşi semn.

178
Capitolul 7

Inegalităţi polinomiale

7.1 Enunţuri
7.1. Dacă a, b, c ∈ R∗ , astfel ı̂ncât a + b + c = 1, atunci
7
5(a2 + b2 + c2 ) + 18abc ≥ .
3

7.2. Găsiţi toate numerele a, b, c, d astfel ı̂ncât:


i) funcţia f (x) = 4x3 − dx pentru x ∈ [−1, 1] să ı̂ndeplinească |f (x)| ≤ 1;
ii) funcţia g(x) = 4x3 + ax2 + bx + c pentru x ∈ [−1, 1] să ı̂ndeplinească
|g(x)| ≤ 1.

7.3. Se consideră un interval [a, b] din axa reală cu b − a > 2 2. Să se de-
cidă dacă există un polinom de forma f (x) = x2 + px + q, p, q ∈ R cu
proprietatea |f (x)| ≤ 1, pentru orice x ∈ [a, b].

7.4. Fie a, b, c ∈ R astfel ı̂ncât a2 + c2 ≤ 4b. Să se arate că:

x4 + ax3 + bx2 + cx + 1 ≥ 0, pentru orice x ∈ R.

7.5. Fie z o rădăcină a ecuaţiei x4 + a1 x3 + a2 x2 + a3 x + a4 = 0 unde ak ∈ C,


|ak | ≤ 1, k = 1, 4. Să se arate că |z| < 2.

7.6. Fie ecuaţia x4 + a1 x3 + a2 x2 + a3 x + a4 = 0 cu a2 , a4 ∈ R+ şi a1 , a3


sunt numere reale de acelaşi semn. Să se arate că dacă |a31 | < |a3 | atunci
rădăcinile ecuaţiei date sunt egale ı̂n modul.

7.7. Demonstraţi că dacă a ∈ R şi a5 − a3 + a = 2 atunci 3 < a6 < 4.

179
) reale ale ecuaţiei 16x − 4x + x − 1 = 0
7.8. Să se arate că toate rădăcinile 5 3
(√ √
6
3 64
sunt ı̂n intervalul , .
2 2

7.9. Dacă P este un polinom de gradul n cu coeficienţi pozitivi, atunci oricare


ar fi numerele reale x1 , x2 , . . . , xm există inegalitatea

m

P (xi ) ≥ mP ( m x1 x2 . . . xm ).
i=1


n
7.10. Dacă ecuaţia ai xn−i = 0, a0 an ̸= 0 are toate rădăcinile reale şi strict
i=0
pozitive, atunci:
1 ∑
n
ak an−k ≥ C2n
n
.
a0 an
k=0

7.11. Fie polinomul P ∈ C[X], P (x) = xn + a1 xn−1 + . . . + an−1 x + an . Să se


arate că dacă rădăcinile sale verifică inegalitatea |xk | ≤ 1, pentru orice
k = 1, n, atunci |a1 | + |a2 | + . . . + |an | ≤ 2n .
7.12. Să se arate că, dacă x1 , x2 , . . . , xn sunt rădăcinile polinomului P ∈ C[X],
P (x) = xn + a1 xn−1 + . . . + an−1 x + an , atunci
√ √
|xk | ≤ |a1 | + |a2 | + . . . + n |an |, pentru orice k = 1, n.

7.13. Să se arate că ecuaţia xn − a1 xn−1 − a2 xn−2 − . . . − an−1 x − an = 0,


ai > 0, i = 1, n are o singură rădăcină pozitivă xn şi că pentru celelalte
rădăcini avem |xi | < xn , pentru orice i = 1, n − 1.
7.14. Fie P ∈ R[X]. Dacă rădăcinile polinomului sunt reale atunci:

(n − 1)[P ′ (x)]2 ≥ nP (x)P ′′ (x),

unde n este gradul polinomului.


7.15. Fie n ∈ N, n ≥ 3. Dacă P ∈ R[X], P (x) = xn+1 + a1 xn + . . . + an x + 1
are cel puţin o rădăcină pozitivă, atunci:

n(a21 + a22 + . . . + a2n ) ≥ 4.

7.16. Fie P ∈ R[X], P (x) = xn + a1 xn−1 + . . . + an−1 x + 1, n = 2k, k ∈ N şi


2
|ai | < . Atunci P (x) > 0, pentru orice x ∈ R.
n−1

180
7.17. Fie P un polinom cu coeficienţii ( 0√sau 1 şi P√(0))̸= 0. Să se arate că P
1 − 5 −1 + 5
nu are rădăcini ı̂n intervalul , .
2 2

7.18. Dacă toate rădăcinile ecuaţiei a0 xn + a1 xn−1 + . . . + an = 0 sunt pozitive,


atunci:
a1 an−1
≥ n2 .
a0 an


n
7.19. Fie polinomul P (x) = xn + ak xn−k , unde n este un număr natural
k=1
par, ak ∈ R, k = 1, n şi an > 0. Dacă

∑ ( √ )
n √
n
n
|ak | ≤ min (n − 1)ann−1 , (n − 1)an
n n

n−1 n−1
k=1

atunci P (x) ≥ 0, pentru orice x ∈ R.

7.20. Fie P ∈ C[X], P (x) = an xn + an−1 xn−1 + . . . + a1 x + a0 , an ̸= 0. Pentru


orice rădăcină α a lui P , avem:

|a0 |
≤ |α|.
|a0 | + max |ai |
i=1,n

7.21. Fie P ∈ C[X], P (x) = an xn + an−1 xn−1 + . . . + a1 x + a0 , an ̸= 0, cu


rădăcinile xi , i = 1, n. Să se arate că dacă |a1 | = |a0 |, atunci există
i ∈ {1, 2, . . . , n} astfel ı̂ncât |xi | ≤ n.

7.22. Fie P (x) = an xn + an−1 xn−1 + . . . + a1 X + a0 ∈ R[X] cu an > 0.


1) Să se arate că dacă toate rădăcinile lui P au partea reală strict nega-
tivă atunci ak ≥ 0 pentru orice k = 1, n.
2) Dacă ak ≥ 0, k = 1, n, este adevărat că toate rădăcinile polinomului
P (x) au partea reală strict negativă?

7.23. Fie ck un şir nedescrescător cu c0 = 0 şi ck − Bck−1 ≤ 1, k = 1, 2, . . .


unde B este constantă, 0 ≤ B ≤ 1. Să se demonstreze inegalitatea
( )2

n ∑
n
c3k xk ≤ ck xk , pentru orice x ≥ 1 şi n ≥ 1.
k=1 k=1

181
7.24. Fie polinomul P ∈ C[X], P (x) = a0 xn + a1 xn−1 + . . . + an cu rădăcinile
xj , j = 1, n. Să se demonstreze inegalitatea

ak
|xj | < 1 + max .
k=1,n a0

7.25. a) Să se găsească polinoamele P ∈ R[X] pentru care


|P (x)| ≤ |x|, pentru orice x ∈ R.

b) Fie Q ∈ R[X] un polinom cu toate rădăcinile reale. Să se găsească


polinoamele P ∈ R[X] pentru care |P (x)| ≤ |Q(x)|, pentru orice x ∈ R.
7.26. Fie P (x) = a0 + a1 x + . . . + an xn , ai ∈ R, i = 1, n. Dacă
min(a0 , a0 + a2 , a0 + a2 + a4 , . . .) > max(|a1 |, |a1 + a3 |, |a1 + a3 + a5 |, . . .)
atunci ecuaţia P (x) = 0 nu are rădăcini ı̂n intervalul [−1, 1].
7.27. Fie P un polinom cu coeficienţi numere reale pozitive şi 0 ≤ c ≤ a ≤ b.
Să se demonstreze inegalitatea:
P (b + c) − P (a + c) ≥ P (b) − P (a) ≥ P (b − c) − P (a − c).

7.28. Fie P un polinom, care nu este o constantă, cu coeficienţi ı̂ntregi, iar


n(P ) numărul tuturor numerelor ı̂ntregi diferite ı̂ntre ele pentru care
(P (k))2 = 1. Să se demonstreze că n(P ) − grad (P ) ≤ 2, unde grad (P )
ı̂nseamnă gradul polinomului P .
7.29. Se dă polinomul
P (x) = a1 x2n −3na1 x2n−1 +a2 x2n−2 +. . .+a2n−2 x2 −16na2n−1 x+9a2n−1
unde a1 , a2 , . . . , a2n−1 ∈ R şi a1 a2n−1 ̸= 0. Ştiind că toate rădăcinile
acestui polinom sunt ı̂n intervalul [1, 3], să se arate că toate rădăcinile
sunt ı̂ntregi.

7.2 Probleme propuse


 7.1. Se dă ecuaţia x3 + x − 1 = 0. Să se arate că:
a) Dacă ecuaţia din enunţ are soluţie reală, aceasta este unică.
b)
( Dacă
) ecuaţia are soluţie, aceasta este un număr iraţional din intervalul
2 3
, .
3 4

182
 7.2. Să se arate că o ecuaţie reciprocă de grad par are toate rădăcinile de
modul 1 dacă şi numai dacă rezolventa sa are toate rădăcinile reale sit-
uate ı̂n intervalul [−2, 2].

 7.3. Fie polinomul x4 −x2 +ax2 +bx+c, a, b, c ∈ R având rădăcinile complexe


nereale z1 , z1 , z2 , z2 . Dacă |z1 |2 = 2Re z2 atunci
√ polinomul are cel puţin
două rădăcini de modul cel puţin egal cu 2 − 1.

 7.4. Fie ecuaţia x3 − 2x2 + x − 1 = 0. Să se arate că:


1) ecuaţia are o singură rădăcină reală x1 ;
1 1
2) 0 < Re xi < şi √ < |xi | < 1, i ∈ {2, 3}.
2 2
 7.5. Se consideră polinomul P (x) = 4x3 + 8ax2 + 4bx + 1, a, b ∈ R. Să se
arate că:
i) Dacă x0 este o rădăcină reală a lui P , atunci x0 ≤ b2 − 2a.
ii) Dacă x1 , x2 sunt rădăcini distincte ale lui P , ambele rădăcini fiind
reale sau ambele complexe nereale, atunci x1 x2 ≥ b − a2 .

 7.6. Dacă a, b, c > 0, atunci ecuaţia x3 − (a2 + b2 + c2 )x − 2abc = 0 are o


rădăcină unică şi pozitivă. Arătaţi că:
2
(a + b + c) ≤ x0 < a + b + c.
3

 7.7. Rădăcinile polinomului P = X 3 + aX 2 + bX + c, P ∈ R[X] sunt ı̂n


intervalul [0, 1] şi au suma egală cu 2. Să se arate că: 8b + 9c ≤ 8.

 7.8. Dacă ecuaţia x3 + ax2 + bx + c = 0 admite rădăcinile reale x1 , x2 , x3


atunci
1 8
x41 + x42 + x43 ≥ a4 + ac − 2b2 .
5 5
În acest caz avem egalitate?

 7.9. Să se arate că rădăcinile x1 , x2 , x3 ale ecuaţiei 2x3 − x2 − 8x − 6 = 0


satisfac condiţiile:

3
a) 0 < Re xk < 1;
k=1

9 ∑ √
3
b) < |xk | < 3 + 2 12.
2
k=1

183
 7.10. Dacă ecuaţia x3 − x2 + ax − b = 0 are rădăcinile reale şi pozitive, să se
8
arate că a − b ≤ .
27
 7.11. Fie ecuaţia x3 − px2 + qx − r = 0 având rădăcinile x1 , x2 , x3 unde 0 <
x1 ≤ x2 ≤ x3 . Să se demonstreze inegalitatea:

x1 x2 x3 pq − 3r
+ + ≥ .
x2 x3 x4 2r

 7.12. Să se demonstreze că dacă x0 este o rădăcină reală a ecuaţiei

x4 + ax3 + bx2 + cx + 1 = 0

unde a2 − b2 + c2 = 4, atunci
{ }
a 1 c b
max x0 + , + ≤ 1 − .
2 x0 2 2

 7.13. Să se arate că, dacă polinomul a0 x2n + a1 x2n−1 + . . . + a2n ∈ Z[X] cu
a0 > 0, are toate rădăcinile reale şi distincte, situate ı̂n intervalul (0, 1),
atunci a0 ≥ 4n + 1.

 7.14. Să se arate că ecuaţia a0 xn + a1 xn−1 + . . . + an−1 x + an = 0 unde


0 < a0 ≤ a1 ≤ . . . ≤ an , are toate rădăcinile de modul mai mare sau egal
cu 1.

n
1
 7.15. Se consideră polinomul P (x) = xk . Să se demonstreze că
n+k+1
k=0
ecuaţia P (x2 ) = P 2 (x) nu are rădăcini reale.

 7.16. Să se arate că oricare ar fi polinomul P ∈ R[X],

P (x) = xn + a1 xn−1 + . . . + an−1 x + an

unde ai ∈ [−1, 1], i = 1, n, n ≥ 1, P (x) are toate rădăcinile de modul


strict mai mic decât 2.

 7.17. Se dă ecuaţia xn+1 + a1 xn−1 + a2 xn−2 + . . . + an = 0, unde ai , i = 1, n


sunt numere complexe şi fie xi rădăcinile ecuaţiei. Să se arate că dacă
r = max{|a1 |, . . . , |an |}, atunci |xi |(|xi | − 1) ≤ r.

184
 7.18. Fie P = X n − a1 X n−1 + a2 X n−2 − . . . + (−1)an un polinom cu coeficienţi
reali şi care are toate rădăcinile reale şi situate ı̂n intervalul [0, 1]. Să se
demonstreze inegalitatea:

ak − ak+1 + ak+2 − . . . + (−1)n−k an ≥ 0, pentru orice k, 1 ≤ k ≤ n.

 7.19. Se consideră ecuaţia cu coeficienţi complexi a0 xn + a1 xn−1 + . . . + an = 0


ı̂n care |a0 | > max(|a1 |, . . . , |an |). Să se arate că dacă această ecuaţie
admite o rădăcină φ astfel ı̂ncât |φ| > 1, atunci |φ| < 2.

 7.20. Fiind dat polinomul f (x) = a0 + a1 x + . . . + an xn cu coeficienţi reali,


x ≥ 1, an ̸= 0 se notează:
|a0 | + |a1 | + . . . + |an−1 |
r=
|an |
şi se cere să se arate că:
1) dacă r ≥ 1, polinomul f nu are nici o rădăcină reală ı̂n |x| > r;
2) dacă r ≤ 1, polinomul f nu are nici o rădăcină reală ı̂n |x| > 1.

 7.21. Fie ecuaţia Z n +Z n−1 +2Z n−2 +. . .+(n−1)Z +n = 0. Să se arate că toate
( )1
n · 2n−1 i
rădăcinile ecuaţiei sunt ı̂n modul mai mici decât max ,n∈
1≤i≤n Cni
N∗ \ {1}.

 7.22. Fie f un polinom


( ) cu coeficienţi reali pozitivi. Să se arate că dacă ine-
1 1
galitatea f ≥ este adevărată pentru x = 1, atunci ea este
x f (x)
adevărată pentru orice x ∈ R+ .

 7.23. Arătaţi că polinomul P (X) = X n + a1 X n−1 + . . . + an−1 X + 1 ∈ C[X]


nu are rădăcini pozitive dacă
√( )n−k
∑ n
|an−k | · k −1 ≤ n.
k

k
1≤k≤n−1


n
 7.24. Fie xi , i = 1, n rădăcinile polinomului P (x) = xn−i , P ∈ R[X]. Să
i=1

n
se arate că dacă xi ≤ 1, i = 1, n atunci a0 ai ≥ 0. În ce caz avem
i=1
egalitate?

185
 7.25. Se consideră şirul de polinoame definite astfel:

P1 (x) = 1, P2 (x) = x, Pn (x) = xPn−1 (x) − Pn−2 (x), n ≥ 3.

Să se arate că polinomul P1619 are două rădăcini reale x1 şi x2 astfel
ı̂ncât |x1 − x2 | < 10−3 .

 7.26. Determinaţi toate polinoamele de forma xn + a1 xn−1 + . . . + an cu ak ∈


{−1, 1} care au toate rădăcinile reale.

 7.27. Dacă rădăcinile zk , k = 1, n ale polinomului P ∈ C[X],


n
P (x) = xn + ak xn−k
k=1

sunt ı̂n discul {z ∈ C; |z| < 1} atunci




n ∑
n

|zk | + 1 + ak ≥ 1.

k=1 k=1

În ce caz avem egalitate?

 7.28. Fie polinomul P ∈ C[X], P (x) = a0 xn + a1 xn−1 + . . . + an cu rădăcinile


xj , j = 1, n. Să se arate că modulele acestora satisfac inegalităţile:

ak
|xj | < φ + max (φ > 0),
k−1
k=1,n a0 φ


ak
|xj | < 2 max n , pentru orice j = 1, n.
k=1,n a0

 7.29. Fie un polinom cu coeficienţi reali

P (x) = xn + an−1 xn−1 + . . . + a0 şi M = |a0 | + |a1 | + . . . + |an−1 | > 1.

Să se demonstreze că:


1) dacă x > M , atunci P (x) > 0,
2) dacă x < −M , atunci (−1)n P (x) > 0.

186
 7.30. a) Dacă x(n) este rădăcina pozitivă a ecuaţiei

xn − xn−1 − . . . − x − 1 = 0 (1)

atunci pentru orice n ∈ N, n ≥ 2 avem inegalităţile

2
2− < x(n) < 2 (2)
n+1

x(n) < x(n + 1). (3)

b) Dacă n ≥ 2 iar x1 , x2 , . . . , xn−1 sunt rădăcinile ecuaţiei (1) diferite de


x(n), atunci

1
√ < |xi | < 1, pentru orice i = 1, n − 1.
3

 7.31. Fie P (x) = a0 + a1 x + . . . + an xn un polinom de gradul n, ai ∈ R, având


toate rădăcinile reale. Să se arate că dacă an = −an−1 = 1, atunci pentru
orice x ≥ 1 + max(|a0 |, |a1 |, . . . , |an−1 |) avem:
( )
nx − 1 n
P (x) ≤ .
n

 7.32. Să se găsească polinoamele reciproce

P (x) = xn + a1 xn−1 + . . . + a1 x + 1 ∈ R[X]

care admite cel puţin o rădăcină pozitivă, iar expresia a21 + a22 + . . . + a2n
[2]
este minimă.

 7.33. Fie polinomul f (x) = a3 x3 + a2 x2 + a1 x + a0 astfel ı̂ncât |f (x)| ≤ 1,


pentru orice |x| ≤ 1. Să se demonstreze că există un număr M astfel
ı̂ncât |a3 | ≤ M oricare ar fi un polinom f care satisface condiţia de mai
sus. Determinaţi cel mai mic număr M care verifică condiţiile problemei.

7.3 Soluţii
7.1. Avem

a2 + b2 + c2 = (a + b + c)2 − 2(ab + bc + ca) = 1 − 2(ab + bc + ca)

187
şi astfel inegalitatea din enunţ devine:
7 5 4
5 − 10(ab + bc + ca) + 18abc ≥ sau (ab + bc + ca) − abc ≤ .
3 9 27
Dacă f este un polinom cu rădăcinile a, b, c şi a + b + c = 1 avem:

f (t) = (t − a)(t − b)(t − c) = t3 − t2 + (ab + bc + ca)t − abc

şi cum:
( )3 ( )2 ( ) ( )( )( )
5 5 5 5 5 5 5
− + (ab+bc+ca)−abc = f = −a −b −c
9 9 9 9 9 9 9
[( ) ( ) ( )]3 [ ]3
1 5 5 5 1 5 8
≤ −a + −b + −c = − (a + b + c) = 2
27 9 9 9 27 3 27
rezultă chiar inegalitatea din enunţ. Cazul de egalitate are loc pentru
1
a=b=c= .
3
Observaţie. Locul acestei probleme nu se justifică la acest capitol, dar
având ı̂n vedere metoda utilizată am plasat-o ı̂n acest capitol.
( )
1
7.2. a) Din condiţia |f (1)| ≤ 1 rezultă 3 ≤ d ≤ 5. Inegalitatea f ≤1
2
implică −1 ≤ d ≤ 3, deci d = 3, iar apoi se verifică uşor că

|f (x)| = |4x3 − 3x| ≤ 1, pentru orice x ∈ [−1, 1].

b) Dacă |g(x)| ≤ 1 atunci |g(−x)| ≤ 1, deci


1 1
|4x3 + bx| = |g(x) + g(−x)| ≤ (|g(x)| + |g(−x)|) ≤ 1.
2 2
De la punctul anterior rezultă b = −3, iar din inegalităţile g(1) ≤ 1,
( ) a+c ≤(
g(−1) ≥ −1 rezultă 0 şi a)+ c ≥ 0 deci a + c = 0. Analog din
1 1
inegalitatea g ≥ −1, g − ≤ 1 rezultă a + 4c = 0, de unde
2 2
a = c = 0.

7.3. Presupunem prin absurd că există un astfel de polinom. Cu notaţia:


b−a
xk = a + k · , k = 0, 2, avem:
2
f (x0 ) f (x1 ) f (x2 )
1= + + sau
(x0 − x1 )(x0 − x2 ) (x1 − x0 )(x1 − x2 ) (x2 − x1 )(x2 − x0 )

188
(b − a)2
= f (x0 ) − 2f (x1 ) + f (x2 ).
2
Aceasta implică:
(b − a)2 √
≤ |f (x0 )| + 2|f (x1 )| + |f (x2 )| ≤ 4 deci b − a ≤ 2 2
2
ceea ce este o contradicţie. În concluzie nu există funcţie polinomială f
cu proprietatea din enunţ.

7.4. Deoarece 4b ≥ a2 + c2 avem succesiv:


a2 2 c2 2
x4 + ax3 + bx2 + cx + 1 ≥ x4 + ax3 + x + x + cx + 1
4 4
( a )2 ( c )2
= x2 x + + x + 1 ≥ 0, pentru orice x ∈ R.
2 2
7.5. Dacă |z| ≤ 1 atunci |z| < 2, iar dacă |z| > 1 notăm q = 1/|z| şi avem
q ∈ (0, 1). Din z 4 = −(a1 z 3 + a2 z 2 + a3 z + a4 ) deducem:

|z|4 ≤ |a1 | · |z|3 + |a2 | · |z|2 + |a3 | · |z| + |a4 |


q 1
deci 1 ≤ q + q 2 + q 3 + q 4 < de unde q > deci |z| < 2.
1−q 2
7.6. Ecuaţia dată are rădăcini egale ı̂n modul dacă se poate descompune
astfel:

x4 + a1 x3 + a2 x2 + a3 x + a4 = (x2 − 2r cos α · x + r2 )(x2 − 2r cos β · x + r2 )

unde r este modulul rădăcinilor, deoarece din x2 − 2r cos α · x + r2 = 0


şi x2 − 2r cos β · x + r2 = 0 rezultă

x1,2 = r(cos α ± i sin α) şi x3,4 = r(cos β ± i sin β)

adică |x1 | = |x2 | = |x3 | = |x4 | = r. Efectuând calculele şi identificând


obţinem:

a1 = −2r(cos α + cos β), a2 = 2r2 (1 + 2 cos α cos β),

a3 = −2r3 (cos α + cos β), a4 = r4 .


Din prima şi a treia relaţie deducem:
a3 a1 a2 a1 a2 − 2a3
r2 = deci 1 + 2 cos α cos β = ⇔ cos α cos β = şi
a1 2a3 4a3

189

a1 a1
cos α + cos β = − ·
2 a3
adică cos α şi cos β sunt rădăcinile ecuaţiei:

a1 a1 a1 a2 − 2a3
t2 + ·t+ =0
2 a3 4a3

şi avem: √


1 3
a1 3
a1 4a1 a2
|t1,2 | = − ± − + 8
4 a3 a3 a3
 √ √ 

1  a31 a31 4a1 a2
< − + ± − + 8  .
4 a3 a3 a3

Din condiţia |a31 | < |a3 | rezultă:



3
a1 3 3 3
< 1 deci − a1 < 1 de unde a1 − 4a1 a2 + 8 < a1 + 8
a3 a3 a3 a3 a3

4a1 a2
deoarece − < 0 şi ţinând cont de condiţiile din enunţ a2 ∈ R+ şi
3
de faptul că a1 , a3 au acelaşi semn deducem:


a3 4a a
± 1

1 2
+ 8 < 3.
a3 a3

Deci |t1,2 | < (1 + 3)/4 = 1, de unde | cos α| < 1, | cos β| < 1 ceea
ce ı̂nseamnă că ecuaţia admite descompunerea cerută şi ı̂n consecinţă
problema este rezolvată. Pentru a1 = a3 = −a, a2 = 0, a4 = 1 se obţine
a doua problemă dată la admiterea ı̂n treapta a doua, sesiunea iulie 1983
profil matematică-fizică.

7.7. Din a5 − a3 + a = 2 rezultă a > 0. Avem a(a4 − a2 + 1) = 2, dar cum


a4 − a2 + 1 ≥ a2 pentru orice a ∈ R rezultă că 2 ≥ a3 deci a6 ≤ 4.
Cazul de egalitate are loc dacă a = 1, dar care nu verifică egalitatea
a5 − a3 + a = 2. Înmulţind relaţia din enunţ cu a obţinem:

a6 − a4 + a2 = 2a ⇔ a6 = a4 − a2 + 2a ⇔ a6 = a4 − a2 + 1 + 2a − 1

190
dar
( )
2 2 1
a4 − a2 + 1 = deci a6 = + 2a − 1 = 2 a + −1≥3
a a a

dar din nou inegalitatea este strictă.

7.8. Observăm că x = 0 nu este rădăcină a ecuaţiei şi ı̂mpărţind cu x3 avem


succesiv:
1 1 1 1 1
16x2 − 4 + 2
− 3 = 0 ⇒ 3 = 16x2 + 2 − 4 > 8 − 4 = 4 deci 6 > 16
x x x x x
de unde √
6
1 4 4
x < 6
= ⇔ x< .
16 64 2
Ecuaţia din enunţ se mai scrie:

x(1 − 4x2 + 16x4 ) − 1 = 0

de unde rezultă:
(4x2 )3 + 1 64x6 + 1 1 (2x − 1)2
x· − 1 = 0 ⇒ = ⇒ 64x 6
= +3>3
4x2 + 1 4x2 + 1 x x
√6
6 3 3
de unde x > deci x > şi astfel inegalitatea din enunţ este
64 2
complet demonstrată.

7.9. Avem:

m ∑
m
P (xi ) = (a0 xni + a1 xin−1 + . . . + an )
i=1 i=1
(m ) (m )
∑ ∑
= a0 xni + a1 xin−1 + . . . + man .
i=1 i=1

Ţinând cont de inegalitatea mediilor avem:


(m ) (m )
∑ ∑
a0 xni + a1 xin−1 + . . . + man
i=1 i=1
√ √
≥ ma0 ( m x1 x2 . . . xm )n + ma1 ( m x1 x2 . . . xm )n−1 + . . . + man

= mP ( m x1 x2 . . . xm ).

191
Observaţie. Folosind metoda inducţiei matematice şi inegalitatea medi-
ilor se poate obţine: Dacă P este un polinom de grad n cu coeficienţi
pozitivi, atunci oricare ar fi numerele pozitive x1 , x2 , . . . , xn există ine-
galitatea:
∏ n

P (xi ) ≥ P m ( m x1 x2 . . . xm ).
i=1

7.10. Vom arăta mai ı̂ntâi că:


ak · an−k
≥ (Cnk ), pentru orice k = 1, n.
a0 · an
Potrivit formulelor lui Viete avem:
∑ ak
x1 x2 . . . xk = (−1)k ·
a0
k

unde xi sunt rădăcinile ecuaţiei din enunţ, iar k, i ∈ {1, 2, . . . , n}. Din
a0 · an ̸= 0 avem:

x1 x2 . . . xn−k
k
∑ 1
=
x1 x2 . . . xn x1 x2 . . . xk
k

şi deci
an−k
∑ (−1)n−k ·
1 a0 an−k
= an
= (−1)k · .
x1 x2 . . . xk (−1)n · an
k
a0
Folosind inegalitatea dintre media aritmetică şi armonică:
( )( )
∑ ∑ 1
x1 x2 . . . xk ≥ (Cnk )2
x1 x2 . . . xk
k k

deoarece ı̂n fiecare sumă avem Cnk termeni. Rezultă deci:


ak · an−k
≥ (Cnk )2 , pentru orice k = 1, n.
a0 · an

Însumând obţinem:

n
ak an−k ∑
n
≥ (Cnk )2 = C2n
n
.
a0 an
k=1 k=1

192

7.11. Avem x1 x2 . . . xk = (−1)k ak şi deci
∑ ∑ ∑

|ak | = x1 x2 . . . xk ≤ |x1 x2 . . . xk | adică |ak | ≤ 1.

Cum suma considerată conţine Cnk termeni vom avea |ak | ≤ Cnk , k = 1, n
şi deci
∑n ∑n ∑
n
|ak | ≤ Cnk = 2n − 1, adică |ak | < 2n .
k=1 k=1 k=1
√ √
7.12. Fie M = |a1 | + |a2 | + . . . + n |an |. Pentru |x| > M avem:
( )
|a1 | |a2 | |an |
|P (x)| ≥ |x| 1 −
n
− 2 − . . . − n şi deci
M M M
( )n
|x|
|P (x)| > (M n − |a1 |M n−1 − |a2 |M n−2 − . . . − |an |).
M
Dar √ √
M n = M n−1 (|a1 | + |a2 | + . . . + n |an |)
√ √
= |a1 |M n−1 + |a2 | · M n−1 + . . . + n |an | · M n−1 şi
√ √
M n−1 k |ak | = M k−1 k |ak | · M n−k ≥ |ak | · M n−k
deci
M n > |a1 |M n−1 + |a2 |M n−2 + . . . + |an |
adică |P (x)| > 0 pentru |x| > M . Aşadar |xk | ≤ M , pentru orice k ∈
{1, 2, . . . , n}.
√ √
Observaţie. Notând M = max(|a1 |, |a2 |, . . . , n |an |) şi ţinând cont de
condiţiile problemei rezultă că |xk | ≤ nM .

7.13. Fie f (x) = xn −a1 xn−1 −a2 xn−2 −. . .−an−1 x−an . Cum f (0) = −an < 0
şi lim f (x) = +∞ rezultă că ecuaţia F (x) = 0 are o rădăcină pozitivă
x→∞
xn = a, a > 0. Vom arăta că pentru x > a, f (x) > 0, iar pentru
0 < x < a avem f (x) < 0, adică f (x) = 0 nu are o altă rădăcinăpozitivă.
Fie x > a deci x = a · k (k > 1) şi obţinem:
( )
a1 · an−1 a2 · an−2 an−1 · a an
f (x) = f (a · k) = k a −
n n
− − ... − − n
k k2 k k−1 k

> k n (an − a1 · an−1 − . . . − an−1 · a − an ) = 0.

193
Pentru 0 < x < a avem x = a · k (k < 1) şi avem:

f (x) = f (ak) < k n (an − a1 · an−1 − . . . − an−1 · a − an ) < 0

şi ı̂n consecinţă rădăcina pozitivă este unică. Fie r modulul uneia din
rădăcinile xi (i = 1, n − 1). Din

xn = a1 · xn−1 + a2 · xn−2 + . . . + an−1 · x − an

deducem că

rn < a1 · rn−1 + a2 · rn−2 + . . . + an−1 · r + an

adică f (r) < 0 dar cum r > 0 rezultă că r < a.

7.14. Fie xi , (i = 1, n) rădăcinile polinomului. Pentru x = xi inegalitatea este


evidentă. Pentru x ̸= xi ,
[ ]′
P ′ (x) ∑ 1 ∑
n n
P ′ (x) 1
= şi deci =− .
P (x) x − xi P (x) (x − xi )2
i=1 i=1

1
Punând = ai , i = 1, n avem:
x − xi

P ′ (x) = (a1 + a2 + . . . + an ) · P (x), iar

P ′′ (x) · P (x) − [P ′ (x)]2


= −(a21 + a22 + . . . + a2n ) sau altfel
P 2 (x)
P ′′ (x) · P (x) − (a1 + a2 + . . . + an )2 · P 2 (x)
= −(a21 + a22 + . . . + a2n ).
P 2 (x)
Astfel inegalitatea din enunţ devine:
 ( n )2 ( n )2 ( n )
∑ ∑ ∑
P 2 (x) (n − 1) ai − n ai + n a2i  ≥ 0 adică
i=1 i=1 i=1

 ( ) ( n )2 

n ∑
P 2 (x) · n a2i − ai  ≥ 0
i=1 i=1

unde al doilea factor este pozitiv datorită inegalităţii lui Cauchy-


Buniakovski-Schwarz.

194
7.15. Mai ı̂ntâi vom demonstra următoarea:
Lemă. Dacă q ∈ (0, 1), atunci:

n+1
1 + q + q2 + . . . + qn ≤ (1 + q n ). (1)
2

Pentru n = 1 inegalitatea (1) este adevărată. Presupunem inegalitatea


(1) adevărată pentru n−1 şi o vom demonstra pentru n. Astfel ne rămâne
să arătăm că:
n n+1
(1 + q n−1 ) + q n ≤ (1 + q n ).
2 2
Vom considera funcţia:

f (q) = nq n−1 − (n − 1)q n − 1, q ∈ (0, 1].

Cum f ′ (q) = n(n − 1)q n−2 (1 − q) şi cum f ′ (q) > 0, dacă q ∈ (0, 1) iar
f ′ (1) = 0 rezultă că q = 1 este punct de maxim. Cum f (1) = 0, rezultă
că f (q) ≤ 0 pentru orice q ∈ (0, 1] şi astfel (1) este demonstrată. Pentru
q = x2 , relaţia (1) devine:

n+1
x2 + x4 + . . . + x2n ≤ (1 + x2n ) − 1
2
şi vom arăta ı̂n continuare că:
n+1 n
(1 + x2n ) − 1 ≤ (1 + xn+1 )2 , pentru orice x ∈ [0, 1].
2 4
Considerăm funcţia:

f (x) = n(1 + xn+1 )2 − 2(n + 1)(1 + x2n ) + 4, pentru orice x ∈ (0, 1].

Avem:
f ′ (x) = 2n(n + 1)xn (1 + xn+1 − 2xn−1 ).
Pentru x ∈ (0, 1) şi n ≥ 3:
n+1 n+1 n+1
xn−1 ≤ x 2 sau 1 + xn+1 − 2xn−1 ≥ 1 + xn+1 − 2x 2 = (1 − x 2 )2 > 0

şi deci f ′ (x) > 0, iar f ′ (1) = 0 deci x = 1 este un punct de minim deci:
f (x) ≥ f (1) = 0, pentru orice x ∈ (0, 1]. Deci
n
x2 + x4 + . . . + x2n ≤ (1 + xn+1 )2 pentru orice x ∈ (0, 1]. (2)
4

195
Pentru x ≥ 1 inegalitatea de mai sus este adevărată deoarece se reduce
la cazul x ≤ 1 prin schimbarea de variabilă x = t−1 cu t ∈ (0, 1].
Fie x0 > 0 o rădăcină a lui P , deci

0 = P (x0 ) = xn+1
0 + a1 xn0 + . . . + an x) + 1 sau

xn+1
0 + 1 = −(a1 xn0 + . . . + an x0 ).
Aplicând inegalitatea lui Cauchy-Buniakovski-Schwarz

(xn+1
0 + 1)2 = (a1 xn0 + . . . + an x0 )2 ≤ (x2n 2 2 2
0 + . . . + x0 )(a1 + . . . + an )

şi ţinând cont de relaţia (2) deducem:


n n+1
(xn+1
0 + 1)2 ≤ (x + 1)2 (a21 + . . . + a2n ) sau n(a21 + a22 + . . . + a2n ) ≥ 4.
4 0

7.16. Punând |x| = a ≥ 0 obţinem


2
P (x) ≥ an + 1 − (an−1 + an−2 + . . . + a)
n−1
(a − 1)2
= [(n − 1)an−2 + 2(n − 2)an−3 + . . . + 2(n − 2)a + n − 1]
n−1
(a − 1)2 ∑
n−1
= k(n − k)ak−1 > 0,
n−1
k=1

dacă a ̸= 1. Cum P (1) > 0 şi P (−1) > 0 rezultă că P (x) nu are rădăcini
reale şi deci P (x) > 0, pentru orice x ∈ R.

7.17. Fie P (x) = a0 + a1 x + . . . + an xn unde ai ∈ {0, 1}, pentru orice i = 0, n.


Avem P (0) = a0 ̸= 0 şi deci a0 = 1. Deoarece pentru |x| < 1,
1
= 1 + x + x2 + . . . + xn + . . .
1−x
rezultă că
1
−2P (x) = −1+(1−2a1 )x+(1−2a2 )x2 +. . .+(1−2an )xn +xn+1 +. . .
1−x
unde |1 − 2ai | = 1, pentru orice i = 1, n. Aşadar

1 |x|

1 − x − 2P (x) + 1 ≤ |x| + |x| + . . . + |x| + . . . = 1 − |x|
2 n

196
deci ( ) ( )
1 1

|2P (x)| = 1 + − − 2P (x) + 1
1−x 1−x

1 1

≥ 1 + − − 2P (x) + 1
1−x 1−x

1 |x| 1 |x|
≥ 1 + − =1+ −
1−x 1 − |x| 1 − x 1 − |x|
deoarece x ∈ (−1, 1) avem

1 |x| 2(1 − |x| − |x|2 )


|2P (x)| ≥ 1 + − = , pentru |x| < 1.
1 + |x| 1 − |x| 1 − |x|2

Dacă x0 este o rădăcină cu proprietatea |x0 | < 1 atunci

2(1 − |x0 | − |x0 |2 )


0 = |2P (x0 )| ≥
1 − x20

5−1
şi deci |x0 | + |x0 | − 1 ≥ 0 adică |x0 | ≥
2 şi ı̂n consecinţă P nu are
( √ √ )2
1 − 5 −1 + 5
rădăcini ı̂n intervalul , .
2 2

7.18. Din inegalitatea dintre media aritmetică şi media armonică avem:
( n )( n )
an−1 a1 ∑ ∑ 1
· = xi ≥ n2 .
an a0 xi
i=1 i=1

Cazul de egalitate are loc dacă şi numai dacă x1 = x2 = . . . = xn .

7.19. Deoarece n este un număr par este suficient să studiem problema numai
pentru valori pozitive ale lui x. Astfel distingem două cazuri:
1) 0 ≤ x ≤ 1. Atunci:
( )

n
P (x) ≥ xn − |a1 |xn−1 − . . . − |an−1 |x + an ≥ xn + an − |ak | x
k=1

Dar
√ ( )n−1 √
an n
x + an ≥ n =x· · (n − 1)ann−1
n n n
xn
n−1 n−1

197
şi ı̂n consecinţă
( √ )
n ∑
n−1
P (x) ≥ (n − 1)an−1 − |ak | xn−1
n
n (2)
n−1
k=1

2) Dacă x > 1 avem


( n )

P (x) ≥ x − |a1 |x
n n−1
− . . . − |an−1 |x + an ≥ x + an −
n
|ak | xn−1
k=1

şi analog deducem


√( )n−1
xn n √
x + an ≥ n · an = xn−1 · (n − 1)an ,
n n n

n−1 n−1
deci ( )
n √ ∑
n−1
P (x) ≥ n
(n − 1)an − |ak | xn−1 (3)
n−1
k=1
Din inegalităţile (1), (2) şi (3) deducem că P (x) ≥ 0, pentru orice x ∈ R.
7.20. Distingem două cazuri:
i) dacă |α| ≥ 1 inegalitatea se verifică deoarece
|a0 |
< 1;
|a0 | + max |ai |
i=1,n

ii) dacă |α| < 1, notăm max |ai | = m şi avem:


i=1,n

an αn + an−1 αn−1 + . . . + a1 α + a0 = 0
de unde
|a0 | = | − an αn − an−1 αn−1 − . . . − a1 α|
≤ |α|(|an | · |α|n−1 + |an−1 | · |α|n−2 + . . . + |a1 |)
≤ |α| max |ai |(1 + |α| + |α|2 + . . . + |α|n−1 )
i=1,n

1 − |α|n |α| · m
= m|α| < ⇒
1 − |α| 1 − |α|
|a0 | − |a0 | · |α| < m|α| ⇔ |a0 | < |α|(|a0 | + m)
|a0 |
deci |α| ≥ .
|a0 | + m

198
7.21. Din relaţiile lui Viete rezultă:

1 1 ∑ 1
n
1
1= + + ... + ≤ .
x1 x2 xn |xi |
i=1

1 1
Dacă |xi | > n, pentru orice i = 1, n ⇒ < , pentru orice i = 1, n
|xi | n
de unde

n
1 1 1 1
1≤ < + + ... + = 1
|xi | n n n
i=1

(avem n termeni ı̂n ultima sumă). Deci presupunerea făcută este falsă şi
ı̂n consecinţă există i ∈ {1, 2, . . . , n} pentru care |xi | ≤ n.

7.22. 1) Fie rădăcinile reale x1 , x2 , . . . , xk şi y1 , y1 , y2 , y2 , . . . , yp , yp rădăcinile


din C \ R. Deci yj = αj + iβj , cu xi < 0, αj < 0 şi avem:


k ∏
p
P = (x − xi ) (x − αj − iβj )(x − αj + iβj )
i=1 j=1


k ∏
p
= (x + x′i ) [(x + αj′2 )2 + βj′2 ]
i=1 j=1

unde x′i = −xi > 0 şi αj′ = −αj > 0 şi ı̂n consecinţă P are coeficienţii
pozitivi.
2) Reciproca nu este adevărată. Fie de exemplu P (x) = x3 + 1.

7.23. Vom utiliza inducţia dublă. Pentru n = 1, inegalitatea (1) se transcrie

c21 · x ≤ c21 · x2 sau c21 · x(c1 − x) ≤ 0

care este adevărată deoarece c1 ≤ 1 şi x ≥ 1. Presupunând (1) adevărată


pentru k = 1, 2, 3, . . . , n va trebui să arătăm că
( n )2

n+1 ∑
n ∑
c3k ·x =
k
c3n+1 ·xn+1
+ c3k ·x ≤
k
c3n+1 ·x n+1
+ ck · x
k

k=1 k=1 k=1

(n+1 )2

≤ ck · xk
k=1

199
sau

n
2 ck · xk ≥ c2n+1 − cn+1 · xn+1 (n = 1, 2, 3, . . .) (2)
k=1

Pentru n = 1, inegalitatea (2) se transcrie c22 − c2 · x2 ≤ 2c1 · x. Dar


x ≥ 1, c22 − x2 c2 ≤ c22 − c2 şi ţinând cont că c2 − Bc1 ≤ 1 rezultă

c22 − c2 ≤ c1 · B · c2 ≤ c1 · B(1 + Bc1 ) ≤ c1 · B(1 + B) ≤ 2c1 ≤ 2c1 x

care este adevărată deoarece pentru 0 ≤ B ≤ 1 avem B(1 + B) ≤ 2.


Presupunând că inegalitatea (2) este adevărată pentru k = 1, . . . , n va
trebui să arătăm că:

n+1
2 ck · xk ≥ 2cn+1 · xn+1 + (c2n+1 − cn+1 · xn+1 ) ≥ c2n+2 − cn+2 · xn+2 ,
k=1

inegalitatea se transcrie astfel

xn+1 (xcn+2 + cn+1 ) ≥ c2n+2 − c2n+1 , n = 1, 2, . . .

Dar ţinând cont că B ≤ 1, −Bcn+1 ≥ −cn+1 şi cn+2 − cn+1 ≤ cn+2 −
Bcn+1 ≤ 1. Prin urmare:

c2n+2 − c2n+1 ≤ cn+2 + cn+1 ≤ xn+1 (xcn+2 + cn+1 ),

fiindcă x ≥ 1. Folosind următoarea:


Lemă. Dacă x, y, p ≥ 2, x > y atunci p(x−y)(xp−1 +y p−1 ) ≥ 2(xp −y p ).
(Exerciţiu)
vom da o generalizare la 7.23. Fie ck un şir nedescrescător cu c0 = 0 şi
ck − Bck−1 ≤ 1, k = 1, 2, . . . , n, unde B ∈ [0, 1]. Atunci pentru x ≥ 1 şi
p = 2, 3, . . . avem
( n )2
∑n
2p−1 k
∑ p−1
2 ck x ≤ p ck x k
, n = 1, 2, . . . (3)
k=1 k=1

Demonstraţia inegalităţii (3) se face la fel ca şi demonstraţia inegalităţii


(1) utilizând dubla inducţie. Pentru p = 2 din inegalitatea (3) rezultă
inegalitatea (1), iar din inegalitatea (1) rezultă inegalitatea:
( n )2
∑n ∑
A3k ≤ Ak , n = 1, 2, . . . (4)
k=1 k=1

200
unde Ak este un şir nedescrescător cu A0 = 0 şi Ak − Ak+1 ≤ 1, iar
pentru Ak = k, inegalitatea (4) dă cunoscuta identitate:
( )2

n ∑
n
k3 = k .
k=1 k=1

7.24. Avem ( )
n a1 an
P (x) = a0 x 1+ + ... +
a0 x a0 xn

ak
şi dacă notăm max = A, atunci pentru |x| > 1 vom avea
k=1,n a0


a1 an

|P (x)| ≥ |a0 x | 1 −
n
+ ... +
a0 x a0 xn
( ) ( )
a1 1 a1 1 A

≥ |a0 x | 1 − ·
n
+ . . . + · n ≥ |a0 x | 1 −
n
a0 |x| a0 |x| |x| − 1
adică ( )
|x| − 1 − A
|P (x)| ≥ |a0 xn |
|x| − 1
ceea ce ne arată că |P (x)| > 0 dacă |x| > 1 + A, afirmaţie echivalentă cu
cea pe care vrem s-o demonstrăm şi aplicând polinomului
( )n ( )
1 x a1 x n−1 an
Q(x) = P (x) = a0 + + ... + n
ρ ρ ρ ρ ρ

o inegalitate anterioară rezultă:



|x| ak ak
≤ 1 + max deci |x| < ρ + max .
ρ k k−1
k=1,n a0 ρ k=1,n a0 ρ

7.25. 1) Pentru x = 0 avem |P (0)| ≤ 0 şi deci P (0) = 0, iar


pentru x ̸= 0 avem
P (x)
|P (x)/x| ≤ 1, pentru orice x ∈ R deci lim ≤ 1 şi ı̂n consecinţă
x→∞ x
rezultă că grad P ≤ 1. Aşadar P (x) = ax + b şi cum P (0) = 0, rezultă că
P (x) = ax. Inegalitatea din enunţ se transcrie |ax| ≤ |x|, pentru orice
x ∈ R şi deducem că |a| ≤ 1. Aşadar polinoamele căutate sunt P (x) = ax
astfel ı̂ncât |a| ≤ 1.

201
2) Fie x1 , x2 , . . . , xk rădăcinile polinomului Q şi n1 , n2 , . . . , nk ordinele
lor de multiplicitate. Pentru x ̸= xi , i = 1, 2, . . . , k avem

P (x)
|P (x)/Q(x)| ≤ 1 şi deci lim ≤1
x→∞ Q(x)

adică grad P ≤ grad Q = n. Pentru x = xi inegalitatea din enunţ devine


|P (xi )| ≤ |Q(xi )| = 0 şi deci P (xi ) = 0 adică P (x) = (x − xi )P1 (x).
Deoarece rădăcina xi are pentru polinomul Q ordinul de multiplicitate
ni avem
Q(x) = (x − xi )ni Q1 (x), Q1 (xi ) ̸= 0.
Aşadar,
|(x − xi )P1 (x)| ≤ |(x − xi )ni Q1 (x)|, pentru orice x ∈ R,
pentru x ̸= xi avem:
|P1 (x)| ≤ |(x − xi )ni −1 Q1 (x)|
şi deci


lim P1 (x) ≤ lim |(x − xi )ni −1 Q1 (x)| = 0.
x→xi x→xi
x̸=xi x̸=xi
Cum funcţiile polinomiale sunt continue rezultă P1 (xi ) = lim P1 (x) = 0
x→xi
şi deci
P1 (x) = (x − xi )P2 (x) adică P (x) = (x − xi )2 P2 (x).
În mod asemănător se arată că
P (x) = (x − xi )ni Pni (x)
şi deci
P (x) = (x − x1 )n1 (x − x2 )n2 · . . . · (x − xk )nk R(x)
deoarece grad P ≤ grad Q rezultă că R(x) constant şi deci P (x) = aQ(x).
Deoarece |aQ(x)| ≤ |Q(x)| pentru orice x ∈ R rezultă |a| ≤ 1.
7.26. Din condiţia iniţială rezultă a0 > 0, a0 > −a1 , a0 + a2 > −a1 , a0 + a2 >
−a1 − a3 , . . ., şi deci


 a0 > 0


 a0 + a1 > 0
a0 + a1 + a2 > 0 (1)



 . . . . . . . . .

a0 + a1 + a2 + . . . + an > 0

202
Folosind inegalitatea lui Abel: Dacă b0 ≥ b1 ≥ . . . ≥ bn > 0 şi dacă
inegalităţile (1) au loc, atunci:

a0 b0 + a1 b1 + . . . + an bn > 0.

Punând x ∈ (0, 1] avem 1 ≥ x ≥ x2 ≥ . . . ≥ xn > 0 şi ı̂n consecinţă


a0 + a1 x + . . . + an xn > 0 adică P (x) > 0. Pentru x = 0 avem P (0) =
a0 > 0 şi deci P (x) > 0 pentru orice x ∈ (0, 1]. Dacă x ∈ [−1, 0] rezultă
analog că P (x) > 0 şi ı̂n consecinţă P (x) > 0 pentru orice x ∈ [−1, 1] şi
ı̂n concluzie ecuaţia P (x) = 0 nu are soluţii ı̂n intervalul [−1, 1].

7.27. Coeficienţii fiind numere reale pozitive, este suficient să demonstrăm
următoarea inegalitate:

(b + c)n − (a + c)n ≥ bn − an ⇔


n−1 ∑
n−1
n
b +c + n
Cnk ·b ·c
k n−k
+a ≥b +a +
n n n
Cnk ak cn−k + cn .
k=1 k=1

La ultima inegalitate am ţinut cont că bk ≥ ak pentru orice k =


1, . . . , n − 1. Cea de-a doua inegalitate se obţine din prima făcând
substituţiile: b − c = b′ , a − c = a′ .

7.28. Să presupunem că fiecare dintre polinoamele P (x) − 1 şi P (x) + 1 are cel
puţin trei rădăcini ı̂ntregi distincte, nici una dintre ele nefiind comună
celor două polinoame. Se notează cu a cel mai mic dintre aceste şase
numere ı̂ntregi şi se consideră, fără a restrânge generalitatea, că a este
rădăcina polinomului P (x) + 1. Este evident că polinomul P (x) + 1 se
poate scrie sub forma P (x)+1 = (x−a)Q(x), unde Q(x) este de asemenea
un polinom cu coeficienţi ı̂ntregi. Fie p, q, r cele trei rădăcini ı̂ntregi şi
diferite ale polinomului P (x) − 1, care sunt toate mai mari decât a, pe
baza modului ı̂n care a fost ales a. Dar P (x) − 1 = (x − a)Q(x) − 2, deci

2 = (p − a)Q(p) = (q − a)Q(q) = (r − a)Q(r),

iar p−a, q−a, r−a sunt numere ı̂ntregi pozitive diferite ı̂ntre ele. Rezultă
că cel puţin unul dintre ele este mai mare decât doi, ceea ce evident
contrazice faptul că acesta este divizor al numărului natural 2. Astfel
presupunerea făcută este falsă şi cel puţin una din ecuaţiile P (x) = 1
sau P (x) = −1 are cel mult două rădăcini ı̂ntregi. Ţinând seama şi de
faptul că numărul rădăcinilor fiecăreia din aceste două ecuaţii nu poate
depăşi grad (P ), rezultă inegalitatea cerută.

203
7.29. Conform relaţiilor lui Viete avem

x1 + x2 + . . . + x2n = 3n (1)

.........
∑ ( )
a2n−1 a2n−1
x1 x2 . . . x2n−1 = (−1)2n−1 −16n = 16n (2)
a1 a1
a2n−1
x1 x2 . . . x2n = 9 (3)
a1
Împărţind pe (2) la (3) obţinem
1 1 1 16n
+ + ... + = (4)
x1 x2 x2n 9

Înmulţind relaţiile (1) şi (4) obţinem


( )
1 1 1 16n2
(x1 + x2 + . . . + x2n ) + + ... + = (5)
x1 x2 xn 3

Se cunoaşte ı̂nsă că dacă ai ∈ [m, M ] atunci


( p )( p )
∑ ∑ 1 p2 (M + m)2
ai ≤ · (6)
ai 4 Mm
i=1 i=1

pentru cazul când p este număr par. Egalitatea se atinge pentru ai ∈


{M, m}. În cazul problemei luăm m = 1, M = 3, p = 2n, ai = xi şi
atunci
p2 (M + m)2 (1 + 3)2 4n2 · 16 16n2
· = (2n)2 = = .
4 Mm 4·3 4·3 3
Rezultă deci că ı̂n (6) are loc egalitate deoarece ı̂n (5) este egalitate şi
ı̂n consecinţă x1 , x2 , . . . , x2n ∈ {1, 3} deci toate rădăcinile polinomului P
sunt ı̂ntregi.

204
Capitolul 8

Inegalităţi trigonometrice

8.1 Enunţuri
[ π] x
8.1. Să se arate că dacă x ∈ 0, , atunci √ ≤ sin x ≤ x.
2 1 + x2

3 1 1
8.2. Să se arate că cos − sin > .
2 2 8
( π)
8.3. Demonstraţi că pentru x ∈ 0, are loc inegalitatea:
2
sin x 3
< < 1.
x 4 − cos x
( π)
8.4. Demonstraţi că pentru x ∈ 0, are loc inegalitatea:
2
2 cos x sin x
< .
1 + cos x x

8.5. Demonstraţi inegalităţile:

3 cos x sin x 3 ( π)
< < , unde x ∈ 0, .
1 + 2 cos x x 4 − cos x 2
( π) √ sin x
8.6. Să se arate că pentru x ∈ 0, are loc inegalitatea 3 cos x < .
2 x
π 3
8.7. Demonstraţi că pentru x ∈ (2, ∞) avem inegalitatea: sin >√ .
x x2 + 9

205
( π) 2
8.8. Demonstraţi că pentru x ∈ 0, , avem sin 2x < .
2 3x − x3
8.9. Să se
( arate că sin(cos x) < cos(sin x) şi sin(sin x) < cos(cos x) pentru
π)
x ∈ 0, .
2
8.10. Să se demonstreze inegalitatea:
√ [ π)
1 1 4 3
( π ) + ( π ) ≥ , pentru x ∈ 0, .
sin x + sin −x 3 3
3 3
sin x − sin y [ π]
8.11. Găsiţi minimul şi maximul expresiei unde x, y ∈ 0, şi
1 − sin x sin y 2
x ̸= y.
x
8.12. Demonstraţi că pentru x ∈ (0, 1) avem: arcsin x < .
1 − x2
[ √ )
π
8.13. Arătaţi că pentru orice x ∈ 0, are loc inegalitatea:
2

sin2 x ≤ sin(x2 ).

8.14. Să se arate că pentru orice x ∈ [0, π] are loc inegalitatea:

| sin(nx)| ≤ n sin x, pentru orice n ∈ N.

8.15. Să se arate că pentru orice x ≥ 0 şi n ∈ N avem:


x3 x5 x4n−1 x3 x5 x4n+1
x− + −...− ≤ sin x ≤ x − + −...+ şi
3! 5! (4n − 1)! 3! 5! (4n + 1)!
x2 x4 x4n−2 x2 x4 x4n
1− + − ... − ≤ cos x ≤ 1 − + − ... − .
2! 4! (4n − 2)! 2! 4! (4n)!
[ π)
8.16. Demonstraţi că pentru x ∈ 0, are loc inegalitatea:
2
2sin x + 2tg x ≥ 2x+1 .

8.17. Demonstraţi că:


tg x x ( π)
1) > , pentru orice x ∈ 0, ;
x sin x 2
sin x + tg x ( π)
2) > x, pentru orice x ∈ 0, .
2 2

206
[ π)
8.18. Demonstraţi că pentru orice x ∈ 0, are loc inegalitatea:
2
2 sin x + tg x ≥ 3x.

[ π] x x
8.19. Dacă x ∈ 0, , atunci: ≤ tg ≤ x.
2 2 2
[ π) x2 tg x x3
8.20. Să se arate că pentru orice x ∈ 0, avem: tg x − x ≥ ≥ .
2 3 3
[ π)
8.21. Să se arate că: ln2 (cos x) ≤ x(tg x − x), pentru orice x ∈ 0, .
2
x [ π)
8.22. Demonstraţi inegalitatea: x·tg +ln(cos x) ≤ 0, pentru orice x ∈ 0, .
2 2
8.23. Să se arate că pentru orice x > 0 are loc inegalitatea:
nx √
arctg ≤ ln n + 1 unde n ∈ N.
1 + (n + 1)x2

( π)
8.24. Demonstraţi că dacă x ∈ 0, şi n ∈ N, atunci:
2

|tg n x − ctg n x| ≥ 2n| cos 2x|.

8.25. Aşezaţi ı̂n ordine crescătoare numerele următoare: 4tg 1◦ , 3tg 2◦ , 2tg 3◦ ,
tg 4◦ .

8.26. Găsiţi fără tabele x◦ ∈ (0◦ , 90◦ ), x ∈ N astfel ı̂ncât:

tg x◦ < 3 < tg (x + 1)◦ .

8.27. Găsiţi minimul şi maximul expresiilor:


1) a sin x + b cos x;
2) a cos2 x + 2b cos x sin x + c sin2 x, unde a, b, c, x ∈ R.

8.28. Să se arate că pentru orice a, b, x ∈ R are loc inegalitatea:


( )2
a+b
(sin x + a cos x)(sin x + b cos x) ≤ 1 + .
2

207
( π)
8.29. Fie x ∈ 0, şi a, b > 0. Să se demonstreze că:
2
( ( ) (
a ) b √ )2
1+ 1+ ≥ 1 + 2ab .
sin x cos x

Când are loc egalitatea?


( π) a − b sin x √
8.30. Dacă a > b > 0 şi x ∈ 0,
, atunci: ≥ a2 − b2 . Să se
2 cos x
b
arate că semnul egal se obţine dacă şi numai dacă sin x = .
a
( π)
8.31. Dacă a, b > 0 şi x ∈ 0, , atunci:
2
√ √ √3
√3 3
a sin x + b cos x ≤ ( a4 + b4 ) 4 .

8.32. Să se demonstreze că:


1 1 1 1 4
+ ≤√ +√ ≤√ ,
a b a2 cos2 x + b2 sin2 x a2 sin2 x + b2 cos2 x 2(a2 + b2 )

pentru a, b > 0.
( π)
8.33. Să se arate că pentru orice x ∈ 0, şi pentru orice n ∈ N, avem:
2
( )( )
1 1 n
1+ n 1+ n
≥ (1 + 2 2 )2 .
sin x cos x

8.34. Dacă a, b, c > 0 şi a cos2 x + b sin2 x < c, atunci:


√ √ √
a cos2 x + b sin2 x < c.
[ π]
8.35. Găsiţi minimul funcţiei f : 0, → R, f (x) = tg 2 (α − x) + tg 2 (α + x),
( π) 2
unde α ∈ 0, .
2
8.36. Demonstraţi inegalitatea: sin x sin y sin z + cos x cos y cos z ≤ 1, pentru
x, y, z ∈ R.

8.37. Demonstraţi inegalitatea:

sin2 x cos2 y + sin2 y cos2 z + sin2 z cos2 x ≤ 1, pentru x, y, z ∈ R.

208
8.38. Să se demonstreze că pentru orice x ∈ R are loc inegalitatea:
2
cos x − sin2 x 2

1 + sin x cos x ≤ √3 .

8.39. Să se demonstreze că:


2
| cos x| + | cos 2x| ≥ √ , pentru orice x ∈ R.
3
8.40. Fie a, b ∈ R cu proprietatea: a sin x + b cos x > 2, pentru orice x ∈ R. Să
se demonstreze că a2 + b2 > 4.
8.41. Să se arate că
√ √
2sin x + 2cos x ≥ 22− 2, pentru orice x ∈ R.

8.42. Să se arate că pentru orice x, y ∈ R are loc inegalitatea:


−9
cos x + cos y + 2 cos(x + y) ≥ .
4
8.43. Să se arate că pentru orice x, y ∈ R avem:
9
1) sin2 x + sin2 y + sin2 (x + y) ≤ ;
4
2) 8 cos x cos y cos(x + y) + 1 ≥ 0.
8.44. Demonstraţi că dacă x, y ∈ R, atunci: cos(x2 ) + cos(y 2 ) − cos(xy) < 3.
8.45. Să se arate că: sin 2x(sin y + cos y + sin y cos yctg x) ≤ 2.
8.46. Fie a1 , a2 , . . . , an ∈ R şi n ∈ N. Ştiind că
|a1 sin x + a2 sin 2x + . . . + an sin nx| ≤ | sin x|, pentru orice x ∈ R,
demonstraţi că: |a1 + 2a2 + . . . + nan | ≤ 1.
8.47. Să se demonstreze că pentru orice a1 , a2 , . . . , an ∈ R avem:
| sin a1 | + | sin a2 | + . . . + | sin an | + | cos(a1 + a2 + . . . + an )| ≥ 1.

8.48. Demonstraţi că pentru orice x ∈ R şi pentru orice n ∈ N are loc inegali-
tatea:
n
| cos x| + | cos 2x| + . . . + | cos 2n−1 x| + | cos 2n x| ≥ .
4
8.49. Găsiţi x ∈ R astfel ı̂ncât cos(2n · x) < 0, pentru orice n ∈ N.

209
8.2 Probleme propuse
π 2 − x2
 8.1. Să se arate că pentru orice x > π avem: sin x ≥ x.
π 2 + x2
π
 8.2. Dacă 0 < x < y < , atunci x − tg x > y − tg y.
2
 8.3. Care număr este mai mare: 4tg 5◦ tg 9◦ sau 3tg 6◦ tg 10◦ ?

 8.4. Pentru care n ∈ Z este ı̂ndeplinită următoarea condiţie:


1
tg n◦ < < tg (n + 1)◦ ?
2

 8.5. Să se arate că pentru orice x, y, α ∈ R are loc inegalitatea:

x2 + y 2 + 1 ≥ 2(x sin α + y cos α).

Când are loc egalitatea?

 8.6. a şi b sunt numere reale astfel ı̂ncât inegalitatea

a cos x + b cos 3x > 1

nu are soluţii. Demonstraţi că |b| ≤ 1.

 8.7. Arătaţi că dacă x, y, z ≥ 0 şi arctg x + arctg y + arctg z < π, atunci:

xyz < x + y + z.
π
 8.8. Fie x, y ≥ 0 şi x + y ≤ . Să se arate că:
2
sin2 x + sin2 y − 1
≤ −1 + sin(x + y).
sin x + sin y + 1
( π) x
 8.9. Dacă x ∈ 0, , atunci xtg + ln cos x < 0.
2 2
 8.10. Demonstraţi că există o singură tripletă a, b, c de numere reale astfel
ı̂ncât:

1+ 3·cos x+a·cos 2x+b·cos 3x+c·cos 4x ≥ 0, pentru orice x ∈ (0, π).
n
 8.11. Dacă x ∈ R şi n ∈ N∗ avem | cos 2x| 2 ≤ | cos2n x − sin2n x|. În ce caz
avem egalitate?

210
 8.12. Fie α, β, γ ≥ 0 astfel ı̂ncât α + β + γ = π. Să se arate că

sin3 α + sin3 β + sin3 γ ≤ 2.

 8.13. Să se determine minimul funcţiei


225π 2
f : [21, 28] → R, f (x) = 4x + + sin3 x + sin x.
x

 8.14. Pentru orice numere reale x1 , x2 , . . . , xn , n ∈ N şi n ≥ 2 notăm



n ∑
n
F (x1 , x2 , . . . , xn ) = {f (xi − xj ) − f (xi + xj )}
i=1 j=1

unde { sin x
, dacă x ̸= 0
f : R → R, f (x) = x
1, dacă x = 0.
Arătaţi că F (x1 , x2 , . . . , xn ) ≥ 0. În ce caz avem egalitate?
[π √
π] 1 sin nα 6
 8.15. a) Dacă n ∈ N, n ≥ 2 şi α ∈ ,π − atunci − ≤ ≤ .
n n 3 n sin α 9
[π π ] 1 sin nα 1
b) Dacă n ∈ N, n ≥ 2 şi α ∈ , , atunci − ≤ ≤ . Apoi să
√ n 2 3 n sin α 5
1 6 1
se arate că − , şi sunt cele mai bune constante posibile.
3 9 5

8.3 Soluţii
8.1. Avem: tg x ≥ x ≥ sin x, pentru orice x ∈ [0, π/2) şi deci:

sin2 x
tg 2 x = ≥ x2 ⇔ sin2 x ≥ x2 cos2 x ⇔ sin2 x ≥ x2 (1−sin2 x) ⇔
cos2 x
x [ π)
(1 + x2 ) sin2 x ≥ x2 ⇔ sin x ≥ √ , pentru orice x ∈ 0, .
1 + x2 2
În x = π/2 ultima inegalitate este evidentă căci sin π/2 = 1.

8.2. Deoarece x ∈ (0, π/2) implică sin x < x şi pentru aceste valori ale lui x
avem:

3 1 3 1 1
2 2
cos x = 1−2 sin x/2 > 1−x /2, şi deci cos −sin > 1− − = .
2 2 8 2 8

211
3
8.3. Inegalitatea < 1 se reduce la cos x < 1. Pentru cealaltă inegal-
4 − cos x
itate, considerăm funcţia:
[ π)
f : 0, → R, f (x) = 3x − sin x(4 − cos x),
2
( π)
va trebui să verificăm: f (x) > 0, pentru orice x ∈ 0, . Avem:
2
f ′ (x) = 3 − cos x(4 − cos x) − sin2 x

= 3 − 4 cos x + cos2 x − (1 − cos2 x) = 2(cos x − 1)2 > 0,


( π)
deci f este strict crescătoare pe 0, şi cum x > 0, rezultă
2
f (x) > f (0) = 0.

8.4. Avem
sin x 2 cos x
> ⇔ tg x(1 + cos x) > 2x.
x 1 + cos x
( π)
Fie f : 0, → R, f (x) = (1 + cos x)tg x − 2x. Vom arăta că f este
2 ( π)
strict crescătoare pe 0, :
2
1 + cos x sin2 x (1 − cos x)(cos x + sin2 x)
f ′ (x) = − − 2 = ≥0
cos2 x cos x 2 cos2 x
rezultă f (x) > f (0) = 0.
Observaţie. Din 8.3 şi 8.4 rezultă:
2 cos x sin x 3 ( π)
< < , pentru orice x ∈ 0, .
1 + cos x x 4 − cos x 2

sin x 3 cos x
8.5. Avem: > ⇔ tg x(1 + 2 cos x) > 3x şi considerând
x[ )
1 + 2 cos x
π
funcţia f : 0, → R, f (x) = (1 + 2 cos x)tg x − 3x se demonstrează
2
uşor că f (x) este strict crescătoare şi rezultă inegalitatea din enunţ.
3 cos x 2 cos x
Observaţie. Se verifică uşor că > , pentru orice x ∈
( π) 1 + 2 cos x 1 + cos x
0, şi deducem că inegalitatea demonstrată este mai fină decât cea
2
din problema 8.4.

212
8.6. Se demonstrează uşor că

x3 x2 x4 ( π)
sin x > x − şi cos x < 1 − + , pentru orice x ∈ 0, .
6 2 24 2
Prin urmare
sin x x2 sin3 x x2 x4 x6
>1− şi deci > 1 − + − .
x 6 x3 2 12 216
Adunând membru cu membru această inegalitate cu inegalitatea

x2 x4
− cos x > −1 + −
2 24
rezultă că
sin3 x 4 9−x
2
− cos x > x · .
x3 216
( π) √
Dar x ∈ 0, rezultă 0 < x < 3 şi deci x2 < 3. Rezultă
2
sin3 x sin x √
3
> cos x, de unde > 3 cos x.
x x

π π π π 3
8.7. Din x > 2 rezultă 0 < < şi deci tg > > . Cum cos2 x =
x 2 x x x
1 π 1
2
rezultă cos2 < ( )2 şi prin urmare
1 + tg x x 3
1+
x

π π x2 9
sin2 = 1 − cos2 > 1 − 2 = 2 .
x x x +9 x +9
√ π ( π)
8.8. I-a soluţie. Se observă că 3 > 1, 6 > şi deci 3x − x3 > 0 pe 0, .
2 2
Inegalitatea de demonstrat se poate scrie succesiv
2 2
> 3x − x3 ⇔ tg x + ctg x > 3x − x3 căci tg x + ctg x = .
sin 2x sin 2x
Demonstraţia ultimei forme de scriere a inegalităţii rezultă imediat din
inegalitatea mediilor.
2 ( π)
A II-a soluţie. Avem ≥ 2 şi 2 ≥ 3x − x3 pentru x ∈ 0, .
sin 2x 2
Ultima inegalitate rezultă din identitatea x − 3x + 2 = (x − 1) (x + 2).
3 2

213
( π)
8.9. Pentru x ∈ 0, , avem sin x < x şi deci sin(cos x) < cos x deoarece
2 ( π)
funcţia cosinus este descrescătoare pe 0, , obţinem: sin(cos x) <
2 ( π)
cos x < cos(sin x). Înlocuind ı̂n această inegalitate pe x ∈ 0, cu
π ( π ) 2
− x ∈ 0, rezultă:
2 2
( (π )) (π )
sin cos − x < cos x sin −x ,
2 2
adică sin(sin x) < cos(cos x).

8.10. Inegalitatea este echivalentă pe rând, cu următoarele:


π
2 sin cos x √
3 4 3
(π ) (π )≥ ⇔
sin + x sin −x 3
3 3
√ √
3 · cos x 4 3
(π ) (π )≥ ⇔
2 sin + x sin −x 6
3 3
cos x 2 2 cos x 2
≥ ⇔ ≥ ⇔
2π 3 2 cos 2x + 1 3
cos 2x − cos
3
3 cos x ≥ 2 cos 2x + 1 ⇔ 3 cos x ≥ 4 cos2 x − 1 ⇔
( )
1
4 cos2 x − 3 cos x − 1 ≤ 0 ⇔ 4(cos x − 1) cos x + ≤0
4
ceea ce este evident căci cos x ≤ 1.

8.11. Vom demonstra că:


sin x − sin y [ π]
−1 ≤ ≤ 1, pentru orice x, y ∈ 0, , x ̸= y.
1 − sin x sin y 2

Avem 1 − sin x sin y > 0 (căci 1 ≥ sin x sin y şi egalitatea ar avea loc
π
numai dacă x = sin y = 1 ⇔ x = y = , fals) şi inegalitatea sin x −
2
sin y ≤ 1 − sin x sin y revine la 0 ≤ (1 − sin x)(1 + sin y), ceea ce este
π
evident. Egalitatea are loc pentri x = . Analog, cealaltă inegalitate
2
π
revine la (1 + sin x)(1 − sin y) ≥ 0 şi se obţine pentru y = .
2

214
x
8.12. Fie f : [0, 1) → R, f (x) = − arcsin x. Deoarece
1 − x2

′ 1 + x2 1 1 + x2 − 1 − x2 · (1 − x2 )
f (x) = −√ = >0
(1 − x2 )2 1 − x2 (1 − x2 )2

(căci 1 + x2 ≥ 1 − x2 (1 − x2 )) rezultă că f este strict crescătoare şi
deci x > 0 implică f (x) > f (0) = 0.

[ √ ] este echivalentă cu arcsin(sin x) ≤


2
8.13. Deoarece inegalitatea de demonstrat
π
x2 , considerăm funcţia f : 0, → R, f (x) = x2 − arcsin(sin2 x).
2
Avem:
1
f ′ (x) = 2x − √ · 2 sin x cos x
1 − sin4 x
( )
x 1
= 2 sin x −√ > 0,
sin x 1 + sin2 x
( π) x 1
pentru orice x ∈ 0, căci ≥ 1 şi 1 ≥ √ implică
2 sin x 1 + sin2 x
x 1
− √ > 0. Rezultă că f (x) ≥ f (0) = 0, pentru orice
sin x 2
[ √ 1)+ sin x
π
x ∈ 0, .
2
8.14. Pentru n = 0 sau n = 1 are loc semnul egal. Presupunând că | sin nx| ≤
n sin x, avem:

| sin(n + 1)x| = | sin(nx + x)| = | sin nx cos x + cos nx sin x|

≤ | sin nx| · | cos x| + | cos nx| · | sin x| ≤ | sin nx| + | sin x|


≤ n sin x + sin x = (n + 1) sin x,
căci sin x ≥ 0, pentru orice x ∈ [0, π].

8.16. Din inegalitatea mediilor avem:



2sin x + 2tg x ≥ 2 2sin x+tg x

şi deci e suficient a verifica inegalitatea


[ π)
sin x + tg x − 2x ≥ 0, pentru orice x ∈ 0, .
2

215
Considerând funcţia
[ π)
f : 0, → R, f (x) = sin x + tg x − 2x,
2
avem
( )
′ 1 1
f (x) = cos x + −2≥ cos x + − 2 ≥ 2 − 2 = 0.
cos2 x cos x
[ π)
Rezultă că f este strict crescătoare pe 0, şi prin urmare
2
f (x) ≥ f (0) = 0.
(
π) x
8.17. 1) Pentru x ∈ 0, avem tg x > x şi 0 < 1 − tg 4 < 1, deci:
2 2
 x x 
tg x x 1  2tg 2tg
− = 2 · 2 − x2 
x sin x x sin x 1 − tg 2 x 1 + tg 2 x
2 2
( )
1 2x 2x
> · − x = 0.
2
x sin x 2 2

Observaţie. Din inegalitatea demonstrată rezultă


[ π)
tg x sin x ≥ x2 , pentru orice x ∈ 0, .
2

2) Folosind observaţia de mai sus şi inegalitatea mediilor, avem:


√ √ [ π)
sin x + tg x ≥ 2 sin xtg x ≥ 2 x2 = 2x, pentru orice x ∈ 0, .
2

Altă soluţie.
x x
2tg 2tg
sin x + tg x = 2 + 2
x x
1 + tg 2 1 − tg 2
2 2
  x
x 1 1 4tg
= 2tg + = 2 > 4tg x ≥ 4 · x = 2x.
2 1 + tg 2 x x x 2 2
1 − tg 2 1 − tg 4
2 2 2

216
[ π)
8.18. Considerând funcţia f : 0, → R, f (x) = 2 sin x + tg x − 3x, avem:
2
1 2 cos3 x − 3 cos2 x + 1
f ′ (x) = 2 cos x + 2
−3=
cos x cos2 x
(cos x − 1)2 (2 cos x + 1)
= ≥ 0.
cos2 x
[ π)
Rezultă că f este strict crescătoare pe 0, şi deci x ≥ 0 ⇒ f (x) ≥
[ π) 2
f (0) = 0, pentru orice x ∈ 0, .
2
[ π] x
8.19. Fie f : 0, → R, f (x) = x − tg . Se obţine:
2 2
1 1 1 cos x
f ′ (x) = 1 − · =1− = ≥ 0.
2 cos2 x 1 + cos x 1 + cos x
2
[ π]
Deci f (x) ≥ f (0) = 0, pentru orice x ∈ 0, .
2
x
Observaţie. Inegalitatea este echivalentă cu arctg x ≥ , pentru orice
[ π] 2 [
π]
x ∈ 0, şi poate fi demonstrată cu ajutorul funcţiei g : 0, → R,
2 2
x
g(x) = arctg x − .
2
[ π)
8.20. Vom demonstra mai ı̂ntâi că pentru orice x ∈ 0, rezultă:
2
x3
f (x) = tg x − x − ≥ 0.
3
1
Într-adevăr, avem f ′ (x) = − 1 − x2 = tg 2 x − x2 ≥ 0 căci tg x ≥ x,
[ π) cos 2x
[ π)
pentru orice x ∈ 0, . Rezultă că f este strict crescătoare pe 0, ,
2 2
deci x ≥ 0 ⇒ f (x) ≥ f (0) = 0. Analog se demonstrează inegalitatea:

x3 2x5 [ π)
g(x) = tg x − x − − ≥ 0, pentru orice x ∈ 0,
3 15 2
2x4
căci g ′ (x) = tg 2 x − x2 −
≥ 0 rezultă din
3
( )2
x3 2x4 x6 2x4
tg x = (tg x) ≥ x +
2 2
= x2 + + ≥ x2 + .
3 3 9 3

217
Inegalitatea din enunţ revine la următoarea:
3x [ π ) (π √ )
tg x ≥ , pentru orice x ∈ 0, < 3 .
3 − x2 2 2
Ea rezultă din faptul că:

x3 2x5 3x
tg x ≥ x + + ≥ .
3 15 3 − x2
Ultima inegalitate revine la următoarele inegalităţi echivalente:

x2 2x4 3 2x4 x4 2x6


1+ + ≥ ⇔ 3 + x 2
+ − x2
− − ≥3 ⇔
3 15 3 − x2 5 3 15
5
10x4 − 5x4 − 2x6 ≥ 0 ⇔ x2 ≤ .
2
π π 2 5
Dar x < implică x2 < < , căci π 2 < 10.
2 4 2
[ π)
8.21. Deoarece tg x ≥ x, pentru orice x ∈ 0, inegalitatea este echivalentă
2
cu următoarele:
√ √
| ln(cos x)| ≤ x(tg x − x) ⇔ x(tg x − x) + ln(cos x) ≥ 0.
[ π) √
Fie f : 0, → R, f (x) = x(tg x − x) + ln(cos x). Avem (pentru
2
x ̸= 0):
1
f ′ (x) = √ [tg x − x + x(1 + tg 2 x − 1)] − tg x
2 x(tg x − x)
√ √ √
tg x − x + xtg 2 x − 2tg x x(tg x − x) ( x · tg x − tg x − x)2
= √ = √ ≥ 0.
2 x(tg x − x) 2 x(tg x − x)
[ π)
Rezultă că f este strict crescătoare pe 0, şi deci f (x) > f (0) = 0,
( π) 2
pentru orice x ∈ 0, . În x = 0 are loc egalitatea.
2
[ π) x
8.22. Fie f : 0, → R, f (x) = xtg + ln(cos x). Avem:
2 2
x
x 1 1 x 2tg ( )
f ′ (x) = tg + x · · − tg x = tg − 2 + x 1 + tg 2 x
2 x 2 2 1 − tg 2 x 2 2
cos2
2 2

218
 x 
( )
x x tg
= 1 + tg 2 − 2 <0
2 2 1 − tg 2 x
2
căci
x
tg x x
2 > tg > .
x 2 2
1 − tg 2
2
nx
8.23. Considerând funcţia f : (0, ∞) → R, f (x) = arctg , avem:
1 + (n + 1)x2

1 n[1 + (n + 1)x2 ] − 2nx2 (n + 1)


f ′ (x) = [ ]2 ·
nx [1 + (n + 1)x2 ]2
1+
1 + (n + 1)x2

n[1 − (n + 1)x2 ]
= .
n2 x2 + [(n + 1)x2 + 1]2
1
Singura rădăcină a derivatei este x0 = √ şi este un punct de maxim
n+1
pe (0, ∞). Rezultă că
nx n
arctg 2
≤ arctg √ , pentru orice x > 0.
1 + (n + 1)x 2 n+1
Mai trebuie verificată inegalitatea:
x √ 1
arctg √ ≤ ln x + 1 = ln(x + 1), pentru orice x ≥ 0.
2 x+1 2
Aceasta rezultă considerând funcţia
1 x
g(x) = ln(x + 1) − arctg √
2 2 x+1
pentru care
√ x
x+1− √
1 1 1 2 x+1
g ′ (x) = − · ·
2(x + 1) x2 2 x+1
1+
4(x + 1)

( x + 1 − 1)2
= ≥ 0.
2(x + 1)(x + 2)

219
8.24. Avem:

|tg n x − ctg n x| = |tg x − ctg x| · |tg n−1 x + tg n−2 x · ctg x + . . . + ctg n−1 x|.

Dar

sin x cos x sin2 x − cos2 x 2| cos 2x|

|tg x−ctg x| = − = = ≥ 2| cos 2x|
cos x sin x sin x cos x | sin 2x|

iar
|tg n−1 x + tg n−2 x · ctg x + . . . + ctg n−1 x|

= |tg n−1 x + tg n−2 x · ctg x + . . . + ctg n−1 x



≥ n n tg n−1 x · . . . · tg x · ctg x · . . . · ctg n−1 x = n
rezultă din inegalitatea mediilor.

8.25. Notăm cu α mărimea ı̂n radiani a unghiului de 1◦ şi considerăm funcţia


tg x
f : (0, 5α) → R, f (x) = . Deoarece
x
x
2x
− tg x 2x − sin 2x
′ cos
f (x) = = > 0,
x2 2x2 cos2 x
f este strict crescătoare şi deci:

tg α tg 4α tg 2α tg 3α
< iar < .
α 4α 2α 3α
Pe de altă parte:

2tg 2◦ 1
3tg 2◦ > tg 4◦ ⇔ 3tg 2◦ > ⇔ tg 2 2◦ < ,
1 − tg 2 2◦ 3

şi ultima inegalitate este evident adevărată. Rezultă că:

4tg 1◦ < tg 4◦ < 3tg 2◦ < 2tg 3◦ .

x3 ( π)
8.26. Din tg x > x + , pentru orice x ∈ 0, (vezi problema 8.20) rezultă
3 2
3 ( π)
ctg x < , pentru orice x ∈ 0, .
3x + x3 2

220
19π 19π
Cum tg 71◦ = ctg 19◦ = ctg , notând α = , avem:
180 180
19 · 3, 14 59, 66
α> = > 0, 33 şi α3 > 0, 33 = 0, 027,
180 180
de unde se obţine că α3 + 3α > 0, 027 + 0, 99 > 1. Rezultă că ctg α < 3
şi prin urmare tg 71◦ < 3. Deoarece
√ √ √
◦ ◦ 10 + 2 5 √
tg 72 = ctg 18 = √ = 5 + 2 5 > 3,
5−1
rezultă că x = 71.
8.27. 1) Din identitatea a2 +b2 = (a sin x+b cos x)2 +(a cos x−b sin x)2 rezultă
că:
√ √
− a2 + b2 ≤ a sin x + b cos x ≤ a2 + b2 , pentru orice x ∈ R.

Egalitatea are loc când a cos x = b sin x.


2) Avem:
a c
a cos2 x + 2b cos x sin x + c sin2 x = (1 + cos 2x) + b sin 2x + (1 − cos 2x)
2 2
a+c a−c
= + b sin 2x + cos 2x
2 2
şi deci (folosind inegalitatea de la punctul 1) minimul şi maximul sunt:
√ ( )2 √ ( )
a+c a − c a + c a−c 2
− b + 2 , respectiv + b + 2 .
2 2 2 2
( )2
a+b
8.28. Dacă cos x = 0, inegalitatea revine la 1 + ≥ sin2 x şi este
2
evidentă. Pentru cos x ̸= 0 ea se scrie:
( )
a+b 2
cos x(tg x + a)(tg x + b) ≤ 1 +
2

2
( )
(tg x + a)(tg x + b) a+b 2
≤1+ .
1 + tg 2 x 2
Notând tg x = t şi efectuând calculele se obţine:
( )2 ( )
2 a+b a+b 2
t +1+ − (a + b)t − ab ≥ 0 ⇔
2 2

221
( )2 ( )2
a+b a−b
·t−1 + ≥ 0,
2 2
ceea ce este evident.

8.29. Efectuând calculul inegalitatea de demonstrat devine:

a b ab √
1+ + + ≥ 1 + 2 2ab + 2ab.
sin x cos x sin x cos x
Dar
ab 2ab
= ≥ 2ab,
sin x cos x sin 2x
iar inegalitatea mediilor implică:
√ √
a b ab 2ab √
+ ≥2 =2 ≥ 2 2ab.
sin x cos x sin x cos x sin 2x

În această inegalitate semnul egal are loc când

a b a
= ⇔ tg x = .
sin x cos x b
Prin urmare ı̂n inegalitatea din enunţ se obţine semnul egal numai dacă
a π
sin 2x = 1 şi tg x = ⇔ x = şi a = b.
b 4

8.30. Enunţul echivalează cu: b sin x + a2 − b2 · cos x ≤ a, şi rezultă din
inegalitatea de la 8.27.1):
√ √ √
b sin x + a2 − b2 · cos x ≤ b2 + ( a2 − b2 )2 = a.

Egalitatea are loc dacă şi numai dacă



b a2 − b2 b
= ⇔ tg x = √ ⇔
sin x cos x a − b2
2

b2 + a2 − b2 a2 1
tg 2 x + 1 = = 2 = ⇔
a −b
2 2 a −b 2 cos2 x
b2 a2 − b2 b
= = a2 ⇔ sin x = .
2
sin x a −b
2 2 a
a2

222
√ √
3
3
8.31. Fie a = A, b = B > 0. Inegalitatea se reduce la:
√ √ 3
A3 · sin x + B 3 · cos x ≤ (A4 + B 4 ) 4 .

Dar:
√ √ √ √
A3 · sin x + B 3 · cos x = A2 (A sin x) + B 2 (B cos x)

≤ (A4 + B 4 )(A2 sin x + B 2 cos x),
din inegalitatea lui Cauchy-Buniakovski. Rămâne să mai verificăm că:
√ √
(A4 + B 4 )(A2 sin x + B 2 cos x) ≤ 4 (A4 + B 4 )3 ⇔

(A4 + B 4 )2 (A2 sin x + B 2 cos x)2 ≤ (A4 + B 4 )3 ⇔



A2 sin x + B 2 cos x ≤ A4 + B 4
ceea ce rezultă tot din inegalitatea Cauchy-Buniakovski:
√ √
A sin x + B cos x ≤ (A4 + B 4 )(sin2 x + cos2 x) = A4 + B 4 .
2 2

8.32. Avem:

√ a2 (1 + cos 2x) b2 (1 − cos 2x)
a2 cos2 x + b2 sin2 x = +
2 2

1 √ a2 + b2 √
= √ · a2 + b2 + (a2 − b2 ) cos 2x = · 1 + ky
2 2
a2 − b2
unde k = ∈ (−1, 1) şi y = cos 2x ∈ [−1, 1]. Analog
a2 + b2

√ a2 + b2 √
a2 sin2 x + b2 cos2 x = · 1 − ky.
2
Considerăm funcţia

2
[(1 + ky)− 2 + (1 − ky) 2 ].
1 1
f (y) =
+b 2 a2

2 2
Pentru k = 0, f (y) = √ . Avem:
a2 + b2
√ √
′ k (1 − ky)3 − (1 + ky)3
f (y) = · √
2 (1 − k 2 y 2 )3

223
k 2 (k 2 y 2 + 3)y
= −√ (√ √ ).
(1 − k 2 y 2 )3 (1 − ky)3 + (1 + ky)3
De aici rezultă că

2 2 1 1
fmax = f (0) = √ şi fmin = f (−1) = f (1) = + .
a2 + b2 a b

8.33. Efectuând calculele inegalitatea de demonstrat se reduce la următoarea:


1 1 1 n
n + n
+ n n
≥ 2 · 2 2 + 2n , dar
sin x cos x sin x cos x
1 2n 2n
= = ≥ 2n
sinn x cosn x (2 sin x cos x)n sinn 2x
1
iar din 1 = sin2 x + cos2 x ≥ 2 sin x cos x avem ≥ 2 şi deci
sin x cos x
(folosind inegalitatea mediilor):

1 1 1 √ n
n + n
≥2 n n
≥ 2 2n = 2 · 2 2 .
sin x cos x sin x cos x

8.34. Folosind inegalitatea Cauchy-Buniakovski şi ipotezam avem:


√ √ √ √
√ √
a · cos2 x + b · sin2 x ≤ ( a · cos x)2 + ( b · sin x)2 · cos2 x + sin2 x
√ √
= a cos2 x + b sin2 x < c.

8.35. Avem:
f (x) + 2 = [tg 2 (α − x) + 1] + [tg 2 (α + x) + 1]
1 1 2
= + ≥
cos (α − x) cos (α + x)
2 2 | cos(α − x) cos(α + x)|
4 4 4
= ≥ ≥
| cos 2x + cos 2α| | cos 2x| + |cos2α| 1 + | cos 2α|
 2 π

 = 2 + 2tg 2 α, 0 < α ≤
cos2 α 4
=

 2 π π
= 2 + 2ctg 2 α, <α<
sin2 α 4 2
Deci:  ( π]

 2tg 2 α, dacă α ∈ 0,
4
f (x) ≥ (π π )

 2ctg 2 α, dacă α ∈ ,
4 2

224
π
Aceste valori se ating pentru x = 0 şi x = . Prin urmare:
2
 π

 2tg 2 α, dacă α ≤
4
min f (x) =
x∈[0, π2 ] 
 2ctg 2 α, dacă π
<α<
π
4 2

8.36. Folosind inegalitatea Cauchy-Buniakovski şi faptul că sin2 x ≤ 1 pentru


orice x ∈ R, avem:

sin x sin y sin z + cos x cos y cos z ≤ | sin x(sin y sin z) + cos x(cos y cos z)|
√ √
≤ sin x + cos x · sin2 y sin2 z + cos2 y cos2 z
2 2


≤ sin2 y · 1 + cos2 y · 1 = 1.

8.37. Fie a = sin2 x, b = sin2 y, c = sin2 z. Rezultă că a, b, c ∈ [0, 1] şi trebuie
demonstrată inegalitatea:

a(1 − b) + b(1 − c) + c(1 − a) ≤ 1 ⇔ a + b + c ≤ ab + bc + ca + 1,

dar ac(1 − b) ≤ ac şi c(1 − a)(1 − b) ≤ (1 − a)(1 − b), deci:

c(1−b) = c(1−b)(1−a+a) ≤ c(1−b)(1−a)+ac(1−b) ≤ (1−a)(1−b)+ac,

adică:

c − bc ≤ 1 − a − b + ab + ac ⇔ a + b + c ≤ ab + bc + ca + 1.

8.38. Inegalitatea se mai poate scrie:


| cos 2x| 1 √
≤ √ ⇔ 3| cos 2x| ≤ 2 + sin 2x ⇔
|2 + sin 2x| 3

3 cos2 2x ≤ 4 + 4 sin 2x + sin2 2x ⇔


3(1 − sin2 2x) ≤ 4 + 4 sin 2x + sin2 2x ⇔
4 sin2 2x + 4 sin 2x + 1 ≥ 0 ⇔ (2 sin 2x + 1)2 ≥ 0
ceea ce este evident. Egalitatea are loc când
1 π kπ
sin 2x = − ⇔ x = (−1)k+1 · + , k ∈ Z.
2 12 2

225
8.39. Fie | cos x| = t ∈ [0, 1]. Cum cos 2x = 2 cos2 x − 1, avem de arătat că:

2
|2t − 1| + t ≥
2
, pentru orice t ∈ [0, 1].
2
[ √ ]
2
Dacă t ∈ 0, , inegalitatea se scrie:
2
√ ( √ )
2 2
1 − 2t2 + t ≥ ⇔ 2t2 − 1 − t− ≤0 ⇔
2 2
( √ )[ ( √ ) ]
2 2
t− 2 t+ −1 ≤0
2 2

şi rezultă din faptul că


√ ( √ )
2 2 √ √
t≤ şi 2 t + − 1 = 2t + 2 − 1 ≥ 2 − 1 > 0.
2 2
(√ ]
2
Dacă t ∈ , 1 , inegalitatea rezultă din:
2
√ ( √ )[ ( √ ) ]
2 2 2
2t2 − 1 + t − = t− 2 t+ + 1 > 0.
2 2 2


8.40. Dacă am avea a2 + b2 ≤ 4, atunci cum a sin x + b cos x ≤ a2 + b2 , ar
rezulta că:
√ √
a sin x + b cos x ≤ a2 + b2 ≤ 4 = 2, pentru orice x ∈ R

ceea ce contrazice ipoteza.

8.41. Folosind inegalitatea mediilor, avem:


√ sin x+cos x

2 π
2sin x + 2cos x ≥ 2 2sin x · 2cos x = 2 · 2 2 = 2 · 2 2 sin(x+ 4 )
√ √ √ √
≥ 2 · 2− 2 = 2 2 = 22− 2 ,
2 2− 2

( π)
căci sin x + ≥ −1.
4

226
8.42. Folosind formula de transformare a sumei de cosinusuri ı̂n produs şi
α
identitatea cos α = 2 cos2 −1, avem următoarele inegalităţi echivalente
2
cu cea din enunţ:
( )
x+y x−y 2 x+y 9
2 cos cos + 2 2 cos −1 ≥− ⇔
2 2 2 4
x+y x+y x−y 9
4 cos2 + 2 cos cos −2+ ≥0 ⇔
2 2 2 4
x+y x+y x−y 1
4 cos2 + 2 cos cos + ≥0 ⇔
2 2 2 4
x + y x + y x − y
16 cos2 + 8 cos cos +1≥0 ⇔
2 2 2
( )
x+y x−y 2 x−y
4 cos + cos + sin2 ≥ 0.
2 2 2

8.43. 1) Inegalitatea este echivalentă cu următoarele:

1 − cos 2x 1 − cos 2y 1 − cos 2(x + y) 9


+ + ≤ ⇔
2 2 2 4
3
cos 2x + cos 2y + cos 2(x + y) ≥ − ⇔
2
3
2 cos2 x − 1 + 2 cos(2y + x) cos x + ≥0 ⇔
2
1
cos2 x + cos(2y + x) cos x + ≥0 ⇔
4
[ ]2
1 1
cos x + cos(2y + x) + sin2 (2y + x) ≥ 0.
2 4

Observaţie. Din demonstraţia de mai sus rezultă inegalitatea:


3
cos x + cos y + cos(x + y) ≥ − , pentru orice x, y ∈ R.
2
3
Din (cos 2x + cos 2y) + cos 2(x + y) ≥ − , rezultă:
2
1
2 cos(x + y)[cos(x − y) + cos(x + y)] ≥ − ⇔
2
8 cos x cos y cos(x + y) + 1 ≥ 0.

227
8.44. Avem cos x2 ≤ 1, cos y 2 ≤ 1 şi − cos(xy) ≤ 1, deci

cos x2 + cos y 2 − cos(xy) ≤ 3.

Va trebui să demonstrăm că sistemul



 cos x2 = 1
cos y 2 = 1

cos(xy) = −1

nu are soluţii reale. Avem:



cos x2 = 1 ⇔ x2 = 2kπ, k ∈ N ⇔ x = ± 2kπ cu k ∈ N şi

cos(xy) = −1 ⇔ xy = ±π + 2lπ, l ∈ Z ⇔ xy = π(2l ± 1) cu l ∈ Z.


Dacă sistemul ar avea
√ o soluţie ar rezulta că există k, p ∈ N şi l ∈ Z

astfel√ ı̂ncât x = ± 2kπ, y = ± 2pπ şi xy = π(2l ± 1). Rezultă că
±2π kp = π(2l ± 1) şi deci 4kp = (2l ± 1)2 , ceea ce evident este fals.
cos x
8.45. Deoarece sin 2x = 2 sin x cos x şi ctg x = , inegalitatea revine la:
sin x
sin x cos x sin y + sin x cos x cos y + sin y cos y cos2 x ≤ 1.

Ea rezultă din inegalitatea ab + bc + ca ≤ a2 + b2 + c2 prin ı̂nlocuirile:


a = sin y cos x, b = cos y cos x, c = sin x.

8.47. Demonstrăm inegalitatea prin metoda inducţiei matematice. Pentru n =


1 avem de verificat că | sin a1 | + | cos a1 | ≥ 1, ceea ce rezultă astfel:

1 = sin2 a1 + cos2 a1 = | sin a1 |2 + | cos a1 |2 ≤ | sin a1 | + | cos a1 |.

Presupunând adevărată inegalitatea din enunţ va trebui să demonstrăm


că pentru orice an+1 ∈ R:

| sin a1 | + . . . + | sin an | + | sin an+1 | + | cos(a1 + . . . + an + an+1 )| ≥ 1.

Ori, folosind ipoteza de inducţie, avem:

(| sin a1 | + . . . + | sin an |) + | sin an+1 | + | cos(a1 + . . . + an + an+1 )|

≥ (1 − | cos(a1 + . . . + an )|) + | sin an+1 | + | cos(a1 + . . . + an + an+1 )|.


Va mai trebui să verificăm inegalitatea

1 − | cos(a1 + . . . + an )| + | sin an+1 | + | cos(a1 + . . . + an + an+1 )| ≥ 1 ⇔

228
| sin an+1 | + | cos(a1 + . . . + an + an+1 )| ≥ | cos(a1 + . . . + an )|.
Notând an+1 = x ∈ R, a1 + . . . + an = y ∈ R verificarea revine la
inegalitatea:
| sin x| + | cos(x + y)| ≥ | cos y|
ceea ce rezultă astfel:

| cos y| = | cos(y + x − x) = | cos(y + x) cos x + sin(y + x) sin x|

≤ | cos(y + x)| · | cos x| + | sin(y + x)| · | sin x| ≤ | cos(x + y)| · 1 + 1 · | sin x|.
1
8.48. Dacă | cos x| ≤ , atunci
2
1 1
| cos 2x| = |2 cos2 x − 1| ≥ 1 − 2 · = .
4 2
Scriem acum membrul stâng al inegalităţii de demonstrat sub forma:

(| cos x| + | cos 2x|) + (| cos 4x| + | cos 8x|) + . . .


[ ]
n+1 1
Obţinem astfel paranteze, fiecare ≥ , deci suma căutată este
[ ] 2 2
n+1 1 n
≥ · ≥ .
2 2 4
1
8.49. Fie x un unghi ı̂ndeplinind condiţia din enunţ. Atunci: cos x ≤ − .
4
1 1
Într-adevăr, dacă cos x > − , atunci cos x <
2 şi deci
4 16
cos 4x = 2 cos2 2x − 1 = 2(2 cos2 x − 1)2 − 1 = 8 cos4 x − 8 cos2 x + 1
1 1
> 1 − 8 cos2 x > 1 − = > 0,
2 2
ceea ce ar contrazice alegerea lui x. Analog se deduce că pentru orice
1
n ∈ N va trebui să avem cos 2n x < − , de unde se obţine că:
4


cos(2 x) − ≥ 3 .
n 1
2 4

Va rezulta:
1 1
cos(2n x) + = 2 cos2 (2n−1 x) −
2 4

229

1 1 3 1
= 2 cos(2n−1 x) − · cos(2 n−1
x) + ≥ cos(2 n−1
x) + ⇒
2 2 2 2
( )n ( )n−1

cos x + 1 ≤ 2 cos(2x) + 1 ≤ . . . ≤ 2 cos(2n x) + 1 ≤ 2 .
2 3 2 3 2 3

1 1

Deci cos x + = 0 ⇔ cos x = − .
2 2

230
Capitolul 9

Inegalităţi diverse

9.1 Enunţuri
9.1. Demonstraţi
√ √ orice x ∈ R
că pentru √ există un triunghi cu lungimile la-
turilor x2 − x + 1, x2 + x + 1, 4x2 + 3 şi aria acestuia nu depinde
de x.

9.2. Să se determine cel mai mare număr natural n cu proprietatea:

xn+2 + xn − xn + 2 ≥ 0, pentru orice x ∈ R.

9.3. 1) Determinaţi m > 0 astfel ı̂ncât inegalitatea

(x + y + z)2 xyz ≥ m(xy + yz + zx)(x + y − z)(x − y + z)(−x + y + z)

să aibă loc pentru orice x, y, z ∈ R.


2) Dar dacă cerem ca inegalitatea să aibă loc pentru orice x, y, z > 0 şi
astfel ı̂ncât y + z − x, z + x − y, x + y − z > 0?

9.4. Dacă x, y, z ∈ R, atunci:


√ √ √
x2 − xy + y 2 + x2 − xz + z 2 ≥ y 2 + yz + z 2 .

9.5. Fie n ∈ N. Să se arate că dacă:

3(xn + y n + z n ) ≥ (x + y + z)n , pentru orice x, y, z ∈ R,

atunci n = 0 sau n = 2.

231
9.6. Care este valoarea maximă a lui k astfel ı̂ncât inegalitatea
∑ (∑ ) (∑ )2
x4 + xyz x ≥k· xy

să aibă loc pentru pentru orice x, y, z ∈ R?

9.7. Dacă a, b, c ∈ R, atunci există inegalitatea:

(a2 + ab + b2 )(b2 + bc + c2 )(c2 + ca + a2 ) ≥ (ab + bc + ca)3 .

9.8. Dacă a, b, c, d ∈ R, atunci:

2(a2 − ab + b2 )(c2 − cd + d2 ) ≥ a2 c2 + b2 d2 .

În ce condiţii avem egalitate?

9.9. Fiind date 5 numere reale x0 , x1 , x2 , x3 , x4 , să se arate că există alte
5∑numere reale y0 , y1 , y2 , y3 , y4 , astfel ı̂ncât xi − yi ∈ N, 0 ≤ i ≤ 4 şi
(yi − yj )2 < 4.
0≤i,j≤4

9.10. Dacă x1 , x2 , . . . , xn ∈ R, atunci:

(n − 1)x2n + 2xn + n − 1
x1 + x1 (x2 − x1 ) + . . . + xn−1 (xn − xn−1 ) ≤ .
2n

În ce caz avem egalitate?

9.11. Dacă a1 , a2 , . . . , an (n > 1) sunt numere reale diferite, atunci:


12
min (ak − ai )2 ≤ (a2 + a22 + . . . + a2n ).
1≤i<k≤n n(n2 − 1) 1

1 4
9.12. Să se arate că dacă x > 0, atunci: x + 3
≥√ √ .
x 3 3
9.13. Dacă a, b > 0, atunci:
( )
a+b n √ n (a n2 − b n2 )2
− ( ab) ≥ , pentru orice n ∈ N∗ .
2 2n

9.14. Demonstraţi că din orice 4 numere pozitive se pot alege două numere x
x−y
şi y astfel ı̂ncât: 0 ≤ ≤ 1.
1 + xy

232
9.15. Să se demonstreze inegalitatea:
[
x y
(x + y + z) +
(2x + y + z)(y + z) (2y + z + x)(z + x)
]
z 9
+ ≥ , unde x, y, z > 0.
(2z + x + y)(x + y) 8

9.16. Să se arate că pentru orice x, y, z > 0 are loc inegalitatea:
∑ x2 3
≥ .
(x + y)(x + z) 4

9.17. Demonstraţi că pentru orice x, y, z > 0 are loc inegalitatea:



∑ y+z ∑√ x
≥2 .
x y+z

∑y+z ∑ x
9.18. Dacă x, y, z > 0, atunci: ≥4 .
x y+z
9.19. Dacă x, y, z > 0, atunci:
( ) ( )
1 1 1 x y z
(x + y + z) + + ≥3+4 + + ≥ 9.
x y z y+z z+x x+y

9.20. Dacă x, y, z > 0, atunci:


( )
2 2 1 2 1 1
(x + y + z ) + +
x2 y 2 z 2
{( ) ( )}
x y z y z x
≥ 3 + 2 max + + , + + .
y z x x y z

9.21. Dacă x, y, z > 0, atunci:


( ) ( )
1 1 1 x y z (x + 1)(y + 1)(z + 1)
3+(x+y+z)+ + + + + + ≥ 3· .
x y z y z x xyz + 1

1 1 1
9.22. Fie a, b, c > 0 şi x = a + , y = b + , z = c + . Să se demonstreze că:
b c a
xy + yz + zx ≥ 2(x + y + z).

233
9.23. Să se demonstreze că pentru orice a, b, c, d > 0 are loc inegalitatea:

a4 b + b4 c + c4 d + d4 a ≥ abcd(a + b + c + d).

9.24. Dacă a, b, c, d > 0, atunci:

∑ (a + b)(a + c)(b + c) (∑ )2
1
≥2 · abcd.
d a

9.25. Dacă x1 , x2 , . . . , xn > 0, atunci:


√ √ √
x1 + x2 + . . . + xn + x2 + x3 + . . . + xn + . . . + xn

≥ x1 + 22 x2 + . . . + n2 xn .

9.26. 1) Demonstraţi că pentru orice a1 , a2 , . . . , an > 0 avem:


( )
1 2 n 1 1 1
+ + ... + <4 + + ... + .
a1 a1 + a2 a1 + a2 + . . . + an a1 a2 an

Arătaţi că constanta 4 din membrul drept:


2) se poate ı̂nlocui cu 2;
3) nu poate fi ı̂nlocuită printr-un număr mai mic.


n
9.27. Dacă S = xk , unde xk > 0, pentru orice k = 1, n, atunci:
k=1
√ √

n
S − xk ∑ n
xk
≥ (n − 1) .
xk S − xk
k=1 k=1

9.28. Fie x, y, z ∈ R cu x + y + z = 1. Să se arate că:

x2 + y 2 + z 2 ≥ 4(xy + yz + zx) − 1.

În ce caz are loc egalitatea?

9.29. Dacă x, y, z > 0 ı̂ndeplinesc condiţia x + y + z = 1, atunci:



12xyz + x2 + y 2 + z 2 ≤ 1.

234
9.30. Dacă x, y, z > 0 ı̂ndeplinesc relaţia x + y + z = 1, atunci:
yz zx xy 1
+ + ≤ .
x+1 y+1 z+1 4

9.31. Dacă x1 , x2 , . . . , xn > 0 şi x1 x2 . . . xn = 1, atunci:


( )( )( n )
∑n ∑ n
1 ∑ ∑
n
1
1+ xk 1+ xk +
xk xk
k=1 k=1 k=1 k=1
( )2 ( )2 

n ∑
n
1
≥ n 1 + xk + 1+ .
xk
k=1 k=1

9.32. Dacă x, y, z ∈ R sunt astfel ı̂ncât xyz = 1 şi n ∈ N, atunci:


n n n
x4 + y 4 + z 4 ≥ x + y + z.

9.33. Să se demonstreze că pentru orice n ∈ N are loc inegalitatea:

(2n2 + 3n + 1)n ≥ 6n (n!)2 .

9.34. Să se arate că pentru orice n ∈ N∗ − {1} are loc inegalitatea:
√ √
3 √
2 + 3 + . . . + n n < 2.

9.35. Dacă a, b, c ∈ R şi a3 + b3 + c3 = 0, atunci:

(a2 + b2 + c2 )3 ≤ [(a − b)2 + (b − c)2 + (c − a)2 ](a4 + b4 + c4 ).

9.36. Dacă a1 , a2 , . . . , an ∈ R (n ∈ N, n ≥ 2) şi

n(n − 2)
a1 + a2 + . . . + an + min{a1 , a2 , . . . , an } ≥ 0,
2n − 1
atunci ( )3
a1 + a2 + . . . + an
a31 + a32 + ... + a3n ≥n .
n

9.37. Demonstraţi că dacă x, y, z > 0 şi x2 + y 2 + z 2 = 3 xyz, atunci:

3 xyz ≤ x + y + z ≤ 3.

235
9.38. Dacă x, y, z > 0 şi x2 + y 2 + z 2 = 1, atunci:

x y z 3 3
+ + ≥ .
1 − x2 1 − y 2 1 − z 2 2

9.39. Demonstraţi că dacă x, y, z > 0 şi x2 + y 2 + z 2 = 1, atunci:



x y z 545
+ + ≥ .
1 − x4 1 − y 4 1 − z 4 4

9.40. Se consideră n numere reale x1 , x2 , . . . , xn astfel ı̂ncât

x21 + x22 + . . . + x2n = 1

şi un ı̂ntreg k ≥ 2. Să se arate că există n numere ı̂ntregi a1 , a2 , . . . , an


nu toate nule astfel ı̂ncât

|ai | ≤ k − 1 pentru orice i ∈ {1, 2, . . . , n} şi



(k − 1) n
|a1 x1 + a2 x2 + . . . + an xn | ≤ .
kn − 1
9.41. Dacă x1 , x2 , . . . , xn > 0 şi x1 + x2 + . . . + xn = 1, atunci:
∏n ( )
1
− 1 ≥ (n2 − 1)n .
x2k
k=1


n ∑
n
9.42. Fie x1 , x2 , . . . , xn ∈ R astfel ı̂ncât |xk | = 1 şi xk = 0. Să se arate
k=1 k=1
că:

∑n 1( )

ak xk ≤ max a − min a , unde a1 , a2 , . . . , an ∈ R.
2 1≤k≤n k 1≤k≤n k
k=1
] [
1
9.43. Să se arate că pentru orice x, y, z ∈ 0, are loc inegalitatea:
2
3
xy + yz + zx − x − y − z + ≥ 0.
4

9.44. Dacă f, g : R → R sunt două funcţii, să se arate că există x, y ∈ [0, 1]
astfel ı̂ncât:
1
|xy − f (x) − g(y)| ≥ .
4

236
9.45. Fie x, y ∈ R astfel ı̂ncât 1 ≤ x2 − xy + y 2 ≤ 2. Să se arate că:
2
1) ≤ x4 + y 4 ≤ 8;
9
2
2) x2n + y 2n ≥ n , pentru orice n ∈ N, n ≥ 3.
3
9.46. Să se demonstreze inegalitatea:

1 − (2x + y + z) + 2xz + xy + yz ≥ 0,

ı̂n ipoteza că x, y, z ∈ [0, 1] şi x ≤ y ≤ z. În ce cazuri avem egalitate?

9.47. Demonstraţi că dacă x1 , x2 , . . . , xn ∈ [0, 1] şi s = x1 + x2 + . . . + xn ,


atunci:
1
1 − s ≤ (1 − x1 )(1 − x2 ) . . . (1 − xn ) ≤ .
1+s
9.48. Fie xk ≥ 1 sau xk ∈ (0, 1], pentru orice k ∈ {1, 2, . . . , n}. Notăm
( )
1 1 1
A = (x1 + x2 + . . . + xn ) + + ... + − n2 şi
x1 x2 xn

B = n(x21 + x22 + . . . + x2n ) − (x1 + x2 + . . . + xn )2 .


Se ştie că A ≥ 0 şi B ≥ 0. Determinaţi max(A, B).

9.49. Dacă xk ∈ [0, 1] şi S = x1 + x2 + . . . + xn , atunci:


n
xk ∏ n
+ (1 − xk ) ≤ 1.
S − xk + 1
k=1 k=1

9.50. Dacă xk ≥ k, pentru orice k ∈ {1, 2, . . . , n}, atunci:


x1 + x2 + . . . + xn √ √ √
≥ x1 − 1 + 2(x2 − 2) + . . . + n(xn − n).
2

9.51. Considerăm numerele a1 , a2 , . . . , an , b1 , b2 , . . . , bn ∈ {1, 2, 3} (nu neapă-


rat distincte). Să se demonstreze că f (n), cea mai mică valoare strict
pozitivă a expresiei
∑ n
√ ∑n

E= ai − bj
i=1 j=1

1 −3
verifică f (n) > n .
4, 15

237
9.52. (Inegalitatea lui Popoviciu) Fie f : [a, b] → R o funcţie convexă. Să se
arate că pentru orice x, y, z ∈ [a, b]:
( )
f (x) + f (y) + f (z) x+y+z
+f
3 3
[ ( ) ( ) ( )]
2 x+y y+z z+x
≥ f +f +f .
3 2 2 2

9.53. Fie a, b, c, d ≥ 0 astfel ı̂ncât a ≥ b şi c ≥ d. Să se arate că:


√ √ √
(a + c)n − (b + d)n ≥ an − bn + cn − dn .
n n n

9.54. Dacă 0 < x1 ≤ x2 ≤ . . . ≤ xn , atunci:


x1 x2 xn x2 x3 x1
+ + ... + ≥ + + ... + .
x2 x3 x1 x1 x2 xn

9.55. Demonstraţi inegalitatea următoare pentru numerele pozitive a1 ≥ a2 ≥


. . . ≥ an :

a21 − a22 + a23 − . . . + (−1)n−1 a2n ≥ (a1 − a2 + . . . + (−1)n−1 an )2 .

√ √ √
9.56. Demonstraţi că dacă a > b > c şi p+ q≤ r, atunci:

q(a − b)(a − c) + p(b − a)(b − c) + r(c − a)(c − b) ≥ 0.

9.57. Fie I ⊂ R un interval, f : I → R+ o funcţie crescătoare şi g : I → R+ o


funcţie descrescătoare. Dacă a, b, c ∈ I, să se demonstreze inegalitatea:

(a − b)f (a)g(b) + (b − c)f (b)g(c) + (c − a)f (c)g(a) ≥ 0.

9.58. Fie f : (0, ∞) → (0, ∞) o funcţie crescătoare şi x1 , x2 , . . . , xn > 0.


Atunci:

n
xk
( ) k=1

n ∏
n n

(f (xk ))xk ≥ f (xk ) .


k=1 k=1

Dacă f este descrescătoare atunci semnul se schimbă.

238
9.59. 1) Găsiţi minimul funcţiei
√ √
f (x) = x2 − 2mx + n2 + x2 − 2px + q 2 ,
unde |m| < |n| şi |p| < |q|.
2) Găsiţi maximul funcţiei
√ √
f (x) = x2 + 2mx + n2 − x2 − 2px + q 2 ,
unde n > m > 0 şi q > p > 0.
2a2 − x2 − y 2
9.60. Găsiţi minimul funţiei pentru x, y ∈ [0, a], a > 0.
a2 − xy
( )
√ 9 2
9.61. Găsiţi valoarea minimă a expresiei (x − y) +
2 2−x − 2 pentru
√ y
0 < x < 2 şi y > 0.
9.62. Să se afle valoarea minimă a funcţiei f : R → [0, ∞):
f (x) = |x − a1 | + |x − a2 | + . . . + |x − an |,
a1 , a2 , . . . , an fiind numere reale distincte.
9.63. Găsiţi minimul funcţiei f : [0, 1) → R dată de legea:
1 1
f (x) = √ + √ ,
n
1+x n
1−x
unde n ≥ 2, n ∈ N.
9.64. Să se demonstreze că o condiţie suficientă ca ı̂ntre
√ numerele naturale a

şi b să existe cel puţin un cub perfect este a − b > 1.
3 3

9.65. Fie k
∑ a1 , a2 , . . . , ak numere naturale distincte astfel ı̂ncât toate cele 2 sume
εi ai (εi = 0 sau 1) sunt distincte. Arătaţi că:
i
( ) ( )

k
1 1
max =2 1− k .
ai 2
i=1

9.66. Notăm prin [a, b] cel mai mic multiplu comun al numerelor a şi b. Să se
arate că pentru orice n + 1 numere naturale nenule a0 < a1 < . . . < an
este adevărată inegalitatea:
1 1 1 1
+ + ... + ≤ 1 − n,
[a0 , a1 ] [a1 , a2 ] [an−1 , an ] 2
dacă: 1) n = 2; 2) n = 3; 3) n ≥ 3.

239
9.2 Probleme propuse
 9.1. Demonstraţi că inegalitatea
(x + 2)(x + 4) . . . (x + 2n) 1
<
(x + 1)(x + 3) . . . (x + 2n + 1) x+n
n2
are loc pentru orice x ≥ (unde n ∈ N).
2
 9.2. Să se arate că dacă x, y, z > 0 are loc inegalitatea:
x y z 1
+ + ≤ .
13x + y + z 13y + z + x 13z + x + y 5
x+y+z xy + yz + zx
 9.3. Dacă x, y, z > 0, atunci: √ ≥ ∑√ .
3 x2 + xy + y 2

 9.4. Demonstraţi că pentru orice x, y, z, t > 0 are loc inegalitatea:


x y z t
+ + + ≥ 2.
y+z z+t t+x x+y

 9.5. Dacă a, b, c, d > 0, atunci: S1 S2 S3 ≥ 3(S14 · S4 + S32 ), unde


∑ ∑ ∑
S1 = a, S2 = ab, S3 = abc şi S4 = abcd.

 9.6. Dacă x, y, z > 0 şi xyz = 1, atunci:


x y z 3
+ + ≥ .
xy + 1 yz + 1 zx + 1 2

 9.7. Arătaţi că:


 
  1
max  min |α1 x1 + α2 x2 + α3 x3 | = √ .
x21 +x22 +x23 =1 αi ∈{−1,0,1} 21
α21 +α22 +α23 ̸=0

Generalizare.

n
 9.8. Fie xk ∈ (0, 1), k = 1, n cu xk = 1. Să se arate că:
k=1


n
xk n
> .
1 − xk n−1
k=1

240
 9.9. Fie n ∈ N∗ şi x1 , x2 , . . . , x2n > 0 cu x1 + x2 + . . . + x2n = 1. Să se arate
că:
1
(x1 x2 . . . xn )2 + (x2 x3 . . . xn+1 )2 + . . . + (x2n x1 . . . xn−1 )2 < .
n2n

 9.10. Fie 0 < a < b şi a ≤ x1 ≤ x2 ≤ . . . ≤ xn ≤ b. Dacă f : [a, b] → R+ este o


funcţie crescătoare, atunci:
( ) ( )
x1 + 2x2 + . . . + nxn (n + 1)(x1 + x2 + . . . + xn )
f ≥f .
n 2n
√ √
 9.11. Fie funcţia f : [a, b] → R, f (x) = x − a + b − x, a < b.
[ ]
a+b
a) Să se arate că f este crescătoare pe a, şi descrescătoare pe
[ ] 2
a+b
,b .
2
b) Să se găsească extremele funcţiei f .

 9.12. Determinaţi numerele reale a şi b astfel ı̂ncât expresia max[x2 + ax + b]


să fie minimă, x ∈ [−1, 1].

 9.13. Fie m ∈ N∗ . Dacă numerele naturale x, y, z verifică ecuaţia

xyz = m(x + y + z)

să se arate că max(x, y, z) ≤ m2 + 2m.


( )2 ( )
1 + x3 + x6 1 + x4
 9.14. Dacă x ≥ 0, atunci ≤ .
3 2
1
 9.15. Fie f : R → R, f (x) = ax2 + bx + c, unde a, b, c ∈ R, a > 0 şi b ≥ . Să
8a
se arate că f (D) ≥ 0, D fiind discriminantul ecuaţiei f (x) = 0.

 9.16. Fie x, y, z ∈ Z astfel ı̂ncât x2 + y 2 + 3 ≥ 6xy. Arătaţi că: x2 + y 2 ≥ 6xy.

 9.17. Dacă −1 ≤ e < d < c < b < a ≤ 1, atunci:


1 1 1 1
+ + + ≥ 8.
a−b b−c c−d d−e
√ √ √ √
 9.18. Dacă x, y, z ≥ 0, atunci: xy + yz + zx ≥ 6
18xyz(x + y + z).

241
 9.19. Numerele b1 , b2 , b3 , b4 , b5 reprezintă o permutare a numerelor a1 , a2 ,
a3 , a4 , a5 ce aparţin intervalului [1, 2]. Să se afle valoarea maximă a
expresiei:
a1 a1 + a2 a1 + a2 + a3 + a4 + a5
+ + ... + .
b1 b1 + b2 b1 + b2 + b3 + b4 + b5
√ √
 9.20. Să se demonstreze inegalitatea: 11 1994 10 > 10 + 1000
10.

 9.21. Dacă a, b, c ≥ 0 astfel ı̂ncât a + b + c = 1, atunci

1
a2 (b + c) + b2 (c + a) + c2 (a + b) ≤ .
4

 9.22. i) Fie x, y, z ≥ 0 astfel ı̂ncât x + y + z = 1.


9
Să se arate că pentru orice a ≤ :
4
1 + (9 − 4a)xyz 9−a
0 ≤ xy + yz + zx − axyz ≤ ≤ .
4 27

9
ii) Numărul a = este cel mai mare număr real cu proprietatea că:
4
9−a
xy + yz + zx − axyz ≤ , pentru orice x, y, z ≥ 0 şi x + y + z = 1.
27

 9.23. Fie n ∈ N, n ≥ 2. Oricare ar fi xi ∈ [1, n−1], i = 1, n, are loc inegalitatea:

n(x1 + x2 + . . . + xn )2 ≥ 4(n − 1)(x21 + x22 + . . . + x2n ).

 9.24. Fie a, b, c ≥ 0 astfel ı̂ncât a + b + c = 1. Să se demonstreze inegalitatea

4(ab + bc + ca)2 + 7(ab + bc + ca) − 21abc ≤ 2.

 9.25. Fie λ > 0 şi xλ rădăcina reală unică şi pozitivă a ecuaţiei x3 +3x+λ = 0.
Să se arate că pentru orice a, b ∈ C, ecuaţia

z 3 + az 2 + bz + a + λ = 0
a
are cel puţin o rădăcină z1 pentru care |z1 | > xλ sau xλ = .
3

242
 9.26. Fie z1 , z2 , . . . , zn numere complexe astfel ı̂ncât |z1 | + |z2 | + . . . + |zn | = 1.
Să se arate că se pot alege dintre acestea k numere astfel ı̂ncât modulul
1
sumei lor să fie mai mare sau egal cu .
6
x z
 9.27. Găsiţi valoarea minimă a expresiei + pentru 1 ≤ x ≤ y ≤ z ≤ t ≤
y t
1994.

 9.28. Se dau numere reale a1 , a2 , . . . , an . Să se determine n numere x1 , x2 , . . .,


xn cu x1 ≤ x2 ≤ . . . ≤ xn astfel ı̂ncât max |ai − xi | să fie minim.
i=1,n

 9.29. Fie 11 numere naturale 1 ≤ a1 < a2 < . . . < a11 cu proprietatea că
pentru orice n ∈ {1, 2, . . . , 1500} există i1 < i2 < . . . < ik astfel ı̂ncât
ai1 + ai2 + . . . + aik = n. Găsiţi cea mai mică valoare posibilă a lui a10 .

 9.30. Să se demonstreze că pentru numerele reale b1 , b2 , . . . , bn ≥ 1 are loc


inegalitatea
( )
b 1 b2 bn 1
+ + ... + ≤ An − Hn + 2n,
b 2 b3 b1 2

unde prin An şi Hn s-au notat media aritmetică, respectiv armonică a


numerelor b1 , b2 , . . . , bn .

 9.31. Fie A ⊂ (0, 1] o mulţime cu patru elemente. Să se arate că există x, y ∈ A
cu x ̸= y astfel ı̂ncât:
√ √ 1
0<x 1 − y2 − y 1 − x2 < .
2
( )2
an + an−1 b + . . . + abn−1 + bn
a+b
 9.32. Dacă a, b > 0, atunci ≤ .
2 n+1
{ xy }
 9.33. Fie A = (x, y) | (x, y) ∈ N∗ × N∗ astfel ı̂ncât |x − y| ≥ . Arătaţi că
25
A conţine cel mult 9 elemente.

 9.34. Fie xi ∈ R, i = 1, 7. Să se arate că există i, j ∈ {1, 2, . . . , 7}, i ̸= j astfel


ı̂ncât 3(xi − xj )2 ≤ (1 + xi xj )2 . Generalizare.

 9.35. Dacă xi , ai , bi ∈ R∗+ , atunci

x21 x22
+ + ...+
(ax2 + bx3 )(ax3 + bx2 ) (ax3 + bx4 )(ax4 + bx3 )

243
x2n n
+ ≥ .
(ax1 + bx2 )(ax2 + bx1 ) (a + b)2
[x] [2x] [nx]
 9.36. Dacă x ≥ 0 şi n ∈ N, atunci: [nx] ≥ + + ... + .
1 2 n
 9.37. Dacă x, y, z > 0, arătaţi că:
2x2 (y + z) 2y 2 (z + x) 2z 2 (x + y)
+ + ≤ x + y + z.
(x + y)(x + z) (y + z)(y + x) (z + x)(z + y)

În ce caz avem egalitate?



n ∑
n
 9.38. Dacă xi ∈ [0, 2], atunci: |xi − xj |2 ≤ n2 . În ce caz avem egalitate?
i=1 j=1

 9.39. Să se demonstreze că 6 < 3 3 < 7 şi apoi să se arate că prima inegalitate
o implică pe a doua.
 9.40. Fiind date numerele reale ai > bi , pentru orice i = 1, n + 1 şi expresiile
( k )( k ) ( k )2
∑ ∑ ∑
Ek = 2
ai bi −
2
ai bi
i=1 i=1 i=1

să se arate că Ek+1 ≥ Ek .


Aplicaţie: Fiind date numerele reale ai , pentru orice i = 1, 2n să se arate
că:
( 2n ) ( 2n ) ( n )( n )
∑ k2 ∑ n2 (n + 1)2 ∑ k2 ∑
ak + ≥ ak + n2 (2n + 1)2 .
ak 4 ak
k=1 k=1 k=1 k=1

 9.41. Fiind date numerele pozitive ai , i = 1, n + 1, se consideră expresiile


v
u n
∑ n
u∏
En = ak − n tn
ak .
k=1 k=1

Să se arate că En+1 ≥ En .


Aplicaţie: 1) Fiind date numerele pozitive ai , bi , i = 1, 2n, să se demon-
streze următoarele inegalităţi:
 vu 2n
v
u 2n 
u ∏ ak u ∏ bk
t
2  2n t
+ 2n 
ak + bk ak + bk
k=1 k=1

244
v v
u n u n
u∏ ak u∏ bk
≤ t
n
+t
n
+ 1 ≤ 2.
ak + bk ak + bk
k=1 k=1
√ √
2) (n + 1) n+1 (n + 1)! − n n n! < n + 1, pentru orice n ∈ N, n ≥ 1.

 9.42. Fie f : I → R o funcţie convexă (concavă), xi ∈ I, i = 1, n + 1 şi notăm


 
∑k
∑k  xi 
 i=1 
Sk = f (xi ) − kf  .
 k 
i=1

Să se demonstreze că Sk+1 ≥ Sk (Sk+1 ≤ Sk ).


( π)
Aplicaţie: Considerând funcţia f : 0, → R,
2
1
f (x) = sin x, xk =
k(k + 1)
deduceţi inegalitatea:
1 1 1
sin ≥ (n + 1) sin − n sin .
(n + 1)(n + 2) n+2 n+1

 9.43. Dacă r, s, t ∈ R astfel ı̂ncât 1 ≤ r ≤ s ≤ t ≤ 4, determinaţi valoarea


minimă a expresiei
(s )2 ( t )2 (
4
)2
(r − 1) +
2
−1 + −1 + −1 .
r s t

 9.44. Determinaţi punctul de coordonate (a, b) ce aparţine dreptei de ecuaţie


2x − 3y − 8 = 0 astfel ı̂ncât expresia a2 + b2 este minimă.

 9.45. Dacă 0 < u < 2 şi v > 0, determinaţi valoarea minimă a expresiei
(√ )
9 2
(u − v)2 + 2 − u2 − .
v

 9.46. Pentru x, y, z ∈ R notăm Sn = xn + y n + z n , n ∈ N∗ . Arătaţi că

(x + y + z)Sp+1 ≥ 2(xy + yz + zx)Sp − 3xyzSp−1 ,

pentru orice p ∈ N număr par.

245
a4 b4 c4 a b c
 9.47. Dacă a, b, c > 0 astfel ı̂ncât abc = 1, atunci: + + ≥ + + .
b c a b c a
π
 9.48. Dacă x, y > 0 şi x + y ≤ , atunci (cos x)sin y + (cos y)sin x > 1.
2
 9.49. Fie P ∈ R[X], P (x) = an xn + an−1 xn−1 + . . . + a1 x + a0 , cu n par.
Presupunem ı̂ndeplinite condiţiile:
a) a0 > 0, an > 0,
b) (n − 1)(a21 + a22 + . . . + a2n ) ≤ 4 min(a20 + a2n ).
Să se arate că P (x) ≥ 0 pentru orice x ∈ R.

9.3 Soluţii
√ √ √
9.1. Se verifică uşor că 4x2 + 3 ≥ x2 + x + 1 ≥ x2 − x + 1 şi deci
condiţia ca cele trei numere să poată reprezenta lungimile laturilor unui
triunghi e suficient să verificăm inegalitatea
√ √ √
x2 − x + 1 + x2 + x + 1 > 4x2 + 3.

Ridicând ambii membri la pătrat ea se reduce la următoarele:



2 (x2 + 1)2 − x2 > 2x2 +1 ⇔ 4(x2 +1)2 −4x2 > 4x4 +4x2 +1 ⇔ 4 > 1.

Dacă notăm cu α unghiul opus laturii cu lungimea 4x2 + 3, avem:

2x2 + 2 − 4x2 − 3
cos α = √
2 x4 + x2 + 1
şi deci √
√ 3
sin α = 1− = √
cos2 α .
2 x4 + x2 + 1
Rezultă că aria triunghiului astfel format este
√ √
1√ 2 √ 3 3
S= x −x+1· x +x+1· √
2 = .
2 2 x4 + x2 + 1 4

9.2. Notând fn (x) = xn+2 + xn − nx + 2 şi ţinând cont că fn (x) ≥ 0, pentru
orice x ∈ R, rezultă: fn (1) ≥ 0 ⇔ 4 − n ≥ 0 ⇔ 4 ≥ n. Apar
următoarele cazuri:
i) Pentru n = 0, f0 (x) = x2 + 3 ≥ 0, pentru orice x ∈ R.

246
ii) Pentru n = 1, f1 (x) = x3 + 2 iar f1 (−2) < 0.
iii) Pentru n = 2, f2 (x) = x4 + x2 − 2x + 2 = x4 + 1 + (x − 1)2 ≥ 0,
pentru orice x ∈ R.
iv) Pentru n = 3, f3 (x) = x5 + x3 − 3x + 2, iar f3 (−2) < 0.
v) Pentru n = 4, f4 (x) = x6 + x4 − 4x + 2 iar f4 (3/4) < 0 (relaţie ce se
verifică prin calcul). Deci răspunsul căutat este n = 2.
9.3. 1) Pentru x = 2, y = z = −1 inegalitatea din enunţ se reduce la 0 ≥ 48m,
deci enunţul nu poate fi adevărat.
2) Luând x = y = z = 1 obţinem 3 ≥ m şi astfel ne rămâne a demonstra
că: (∑ ) (∑ ) ∏
x xyz ≥ 3 xy (−x + y + z).
Inegalitatea rezultă din următoarele:
(∑ )2 (∑ ) ∏
x ≥3 xy şi xyz ≥ (−x + y + z).
( √ )
x 3 x
9.4. Într-un reper cartezian considerăm punctele A , , (0, y) şi
2 2
( √ )
z 3 z
C − , . Inegalitatea din enunţ este echivalentă cu AB + BC ≥
2 2
AC, ceea ce are loc pentru orice trei puncte din plan.
9.5. Punând x = y = 1 şi z = 0 deducem că 6 ≥ 2n . Rezultă că n ∈ {0, 1, 2},
căci pentru n ≥ 3 avem: 2n > 6. Cazul n = 1 revine la

3(x + y + z) ≥ x + y + z ⇔ 2(x + y + z) ≥ 0,

inegalitate care nu este adevărată pentru orice x, y, z ∈ R căci luând de


exemplu x = y = z = −1 inegalitatea este falsă. Pentru n = 0 şi n = 2
inegalitatea are loc pentru orice x, y, z ∈ R.
2 2
9.6. Pentru x = y = z = 1 inegalitatea conduce la k ≤ . Dar pentru k =
3 3
inegalitatea se scrie
[ )]
1 5 (∑ 2 ∑
(x − y 2 )2 + (x2 − yz)2 ≥ 0
6 2
ceea ce evident are loc pentru orice x, y, z ∈ R. Valoarea căutată este
2
deci k = .
3

247
3 3
9.7. Deoarece a2 + ab + b2 ≥ (a + b)2 , b2 + bc + c2 ≥ (b + c)2 , c2 + ca + a2 ≥
4 4
3 2
(c + a) , este suficient să demonstrăm inegalitatea:
4
27(a + b)2 (b + c)2 (c + a)2 ≥ 64(ab + bc + ca)3 ,

echivalentă cu
27(S1 · S2 − S3 )2 ≥ 64S23 (1)
unde S1 = a + b + c, S2 = ab + bc + ca şi S3 = abc. Dacă a, b, c > 0, din
inegalităţile cunoscute
∑ ∑
S1 S2 ≥ 9S3 ⇔ a(b − c)2 ≥ 0 şi S12 ≥ 3S2 ⇔ (a − b)2 ≥ 0,

rezultă:
( )2
1 64 2 2
27(S1 S2 − S3 ) ≥ 27 S1 S2 − S1 S2
2
= S S ≥ 64S23 .
9 3 1 2

Ţinând seama de faptul că inegalitatea (1) este simetrică ı̂n raport cu
a, b, c şi rămâne neschimbată prin ı̂nlocuirea numerelor a, b, c respectiv cu
−a, −b, −c, ı̂n continuare vom considera numai cazul a < 0, b ≥ 0, c ≥ 0.
În plus, presupunem că S2 > 0, fiindcă ı̂n cazul S2 ≤ 0, inegalitatea (1)
devine evidentă. Deoarece inegalitatea S2 > 0 implică b > 0, c > 0,
bc
a>− , rezultă
b+c

bc b2 + bc + c2
S1 > − +b+c= >0 şi S3 < 0.
b+c b+c
Prin urmare:

27(S1 S2 − S3 )2 − 64S23 > 27S12 S22 − 64S23

= S22 [27(a2 + b2 + c2 ) − 10(ab + bc + ca)]


= S22 [27a2 + 22(b2 + c2 ) + 5(b − c)2 + 10(−a)(b + c)] > 0.
Atât ı̂n (1), cât şi ı̂n inegalitatea din enunţ, avem egalitate numai când
a = b = c sau când două din numerele a, b, c sunt egale cu zero.

9.8. După desfacerea parantezelor se obţine:

a2 [d2 + (c − d)2 ] − 2ab[(c − d)2 + cd] + b2 [(c − d)2 + c2 ] ≥ 0 ⇔

248
(c − d)2 (a2 − 2ab + b2 ) + a2 d2 − 2abcd + b2 c2 ≥ 0 ⇔

(c − d)2 (a − b)2 + (ad − bc)2 ≥ 0,


ceea ce este evident adevărat. Egalitatea are loc ı̂n următoarele cazuri:
1) c = d ̸= 0 şi a = b sau 2) c = d = 0 sau
3) a = b ̸= 0 şi c = d sau 4) a = b = 0.

9.9. Pentru orice y ∈ R avem:

∑ ∑ ∑
4
(yi − yj )2 = (yi − y + y − yj )2 ≤ 5 (yi − y)2 .
0≤i<j≤4 0≤i<j≤4 i=1

Luând yi astfel ı̂ncât |yi | < 1 şi xi − yi ∈ N, rezultă că numerele yi se


4
găsesc ı̂ntr-un interval de lungime (la ı̂nceput le alegem pozitive, iar
5
4
apoi dacă lungimea intervalului este mai mare ca scădem 1 din cel mai
5
1
mare număr yi ). Deci există 0 ≤ r, s ≤ 4 astfel ı̂ncât |yr − ys | ≤ şi
5
yr + ys 1 1
notăm y = . Rezultă |yr − y| ≤ şi |ys − y| ≤ . Apoi adunăm
2 10 10
1
sau scădem la toţi yi pentru a avea |yi − y| ≤ . Va rezulta:
2
[ ( ) ( )2 ]
∑ 1 2
1
(yi − yj )2 ≤ 5 2 +3 < 4.
10 2
0≤i<j≤4

9.10. Pornind de la inegalitatea evidentă:

[kxk+1 − (k + 1)xk + 1]2 ≥ 0, pentru orice k = 1, n − 1

după efectuarea calculelor obţinem:

kx2k+1 + 2xk+1 + k (k − 1)x2k + 2xk + k − 1


xk (xk+1 − xk ) ≤ − .
2(k + 1) 2k

Însumând ı̂n inegalitatea de mai sus scrisă pentru k de la 1 la n − 1


obţinem inegalitatea din enunţ. Egalitatea are loc numai dacă

kxk+1 − (k + 1)xk + 1 = 0, pentru orice k = 1, n − 1.

249
9.11. Fie a1 < a2 < . . . < an , 1 ≤ i < k ≤ n, min(ak − ai )2 = d2 , d > 0,
min a2i = a2j unde j este fixat. Astfel pentru i > j, ai > 0 iar pentru i < j,
ai < 0. Pentru i = 1, n notând bi = ai + (i − j)d, avem bi = b1 + (i − 1)d.
Astfel pentru i > j şi analog pentru i < j: ai ≤ bi ≤ −ai . Din cele două
inegalităţi de mai sus rezultă că a2i ≥ b2i , deci:

n ∑
n ∑
n
a2i ≥ b2i = (b21 + 2b1 d(i − 1) + (i − 1)2 d2 )
i=1 i=1 i=1
( )2
n−1 n(n2 − 1) 2 n(n2 − 1) 2
= n b1 + d + d ≥ d
2 12 12
(am folosit ı̂n calcul identităţile
k(k + 1) k(k + 1)(2k + 1)
1 + 2 + ... + k = şi 12 + 22 + . . . + k 2 = ).
2 6

Observaţie. Din egalitatea din enunţ rezultă că:

12 ∑ n
min (ak − ai ) ≤ 2
(ai + t)2
1≤i<k≤n n(n2 − 1)
i=1

are loc pentru orice t ∈ R.

9.12. Folosind inegalitatea mediilor avem:



1 x x x 1 4 x3 4
x+ 3 = + + + 3 ≥4· =√ √ .
x 3 3 3 x 27x3 3 3
x 1 √ √
= 3 ⇔ x2 = 3 ⇔ x = 3.
4
Egalitatea are loc numai când
3 x
9.13. Fie a = x2 , b = y 2 , cu x, y > 0. Folosind formula binomului lui Newton
şi identitatea

n−1
Cnk = 2n − 2,
k=1
inegalitatea de demonstrat se scrie succesiv astfel:

(x2 + y 2 )n − 2n xn y n ≥ x2n + y 2n − 2xn y n ⇔


n−1
x2n + y 2n + Cnk x2n−2k y 2k − (2n − 2)xn y n ≥ x2n + y 2n ⇔
k=1

250

n−1 ∑
n−1
Cnk x2n−2k y 2k − Cnk xn y n ≥ 0 ⇔
k=1 k=1


n−1 ∑
n−1 ∑
n−1
Cnk x2n−2k y 2k + Cnk y 2n−2k x2k −2 Cnk xn y n ≥ 0 ⇔
k=1 k=1 k=1


n−1 ∑
n−1
Cnk xn y 2k (xn−2k − y n−2k ) + Cnk x2k y n (y n−2k − xn−2k ) ≥ 0 ⇔
k=1 k=1


n−1
Cnk (xn−2k − y n−2k )x2k y 2k (xn−2k − y n−2k ) ≥ 0 ⇔
k=1


n−1
Cnk (xn−2k − y n−2k )2 x2k y 2k ≥ 0, pentru orice n ≥ 2.
k=1
Pentru n = 1 inegalitatea revine la
√ √
a+b √ ( a − b)2
− ab ≥
2 2
ceea ce este evident.

9.14. Fie α, β, γ, δ arctangentele numerelor date aranjate ı̂n ordine crescătoare,


atunci:
π π
− < α ≤ β ≤ γ ≤ δ < < α + π.
2 2
Numerel β, γ, δ ı̂mpart segmentul [α; α +4] ı̂n patru segmente. Lungimea
π
oricăruia dintre aceste segmente nu depăseşte ; ı̂n rolul lui x şi y putem
4
lua tangentele capătului din stânga şi din dreapta. Într-adevăr, dacă de
π
exemplu β − α ≤ , atunci cum β − α ≥ 0, avem:
4
tg β − tg α x−y
0 ≤ tg (β − α) ≤ 1, tg (β − α) = = ,
1 + tg βtg α 1 + xy
π
inegalitatea rezultă. În cazul (α + π) − δ ≤ folosim mai ı̂ntâi egalitatea
4
tg (α + π − δ) = tg (α − δ).

9.15. Simplificând cele trei fracţii cu (x + y + z)2 expresia din membrul stâng
al inegalităţii devine:
α β γ
E= + + ,
(2α + β + γ)(β + γ) (2β + α + γ) (2γ + α + β)(α + β)

251
x y z
α= , β= , γ= şi α + β + γ = 1.
x+y+z x+y+z x+y+z
Putem scrie:
α β γ
E= + +
(1 + α)(1 − α) (1 + β)(1 − β) (1 + γ)(1 − γ)
α β γ
= + + .
1−α 2 1−β 2 1 − γ2
x
Considerăm funcţia f : (0, 1) → R, f (x) = . Avem
1 − x2
2x3 + 6x
f ′′ (x) = > 0, pentru orice x ∈ (0, 1)
(1 − x2 )3
şi din inegalitatea lui Jensen
( )
f (α) + f (β) + f (γ) α+β+γ
≥f ,
3 3
( )
1 9
rezultă E ≥ 3f = , ceea ce trebuia demonstrat.
3 8
9.16. Avem

∑ x2 ∑ x2 (y + z) x2 (y + z)
= =
(x + y)(x + z) (x + y)(y + z)(z + x) (x + y)(y + z)(z + x)
(x + y)(y + z)(z + x) − 2xyz 2xyz 3
= =1− ≥ ,
(x + y)(y + z)(z + x) (x + y)(y + z)(z + x) 4
√ √ √
deoarece (x + y)(y + z)(z + x) ≥ 2 xy · 2 yz · 2 zx = 8xyz şi deci
2xyz 1
≤ .
(x + y)(y + z)(z + x) 4
Egalitatea se obţine evident dacă şi numai dacă x = y = z.
9.17. Folosim pentru rezolvarea problemei inegalitatea lui Abel ı̂n cazul partic-
ular n = 3. Dacă a1 ≥ a2 ≥ a3 > 0 şi b1 ≥ 0, b1 + b2 ≥ 0, b1 + b2 + b3 ≥ 0,
atunci:
a1 b1 + a2 b2 + a3 b3 ≥ 0.
Demonstraţia rezultă imediat notând α = a2 − a3 ≥ 0 şi β = a1 − a2 =
a1 − a3 − α ≥ 0 şi observând că:

a1 b1 + a2 b2 + a3 b3 = (a3 + α + β)b1 + (a3 + α)b2 + a3 b3

252
= a3 (b1 + b2 + b3 ) + α(b1 + b2 ) + βb1 ≥ 0.
Fie acum, fără a micşora generalitatea problemei: 0 < x ≤ y ≤ z. Notăm
√ √ √
y+z x+z x+y
= a, = b, = c.
x y z

Relaţia de demonstrat se scrie succesiv:


∑ ∑1 ∑ a2 − 2 ∑ y + z − 2x
a≥2 ⇔ ≥0 ⇔ √ ≥ 0.
a a x(y + z)

Punând
1 1 1
a1 = √ , a2 = √ , a3 = √ ,
x(y + z) y(z + x) z(x + y)

b1 = y + z − 2x, b2 = x + z − 2y, b3 = x + y − 2z
sunt verificate ipotezele din inegalitatea lui Abel şi deci:

a1 b1 + a2 b2 + a3 b3 ≥ 0.

√ 4x 2x
9.18. Deoarece y + z ≥ 2 yz rezultă ≤ √ . Punând x = a2 , y = b2 ,
y+z yz
z = c2 rămâne să arătăm că:
∑ b2 + c2 ∑ 2a2 ∑ ∑
≥ ⇔ b2 c2 (b2 + c2 ) ≥ 2 a4 bc ⇔
a2 bc
∑ ∑ ∑
a4 (b2 + c2 ) ≥ 2 a4 · bc ⇔ a4 · (b − c)2 ≥ 0,
ceea ce este evident. Semnul egal are loc dacă şi numai dacă a = b =
c ⇔ x = y = z.

9.19. Prima inegalitate se reduce la inegalitatea din problema 9.18. A doua se


reduce la cunoscuta inegalitate
x y z 3
+ + ≥ .
y+z z+x x+y 2

9.20. Folosind inegalitatea Cauchy-Buniakovski, avem:


( ) ( )2
1 1 1 x y z
2 2
(x + y + z ) 2
2
+ 2+ 2 ≥ + + şi
y z x y z x

253
( ) ( )
1 1 1 y z x 2
2 2
(y + z + z ) 2
+ + ≥ + + .
x2 y 2 z 2 x y z
x y z y z x
Notăm a = + + şi b = + + . Din inegalitatea mediilor avem:
y z x x y z
√ √
x y z x y z y z x y z x
a= + + ≥3 3 · · = 3 şi b = + + ≥ 3 3 · · ≥ 3.
y z x y z x x y z x y z

Fără a restrânge generalitatea problemei presupunem că a ≥ b. Cazul


a ≤ b se tratează analog. Ţinând cont de inegalităţile de mai sus rămâne
să demonstrăm că dacă a ≥ 3, atunci a2 ≥ 3 + 2a. Dar a ≥ 3 implică
a2 ≥ 3a = 2a + a ≥ 2a + 3. Egalitatea se obţine numai dacă x = y = z.
Observaţie. Ţinând seama de faptul că a, b ≥ 3 deducem că
{( ) ( )}
x y z y z x
3 + 2 max + + , + + ≥9
y z x x y z

şi astfel inegalitatea din enunţ o ı̂mbunătăţeşte pe cunoscuta inegalitate:


( )
1 1 1
2 2
(x + y + z ) 2
+ + ≥ 9.
x2 y 2 z 2

9.21. Înmulţind inegalitatea de demonstrat cu xyz(1 + xyz) şi efectuând apoi


calulele ajungem la forma:

xy(x + 1)(z − 1)2 ≥ 0

ı̂n care egalitatea are loc dacă şi numai dacă x = y = z = 1.


1
9.22. Avem xyz − x − y − z = abc + ≥ 2, cu egalitate dacă şi numai dacă
abc
abc = 1. Deoarece
a 1 1 (b − 1)2
xy − 1 = ab + + > 0 şi x + y − 2 = a + + > 0,
c bc c b
x+y+2
inegalitatea xyz − x − y − z ≥ 2 se scrie sub forma z ≥ , iar
xy − 1
2(x + y) − xy
inegalitatea de demonstrat sub forma z ≥ . Vom arăta că
x+y−2

x+y+2 2(x + y) − xy
≥ .
xy − 1 x+y−2

254
Sub altă formă, această inegalitate devine:

(x + y)2 + 2(1 − xy)(x + y) + x2 y 2 − xy − 4 ≥ 0 ⇔

(x + y + 1 − xy)2 + xy − 5 ≥ 0. (1)
Deoarece
√ √ √
x+y +1−xy ≥ 2 xy +1−xy şi 2 xy +1−xy ≥ 0 pentru xy ≤ 3+2 2,
√ √
apar două cazuri: xy ∈ (3 + 2 2, ∞) şi xy ∈ (1, 3 + 2 2]. În primul caz
inegalitatea (1) este evidentă, deoarece xy − 5 > 0. În al doilea caz, este

suficient să arătăm că (2 xy + 1 − xy)2 + xy − 5 ≥ 0. Această inegalitate

rezultă scriind-o sub forma (xy − 1)( xy − 2)2 ≥ 0. Egalitatea se obţine
din inegalitatea din enunţ când abc = 1, x = y şi xy = 4, adică atunci
când a = b = c.

9.24. Din inegalitatea Cauchy-Buniakovski avem:


( )2 ( )
1 1 1 1 1 1 1 1
·1+ ·1+ ·1+ ·1 ≤4 + + +
a b c d a2 b2 c2 d2
(∑ )2 ∑
1 8abc ∑ (a + b)(b + c)(c + a)
2abcd ≤ ≤
a d d
căci 8abc ≤ (a + b)(b + c)(c + a) din inegalitatea mediilor. Semnul egal
se obţine dacă şi numai dacă a = b = c = d.

9.25. Demonstrăm prin inducţie următoarea inegalitate: Dacă a1 ≥ a2 ≥ . . . ≥


an ≥ 0, atunci

a21 + 3a22 + . . . + (2n − 1)a2n ≤ (a1 + a2 + . . . + an )2 .

Inegalitatea este evidentă pentru n = 1. Presupunând că a1 ≥ . . . ≥


an ≥ an+1 ≥ 0, avem:

a21 +3a22 +. . .+(2n−1)a2n +(2n+1)a2n+1 ≤ (a1 +a2 +. . .+an )2 +(2n+1)a2n+1 .

Va trebui să mai verificăm că

(a1 + . . . + an )2 + (2n + 1)a2n+1 ≤ (a1 + . . . + an + an+1 )2

= (a1 + . . . + an )2 + 2an+1 (a1 + . . . + an ) + a2n+1 ,

255
ceea ce revine la nan+1 ≤ a1 + . . . + an şi rezultă din an+1 ≤ an , . . .,
an+1 ≤ a1 . Luând ı̂n inegalitatea de mai sus
√ √ √
a1 = x1 + x2 + . . . + xn , a2 = x2 + . . . + xn , . . . , an = xn

deducem că:

(x1 + x2 + . . . + xn ) + 3(x2 + x3 + . . . + xn ) + . . . + (2n − 1)xn


√ √ √
≤ ( x1 + x2 + . . . + xn + x2 + x3 + . . . + xn + . . . + xn )2 ⇔
√ √
x1 + 22 x2 + . . . + n2 xn ≤ x1 + x2 + . . . + xn
√ √
+ x2 + x3 + . . . + xn + . . . + xn
căci 1 + 3 + 5 + . . . + (2k − 1) = k 2 pentru orice k ∈ N∗ .

9.26. 1) Fie b1 , b2 , . . . , bn numerele a1 , a2 , . . . , an aşezate ı̂n ordine crescătoare.


Deoarece pentru orice i de la 1 la n:
i i
≤ şi
a1 + a2 + . . . + ai b1 + b2 + . . . + b i
1 1 1 1 1 1
+ + ... + = + + ... + ,
a1 a2 an b1 b2 bn
e suficient a demonstra inegalitatea pentru numerele bi :
( )
1 2 n 1 1
+ + ... + ≤4 + ... + .
b1 b1 + b2 b1 + b 2 + . . . + bn b1 bn

Avem evident
2 2 1
≤ =
b1 + b2 b1 + b1 b1
n+1
şi pentru orice k, 2 ≤ k ≤ :
2
2k − 1 2k − 1 2k − 1 2
≤ ≤ < şi
b1 + b2 + . . . + b2k−1 bk + bk+1 + . . . + b2k−1 kbk−1 bk−1
2k 2k 2k 2
≤ ≤ = ,
b1 + b2 + . . . + b2k bk+1 + . . . + b2k kbk bk
de unde rezultă inegalitatea:
1 2 n
+ + ... +
b1 b1 + b 2 b 1 + b2 + . . . + bn

256
( )
1 1 2 2 2 2 2 1 1 1
< + + + + + + + ... = 4 + + ... + .
b1 b 1 b1 b2 b2 b3 b3 b1 b2 bn
ai
2) Fie ti = , 1 ≤ i ≤ n. Folosind inegalitatea
i
( )
1 1 1
(x1 + x2 + . . . + xn ) + + ... + ≥ n2 ,
x1 x2 xn

pentru orice x1 , x2 , . . . , xn > 0 vom avea pentru orice m ≤ n:


m m m
= =
a1 + a2 + . . . + am t1 + 2t2 + . . . + mtm t1 + t2 + t2 + . . . + tm
( )
m 1 1 1 1 1
≤[ ] + + + + ... +
m(m + 1) 2 t1 t2 t2 t3 tm
2
( )
4 1 2 m
= + + ... +
m(m + 1)2 t1 t2 tm
( )
4 1 22 m2
= + + ... + .
m(m + 1)2 a1 a2 am
1
Adunând aceste relaţii pentru m de la 1 la n, coeficientul lui va fi
ai
[ ]
2 1 1 1
βi = 4i + + ... +
i(i + 1)2 (i + 1)(i + 2)2 n(n + 1)2

şi cum ( )
1 1 2j + 1 1 1 1
2
< · 2 2
= 2

j(j + 1) 2 j (j + 1) 2 j (j + 1)2
rezultă că ( )
1 1 1
βi < 4i ·
2
2
− < 2.
2 i (n + 1)2

3)( Fie ai = i. Atunci ) membrul stâng al( inegalităţii este) egal cu


1 1 1 1 1
2 + + ... + iar cel drept cu c 1 + + . . . + , unde c
2 3 n+1 2 n
este constanta cu care l-am ı̂nlocuit pe 4. Dacă c ≤ 2, atunci diferenţa:
( ) ( )
1 1 1 1 1
2 + + ... + − c 1 + + ... +
2 3 n+1 2 n

257
( )
1 1 1 2
= (2 − c) + + ... + + −c
2 3 n n+1
1 1 1
pentru n destul de mare va fi pozitivă deoarece şirul + + . . . +
2 3 n
este nemărginit.

9.27. Inegalitatea din enunţ se scrie succesiv:


∑n √ ∑ n √
S − xk xk
≥ (n − 1) ⇔
xk S − xk
k=1 k=1
(√ √ ) n √

n
S − xk xk ∑ xk
+ ≥n ⇔
xk S − xk S − xk
k=1 k=1


n n √

1 xk
S √ ≥n .
xk (S − xk ) S − xk
k=1 k=1

Fără a restrânge generalitatea problemei presupunem că x1 ≥ x2 ≥ . . . ≥


xn şi atunci avem

x1 (x2 + x3 + . . . + xn ) ≥ x2 (x1 + x3 + . . . + xn ),

deci
x1 (S − x1 ) ≥ x2 (S − x2 )
şi analoagele. Inegalitatea din enunţ rezultă acum din inegalitatea lui
Cebâşev:
( n )( n ) n √
∑ ∑ 1 ∑ xk
xk √ ≥n .
xk (S − x k ) S − xk
k=1 k=1 k=1

Observaţie. Problema reprezintă o generalizare a inegalităţii 9.17 care se


obţine pentru n = 3.

9.28. Din x + y + z = 1 rezultă x2 + y 2 + z 2 + 2(xy + yz + zx) = 1 şi deci


inegalitatea de demonstrat se poate scrie sub forma:

x2 + y 2 + z 2 ≥ 4(xy + yz + zx) − (x2 + y 2 + z 2 + 2xy + 2yz + 2zx) ⇔

2(x2 + y 2 + z 2 ) ≥ 2(xy + yz + zx) ⇔ (x − y)2 + (y − z)2 + (z − x)2 ≥ 0.


1
Egalitatea are loc dacă şi numai dacă x = y = z = .
3

258
9.29. Notând x = a2 , y = b2 , z = c2 inegalitatea din enunţ devine

12 · abc + a4 + b4 + c4 ≤ 1 ı̂n ipoteza a2 + b2 + c2 = 1.

Ţinând seama de ipoteză, inegalitatea de mai sus se poate scrie sub


forma:

12a2 b2 c2 (a2 + b2 + c2 ) ≤ [(a2 + b2 + c2 )2 − (a4 + b4 + c4 )]2 ,

şi avem succesiv:

12a2 b2 c2 (a2 +b2 +c2 ) ≤ [a4 +b4 +c4 +2a2 b2 +2b2 c2 +2c2 a2 −(a4 +b4 +c4 )]2 ⇔

3a2 b2 c2 (a2 + b2 + c2 ) ≤ (a2 b2 + b2 c2 + c2 a2 )2 .


Egalitatea se obţine dacă şi numai dacă
1
a2 = b2 = c2 ⇔ x = y = z = .
3
9.30. I-a soluţie. Inegalitatea din enunţ este echivalentă cu:
1 1 1 1
+ + ≤ sau
x(x + 1) y(y + 1) z(z + 1) 4xyz
1 1 1 1 1 1 1
+ + ≤ + + + .
x y z x + 1 y + 1 z + 1 4xyz
În continuare vom demonstra că:
1 1 1 9 1 1 1 1 1
+ + ≤ + ≤ + + + .
x y z 4 4xyz x + 1 y + 1 z + 1 4xyz
Prima inegalitate este echivalentă cu
(∑ ) (∑ ) (∑ )3
4 a ab ≤ a + 9abc

a b c
dacă luăm x = ,y= ,z= . Iar inegalitatea
a+b+c a+b+c a+b+c
de mai sus este echivalentă, după efectuarea calculelor cu

(−a + b + c)(a − b + c)(a + b − c) ≤ abc,

adică inegalitatea de la 9.3.2. A doua inegalitate revine la a demonstra


că dacă x = y = z = 1, atunci
1 1 1 9
+ + ≥ ,
x+1 y+1 z+1 4

259
ceea ce rezultă din inegalitatea mediilor:
(∑ ) (∑ )
1
(x + 1)
x+1

1 √
≥3· 3 · 3 3 (x + 1)(y + 1)(z + 1) = 9
(x + 1)(y + 1)(z + 1)
∑ (∑ ) ∑ 1 9
şi cum (x + 1) = x + 3 = 4, obţinem ≥ . Semnul
x+1 4
1
egal este evident are loc dacă şi numai dacă x = y = z = .
3
A II-a soluţie. Inegalitatea de demonstrat se mai scrie:
∑ (∑ )
1 1
xy − xyz ≤ .
x+1 4
Ţinând cont că media armonică este mai mică sau cel mult egală cu cea
aritmetică, avem:

3 (x + 1) 4 ∑ 1 9
∑ 1 ≤ 3
= sau
3 x+1
≥ .
4
x+1
Este suficient să arătăm că
9xyz 1
E(x, y, z) = xy + yz + zx − ≤ .
4 4
Vom demonstra această inegalitate cu ajutorul metodei lui Sturm. Pre-
supunem z fixat şi variem x şi y fără a schimba suma. Avem:
( )
9
E(x + a, y − a, z) − E(x, y, z) = a(y − x − z) 1 − z .
4
Dacă x < y şi 0 < a < y − x, ”apropiind” pe x de y, E(x, y, z) creşte
4 4 4
dacă z < şi scade dacă z > . Presupunem că x ≤ y ≤ z. Dacă z > ,
9 9 9
ı̂ndepărtând x şi y obţinem:

E(x, y, z) ≤ E(0, 1 − z, z).

Apropiiind acum pe 1 − z de z găsim:


( )
1 1 1
E(0, 1 − z, z) ≤ E 0, , = .
2 2 4

260
4 1
Dacă z < , atunci apropiind x de y, E(x, y, z) creşte şi deoarece x ≤
9 3
1
şi y ≥ , vom avea
3
( )
1 1
E(x, y, z) ≤ E ,y + x − ,z .
3 3
1 4 1
Fixând acum pe z = < şi apropiind y + z − de z, deducem că:
3 9 3
( ) ( )
1 1 1 1 1 1
E(x, y, z) ≤ E ,y + x − ,z ≤ E , , = .
3 3 3 3 3 4

Urmând raţionamentul de mai sus se poate vedea că maximul este atins
numai dacă
( ) { }
1 1 1 1 1
(x, y, z) = , , sau {x, y, z} = 0, , .
3 3 3 2 2

9.35. Fie sn = an + bn + cn , n ∈ N. Avem (cu ajutorul determinantului Van-


dermonde):

1 1 1 1 a a2 3 s1 s2

∆ = (a−b)2 (b−c)2 (c−a)2 = a b c · 1 b b2 = s1 s2 s3

a2 b2 c2 1 c c2 s2 s3 s4

= s4 (3s2 − s21 ) − s3 (3s3 − s1 s2 ) + s2 (s1 s3 − s22 ),


Ţinând seama că

(a − b)2 + (b − c)2 + (c − a)2 = 3s2 − s21 ,

obţinem:

[(a − b)2 + (b − c)2 + (c − a)2 ](a4 + b4 + c4 ) − (a2 + b2 + c2 )3

= s4 (2s2 − s21 ) − s32 = ∆ + s3 (3s3 − s1 s2 ).


Dar s3 = 0 şi ∆ ≥ 0, ceea ce demonstrează enunţul.

9.36. Vom utiliza pentru demonstraţie metoda inducţiei matematice. Pentru


n = 2, avem a1 + a2 ≥ 0 şi trebuie demonstrat că

4(a31 + a32 ) ≥ (a1 + a2 )3 ⇔ 3(a1 + a2 )(a1 − a2 )2 ≥ 0,

261
ceea ce este evident adevărat. În continuare presupunem că a1 ≥ a2 ≥
. . . ≥ an . Considerăm proprietatea adevărată pentru n = k ≥ 2, adică:
dacă
k2 − 1
a1 + a2 + . . . + ak−1 + · ak ≥ 0,
2k − 1
atunci ( )
a1 + a2 + . . . + ak 3
a1 + a2 + . . . + ak ≥ k
3 3 3
k
şi demonstrăm proprietatea pentru n = k + 1, adică: dacă

k 2 + 2k
a1 + a2 + . . . + ak + · ak+1 ≥ 0,
2k + 1
atunci
( )3
a1 + a2 + . . . + ak+1
a31 + ... + a3k+1 ≥ (k + 1) .
k+1
Vom arăta mai ı̂ntâi că dacă
k 2 + 2k
a1 + a2 + . . . + ak + ak+1 ≥ 0,
2k + 1
atunci
k2 − 1
a1 + a2 + . . . + ak−1 + · ak ≥ 0.
2k − 1
În cazul ak ≥ 0, deci a1 ≥ a2 ≥ . . . ≥ ak ≥ 0 inducţia este evidentă.
Dacă ak < 0, deci ak+1 ≤ ak < 0, este suficient să arătăm că:
( ) ( )
a1 + a2 + . . . + ak 3 a1 + a2 + . . . + ak+1 3
k + ak+1 ≥ (k + 1)
3
k k+1
ceea ce rezultă scriind această inegalitate sub forma
( )( )2
a1 + a2 + . . . + ak k+1 a1 + a2 + . . . + ak
+ · ak+1 − ak+1 ≥ 0.
k 2k + 1 k

9.37. Fie xyz = a. Atunci, din inegalitatea mediilor, rezultă că


√ √ √
3
3 a = x2 + y 2 + z 2 ≥ 3 · 3 x2 y 2 z 2 = 3 · a2 .

De aici deducem că


√ √ 3 √
a ≥ a2 ⇔ a3 ≥ a4 ⇔ a ≤ 1 ⇒ 3 a ≥ a

262
şi deci: √ √ √
x + y + z ≥ 3 3 a ≥ 3 a = 3 xyz.
Apoi din xy + yz + zx ≤ x2 + y 2 + z 2 rezultă:
(∑ )2 ∑ ∑ (∑ ) √
x = x2 + 2 xy ≤ 3 x2 = 9 a ≤ 9,

de unde x + y + z ≤ 3.

9.38. Vom demonstra mai ı̂ntâi că pentru orice x ∈ (0, 1) are loc inegalitatea:
2
x(1 − x2 ) ≤ √ .
3 3

Inegalitatea se scrie, folosind notaţia y = x 3, sub următoarele forme
echivalente:
2 √ √
x − x3 ≤ √ ⇔ 3x 3 − ( 3x)3 ≤ 2 ⇔ 3y − y 3 ≤ 2 ⇔
3 3

y 3 − 3y + 2 ≥ 0 ⇔ (y − 1)2 (y + 2) ≥ 0,
ceea ce√este evident. Semnul egal are loc dacă şi numai dacă y = 1 ⇔
x = 1/ 3. Din inegalitatea demonstrată rezultă că

x 3 3 2
≥ ·x
1 − x2 2
1
şi deci semnul egal are loc dacă şi numai dacă x = y = z = √ .
3

x 545 2
9.39. Demonstrăm mai ı̂ntâi că ≥ x , pentru orice x ∈ (0, 1).
1 − x4 4
Într-adevăr, considerând funcţia f : (0, 1) → R, f (x) = x(1 − x4 ), avem
1
f ′ (x) = 1 − 5x4 şi deci f ′ (x) = 0 ⇔ x = √ 4
.
5
( )
′ 1
Deoarece f (x) > 0 pentru x ∈ 0, √ 4
şi f ′ (x) < 0 pentru x ∈
( ) 5
1 1

4
, 1 , funcţia f admite un maxim ı̂n x = √ 4
şi deci
5 5
( )
4 1
f (x) = x(1 − x ) ≤ √
4
=f √ , pentru orice x ∈ (0, 1).
545 4
5

263
Dar √
x 545 2 4
≥ x ⇔ x(1 − x4 ) ≤ √ .
1−x 4 4 545
Prin urmare vom avea:
√ √
x y z 545 2 545
+ + ≥ 2 2
(x + y + z ) = .
1 − x4 1 − y 4 1 − z 4 4 4
{ }
k−1
9.40. Fie A = r − | r = 0, 1, . . . , k − 1 şi B mulţimea numerelor de
2
∑n
forma bi xi unde bi ∈ A. Evident B conţine k n elemente. Avem:
i=1
( )2 ( )( n )

n ∑
n ∑ ∑
n
k−1
bi xi ≤ b2i x2i = b2i şi |bi | ≤ , deci
2
i=1 i=1 i=1 i=1
( )2 ( )2

n
k−1 k−1 ∑ √ √ k−1 n
bi xi ≤n , de unde − n· ≤ bi xi ≤ n· .
2 2 2
i=1 i=1
Notând elementele mulţimii B cu c1 , c2 , . . . , c√ kn , rezultă că există s, t ∈
n · (k − 1)
{1, 2, . . . , k n } cu s ̸= t astfel ı̂ncât |cs − ct | ≤ . Deoarece
kn − 1

n ∑
n
cs − ct = bi x i − b′i xi
i=1 i=1

k−1 k−1
cu bi , b′i ∈ A, 0 ≤ i ≤ n şi bi = − + r, b′i = − + r′ cu
2 2
0 ≤ r, r′ ≤ k − 1, rezultă că

bi − b′i ∈ {−k + 1, −k + 2, . . . , k − 1}

şi notând ai = bi − b′i , 1 ≤ i ≤ n problema este rezolvată.

9.41. Din inegalitatea mediilor avem:


n (
∏ ) ∏ n ( )
1 1 − xk 1 + xk
−1 = ·
x2k xk xk
k=1 k=1


n
x1 + x2 + . . . + xk−1 + xk+1 + . . . + xn
=
xk
k=1

264

n
x1 + x2 + . . . + xk−1 + 2xk + xk+1 + . . . + xn
×
xk
k=1


n √
(n − 1) n−1 x1 x2 . . . xk−1 xk+1 . . . xn

xk
k=1

∏n (n + 1) n+1 x x . . . x
1 2
2
k−1 xk xk+1 . . . xn
×
xk
k=1
√ √
n−1
(x1 . . . xn )n−1 · n+1 (x1 . . . xn )n+1
= (n − 1) (n + 1) ·
n n
= (n2 − 1)n .
(x1 x2 . . . xn )2
1
Cazul de egalitate are loc dacă şi numai dacă xi = pentru orice i =
n
1, n.

9.42. Fără a restrânge generalitatea problemei putem presupune că

min ak = a1 şi max ak = an .


1≤k≤n 1≤k≤n


n
Astfel, ı̂n virtutea ipotezei xk = 0, avem:
k=1
n n
∑ 1 ∑ 1∑n

ak xk = (2ak − an − a1 )xk ≤ |2ak − an − a1 | · |xk | (∗)
2 2
k=1 k=1 k=1

Dar |2ak − an − a1 | ≤ an − a1 căci ea se reduce la

a2k − (a1 + an )ak + a1 an ≤ 0, k = 1, n



n
şi rezultă din a1 ≤ ak ≤ an . Ţinând seama că |xk | = 1, din (∗) rezultă
k=1
inegalitatea din enunţ.
1 1
9.43. Din ipoteză rezultă x − ≤ 0 şi y − ≤ 0, deci
2 2
( )( )
1 1 x y 1
x− y− ≥ 0, adică xy − − + ≥ 0.
2 2 2 2 4
Adunând inegalitatea de mai sus cu ı̂ncă două inegalităţi analoage,
rezultă enunţul.

265
9.44. Presupunem prin absurd că
1
|f (x) + g(y) − xy| < , pentru orice x, y ∈ [0, 1].
4
Atunci, ı̂n particular, rezultă că:
1 1 1
|f (0) + g(1)| < , |f (0) + g(0)| < şi |f (1) + g(0)| < .
4 4 4
Folosind inegalitatea |a + b| ≤ |a| + |b|, pentru orice a, b ∈ R se va obţine:

|1 − f (1) − g(1)| ≥ 1 − |f (1) + g(1)| ≥ 1 − |f (1) + g(0) − g(0) + g(1)|

≥ 1 − |f (1) + g(0)| − | − g(0) + g(1)|


1 1
≥ 1 − |f (1) + g(0)| − | − g(0) − f (0)| − |g(1) − f (0)| > 1 − 3 · =
4 4
ceea ce constituie o contradicţie.

9.46. Inegalitatea dată se mai scrie sub forma:

x(2z + y − 2) + yz − y − z + 1 ≥ 0.

Notând y = b, z = c şi considerând funcţia

f : [0, b] → R, f (x) = x(2c + b − 2) + bc − b − c + 1,

avem:
f (x) ≤ min(f (0), f (b)), pentru orice x ∈ [0, b]
deoarece funcţia este liniară. Vom arăta că f (0), f (b) ≥ 0, de unde
rezultă inegalitatea din enunţ. Avem:

f (0) = bc − b − c − 1 = (1 − b)(1 − c) ≥ 0 şi

f (b) = b(2c + b − 2) + bc − b − c + 1 = b2 + 3bc − 3b − c + 1.


Considerăm funcţia g : [b, 1] → R, g(x) = (3b − 1)x + b2 − 3b + 1. Avem
g(x) ≥ min(g(b), g(1)) şi cum g(b) = (3b−1)b+b2 −3b+1 = (2b−1)2 ≥ 0
iar g(1) = b2 ≥ 0, rezultă că g(x) ≥ 0, pentru orice x ∈ [b, 1]. Cum
f (b) = g(c), a rezultat că f (b) ≥ 0. Pentru a obţine cazurile de egalitate
analizăm situaţiile:
1) 2c + b − 2 > 0. În acest caz f este strict crescătoare. Egalitatea are
loc dacă şi numai dacă x = 0 şi f (b) = (b − 1)(c − 1) = 0. Cum b ≤ c,
rezultă x = 0, z = 1, y ∈ (0, 1).

266
2) 2c + b − 2 = 0. Semnul egal are loc ⇔ (b − 1)(c − 1) = 0, deci z = 1
şi y = x = 0.
3) 2c + b − 1 < 0. În acest caz f este strict crescătoare. Semnul egal
are loc dacă şi numai dacă x = b şi f (b) = 0 ceea ce implică g(c) =
(3b − 1)c + b2 − 3b + 1 = 0.
Apar subcazurile:
1
a) b < . Atunci g este strict descrescătoare şi deci egalitatea are loc
3
⇔ c = 1, g(1) = 0 ceea ce implică b = 0 şi cazul nu convine.
1
b) b = , e un caz ce nu convine.
3
1 1
c) b > implică c = b şi g(b) = 0, deci b = . Prin urmare egalitatea ı̂n
3 2
inegalitatea enunţată are loc dacă şi numai dacă x = 0, y ∈ [0, 1], z = 1
sau x = y = z = 1/2.

9.47. Vom demonstra inegalitatea


1
(1 − x1 )(1 − x2 ) . . . (1 − xn ) ≤
1+s
prin inducţie, cealaltă inegalitate din enunţ fiind evidentă. Fie

sn = x1 + x2 + . . . + xn şi tn = (1 − x1 )(1 − x2 ) . . . (1 − xn ), n ≥ 1.

Pentru n = 1, s = x1 şi inegalitatea revine la


1
1 − x1 ≤ ⇔ 1 − x21 ≤ 1,
1 + x1

evident. Înmulţind inegalitatea de demonstrat cu 1 − xn+1 , avem:


1
(1 − x1 )(1 − x2 ) . . . (1 − xn )(1 − xn+1 ) ≤ (1 − xn+1 )
1 + sn
1 − x2n+1 1 1
= ≤ ≤
(1 + sn )(1 + xn+1 ) (1 + sn )(1 + xn+1 ) 1 + sn+1
ceea ce ı̂ncheie demonstraţia prin inducţie.

9.48. Avem:
( n )2 ( n )( n ) ( n )
∑ ∑ ∑ 1 ∑
A−B = xi + xi − n2 − n x2i
x1
i=1 i=1 i=1 i=1

267
( ) n ( ) ( )

n ∑ 1 ∑
n
1
= xi xi + −n xi xi + .
xi xi
i=1 i=1 i=1

1
Notăm yi = + xi , i = 1, n şi deducem că
xi
( n )( n )
∑ ∑ ∑
n
A−B = xi yi − n xi yi .
i=1 i=1 i=1

De asemenea se observă că:


1 1 (xi − xj )(xi xj − 1)
yi − yj = − + xi − xj = , adică
xi xj xi xj

(xi xj − 1)(xi − xj )2
(xi − xj )(yi − yj ) = .
xi xj
Dar
n ∑
∑ n
S= (xi − xj )(yi − yj )
j=1 i=1
( n )

n ∑ ∑
n ∑
n
= xi yi − yj xi − xj yi + nxj yj
j=1 i=1 i=1 i=1

( n ) n  ( ) n 

n ∑ ∑ ∑
n ∑ ∑
n
=n xi yi − xi  yj  − yi  xj  + n xj yj
i=1 i=1 j=1 i=1 j=1 j=1

( n ) n 

n ∑ ∑
= 2n xi y i − 2 xi  yj  ,
i=1 i=1 j=1

deci S = 2(B − A). Astfel distingem două cazuri:


1) Dacă xi ∈ (0, 1], i = 1, n, atunci pentru orice pereche (i, j) avem:

(xi − xj )(yi − yj ) ≤ 0,

ceea ce implică S ≥ 0 ⇔ A ≥ B.
2) Dacă xi ≥ 1, i = 1, n, atunci pentru orice pereche (i, j) avem:

(xi − xj )(yi − yj ) ≥ 0,

268
ceea ce implică S ≥ 0, adică B ≥ A. De asemenea observăm că S = 0 ⇔
A = B ⇔ x1 = x2 = . . . = xn sau y1 = y2 = . . . = yn . În concluzie,
avem:
{
A, dacă pentru orice i = 1, n avem xi ∈ (0, 1]
max(A, B) =
B, dacă pentru orice i = 1, n avem xi ≥ 1.

9.49. Pentru n = 1 are loc semnul egal. Pentru n = 2 obţinem:


x1 x2 x1 x2 (x1 x2 − 1)
+ + (1 − x1 )(1 − x2 ) ≤ 1 ⇔ ≤ 0,
x2 + 1 x1 + 1 (x1 + 1)(x2 + 1)
ceea ce are loc. Egalitatea se obţine pentru x1 = 0 sau x2 = 0 sau
x1 x2 = 1. Presupunem inegalitatea adevărată pentru n, adică: Dacă
xi ∈ [0, 1], i = 1, n + 1 şi S = x1 + . . . + xn , atunci:

n
xi ∏ n
+ (1 − xi ) ≤ 1.
S − xi + 1
i=1 i=1

Rezultă că:

n ∑
n
xi
(1 − xi ) ≤ 1 − .
S − xi + 1
i=1 i=1
Pentru a demonstra inegalitatea corespunzătoare lui n + 1 considerăm
numerele xi ∈ [0, 1], i = 1, n + 1 şi (fără a restrânge generalitatea) pre-
supunem că xn+1 ≤ xi , i = 1, n + 1. Avem:
[ n ] ( )
∏ ∑n
xi
(1 − xi ) (1 − xn+1 ) ≤ 1 − (1 − xn+1 )
S − xi + 1
i=1 i=1


n
xi ∑ n
xi not
=1− − xn+1 + xn+1 = E.
S − xi + 1 S − xi + 1
i=1 i=1
Pentru a ı̂ncheia demonstraţia va trebui să mai verificăm inegalitatea:

n
xi xn+1
E ≤1− − .
S + xn+1 + 1 − xi S + 1
i=1

Această inegalitate se scrie succesiv:


( )
1 ∑ xi
xn+1 −1+
S+1 S − xi + 1
i=1

269

n
xi xn+1
≤ ⇔
(S − xi + 1)(S + xn+1 + 1 − xi )
i=1
[ ]

n
xi S + xn+1 − xi S
xn+1 · − ≤ 0.
S + 1 − xi S + 1 + xn+1 − xi S + 1
i=1

S + xn+1 − xi S
Dar ≤ deoarece este echivalentă cu xn+1 ≤ xi .
S + 1 + xn+1 − xi S+1
Putem deci scrie că:
( n )
∑n
xi S + xn+1 − xi S S ∑ xi
· − ≤ −1
S − xi + 1 S + 1 + xn+1 − xi S + 1 S + 1 S − xi + 1
i=1 i=1
( n )
S ∑ xi
≤ −1 = 0,
S+1 S
i=1

ceea ce ı̂ncheie soluţia problemei. Cazul de egalitate se realizează dacă


şi numai dacă toate cele n numere sunt egale cu 1 sau n − 1 dintre ele
sunt 0.

9.50. Avem:
√ √ √
x1 + x2 + . . . + xn ≥ 2 x1 − 1 + 2 2(x2 − 2) + . . . + 2 n(xn − n) ⇔
√ √
(x1 − 1 − 2 x1 − 1 + 1) + (x2 − 2 − 2 2(x2 − 2) + 2) + . . . +

+(xn − n − 2 n(xn − n) + n) ≥ 0 ⇔
√ √ √ √ √
( x1 − 1 − 1)2 + ( x2 − 2 − 2)2 + . . . + ( xn − n − n)2 ≥ 0,
ceea ce este evident. Egalitatea are loc dacă şi numai dacă xk − k = k ⇔
xk = 2k, pentru orice k ∈ {1, 2, . . . , n}.

9.51. Fie k, l, p numărul elementelor din şirul a1 , a2 , . . . , an egale cu 1, 2, 3


respectiv şi analog r, s, t pentru şirul b1 , b2 , . . . , bn . Astfel:
√ √ √ √
E = (k − r) + (l − s) 2 + (p − t) 3 = a + b 2 + c 3
√ √
unde a, b, c ∈ Z şi |a|, |b|, |c| ≤ n. Rezultă că |E| ≤ n(1 + 2 + 3). Fie
√ √
Ei = a ± b 2 ± c 3, i = 1, 3,

√ lui E. Atunci: P = E1 E2 E3 E ∈ Z. Dacă P = 0,


expresiile ”conjugate”

atunci a ± b 2 ± c 3 = 0 implică a = b = c = 0. Considerăm E minim

270
valoarea minimă strict pozitivă, deci f (n). Astfel E > 0 şi deci |P | ≥ 2
şi avem:

|P | 1 1 1
f (n) ≥ ≥ ≥ √ √ > .
|E1 E2 E3 | |E1 | · |E2 | · |E3 | (1 + 2 + 3)n3 4, 15n3

x+y+z
9.52. Putem presupune x ≤ y ≤ z. Dacă y ≤ atunci
3
x+y+z x+z x+y+z y+z
≤ ≤z şi ≤ ≤ z.
3 2 3 2
Prin urmare, există s, t ∈ [0, 1] astfel ı̂ncât să avem

x+z x+y+z y+z x+y+z


=s· + (1 − s)z şi =t· + (1 − t)z.
2 3 2 3
Adunând relaţiile de mai sus, deducem că

x + y − 2z x + y − 2z
= (s + t) · , deci s + t = 3/2.
2 3
Funcţia f fiind convexă, avem:
( ) ( )
x+z x+y+z
f ≤s·f + (1 − s) · f (z)
2 3
( ) ( )
y+z x+y+z
f ≤t·f + (1 − t) · f (z)
2 3
( )
x+y 1 1
f ≤ f (x) + f (y)
2 2 2
Adunând aceste trei inegalităţi şi ı̂nmulţind cu 2/3 rezultă enunţul.
x+y+z
Cazul < y se tratează analog, ţinând cont de faptul că atunci
3
x+z x+y+z y+z x+y+z
x≤ ≤ şi x≤ ≤ .
2 3 2 3

9.53. Inegalitatea se mai poate scrie


√( ) √( ) √( )
a+c n a n c n
(b + d) n
−1≥b n
−1+dn −1 ⇔
b+d b d

271
√( )n √( ) √
a+c b a n d n ( c )n
n
−1≥ n
−1+ − 1.
b+d b+d b b+d d

Fie f : [1, ∞) → R, f (x) = n
xn − 1. Avem

f ′ (x) = xn−1 (xn − 1)−


n−1
n şi

f ′′ (x) = −(n − 1)xn−2 (xn − 1)−


2n−1
n < 0, pentru orice x ∈ (1, ∞)
şi deci f este concavă pe (1, ∞). Din definiţia concavităţii rezultă că:
(a) (c) ( ) ( )
b d b a d c a+c
·f + ·f ≤f · + · =f
b+d b b+d d b+d b b+d d b+d
de unde rezultă enunţul.

9.54. Demonstrăm inegalitatea prin inducţie. Pentru n = 2 avem 0 < x1 ≤ x2


şi inegalitatea revine la:
x1 x2 x2 x1
+ ≥ + ,
x2 x1 x1 x2
ceea ce evident are loc. Presupunând că 0 < x1 ≤ x2 ≤ . . . ≤ xn implică
x1 x2 xn x2 x3 x1
+ + ... + ≥ + + ... + ,
x2 x3 x1 x1 x2 xn
va trebui să demonstrăm că dacă 0 < x1 ≤ x2 ≤ . . . ≤ xn ≤ xn+1 ,
atunci:
x1 xn−1 xn xn+1 x2 x3 xn+1 x1
+ ... + + + ≥ + + ... + + .
x2 xn xn+1 x1 x1 x2 xn xn+1
Avem: ( )
x1 xn−1xn xn+1
+ ... + + +
x2 xnxn+1 x1
( )
x2 x3 x1 xn xn xn+1
≥ + + ... + − + + ,
x1 x2 xn x1 xn+1 x1
din ipoteza de inducţie. Inegalitatea va fi demonstrată dacă vom verifica
inegalitatea:
x2 x3 x1 xn xn xn+1 x2 xn xn+1 x1
+ +. . .+ − + + ≥ +. . .+ + + ⇔
x1 x2 xn x1 xn+1 x1 x1 xn−1 xn xn+1
(xn+1 − xn )(x1 − xn )(x1 − xn+1 ) ≥ 0,
ceea ce evident are loc.

272
9.55. Demonstrăm inegalitatea prin inducţie. Pentru n = 2 ea se reduce la

a21 − a22 ≥ (a1 − a2 )2 ⇔ (a1 − a2 )2a2 ≥ 0,

evident. Considerând funcţia

fn (x) = a21 − a22 + . . . + (−1)n−2 a2n−1 + (−1)n−1 x2

−(a1 − a2 + . . . + (−1)n−2 an−1 + (−1)n−1 x)2 ,


fn : [0, an−1 ] → R. Se observă că dacă n este impar funcţia este liniară
iar pentru n par este o funcţie de gradul doi cu coeficientul lui x2 egal
cu −2. Prin urmare demonstrarea inegalităţii fn (x) ≥ 0 pentru orice
x ∈ [0, an−1 ] se reduce la a verifica condiţiile fn (0) ≥ 0 şi fn (an−1 ) ≥ 0
care se reduc la

a21 − a22 + . . . + (−1)n a2n−1 − (a1 − a2 + . . . + (−1)n an−1 )2 ≥ 0

şi respectiv

(a1 − a2 + . . . + (−1)n−3 an−2 )2 ≤ a21 − a22 + . . . + (−1)n−3 a2n−2 ,

ceea ce rezultă din ipoteza de inducţie.


a−c a−c
9.56. Scriem inegalitatea sub forma p ≤ q · +r· şi considerăm
b−c a−b
funcţia
a−c a−c
f : (c, a) → R, f (x) = q · +r· .
x−c a−x
Trebuie demonstrat că f (b) ≥ p. Avem:
( )
′ r q
f (x) = (a − c) −
(a − x)2 (x − c)2
şi deci
f ′ (x) > 0 ⇔ r(x − c)2 > q(a − x)2 ⇔
√ √
√ √ a q+c r
r(x − c) > q(a − x) ⇔ x > √ √
q+ r
√ √
(dacă q + r = 0 rezultă r = q = p = 0 şi inegalitatea de demonstrat
√ √
a q+c r
este evidentă). În punctul x0 = √ √ funcţia admite minim, deci
q+ r
cum √ √
(a − c) q (a − c) r
x0 − c = √ √ şi a − x0 = √ √ ,
q+ r q+ r

273
rezultă:
√ √ √ √
q+ r q+ r √ √
f (b) ≥ f (x0 ) = q · √ +r· √ = ( q + r)2 ≥ p.
q r

9.57. Presupunem c = max(a, b, c) şi scriem inegalitatea sub forma:

(a − b)[f (a)g(b) − f (b)g(a)]

+(c − b)[g(a) − g(c)]f (b) + (c − a)[f (c) − f (b)]g(a) ≥ 0.


Deoarece (c − b)f (b)[g(a) − g(c)] > 0 şi (c − a)[f (c) − f (b)]g(a) ≥ 0,
rămâne să arătăm că: (a − b)[f (a)g(b) − f (b)g(a)] ≥ 0. În cazul a ≥ b,
din f (a) ≥ f (b) şi g(b) ≥ g(a) rezultă f (a)g(b) ≥ f (b)g(a), deci (a −
b)[f (a)g(b) − f (b)g(a)] ≥ 0, iar ı̂n cazul a ≤ b, din f (a) ≤ f (b) şi g(b) ≤
g(a) rezultă f (a)g(b) ≤ f (b)g(a), deci (a − b)[f (a)g(b) − f (b)g(a)] ≥ 0.

9.58. Dacă xk > 0 atunci xk şi ln xk sunt la fel ordonate, deci putem aplica
inegalitatea lui Cebâşev:
( n )( n )
∑n
1 ∑ ∑
xk · ln xk ≥ xk ln xk ⇔
n
k=1 k=1 k=1
( ) ( )

n
1 ∑
n ∏
n
ln xxk k ≥ xk ln xk ⇔
n
k=1 k=1 k=1

n
xk
( ) ( ) k=1

n ∏
n n

ln xxk k ≥ ln xk .
k=1 k=1

Considerând numerele xk , ln f (xk ) care sunt la fel ordonate, din inegal-


itatea lui Cebâşev deducem:

n
xk
( ) k=1

n ∏
n n

ln [f (xk )]xk ≥ ln f (xk ) ,


k=1 k=1

ceea ce demonstrează enunţul.

9.59. 1) Folosind inegalitatea lui Minkowski, avem:


√ √
f (x) = x2 − 2mx + n2 + x2 − 2px + q 2

274
√ √ √ √
= (x − m) + ( n − m ) + (p − x)2 + ( q 2 − p2 )2
2 2 2 2

√ √ √
≥ (x − m + p − x)2 + ( n2 − m2 + q 2 − p2 )2
√ √ √
= (p − m)2 + ( n2 − m2 + q 2 − p2 )2

deci minimul funcţiei este această constantă.


2) Avem:
√ √ √ √ √
(x − p) + ( q − p ) + (m + p)2 + ( n2 − m2 − q 2 − p2 )2
2 2 2 2

√ √ √ √
≥ (x − p + m + p)2 + ( n2 − m2 − q 2 − p2 + q 2 − p2 )2

= x2 + 2mx + n2

de unde √ √
f (x) = x2 + 2mx + n2 − x2 − 2px + q 2
√ √ √
≤ (m + p)2 + ( n2 − m2 − q 2 − p2 )2 .

Acest ultim număr este evident maximul funcţiei considerate.

9.60. Fie x = aα, y = aβ cu 0 ≤ α, β < 1. Avem:

2a2 − x2 − y 2 2a2 − a2 (α2 + β 2 ) 2 − (α2 + β 2 )


= = ≤2
a2 − xy a2 − a2 αβ 1 − αβ

care revine la

2 − (α2 + β 2 ) ≤ 2 − 2αβ ⇔ α2 + β 2 ≥ 2αβ ⇔ (α − β)2 ≥ 0,

ceea ce este evident. Semnul egal se obţine când x = y.

9.61. Expresia dată reprezintă pătratul distanţei dintre sfertul de cerc x2 +


y 2 = 2 din primul cadran şi jumătatea de hiperbolă xy = 9 din acelaşi
prim cadran al axelor de coordonate. Cum tangentele la cele două curbe
ı̂n (1,1) şi (3,3) separă curbele şi sunt amândouă perpendiculare pe
dreapta x = y, rezultă că minimul pătratului distanţei căutate este
(3 − 1)2 + (3 − 1)2 = 8.

275
9.62. Să presupunem reordonând că a1 < a2 < . . . < an . Atunci pentru orice
x ∈ [ak , ak+1 ] avem:
f (x) = (x − a1 ) + . . . + (x − ak ) + (ak+1 − x) + . . . + (an − x),
deci


 −nx + (a1 + a2 + . . . + an ), x < a1



(2k − n)x + (−a1 − a2 − . . . − ak + ak+1 + . . . + an ),
f (x) =

 x ∈ [ak , ak+1 ]



nx − (a1 + a2 + . . . + an ), x > an
de unde: 

 −n, x < a1

f ′ (x) = 2k − n, ak < x < ak+1



n, n > an .
Calculând derivatele laterale ı̂n ak , k = 1, n obţinem
fd′ (ak ) = 2k − n şi fs′ (ak ) = 2(k − 1) − n,
de unde rezultă că f nu este derivabilă ı̂n ak . Studiem acum semnul lui
f ′ pe R − {a1 , a2 , . . . , an }.
1) Dacă n = 2s, s ∈ N, avem:

 < 0, x ∈ (−∞, a1 ) ∪ (a1 , a2 ) ∪ . . . ∪ (a[ n2 ]−1 , a[ n2 ] )


f ′ (x) = = 0, x ∈ (a[ n2 ] , a[ n2 ]+1 )



> 0, x ∈ (a[ n2 ]+1 , a[ n2 ]+2 ) ∪ . . . ∪ (an−1 , an ) ∪ (an , ∞)
deci f este strict descrescătoare pe (−∞, a[ n2 ] ), constantă pe
(a[ n2 ] , a[ n2 ]+1 ) şi strict crescătoare pe (a[ n2 ]+1 , ∞).
2) Dacă n = 2s + 1, s ∈ N, avem:
{
< 0, x ∈ (−∞, a1 ) ∪ (a1 , a2 ) ∪ . . . ∪ (a[ n2 ] , a[ n2 ]+1 ),
f ′ (x) =
> 0, x ∈ (a[ n2 ]+1 , a[ n2 ]+2 ) ∪ . . . ∪ (an−1 , an ) ∪ (an , ∞),
deci f este strict descrescătoare pe (−∞, a[ n2 ]+1 ) şi strict crescătoare pe
(a[ n2 ]+1 , ∞). Rezultă din consideraţiile de mai sus că minimul funcţiei
din enunţ, ı̂n ipoteza a1 < a2 < . . . < an este atins ı̂n x0 = a[ n2 ]+1 , deci

n
min f (x) = |a[ n2 ]+1 − ai |.
x∈R
i=1

276
9.63. Folosind inegalitatea mediilor, avem

1 1 1 √
f (x) = √ + √ ≥2 √ ≥ 2 1 = 2,
n
1+x n
1−x n
1 − x2
√ 1
căci n
1 − x2 ≤ 1 implică √ ≥ 1. Se atinge minimul dacă şi numai
n
1 − x2
dacă 1 − x2 = 1 ⇔ x = 0.
Observaţie. O a doua soluţie se poate obţine observând că

f (x) = (1 + x)− n + (1 − x)− n


1 1

are derivata
1
f ′ (x) = − [(1 + x)− n −1 − (1 − x)− n −1 ]
1 1

n
şi deci
f ′ (x) = 0 ⇔ 1 + x = 1 − x ⇔ x = 0.
Cum f ′ (x) > 0, pentru orice x ∈ (0, 1) rezultă că x = 0 este punct minim
şi deci f (x) ≥ f (0) = 2, pentru orice x ∈ [0, 1).

9.64. Într-adevăr, dacă ı̂ntre a şi b nu există nici un cub perfect


√ atunci există

n ∈ N astfel ı̂ncât n3 ≤ √ b < a < (n + 1)3 , adică n ≤ 3 b şi 3 a ≤ n + 1,

de unde obţinem 3 a − 3 b ≤ 1, ceea ce contrazice ipoteza.
Aplicaţie. Dacă n ∈ N, n ≥ 10 atunci ı̂ntre n şi 3n există cel puţin un
cub perfect (T. Andreescu).
Pentru n = 10, 11, . . . , 15 afirmaţia se verifică direct. Pentru n ≥ 16
avem:
1 1
n > (2, 5)3 = > √ ,
(1, 4 − 1)3
( 3 − 1)3
3

de unde rezultă:
√ 1 √ √
n> √ , adică 3n − 3 n > 1.
3 3
3
3−1
Ţinând cont de proprietatea din enunţul problemei deducem că ı̂ntre n
şi 3n există cel puţin un cub perfect.

9.65. Fără a restrânge generalitatea problemei presupunem că 1 ≤ a1 < a2 <


. . . < ak . Deoarece sumele sunt distincte, rezultă că:

a1 + a2 + . . . + ai ≥ 2i − 1 = 1 + 2 + . . . + 2i−1 , i = 1, k

277
şi astfel rezultatul dorit se obţine particularizând ı̂n lema următoare prin
xi = ai , yi = 2i−1 .
Lemă. Dacă 0 < x1 y1 < . . . < xk yk şi

x1 + x2 + . . . + xi ≥ y1 + y2 + . . . + yi , i = 1, k,

atunci

k
1 ∑
k
1

xi yi
i=1 i=1

cu egalitate dacă şi numai dacă xi = yi , i = 1, k.



i
1
Demonstraţie. Notăm Si = (xl − yl ), Zi = , i = 1, k şi fie
xi y i
l=1
Zk+1 = 0. Rezultă Si ≥ 0 şi Zi > Zi+1 , i = 1, k, deci:
k (
∑ ) ∑
k ∑
k
1 1
− = (xi − yi )Zi = Si (Zi − Zi+1 ) ≥ 0,
yi xi
i=1 i=1 i=1

cu egalitate ⇔ Si = 0, i = 1, k ⇔ xi = yi , pentru orice i = 1, k.

278
Indice de autori

— A — 9.50, 237 2.22, 39 Damian, M. 2.41, 51 2.52, 42


Alazian, M.E. Bătineţu,D.M. 2.66, 54 7.20, 181 2.45, 51 9.26, 234
2.42, 41 5.23, 150 2.81, 45 Darga, Gh. Giol, A. 9.55, 238
Alexandrescu,P. 4.10, 126 2.90, 46 1.46, 9 1.67, 10 9.56, 238
9.53, 238 4.22, 127 3.18, 105 Deneş, C. Gonciulea, N.
Alihanov,G.G. 4.26, 127 7.16, 180 3.17, 109 3.8, 108
— L —
2.36, 40 Bătineţu,Maria 9.22, 233 7.8, 180 3.13, 104
Lambă,Cristina
2.44, 41 2.38, 51 9.36, 235 Diaz, I. Grigorescu, D.
1.36, 8
Anca, D. Balaj, M. 9.57, 238 3.27, 106 4.26, 127
Lascu, M.
2.48, 42 5.4, 148 9.7, 232 Dicu, M.
2.39, 51
Andras, S. Bazon, V. 9.8, 232 2.45, 41 — H — 2.72, 54
2.77, 55 2.13, 38 Caragea, C. Dinică, M. Hârţan, N. 2.73, 54
5.12, 148 Becheanu, M. 2.19, 48 2.27, 40 9.5, 231 2.74, 55
5.20, 150 7.18, 185 2.71, 54 Douglas,A.J.2.79, Haivas, M. 2.75, 55
5.21, 150 9.9, 241 2.43, 41 55 3.2, 108 2.77, 55
8.12, 211 Bencze, M. 2.46, 42 Drâmbe,M.O. 7.9, 183 2.82, 56
9.40, 244 5.16, 149 2.69, 44 2.61, 43 Hammer, F.D. 3.17, 109
9.41, 244 4.28, 127 Ceteraş, M. 9.23, 234 9.39, 244 4.20, 130
9.42, 245 7.9, 180 1.19, 6 Harris, L. 4.26, 131
Andreescu, T. 9.27, 234 Chiş, M. 5.28, 151
— E — 5.16, 149
3.6, 108 Berindeanu, 8.9, 210
Eckstein, Gh. 5.21, 150
7.12, 184 Mihaela Chiriţă, M.
2.13, 48 — I — 5.4, 148
7.22, 185 1.66, 10 5.15, 149
2.23, 39 Ionescu Ţiu,C. 5.6, 148
9.15, 241 Berkolaiko,L.K. 7.17, 184
4.12, 126 1.31, 12 5.7, 148
9.17, 241 2.84, 46 9.21, 242
Enescu, F. 1.41, 13 7.23, 185
2.67, 44 2.91, 46 4.16, 126
7.3, 183 1.22, 7 7.24, 185
4.17, 126 Blaga, Al. 9.53, 238
Erdös, P. 1.23, 7 7.27, 186
7.4, 179 1.13, 11 Cocea, C.
9.65, 239 9.10, 241
9.64, 239 2.72, 45 4.11, 126
9.66, 239 9.12, 241
Andrei, Gh. 6.8, 163 Constantin, A. — J —
Ermilov, A. 9.18, 241
2.56, 43 9.2, 231 1.47, 9 Janons, W.
2.41, 41 9.23, 242
2.87, 46 9.21, 233 Constantinescu,Al. 2.83, 56 9.32, 243
4.14, 126 Brânzănescu,V. 1.30, 7 7.7, 183 9.35, 243
4.32, 128 9.28, 234 Crâşmăreanu,M. — F — 7.22, 181 9.40, 244
7.4, 179 Buşneag, D. 1.14, 11 Fianu, Gh. Jovanovic,M. 9.41, 244
Andrica, D. 2.59, 43 5.14, 149 2.57, 53 7.19, 181 9.42, 245
7.15, 184 Bucicowski,O. 1.27, 7 Fianu, M. Just, E. 9.48, 246
1.65, 10 3.19, 105 Crânganu, J. 2.71, 54 6.21, 167 9.49, 246
4.18, 126 Buliga, L. 9.16, 241 Fomin, S.
1.20, 6
Aoyagi, M. 2.80, 45 2.86, 46 2.56, 52
— K — 1.6, 5
2.70, 54 6.11, 164 Cseh, L. Frangulea,Şt.
Kantor, I.L. 1.70, 10
Apostol, C. Burcă, I. 2.31, 50 5.24, 150
2.83, 46 2.15, 38
4.25, 131 1.3, 10 Cucurezeanu,I. Fransen, A.
9.26, 234 2.28, 40
Askey, R.A. Bursuc, I. 9.13, 241 7.18, 181
Klamkin, M.S. 2.65, 44
8.15, 211 1.8, 5 5.13, 146 Funar, L.
2.81, 55 2.66, 44
Asparuhov, T. Curea, A. 9.51, 237
4.27, 131 2.74, 45
7.1, 179 1.1, 5
— C — 9.46, 245 2.75, 45
Atanasiu, I.
Cârjan, F. — G — Kolumban, J. 2.81, 45
1.6, 11
1.4, 5 — D — Galperin,L. 9.30, 243 2.89, 46
Cârtoaje, V. Dăncilă, I. 2.63, 44 Kurliandcik,L.D. 2.90, 46
— B — 2.9, 47 2.27, 49 Georgescu,R. 2.44, 51 2.92, 46
Băndilă, V. 9.23, 242 9.20, 233 1.5, 11 9.4, 240 3.16, 105
1.55, 9 2.11, 38 Daia, L. Ghenghiu, R. 1.69, 10 3.18, 105
8.42, 209 2.12, 38 7.21, 181 1.8, 11 2.49, 42 3.28, 106

279
3.8, 104 2.31, 50 9.24, 234 9.24, 242 Soncodi, A. 9.6, 240
4.15, 126 Metcalf, V. Niculescu,Liliana 9.46, 237 7.21, 181 2.73, 45
4.27, 127 3.27, 106 3.7, 108 Popescu, G. Stănean, M.
5.22, 147 Miculescu,Gh. 1.43, 13 2.82, 56
7.15, 180 4.24, 127 Popescu-P.P. 4.20, 130 — U —
— O —
7.6, 179 Miheţ, D. 1.6, 5 4.26, 131 Uşakov,R.P.
Onofraş, E.
9.17, 233 4.10, 129 7.5, 179 2.9, 38 9.18, 241 2.60, 43
9.18, 233 4.23, 130 9.35, 243 9.37, 235
9.19, 233 3.15, 105 4.23, 127 Ursu, I.
— R —
9.25, 234 3.26, 106 — P — 7.29, 182 2.64, 53
Pârşan, L. Rădulescu,M.
9.31, 235 3.30, 106 9.31, 235 2.88, 46
1.32, 7 8.45, 209
9.36, 235 4.31, 128 Stoica, Gh. 9.9, 232
1.37, 8 Radu, D.
9.48, 237 Mitrinovic̀,D.S. 7.19, 185 2.20, 39
Leggett, J. 2.60, 53 Păltănea, E. Stroeker, R.J.
9.21, 233 Rotar, C. — V —
2.78, 55 9.11, 232 9.47, 246 8.14, 211
Lohne, J. 9.3, 231 Păuna, N. Szölösy, Gh. Vâjâitu,V.
2.62, 53 Rudniki, R. 2.43, 41
7.18, 181 9.42, 236 1.30, 12 2.23, 49
Love, J.B. 9.6, 232 Panaitopol, L. Szörös, Al. 9.30, 235
2.55, 52 2.32, 40 Vickers,G.T.
7.14, 180 Miu, I. 2.48, 52
Lupaş, Al. 1.9, 11 3.18, 110 1.24, 7 2.79, 55
3.11, 109 Mnich, W. 4.14, 130 — S — 1.43, 8 Vlaicu, L.
3.12, 109 2.17, 48 5.22, 150 Saffert,H.J. 2.16, 38 7.13, 184
3.13, 109 Mocanu, P. 9.11, 241 6.19, 167 2.25, 39
7.30, 187 4.20, 126 9.28, 243 Safta, I. 2.47, 42
8.10, 210 Mogoşanu,M. 9.48, 246 2.82, 46 — T — 3.28, 106
3.24, 105 6.10, 167 9.49., 246 Satnoianu,R. Tănasie, D. 7.7, 179
2.5, 37 2.59, 53 7.32, 187 Vulpescu
Moldovan, C.
3.32, 106 6.13, 164 Tereşin, D. Jalea,F.
2.22, 49
— M — 4.19, 126 Schaumberger,N. 2.26, 49 4.9, 129
Molea, Gh.
Müller, H. 4.3, 125 5.27, 151 2.35, 50 6.9, 167
2.24, 39
9.49, 237
Mortici, C. 6.2, 163 6.21, 167 Tifui, V. 4.10, 126
Mârşacova,T.G. 7.25, 182 Schwartz,D. 2.10, 48
2.35, 40
9.59, 239
9.12, 232 7.26, 182 7.16, 184 Tiotoi, I.
Marghescu, 9.45, 237 Seclăman,D. 2.54, 52 — W —
Murty,V.N.
Gh. Papacu, N. 3.5, 108 Tomescu, I. Walsh,J.L.
6.16, 167
2.28, 49 7.27, 182 4.25, 127 7.25, 186 7.12, 180
6.17, 167
2.19, 39 Paterău, V. Sergheev,I.N. 9.45, 237 Weiss, M.
9.37, 244
2.39, 41 7.2, 179 9.14, 232 Toteanu, T. 6.14, 164
7.6, 179
Marinescu, D. Peiţa, V. Shafer,R.E. 9.8, 240 Wilansky,A.
9.2, 240 2.18, 48 6.15, 167 Tsintsifas,G. 6.20, 167
Marinescu,D.Şt. — N — Pop, O. Skatoniuk,M.B. 2.80, 55
9.22, 242 Neacşu, C. 2.73, 54 2.53, 42 Tuţescu, L.
9.31, 243 9.32, 243 2.74, 55 Smarandache,S. 1.3, 5 — Z —
2.70, 44 Nedelcu,C.T. 2.75, 55 1.38, 13 1.61, 10 Zaharescu,A
5.23, 147 2.15, 48 1.45, 8 1.39, 13 9.5, 231 2.43, 51
Mascioni, V. 1.54, 9 9.21, 233 1.42, 13 9.58, 238 9.30, 235
8.11, 210 Nesbitt, J. Popa, C. 2.11, 48 Tudor, I. Zeitlin,D.
Matizen,V.E. 2.83, 56 7.11, 184 1.63, 10 6.12, 164 7.23, 181
9.38, 236 Nicula, V. Popa, E. 2.6, 37 Tudor, V. Zidaru,V.
9.39, 236 3.19, 110 7.8, 183 Smoliakov,A.N. 1.2, 10 2.72, 54
Mavlo,D.P. 9.25, 242 Popa, S. 2.37, 41 2.53, 52 7.31, 187
9.13, 232 9.5, 240 3.16, 109 Solomon,I. 2.14, 38 2.62, 44
Mereni, I. 2.31, 40 Popa, V. 5.25, 150 Tuszon, Z. 3.1, 103

280
Bibliografie

[1] Andrei, Gh., Caragea, C., Cucurezeanu, I., Bordea, Gh.: Probleme de Algebră
pentru concursuri de admitere şi olimpiade şcolare vol.I, vol.II, Editura Didactică
şi Pedagogică (1993).
[2] Beckenbach, E., Bellman, R.: Inequalities, Springer-Verlag, Berlin (1961).
[3] Hardy, G.H., Litllewood, J.E., Polya, G.: Inequalities, Cambridge, University Press
(1967).
[4] Lascu, M.: Inegalităţi. Aplicaţii - Lucrare de diplomă, Universitatea Babeş-Bolyai,
Cluj-Napoca (1982).
[5] Lascu, M.: Inegalităţi, Lucrare metodico-ştiinţifică pentru obţinerea gradului di-
dactic I, Universitatea Babeş-Bolyai, Cluj-Napoca (1994).
[6] Lascu, M.: Inegalităţi, Editura GIL (1994).
[7] Miheţ, D.: Principiul trinomului, Caiete metodice, Timişoara.
[8] Mitrinovic, D.S.: Analytic Inequalities, Springer-Verlag (1970).
[9] Năstăsescu, C., Niţă, C.: Exerciţii şi probleme de algebră, Editura Didactică şi
Pedagogică (1983).
[10] Panaitopol, L., Ottescu, C.: Probleme date la Olimpiadele de matematică (1968-
1974), Editura Didactică şi Pedagogică, Bucureşti (1976).
[11] Panaitopol, L., Drăghicescu, I.C.: Polinoame şi ecuaţii algebrice, Editura Alba-
tros (1980).
[12] Tomescu, I. şi colaboratori: Probleme date la olimpiadele de matematică pentru
licee (1950-1990), Editura Didactică şi Pedagogică, Bucureşti (1991).
[13] Colecţia Gazetei Matematice (1950-1994).
[14] Colecţia Revistei de matematică a elevilor din Timişoara (1972-1992).

281

S-ar putea să vă placă și